Sie sind auf Seite 1von 244

Arbeitsbuch Grundwissen Mathematikstudium

Martin Brokate Norbert Henze Frank Hettlich Andreas Meister


Gabriela Schranz-Kirlinger Thomas Sonar

Arbeitsbuch Grundwissen
Mathematikstudium
Höhere Analysis, Numerik und Stochastik

Aufgaben, Hinweise, Lösungen und Lösungswege

unter Mitwirkung von Daniel Rademacher


Autoren
Martin Brokate, TU München Zentrum Mathematik (M10), Garching, brokate@ma.tum.de
Norbert Henze, Karlsruher Institut für Technologie (KIT), norbert.henze@kit.edu
Frank Hettlich, Karlsruher Institut für Technologie (KIT), frank.hettlich@kit.edu
Andreas Meister, Universität Kassel, meister@mathematik.uni-kassel.de
Gabriela Schranz-Kirlinger, TU Wien, g.schranz-kirlinger@tuwien.ac.at
Thomas Sonar, TU Braunschweig FB 1 Mathematik und Informatik, Braunschweig, t.sonar@tu-bs.de

ISBN 978-3-642-54945-8 ISBN 978-3-642-54946-5 (eBook)


DOI 10.1007/978-3-642-54946-5
Die Deutsche Nationalbibliothek verzeichnet diese Publikation in der Deutschen Nationalbibliografie; detaillierte biblio-
grafische Daten sind im Internet über http://dnb.d-nb.de abrufbar.

Springer Spektrum
© Springer-Verlag Berlin Heidelberg 2016

Das Werk einschließlich aller seiner Teile ist urheberrechtlich geschützt. Jede Verwertung, die nicht ausdrücklich vom Ur-
heberrechtsgesetz zugelassen ist, bedarf der vorherigen Zustimmung des Verlags. Das gilt insbesondere für Vervielfältigungen,
Bearbeitungen, Übersetzungen, Mikroverfilmungen und die Einspeicherung und Verarbeitung in elektronischen Systemen.

Die Wiedergabe von Gebrauchsnamen, Handelsnamen, Warenbezeichnungen usw. in diesem Werk berechtigt auch ohne be-
sondere Kennzeichnung nicht zu der Annahme, dass solche Namen im Sinne der Warenzeichen- und Markenschutz-
Gesetzgebung als frei zu betrachten wären und daher von jedermann benutzt werden dürften.

Der Verlag, die Autoren und Herausgeber gehen davon aus, dass die Angaben und Informationen in diesem Werk zum
Zeitpunkt der Veröffentlichung vollständig und korrekt sind. Weder der Verlag noch die Autoren oder die Herausgeber
übernehmen, ausdrücklich oder implizit, Gewähr für den Inhalt des Werkes, etwaige Fehler oder Änderungen.

Planung und Lektorat: Dr. Andreas Rüdinger, Bianca Alton


Satz: EDV-Beratung Frank Herweg, Leutershausen
Einbandabbildung: © Jos Leys
Einbandentwurf: deblik, Berlin

Gedruckt auf säurefreiem und chlorfrei gebleichtem Papier

Springer-Verlag GmbH Berlin Heidelberg ist Teil der Fachverlagsgruppe Springer Science+Business Media
(www.springer.com)
Vorbemerkungen
Auf verschiedentlichen Wunsch bieten wir alle Aufgaben des Ein Punktesystem unterscheidet leichte Aufgaben •, mittel-
Buchs Brokate et al., Grundwissen Mathematikstudium – schwere •• und anspruchsvolle ••• Aufgaben. Die Lösungs-
Höhere Analysis, Numerik und Stochastik mit Hinweisen, hinweise helfen Ihnen, falls Sie bei einer Aufgabe partout
Lösungen und Lösungswegen als gedrucktes Buch. Die In- nicht weiterkommen. Für einen optimalen Lernerfolg schla-
halte des Buchs stehen als PDF-Dateien auch auf der Website gen Sie die Lösungen und Lösungswege bitte erst nach, wenn
matheweb zur Verfügung. Sie selber zu einer Lösung gekommen sind.
Verweise auf Seiten, Formeln, Abschnitte und Kapitel bezie-
Die Aufgaben gliedern sich in drei Kategorien: Anhand der
hen sich auf das Buch Grundwissen Mathematikstudium –
Verständnisfragen können Sie prüfen, ob Sie die Begriffe
Höhere Analysis, Numerik und Stochastik von Brokate et al.
und zentralen Aussagen verstanden haben, mit den Rechen-
aufgaben üben Sie Ihre technischen Fertigkeiten und die Be- Wir wünschen Ihnen viel Freude und Spaß mit diesem Ar-
weisaufgaben geben Ihnen Gelegenheit, zu lernen, wie man beitsbuch und in Ihrem Studium.
Beweise findet und führt.
Der Verlag und die Autoren
Inhaltsverzeichnis

Kapitel 2: Lineare Differenzialgleichungen – Kapitel 9: Fredholm-Gleichungen – kompakte


Systeme und Gleichungen höherer Ordnung 1 Störungen der Identität 69
Aufgaben . . . . . . . . . . . . . . . . . . . . . . . 1 Aufgaben . . . . . . . . . . . . . . . . . . . . . . . 69
Hinweise . . . . . . . . . . . . . . . . . . . . . . . 3 Hinweise . . . . . . . . . . . . . . . . . . . . . . . 70
Lösungen . . . . . . . . . . . . . . . . . . . . . . . 4 Lösungen . . . . . . . . . . . . . . . . . . . . . . . 70
Lösungswege . . . . . . . . . . . . . . . . . . . . . 5 Lösungswege . . . . . . . . . . . . . . . . . . . . . 71

Kapitel 3: Randwertprobleme und nichtlineare Kapitel 10: Hilberträume – fast wie im


Differenzialgleichungen – Funktionen sind Anschauungsraum 76
gesucht 13 Aufgaben . . . . . . . . . . . . . . . . . . . . . . . 76
Aufgaben . . . . . . . . . . . . . . . . . . . . . . . 13 Hinweise . . . . . . . . . . . . . . . . . . . . . . . 77
Hinweise . . . . . . . . . . . . . . . . . . . . . . . 14 Lösungen . . . . . . . . . . . . . . . . . . . . . . . 78
Lösungen . . . . . . . . . . . . . . . . . . . . . . . 15 Lösungswege . . . . . . . . . . . . . . . . . . . . . 78
Lösungswege . . . . . . . . . . . . . . . . . . . . . 16

Kapitel 11: Warum Numerische Mathematik? –


Kapitel 4: Qualitative Theorie – jenseits von Modellierung, Simulation und Optimierung 85
analytischen und mehr als numerische Lösungen 24 Aufgaben . . . . . . . . . . . . . . . . . . . . . . . 85
Aufgaben . . . . . . . . . . . . . . . . . . . . . . . 24 Hinweise . . . . . . . . . . . . . . . . . . . . . . . 86
Hinweise . . . . . . . . . . . . . . . . . . . . . . . 25 Lösungen . . . . . . . . . . . . . . . . . . . . . . . 86
Lösungen . . . . . . . . . . . . . . . . . . . . . . . 26 Lösungswege . . . . . . . . . . . . . . . . . . . . . 86
Lösungswege . . . . . . . . . . . . . . . . . . . . . 26

Kapitel 12: Interpolation – Splines und


Kapitel 5: Funktionentheorie – Analysis im
mehr 89
Komplexen 33
Aufgaben . . . . . . . . . . . . . . . . . . . . . . . 89
Aufgaben . . . . . . . . . . . . . . . . . . . . . . . 33
Hinweise . . . . . . . . . . . . . . . . . . . . . . . 90
Hinweise . . . . . . . . . . . . . . . . . . . . . . . 34
Lösungen . . . . . . . . . . . . . . . . . . . . . . . 90
Lösungen . . . . . . . . . . . . . . . . . . . . . . . 35
Lösungswege . . . . . . . . . . . . . . . . . . . . . 91
Lösungswege . . . . . . . . . . . . . . . . . . . . . 36

Kapitel 6: Differenzialformen und der allgemeine Kapitel 13: Quadratur – numerische


Satz von Stokes 41 Integrationsmethoden 99
Aufgaben . . . . . . . . . . . . . . . . . . . . . . . 41 Aufgaben . . . . . . . . . . . . . . . . . . . . . . . 99
Hinweise . . . . . . . . . . . . . . . . . . . . . . . 42 Hinweise . . . . . . . . . . . . . . . . . . . . . . . 100
Lösungen . . . . . . . . . . . . . . . . . . . . . . . 42 Lösungen . . . . . . . . . . . . . . . . . . . . . . . 100
Lösungswege . . . . . . . . . . . . . . . . . . . . . 43 Lösungswege . . . . . . . . . . . . . . . . . . . . . 101

Kapitel 7: Grundzüge der Maß- und Integrations- Kapitel 14: Numerik linearer Gleichungs-
theorie – vom Messen und Mitteln 47 systeme – Millionen von Variablen im Griff 104
Aufgaben . . . . . . . . . . . . . . . . . . . . . . . 47 Aufgaben . . . . . . . . . . . . . . . . . . . . . . . 104
Hinweise . . . . . . . . . . . . . . . . . . . . . . . 50 Hinweise . . . . . . . . . . . . . . . . . . . . . . . 105
Lösungen . . . . . . . . . . . . . . . . . . . . . . . 51 Lösungen . . . . . . . . . . . . . . . . . . . . . . . 105
Lösungswege . . . . . . . . . . . . . . . . . . . . . 52 Lösungswege . . . . . . . . . . . . . . . . . . . . . 106

Kapitel 8: Lineare Funktionalanalysis – Kapitel 15: Numerische Eigenwertberechnung –


Operatoren statt Matrizen 60 Einschließen und Approximieren 110
Aufgaben . . . . . . . . . . . . . . . . . . . . . . . 60 Aufgaben . . . . . . . . . . . . . . . . . . . . . . . 110
Hinweise . . . . . . . . . . . . . . . . . . . . . . . 61 Hinweise . . . . . . . . . . . . . . . . . . . . . . . 111
Lösungen . . . . . . . . . . . . . . . . . . . . . . . 62 Lösungen . . . . . . . . . . . . . . . . . . . . . . . 111
Lösungswege . . . . . . . . . . . . . . . . . . . . . 62 Lösungswege . . . . . . . . . . . . . . . . . . . . . 112
VIII INHALTSVERZEICHNIS

Kapitel 16: Lineare Ausgleichsprobleme – Lösungen . . . . . . . . . . . . . . . . . . . . . . . 165


im Mittel das Beste 115 Lösungswege . . . . . . . . . . . . . . . . . . . . . 166
Aufgaben . . . . . . . . . . . . . . . . . . . . . . . 115
Hinweise . . . . . . . . . . . . . . . . . . . . . . . 116
Kapitel 22: Stetige Verteilungen und allgemeine
Lösungen . . . . . . . . . . . . . . . . . . . . . . . 116
Betrachtungen – jetzt wird es analytisch 179
Lösungswege . . . . . . . . . . . . . . . . . . . . . 117
Aufgaben . . . . . . . . . . . . . . . . . . . . . . . 179
Hinweise . . . . . . . . . . . . . . . . . . . . . . . 183
Kapitel 17: Nichtlineare Gleichungen und Lösungen . . . . . . . . . . . . . . . . . . . . . . . 184
Systeme – numerisch gelöst 121 Lösungswege . . . . . . . . . . . . . . . . . . . . . 185
Aufgaben . . . . . . . . . . . . . . . . . . . . . . . 121
Hinweise . . . . . . . . . . . . . . . . . . . . . . . 122
Kapitel 23: Konvergenzbegriffe und Grenzwert-
Lösungen . . . . . . . . . . . . . . . . . . . . . . . 123
sätze – Stochastik für große Stichproben 200
Lösungswege . . . . . . . . . . . . . . . . . . . . . 123
Aufgaben . . . . . . . . . . . . . . . . . . . . . . . 200
Hinweise . . . . . . . . . . . . . . . . . . . . . . . 203
Kapitel 18: Numerik gewöhnlicher Differenzial- Lösungen . . . . . . . . . . . . . . . . . . . . . . . 204
gleichungen – Schritt für Schritt zur Trajektorie 128 Lösungswege . . . . . . . . . . . . . . . . . . . . . 205
Aufgaben . . . . . . . . . . . . . . . . . . . . . . . 128
Hinweise . . . . . . . . . . . . . . . . . . . . . . . 129
Kapitel 24: Grundlagen der Mathematischen
Lösungen . . . . . . . . . . . . . . . . . . . . . . . 130
Statistik – vom Schätzen und Testen 216
Lösungswege . . . . . . . . . . . . . . . . . . . . . 130
Aufgaben . . . . . . . . . . . . . . . . . . . . . . . 216
Hinweise . . . . . . . . . . . . . . . . . . . . . . . 220
Kapitel 19: Wahrscheinlichkeitsräume – Modelle Lösungen . . . . . . . . . . . . . . . . . . . . . . . 221
für stochastische Vorgänge 136 Lösungswege . . . . . . . . . . . . . . . . . . . . . 222
Aufgaben . . . . . . . . . . . . . . . . . . . . . . . 136
Hinweise . . . . . . . . . . . . . . . . . . . . . . . 139
Lösungen . . . . . . . . . . . . . . . . . . . . . . . 140
Lösungswege . . . . . . . . . . . . . . . . . . . . . 140

Kapitel 20: Bedingte Wahrscheinlichkeit und


Unabhängigkeit – Meister Zufall hängt (oft) ab 146
Aufgaben . . . . . . . . . . . . . . . . . . . . . . . 146
Hinweise . . . . . . . . . . . . . . . . . . . . . . . 149
Lösungen . . . . . . . . . . . . . . . . . . . . . . . 150
Lösungswege . . . . . . . . . . . . . . . . . . . . . 151

Kapitel 21: Diskrete Verteilungsmodelle – wenn


der Zufall zählt 160
Aufgaben . . . . . . . . . . . . . . . . . . . . . . . 160
Hinweise . . . . . . . . . . . . . . . . . . . . . . . 164
Kapitel 2 Aufgabe 2.8 •• Berechnen Sie eA für
⎛ ⎞
−1 −1 0
A = ⎝ 0 −1 0 ⎠ .
Aufgaben 0 0 −2

Verständnisfragen Aufgabe 2.9 •• Geben Sie für die lineare Differenzial-


gleichung
Aufgabe 2.1 • Welche der folgenden skalaren Diffe- y  (x) = Ai y(x), i = 1, 2, 3
renzialgleichungen 1. Ordnung sind linear?
mit den folgenden zweidimensionalen Matrizen
     
a) y(x)y  (x) − 2x = 0 c) xy  (x) + 3y(x) = ex 2 0 2 0 −2 2
b) y  (x) + xy(x) = 0 d) y  + (tan x)y = 2 sin x A1 = , A2 = , A3 =
0 2 1 2 0 2

Aufgabe 2.2 • Gibt es eine reelle 2 × 2-Matrix A mit jeweils eine Fundamentalmatrix an.
 
−1 0 Aufgabe 2.10 •• Bestimmen Sie jeweils ein reelles
eA = ?
0 −4 Fundamentalsystem für die Differenzialgleichung y  =
Ai y , i = 1, 2, 3 mit
Aufgabe 2.3 •• Die Funktionen y1 (x) = x 3 und ⎛ ⎞ ⎛ ⎞
−1 1 −1 3 −3 2
y2 (x) = |x|3 sind linear unabhängig auf (−1, 1), aber A1 = ⎝ 2 −1 2 ⎠ , A2 = ⎝ −1 5 −2 ⎠ ,
W [y1 , y2 ](x) = 0. Wie ist das möglich? 2 2 −1 −1 3 0
 
Aufgabe 2.4 • Welche der folgenden Funktionen 6 −17
A3 = .
a) y(x) = (2ex + e−x , e2x ) 1 −2
b) y(x) = (2ex + e−x , ex )
Aufgabe 2.11 • Berechnen Sie jeweils die Lösungen
c) y(x) = (2ex + e−x , xex )
der Differenzialgleichungen zu den Anfangsbedingungen
d) y(x) = (ex + 3e−x , ex + 3e−x )
y(0) = 1 und y  (0) = 0:
kann eine Lösung einer Differenzialgleichung
a) y  (x) = −y(x), b) y  (x) = y(x).
y  (x) = Ay(x)

mit A ∈ R2×2 sein? Aufgabe 2.12 •• Wie muss die rechte Seite f gewählt
werden, damit bei der linearen Differenzialgleichung
Aufgabe 2.5 • Formulieren Sie die Differenzialglei-
y  (x) + 2y  (x) + y  (x) + 2y(x) = f (x)
chungen
a) y  − (y  )2 y sin x = cosh x − y 2 , Resonanz auftritt?
b) y  + 2y  + y  = 2e3x ,
Aufgabe 2.13 •• Bestimmen Sie die Lösung der Diffe-
jeweils als ein System 1. Ordnung. renzialgleichung

y  (x) = x(1 + y(x)) ,


Rechenaufgaben
mit der Anfangsbedingung y(0) = 2.
Aufgabe 2.6 • Lösen Sie das Anfangswertproblem
Aufgabe 2.14 •• Betrachten Sie den allgemeinen har-
y  (x) = 4x, y(0) = 1.
monischen Oszillator mit Reibung, aber ohne äußere Anre-
gung
Aufgabe 2.7 • Zeigen Sie, dass die Funktion y  (x) + 2by  (x) + cy(x) = 0, b, c ∈ R .
cx
y(x) = , x ∈ I ⊆ R \ {−1}, c ∈ R Geben Sie in Abhängigkeit von b, c jeweils ein Fundamen-
1+x
talsystem an.
Lösung der Differenzialgleichung
Aufgabe 2.15 •• Bestimmen Sie die Lösung des An-
x(1 + x)y  (x) − y(x) = 0 fangswertproblems

ist. y  (x) + 5y  (x) + 6y(x) = cos x


M. Brokate et al., Arbeitsbuch Grundwissen Mathematikstudium – Höhere Analysis, Numerik und
Stochastik, DOI 10.1007/978-3-642-54946-5_1, © Springer-Verlag Berlin Heidelberg 2016
2 Aufgaben zu Kapitel 2

zu den Anfangsbedingungen y(0) = y  (0) = 1.1. Wählen Aufgabe 2.22 •• Die logistische Gleichung
Sie für die Partikulärlösung den Ansatz
y  (x) = λy(x)(K − y(x)) = λKy(x) − λy 2 (x) ,
yp (x) = d cos(x + δ) .
die wir zu Beginn dieses Kapitels kennengelernt haben, ist
Aufgabe 2.16 •• Gesucht ist eine Lösung der Differen- eine Bernoulli’sche Differenzialgleichung mit γ = 2. Lö-
zialgleichung sen Sie die logistische Differenzialgleichung und verifizieren
Sie, dass
y  (x) + y  (x) = x + 1 . K
y(x) =
1 + ( y0 − 1)e−λKx
K
Aufgabe 2.17 •• Bestimmen Sie die allgemeine Lö-
sung der Differenzialgleichung 3. Ordnung die Lösung zur Anfangsbedingung y(0) = y0 ist.

y  (x) − 3y  (x) + 2y(x) = 9ex .


Beweisaufgaben
Aufgabe 2.18 •• Berechnen Sie die Lösung der Diffe-
renzialgleichung Aufgabe 2.23 •• Sei Y ∈ C 1 (I, Rn×n ) eine Funda-
mentalmatrix für das lineare System y  (x) = A(x)y(x). Zei-
y  (x) = λy(x) , y(0) = 1, λ ∈ R gen Sie:
mithilfe eines Potenzreihenansatzes. a) Die Matrix X ∈ C 1 (I, Rn×n ) ist genau dann eine Funda-
mentalmatrix, wenn es eine reguläre Matrix B ∈ Rn×n
Aufgabe 2.19 •• Die Methode der sukzessiven Appro- gibt mit X(x) = Y (x) B für alle x ∈ I .
ximation oder auch Picard-Iteration ist nicht das einzige Itera- b) Die Matrix X(x) := Y (x)(Y (x0 ))−1 ist ein Hauptfunda-
tionsverfahren um (approximative) Lösungen von Anfangs- mentalsystem, d. h. ein Fundamentalsystem Y (x) mit der
wertproblemen zu erhalten. Ein klassisches Vorgehen ist, Eigenschaft Y (x0 ) = I n .
beim Startpunkt x0 eine Taylorreihe der Lösung y zu finden.
Die Idee ist durch Differenzieren der Differenzialgleichung Aufgabe 2.24 ••
y  (x) = f (x, y(x)) a) Seien A , B ∈ Rn×n mit AB = BA. Zeigen Sie eA+B =
eA eB = eB eA und e(s+t)A = es A et A für alle s, t ∈ R .
nach x die Werte y (n) (x0 ) , n = 0, 1, 2, ... zu bestimmen. b) Im allgemeinen gilt nicht eA+B = eA eB . Geben Sie ein
Geben Sie die Formeln für die gesuchten Werte y (n) (x0 ) für Gegenbeispiel an.
n = 1, 2, 3 an. Betrachten Sie die Differenzialgleichung

y = y2 , y(0) = 1 . Aufgabe 2.25 •• Sei A eine reelle n×n-Matrix. Zeigen


Sie:
Um die Koeffizienten der der Taylorreihe von y bei
 x0 zu be-n
rechnen, ist es geschickt mit dem Ansatz y(x) = ∞ a) det eA = eSp A
n=0 yn x T
zu arbeiten. Geben Sie eine Rekurrenz für die Koeffizienten b) eA = (eA )T
yn an. Konvergiert die Taylorreihe? Geben Sie das Konver- c) Aus AT = −A (d. h. A ist schiefsymmtrisch) folgt, dass
genzintervall an. eAx orthogonal ist und det eAx = 1 .

Aufgabe 2.20 •• Eine Gleichung der Gestalt Aufgabe 2.26 •

y  (x) = a(x)y(x) + b(x)y γ (x) a) Zeigen Sie, dass die m × m-Matrix N


⎛ ⎞
mit γ ∈ R und a, b stetigen Funktionen auf I heißt Ber- 0 1 0 ... 0 0
noulli’sche Differenzialgleichung. Zeigen Sie, dass für γ  = 0 ⎜ ⎟
⎜ 0 0 1 ... 0 0 ⎟
oder γ = 1 , der Ansatz z(x) = y 1−γ (x) auf eine lineare ⎜ ⎟
⎜ ⎟
⎜ . ⎟
Differenzialgleichung führt. ⎜ 0 0 0 .. 0 0 ⎟
N =⎜ . . ⎟,
⎜ . . .. .. .. .. ⎟
Aufgabe 2.21 •• ⎜ . . . . . . ⎟
Eine Gleichung der Gestalt ⎜ ⎟
⎜ . ⎟
⎝ 0 0 0 .. 0 1 ⎠
y  (x) = q(x) + p(x)y(x) + r(x)y 2 (x),
0 0 0 ... 0 0
wobei q, p und r stetige Funktionen auf I sind und r(x)  = 0
für jedes x ∈ I , heißt Riccati’sche Differenzialgleichung. nilpotent ist, d. h., das N k = 0 für alle k ≥ m , m ∈ N
Beweisen Sie, dass die Gleichung durch den Ansatz y(x) = gilt.
u(x) + v(x) in eine Bernoulli’sche Differenzialgleichung in b) Bestimmen Sie alle Eigenwerte einer solchen nilpotenten
v(x) überführt werden kann, falls eine spezielle Lösung u(x) Matrix N .
bekannt ist. c) Lösen Sie die Differenzialgleichung y  = N y für m = 3 .
Hinweise zu Kapitel 2 3

Aufgabe 2.27 •• Gegeben sei eine stetige Funktion Rechenaufgaben


f : R → R. Zeigen Sie, dass das Anfangswertproblem
Aufgabe 2.6 • Integration.
y  (x) = f (x), y(x0 ) = y0 , y  (x0 ) = y1
Aufgabe 2.7 • Differenzieren und Einsetzen.
eine eindeutige Lösung y : R → R besitzt.
Aufgabe 2.8 •• Berechnen Sie die Eigenwerte und die
Aufgabe 2.28 •• Für die Wahl q(x) = 2x, p(x) = zugehörigen Eigenvektoren und Hauptvektoren.
(1 − 2x) und r(x) = −1 lässt sich eine spezielle Lösung
dieser Riccati’schen Gleichung besonders einfach finden. Aufgabe 2.9 •• Matrixexponentialfunktion.
Berechnen Sie die dieser Gleichung entsprechenden Ber-
noulli’sche Differenzialgleichung und lösen Sie diese. Aufgabe 2.10 •• Eigenwerte und Eigenvektoren aus-
rechnen.
Aufgabe 2.29 •• Beweisen Sie: Erfüllt eine Funktion F
die Voraussetzungen des Satzes von Picard-Lindelöf für je- Aufgabe 2.11 • –
des a. I = [x0 − a, x0 + a] und Q = Cn , dann existiert eine
auf ganz R definierte eindeutige Lösung des Anfangswert- Aufgabe 2.12 •• Resonanz tritt auf, wenn die charak-
problems y  (x) = F (x, y(x)) und y(x0 ) = y 0 . teristische Gleichung p(λ) = 0 entweder eine mehrfache
Nullstelle hat oder in der Inhomogenität f ein Term eμx auf-
Aufgabe 2.30 ••• Beweisen Sie den Satz von Liouville: tritt und p(μ) = 0 ist.
Die Wronski-Determinante W (x) erfüllt die skalare Diffe-
renzialgleichung Aufgabe 2.13 •• Hier liegt eine lineare skalare inho-
mogene Differenzialgleichung 1. Ordnung vor, y  (x) =
W  (x) = Sp A(x)W (x) , x∈I, xy(x) + x.

daher gilt für x, x0 ∈ I Aufgabe 2.14 •• Berechnen Sie die charakteristische


x Gleichung und machen Sie eine Fallunterscheidung.
Sp A(u) du
W (x) = W (x0 ) e x0
.
Aufgabe 2.15 •• –

Aufgabe 2.16 •• Wählen Sie einen geeigneten Ansatz


zur Berechnung einer Partikulärlösung.
Hinweise
Aufgabe 2.17 •• Die Methode der Variation der Kon-
Verständnisfragen stanten zur Berechnung einer Partikulärlösung führt auf ein
dreidimensionales lineares Gleichungssystem mit der Fun-
Aufgabe 2.1 • Eine skalare lineare Differenzialglei- damentalmatrix als Koeffizientenmatrix. Einfacher berech-
chung 1. Ordnung ist von der Gestalt net man hier mittels Ansatzmethode eine Partikulärlösung.
Achtung, es tritt Resonanz auf!
a1 (x)y  (x) + a2 (x)y(x) = f (x).

Die stetigen Funktionen a1 , a2 und f hängen nicht von der Aufgabe 2.18 •• Eine weitere Methode, Differenzial-
gesuchten Funktion y ab. gleichungen höherer Ordnung zu lösen, ist der Potenzreihen-
ansatz. Ist
Aufgabe 2.2 • Matrixexponentialfunktion.
y (n) (x) = f (x, y  (x), . . . , y (n−1) (x))
Aufgabe 2.3 •• Es gilt: Für zwei stetig differenzierbare mit den Anfangsbedingungen y(x0 ), . . . , y (n−1) (x0 ) gege-
linear abhängige Funktionen y1 und y2 ist W [y1 , y2 ](x) = ben, so setzt man als Lösung eine Potenzreihe der Form
0, x ∈ I . Gilt die Umkehrung?


Aufgabe 2.4 • Jede Lösung einer solchen Differenzi- y(x) = ak (x − x0 )k ,
k=0
algleichung lässt sich in der Form
(k)(x0 )
y(x) = c1 v 1 eλ1 x + c2 v 2 eλ2 x mit ak = y k! für k ≤ n − 1 an. Durch das Einsetzen der
Potenzreihe in die Differenzialgleichung und anschließenden
mit Eigenwerten λi und Eigenvektoren v i i = 1, 2 von A Koeffizientenvergleich bekommt man die Koeffizienten ak
darstellen. für k ≥ n rekursiv.

Aufgabe 2.5 • – Aufgabe 2.19 •• –


4 Lösungen zu Kapitel 2

Aufgabe 2.20 •• – Aufgabe 2.5 •


   
u1 u2
a) =
Aufgabe 2.21 •• – u2 u1 u2 sin x + cosh x − u21
⎛ ⎞ ⎛ 2 ⎞
u1 u2
Aufgabe 2.22 •• Transformieren Sie die logistische b) ⎝u2 ⎠ = ⎝ u3 ⎠
Differenzialgleichung in eine lineare Differenzialgleichung. u3 −2 u3 − u2 + 2 e3x

Beweisaufgaben Rechenaufgaben
Aufgabe 2.23 •• – Aufgabe 2.6 • y(x) = 2x 2 + 1

Aufgabe 2.24 •• Binomischer Lehrsatz, Cauchy-Pro- Aufgabe 2.7 • –


dukt.
Aufgabe 2.8 ••
Aufgabe 2.25 •• Verwenden Sie die Darstellung ⎛ ⎞
e−1 − e−1 0
A = T J T −1 , wobei J die Jordan’sche Normalform von A
eA = T eJ T −1 =⎝ 0 e−1 0 ⎠.
ist.
0 0 e−2
Aufgabe 2.26 • Die Matrix N mit sich selbst multi-
plizieren und beobachten, was mit den von 0 verschiedenen ••
Aufgabe 2.9  
Einträgen passiert. 1 0
Y 1 (x) = e2x
0 1
Aufgabe 2.27 •• Transformation auf ein zweidimen-  
0 1
sionales System 1. Ordnung und Satz von Picard-Lindelöf Y 2 (x) = e2x
1 x
anwenden.  
e−2x e2x
Y 3 (x) =
Aufgabe 2.28 •• Eine spezielle Lösung ist hier u(x)=1. 0 2e2x

Aufgabe 2.29 •• Schauen Sie sich die Parameter a und


Aufgabe 2.10 ••
b im Satz von Picard-Lindelöf an.
A1 : y 1 (x) = v 1 e−x , y 2 (x) = v 2 ex , y 3 (x) = v 3 e−3x mit
⎛ ⎞ ⎛ ⎞ ⎛ ⎞
Aufgabe 2.30 ••• Gehen Sie von einer Fundamentalma- −1 0 2
trix Y (x) aus und verwenden Sie außerdem die Linearität der v1 = ⎝ 1 ⎠ , v2 = ⎝ 1 ⎠ , v 3 = ⎝ −3 ⎠
Determinantenfunktion als Funktion jeder Spalte. 1 1 1

A2 : y 1 (x) = v 1 e4x , y 2 (x) = v 2 e2x , y 3 (x) = h1 e4x mit


⎛ ⎞ ⎛ ⎞ ⎛ ⎞
−1 −2 3
Lösungen v1 = ⎝ 1 ⎠ , v2 = ⎝ 0 ⎠ , h1 = ⎝ 1 ⎠
1 1 0
Verständnisfragen
    
Aufgabe 2.1 • 4 1
A3 : y 1 (x) = e2x cos x − sin x ,
a) nichtlinear 1 0
b) linear     
1 4
c) linear y 2 (x) = e2x cos x + sin x
0 1
d) linear

Aufgabe 2.2 • Es gibt keine reelle 2 × 2-Matrix, aber Aufgabe 2.11 •


eine entsprechende komplexe Matrix. a) y(x) = cos x,
b) y(x) = cosh x.
Aufgabe 2.3 •• –
Aufgabe 2.12 •• f (x) = ce−1, f (x) = ceix oder
Aufgabe 2.4 • f (x) = ce−ix , c ∈ C
a) nein
x2
b) ja Aufgabe 2.13 •• y(x) = ce 2 −1
c) nein
d) nein Aufgabe 2.14 •• –
Lösungswege zu Kapitel 2 5

Aufgabe 2.15 •• Lösungswege


y(x) = yh (x) + yp (x)

−2x −3x 2 π Verständnisfragen
= 4e − 3e + cos(x − )
10 4
Aufgabe 2.1 •

Aufgabe 2.16 •• y(x) = c1 +c2 e−x + 21 x 2 , c1 , c2 ∈ C a) Durch das Produkt y(x)y  (x) liegt hier eine nichtlineare
Differenzialgleichung vor.
b) Das ist eine lineare, homogene Differenzialgleichung 1.
Aufgabe 2.17 •• Ordnung mit a1 (x) = 1 und a2 (x) = x.
3 c) Hier liegt eine lineare, inhomogene Differenzialglei-
y(x) = c1 ex + c2 xex + c3 e−2x + x 2 ex , c1 , c2 , c3 ∈ C
2 chung 1. Ordnung vor, wobei a1 (x) = x, a2 (x) = 3.
d) Hier liegt ebenfalls eine lineare, inhomogene Differenzi-
algleichung 1. Ordnung vor.
Aufgabe 2.18 •• y(x) = eλx
Aufgabe 2.2 • Aus der Definition der Matrixexponen-
Aufgabe 2.19 •• tialfunktion folgt, dass Diagonalgestalt vom eA nur möglich
1
n−1
ist, falls A eine Diagonalmatrix mit a1 , a2 ∈ C
y0 = 1 , yn = yk yn−1−k
n
k=0  
a1 0

A=
1 0 a2
y(x) = xn = , |x| < 1
1−x
n=0
ist, also
   
Aufgabe 2.20 •• z (x) = (1 − γ )a(x)z(x) + −1 0 ea1 0
eA = = .
(1 − γ )b(x) 0 −4 0 ea2

Es gilt
Aufgabe 2.21 •• v  (x) = (2r(x)u(x) + p(x))v(x) +
r(x)v 2 (x)
ea1 = −1 ⇒ a1 = iπ, da eiπ = cos π + i sin π = −1,

Aufgabe 2.22 •• – ea2 = −4 ⇒ a1 = ln 4 + iπ, da eln 4+iπ


= 4 (cos π + i sin π ) = −4 .

Beweisaufgaben Die skalare Exponentialfunktion nimmt für reelle Argumente


nur positve Werte an.
Aufgabe 2.23 •• –
Aufgabe 2.3 •• Aus ax 3 + b|x|3 = 0 folgt im Fall
Aufgabe 2.24 •• – 0 < x < 1 die Beziehung a + b = 0 , für −1 < x < 0
allerdings gilt −a + b = 0 . Also ist a = b = 0 , und die
Aufgabe 2.25 •• – beiden Funktionen sind linear unabhängig. Für die Wronski-
Determinante gilt
Aufgabe 2.26 • Alle Eigenwerte sind 0.  
x3 |x|3
W [y1 , y2 ](x) = det 2 2 = 0.
3x 3x sgn x
Aufgabe 2.27 •• –
Nur für linear unabhängige Lösungen y1 und y2 einer Dif-
Aufgabe 2.28 •• ferenzialgleichung 2. Ordnung kann man aus der linearen
1 Unabhängigkeit von y1 und y2 schließen, dass W [y1 , y2 ](x)
y(x) = 1 + 2 2 x 2 ungleich 0 ist. Offensichtlich sind die vorliegenden Funktio-
cex+x + ex+x x0 et+t dt nen keine solchen Lösungen.

Das Integral ist in geschlossener Form nicht darstellbar. Aufgabe 2.4 •


a) Die Matrix A besitzt höchstens zwei verschiedene Eigen-
Aufgabe 2.29 •• – werte λ1 und λ2 . Jede Lösungskomponente ist eine
Linearkombination von eλ1 x und eλ2 x . Daher kann die
Aufgabe 2.30 ••• – angegebene Funktion y(x) keine Lösung sein.
6 Lösungswege zu Kapitel 2

b) Falls y(x) eine Lösung ist, dann sind λ1 = 1 und Aufgabe 2.8 •• Die Eigenwerte der Matrix A sind
λ2 = −1 die Eigenwerte der Matrix A und da sich λ1 = −1 mit der algebraischen Vielfachheit 2 und der geo-
jede Lösung von y  (x) = Ay(x) in der Form y(x) = metrischen Vielfachheit 1, sowie λ2 = −2. Die Eigenvekto-
c1 v 1 ex +c2 v 2 e−x mit Eigenvektoren v 1 und v 2 schreiben ren und Hauptvektoren sind gegeben durch
lässt, folgt aus y(x), dass v 1 = (3, 1) und v 2 = (1, 0) . ⎛ ⎞ ⎛ ⎞ ⎛ ⎞
Daraus kann die Matrix A berechnet werden. 1 0 0
c) y(x) ist keine Lösung, da ein Term xex nur im Fall von v 1 = ⎝ 0 ⎠ , h1 = ⎝ −1 ⎠ , v 2 = ⎝ 0 ⎠ .
Resonanz, also bei einem doppelten Eigenwert von A 0 0 1
auftreten kann.
d) Die Eigenwerte sind verschieden, und falls y(x) eine Lö- Es gilt eA = T eJ T −1 mit T = (v 1 , h1 , v 2 ) und
sung wäre, dann müssten die beiden Eigenvektoren linear ⎛ ⎞
−1 1 0
abhängig sein, was unmöglich ist. J = ⎝ 0 −1 0 ⎠ = D + N
0 0 −2
Aufgabe 2.5 • Da die Differenzialgleichungen auch in ⎛ ⎞ ⎛ ⎞
expliziter Form geschrieben werden können, ist es möglich, −1 0 0 0 1 0
sie als Systeme 1. Ordnung zu formulieren. Wir setzen ui = = ⎝ 0 −1 0 ⎠ + ⎝ 0 0 0 ⎠ ,
y (i−1) für i = 1 . . . n , wobei n die Ordnung der skalaren 0 0 −2 0 0 0
Differenzialgleichung höherer Ordung ist.
wobei N eine nilpotente Matrix mit N 2 = 0 ist und daher
a) y  = (y  )2 y sin x + cosh x − y 2 , gilt eN = I + N . Also ist eJ = eD+N = eD eN , und
u1 = y ⎛ −1 ⎞
e − e−1 0
u2 = y  eA = T eJ T −1 = ⎝ 0 e−1 0 ⎠.
    0 0 e −2
u1 u2
⇒ =
u2 u22 u1 sin x + cosh x − u21
Aufgabe 2.9 •• Es gilt Y (x) = T eJ x .
b) y  = −2y  − y + 2e3x ,
Die Matrix A1 hat den doppelten Eigenwert λ = 2 und schon
u1 = y Diagonalgestalt, also gilt T = I,
u2 = y   
1 0
u3 = y  Y 1 (x) = T eAx = eAx = e2x .
0 1
⎛ ⎞ ⎛ ⎞
u1 u2
⇒ ⎝u2 ⎠ = ⎝ u3 ⎠
A2 hat auch den doppelten Eigenwert λ = 2, aber mit geo-
u3 −2 u3 − u2 + 2 e 3x
metrischer Vielfachheit 1. Also gilt A = T J T −1 mit
   
0 1 2 1
Rechenaufgaben T = (v, h) = und J = =D+N,
1 0 0 2
Aufgabe 2.6 • Die allgemeine Lösung ist  
Y 2 (x) = T e Jx
= Te Dx N x
e =e 2x 0 1
.
 1 x
y(x) = y (x) dx = 4x dx = 2x 2 + c , c ∈ R.

Wir bestimmen die Konstante c durch Einsetzen der Anfangs- A3 hat die Eigenwerte λ1 = −2 und λ2 = 2. Es gilt A =
bedingung y(0) = 1, aus der c = 1 folgt. Die gesuchte Lö- T DT −1 mit
sung ist daher y(x) = 2x 2 + 1.    
1 1 −2 0
T = (v 1 , v 2 ) = und D = ,
Aufgabe 2.7 • Diese Aufgabe lösen wir, indem wir die 0 2 0 2
Lösung einmal ableiten und dann y  und y in die Differen-  
Dx e−2x e2x
zialgleichung einsetzen, Y 3 (x) = T e = .
0 2e2x
c(1 + x) − cx c
y  (x) = = .
(1 + x)2 (1 + x)2 Aufgabe 2.10 •• Das Fundamentalsystem ist durch die
Einsetzen in die gegebene Differenzialgleichung führt auf Spalten der Fundamentalmatrix Y (x) = T eJ x gegeben, wo-
bei J die Jordansche Normalform von A = T J T −1 ist.
x(1 + x)y  (x) − y(x)
c cx cx cx A1 : Die Eigenwerte von A1 , die wir mithilfe des charakte-
= x(1 + x) − = − = 0, ristischen Polynoms
(1 + x) 2 1+x 1+x 1+x
also liegt wirklich eine Lösung der gegebenen Gleichung vor. p(λ) = det(A1 − λI ) = (−1 − λ)(1 − λ)(−3 − λ2 )
Lösungswege zu Kapitel 2 7

berechnen, sind λ1 = −1, λ2 = 1, λ3 = −3. Zu diesen b) Die charakteristische Gleichung ist p(λ) = λ2 − 1 = 0
drei verschiedenen Eigenwerten finden wir drei linear mit den Lösungen λ1 = 1 und λ2 = −1. Die allgemeine
unabhängige Eigenvektoren v 1 , v 2 , v 3 mit Lösung ist y(x) = c1 e−x + c2 ex mit Ableitung y  (x) =
⎛ ⎞ ⎛ ⎞ ⎛ ⎞ −c1 e−x + c2 ex . Aus den Anfangsbedingungen y(0) = 1
−1 0 2
und y  (0) = 0 folgt schließlich c1 = c2 = 21 , also
v 1 = ⎝ 1 ⎠ , v 2 = ⎝ 1 ⎠ , v 3 = ⎝ −3 ⎠ .
1 1 1 1 −x
y(x) = (e + ex ) = cosh x.
2
Daraus ergibt sich ein Fundamentalsystem
Aufgabe 2.12 •• Der Ansatz y(x) = eλx führt bei die-
y 1 (x) = v 1 e−x , y 2 (x) = v 2 ex , y 3 (x) = v 3 e−3x . sem Beispiel auf die charakteristische Gleichung
A2 : Die Eigenwerte von A2 berechnen sich mithilfe des λ3 + 2λ2 + λ + 2 = 0
charakteristischen Polynoms
mit den Lösungen λ1 = −2, λ2 = i, λ2 = −i. Resonanz
p(λ) = det(A2 − λI ) = −(x − 4)(x − 2)2 tritt hier auf, wenn die Funktion f (x) = ceμx , c ∈ C und
μ = λi für ein i = 1, 2 oder 3 ist oder einen solchen Term
zu λ1 = 4 und λ2 = λ3 = 2. Zu λ1 finden wir einen Ei- enthält.
genvektor v 1 , zu λ2 = λ3 gibt es nur einen Eigenvektor
v 2 und einen zugehörigen Hauptvektor h1 : Aufgabe 2.13 •• Zunächst lösen wir die homogene Dif-
⎛ ⎞ ⎛ ⎞ ⎛ ⎞ ferenzialgleichung durch getrennte Integration auf der linken
−1 −2 3
v 1 = ⎝ 1 ⎠ , v 2 = ⎝ 0 ⎠ , h1 = ⎝ 1 ⎠ . und rechten Seite
dy
1 1 0 y  (x) = = xy(x)
dx
Daraus erhalten wir ein Fundamentalsystem dy
= xdx
y
y 1 (x) = v 1 e4x , y 2 (x) = v 2 e2x , y 3 (x) = hx e4x .
ln y = x 2 + c
A3 : Die Eigenwerte von A3 berechnen sich mithilfe des
x2
charakteristischen Polynoms y(x) = ce 2 .
p(λ) = det(A3 − λI ) = (2 + i − λ)(2 − i − λ) Die Partikulärlösung erhalten wir durch Variation der Kon-
x2
zu λ1 = 2 + i und λ2 = 2 − i mit den Eigenvektoren stanten mit dem Ansatz yp (x) = c(x)e 2 ,
v 1 und v 2 = 
v1 x2 x2
      yp (x) = c (x)e 2 + c(x)xe 2
4+i 4 1
v1 = = +i , = xy(x) + x = xc(x)e
x2
+x.
1 1 1 2
     
4−i 4 1 x2 2
v2 = = −i . Aus c (x) = xe 2 folgt durch Substitution − x2 = u und
1 1 1
x2 −x 2 x2
Integration von 0 bis x yp (x) = c(x)e 2 = −e 2 e 2 = −1.
Wir erhalten ein reelles Fundamentalsystem als Real- x2
und Imaginärteil von e1λ v 1 Die allgemeine Lösung ist daher y(x) = − 1 und nach
ce 2
Einsetzen der Anfangsbedingung folgt c = 3.
y 1 (x) = Re (e(2+i)x v 1 )
     Aufgabe 2.14 •• Die charakteristische Gleichung lau-
2x 4 1
=e cos x − sin x , tet
1 0
λ2 + 2bλ + c = 0
y 2 (x) = Im (e(2+i)x v 1 )
     mit Lösung
2x 1 4 
=e cos x + sin x . λ1,2 = −b ± b2 − c = −b ± B .
0 1
Wir unterscheiden drei Fälle
Aufgabe 2.11 •
b2 = c ⇒
a) Die charakteristische Gleichung ist p(λ) = λ2 + 1 = 0
mit den Lösungen λ1 = i und λ2 = −i. Die allge- λ1 = λ2 = −b y1 (x) = e−bx , y2 (x) = xe−bx
meine Lösung dieser homogenen Differenzialgleichung b2 > c ⇒
reell geschrieben ist y(x) = c1 cos x + c2 sin x mit Ab-
λ1 , λ2 ∈ R y1 (x) = e(−b+B)x , y2 (x) = e(−b−B)x
leitung y  (x) = −c1 sin x + c2 cos x. Aus den Anfangs-
bedingungen y(0) = 1 und y  (0) = 0 folgt schließlich b2 < c ⇒
c1 = 1 und c2 = 0, also λ1 , λ2 ∈ C y1 (x) = e−bx cos(Bx),
y(x) = cos x . y2 (x) = e−bx sin(Bx)
8 Lösungswege zu Kapitel 2

Aufgabe 2.15 •• Die Nullstellen der charakteristischen ein Fundamentalsystem der homogenen Differenzialglei-
Gleichung λ2 + 5λ + 6 = 0 sind λ1 = −2 und λ2 = −3. chung. Für die Inhomogenität gilt f (x) = x + 1 = q(x) e0x
Die allgemeine Lösung yh der entsprechenden homogenen mit q(x) = x + 1. Da hier Resonanz auftritt, wählen
Differenzialgleichung ist daher wir den Ansatz yp (s) = x(bx + a) e0x = bx 2 + ax zur
Berechnung einer Partikulärlösung. Es gilt yp (x) = 2bx + a
yh (x) = c1 e−3x + c2 e−2x , c1 , c2 ∈ R . und yp (x) = 2b . Einsetzen in die Differenzialgleichung
führt auf
Ansatz für die Partikulärlösung: yp (x) = d cos(x + δ),
2b + 2bx + a = x + 1 ,
y  (x) + 5y  (x) + 6y(x) = cos x durch Koeffizientenvergleich erhalten wir a = 0 und b =
1 1 2
d(− cos(x + δ) − 5 sin(x + δ) + 6 cos(x + δ)) = cos x 2 . Eine Partikulärlösung ist daher yp (x) = 2 x und die
allgemeine Lösung ist
5d(cos(x + δ) − sin(x + δ)) = 5dD(x) = cos x ,
1
wobei y(x) = c1 + c2 e−x + x 2 , c1 , c2 ∈ C.
2
D(x) = cos(x + δ) − sin(x + δ)
= cos x cos δ − sin x sin δ − (sin x cos δ + cos x sin δ) Aufgabe 2.17 •• Für die charakteristische Gleichung
= sin x(− sin δ − cos δ) + cos x(cos δ − sin δ) . gilt
λ3 − 3λ + 2 = (λ − 1)2 (λ + 2) = 0 ,
Wir wollen, dass nur ein cos–Term übrigbleibt, also wählen die allgemeine homogene Lösung ist daher
wir δ so, dass
yh (x) = c1 ex + c2 xex + c3 e−2x , c1 , c2 , c3 ∈ C .
sin δ = − cos δ ⇐⇒ tan δ = −1 ,
Es liegt Resonanz vor, und da 1 eine doppelte Nullstelle der
im Intervall [0, 2π] ist das für charakteristischen Gleichung ist, ist der Ansatz für die Parti-
3π √ kulärlösung
δ= ⇒ cos δ − sin δ = − 2 und yp (x) = cx 2 ex .
4
7π √ Für die Ableitungen erhalten wir
δ= ⇒ cos δ − sin δ = + 2
4
yp (x) =ex (2cx + cx 2 )
erfüllt. Dann ist
yp (x) =ex (2c + 4cx + cx 2 )
√ 3π
5dD(x) = − 5d 2 cos x für δ= und yp (x) =ex (6c + 6cx + cx 2 ) ,
4
√ 7π
5dD(x) = + 5d 2 cos x für δ= . und nach Einsetzten in die inhomogene Differenzialglei-
4
chung erhalten wir c = 23 . Die allgemeine Lösung der gege-

Aus 5dD(x) = cos x folgt d = − 102 für δ = 3π
und benen Differenzialgleichung ist daher
√ 4
d= + 102 für δ = 7π
4 . Für die Partikulärlösung folgt 3
y(x) = c1 ex + c2 xex + c3 e−2x + x 2 ex , c1 , c2 , c3 ∈ C .
√   √   2
2 3π 2 7π
yp (x) = − cos x + =+ cos x +
10 4 10 4
√ Aufgabe 2.18 •• Zum Lösen des gegebenen Anfangs-
2 π wertproblems setzen wir
=+ cos(x − ) ,
10 4


die allgemeine Lösung ist y(x) = ak x k
√   k=0
2 π
y(x) = yh (x)+yp (x) = c1 e−2x +c2 e−3x + cos x− an. Aus der Anfangsbedingung y(0) = 1 folgt a0 = 1. Für
10 4
die Ableitung gilt
und nach Einsetzen der Anfangsbedingungen folgt c1 = 4



und c2 = −3.
y  (x) = lal x l−1 = (k + 1)ak+1 x k .
Es zeigt sich eine starke Dämpfung gegen die Partikulärlö- l=1 k=0
sung hin für wachsendes x.
Einsetzen in die Differenzialgleichung ergibt
Aufgabe 2.16 •• Das charakteristische Polynom ∞


p(λ) = λ2 + λ hat die einfachen Nullstellen λ1 = 0 und (k + 1)ak+1 x k = λak x k .
λ2 = −1 . Daher bilden y1 (x) = e0x = 1 und y2 (x) = e−x k=0 k=0
Lösungswege zu Kapitel 2 9

Mittels Koeffizientenvergleich erhalten wir die Rekursions- Wir berechnen


gleichung
λ y1 = y02 = 1 ,
ak+1 = ak ,
k+1 1
y2 = (y0 y1 + y1 y0 ) = 1 ,
2
aus der
1
λk λk y3 = (y0 y2 + y1 y1 + y2 y0 ) = 1 ,
ak = a0 = 3
k! k!
..
folgt, also ist .
yn = 1
∞ k


λ (λx)k
y(x) = xk = = eλx . und erhalten als Taylorreihe der Lösung y(x) die unendliche
k! k!
k=0 k=0 geometrische Reihe

1
Aufgabe 2.19 •• Wir berechnen zunächst die Ableitun- y(x) = xn = ,
1−x
n=0
gen y (n) (x0 ) für n = 1, 2, 3
die für |x| < 1 konvergiert.
y  (x) = f (x, y(x))
y  (x) = fx (x, y(x)) + fy (x, y(x))y  (x) , Aufgabe 2.20 ••
= fx (x, y(x)) + fy (x, y(x))f (x, y(x)) , z (x) = (y 1−γ (x)) = (1 − γ )y −γ (x)y  (x)
 2
y = fxx + fxy f + fyx f + fyy f + fy fx + fy2 f . = (1 − γ )y −γ (x)[a(x)y(x) + b(x)y γ (x)]

Nun betrachten wir die Differenzialgleichung = (1 − γ )[a(x)y 1−γ (x) + b(x)]


= (1 − γ )a(x)z(x) + (1 − γ )b(x)
y = y2 , y(0) = 1
Das ist eine lineare inhomogene Differenzialgleichung in
und machen für die gesuchte Lösung den Ansatz z(x).


Aufgabe 2.21 •• Sei u(x) eine spezielle Lösung von
y(x) = yn x n y  (x) = q(x) + p(x)y(x) + r(x)y 2 (x). Mit
n=0
y(x) = u(x) + v(x) ,
als Taylorreihe mit den Koeffizienten yn . Zur Berechnung y  (x) = u (x) + v  (x) und
dieser Koeffizienten verwenden wir die Differenzialglei-
chung und das Cauchy-Produkt zweier Reihen. Es gilt y 2 (x) = u2 (x) + v 2 (x) + 2u(x)v(x) ,

∞ folgt

n
y(x) = yn x y  (x) = u (x) + v  (x)
n=0
∞ = q(x) + p(x)(u(x) + v(x))

 n−1
y (x) = n yn x + r(x)(u2 (x) + v 2 (x) + 2u(x)v(x)) .
n=1
 ∞
2 Wir erhalten eine Bernoulli’sche Differenzialgleichung in v

2 n
= y (x) = yn x
v  (x) = ((2r(x)u(x) + p(x))v(x) + r(x)v 2 (x)) .
n=0

n
= yk x k yn−k x n−k Diese Differenzialgleichung kann mittels z(x) = v −1 (x) in
n=0 k=0 eine inhomogene lineare Differenzialgleichung in z


n−1
z (x) = −(2r(x)u(x) + p(x))z(x) − r(x)
= x n−1 yk yn−1−k .
n=1 k=0 transformiert werden, die schließlich mit den Standardme-
thoden gelöst werden kann.
Ein Koeffizientenvergleich und die Anfangsbedingung
y(0) = 1 führt auf die Rekurrenz Aufgabe 2.22 •• Die logistische Gleichung

1
n−1
y  (x) = λy(x)(K − y(x))
y0 = 1 , yn = yk yn−1−k .
n
k=0 = λKy(x) − λy 2 (x)
10 Lösungswege zu Kapitel 2

ist eine Bernoulli’sche Differenzialgleichung mit a(x) = b) Für die Matrizen


λK, b(x) = −λ und γ = 2. Die Transformation y(x) =    
u−1 (x) führt auf die lineare Differenzialgleichung 1 1 1 0
A= und B =
0 1 1 1
u (x) = −λKu(x) + λ
gilt
mit der Lösung
    2 2
1 e e e 0 2e e
u(x) = ce −λKx
+ eA eB = =
K 0 e e e e2 e2
 
und
3
e+e 3−e+e
−1 K = 2
−e+e3
2
e+e3
= eA+B .
y(x) = u (x) = .
1 + Kce−λKx 2 2
Mithilfe von y(0) kann die Konstante c ausgedrückt werden
und es folgt Aufgabe 2.25 •• Sei J die Jordan’sche Normalform
von A = T J T −1 .
K
y(x) = u−1 (x) = . a)
K
1 + ( y(0) − 1)e−λKx
−1
det eA = det eT J T = det(T eJ T −1 )
Beweisaufgaben = det T det eJ det T −1 = det eJ
Aufgabe 2.23 •• 
n n
= e λj = e j =1 λj = eSp A
a) Wir zeigen beide Richtungen: j =1
„⇐“: X(x) ist als Produkt zweier regulärer Matrizen re-
gulär, die Spalten von X(x) sind daher linear unabhängig. b)
Des Weiteren sind die Spalten von X(x) Linearkombina- −1
)T = (T eJ T −1 )T
T
eA = (eT J T
tion der Spalten von Y (x) und nach dem Superpositions-
= (T −1 )T (eJ )T T T = (T −1 )T e(J ) T T
T
prinzip daher Lösungen des linearen Systems. Somit ist
X(x) eine Fundamentalmatrix. −1 )T J T T T T)
= e(T = e(A
„⇒“: Wegen der Eindeutigkeit der Lösung des Systems
y  (x) = A(x)y(x) hat jede Lösung die Gestalt y(x) = c) Aus AT = −A folgt
Y (x) c mit c ∈ Rn . Da die Spalten von X(x) Lösungen
= e−Ax = (eAx )−1 ,
T Tx
sind, muss X(x) = Y (x) B gelten und wegen der linea- (eAx )T = e(Ax) = eA
ren Unabhängigkeit der Spalten von X(x) muss B regulär und die Orthogonalität von eAx . Wegen
sein.
b) Aus Punkt a) folgt, dass X(x) eine Fundamentalmatrix Sp A = Sp AT = Sp(−A) = 0
ist und wegen X(x0 ) = Y (x0 )(Y (x0 ))−1 = I n ist X(x)
Hauptfundamentalmatrix. und a) gilt

Aufgabe 2.24 •• det eAx = eSp(A)x = e0 = 1 .

a) Da A und B kommutieren, gilt mit dem Binomischen


Aufgabe 2.26 •
Lehrsatz und dem Cauchy-Produkt
n   a) Wie wir leicht erkennen, rutschen aufgrund der speziellen
(A + B)n 1 n k n−k Struktur der Matrix bei der Multiplikation von N mit N
= A B
n! n! k die vorhandenen Einser in jeder Zeile um eine Position
k=0

n nach rechts. Da die Matrix N m Spalten (und auch m
Ak B n−k
= , Zeilen) hat, ist beim (m − 1)-fachen Produkt N m−1 der
k! (n − k)! letzte noch vorhandene Einser in der ersten Zeile und m-
k=0

(A + B)n ten Spalte, sodass schließlich N k = 0 für k ≥ m .
eA+B = b) Für die Eigenwerte λ von N mit Eigenvektoren v gilt
n!
n=0
∞ n N v = λv .
Ak B n−k
=
k! (n − k)! Multiplikation mit N k−1 führt auf
n=0 k=0
∞  ∞ 
An Bn 0 = N k v = λN k−1 v = λk v .
=
n! n!
n=0 n=0 Da Eigenvektoren v = 0 sind, folgt, dass alle Eigenwerte
=e e .
A B 0 sind.
Lösungswege zu Kapitel 2 11

c) Aus dem System für die Partikulärlösung folgt


⎛ ⎞ x
0 1 0 2
c(x) = et+t dt .
y = ⎝ 0 0 1 ⎠ y x0
0 0 0
Da dieses Integral nicht in geschlossener Form darstellbar
folgt ist, folgt für die allgemeine Lösung

x2 x
y1 = y2 ⇒ y1 (x) = c1
2 2 2
+ c2 x + c3 , z(x) = cex+x + ex+x et+t dt
2 x0

y2 = y3 ⇒ y2 (x) = c1 x + c2 , 1 1
v(x) = = 2 x t+t 2
und
y3 = 0 ⇒ y3 (x) = c1 . z(x) x+x 2
+e x+x
ce x0 e dt
1
y(x) = 1 + v(x) = 1 +
Aufgabe 2.27 •• Mit z(x)
1
=1+ 2 x
,
y(x) = u1 (x) 2
cex+x + ex+x
2
et+t dt
x0

y (x) = u2 (x)
wobei die Konstante c mittels einer Anfangsbedingung y(x0 )
wird die gegebene Differenzialgleichung 2. Ordnung in ein berechnet werden kann.
zweidimensionales System 1. Ordnung transformiert
Aufgabe 2.29 •• Der Satz von Picard-Lindelöf kann
u (x) = Au(x) + g(x) ,
mit jedem b > 0 angewandt werden, insbesondere auch mit
b = aR und dann folgt α = a . Es existiert also eine Lö-
wobei
sung auf jedem Intervall [x0 − a, x0 + a]. Da die Lösung
      jeweils eindeutig ist, handelt es sich um Einschränkungen
u1 0 0 1
u= , g(x) = und A = . derselben stetig differenzierbaren Funktion y : R → Cn auf
u2 f (x) 0 0
die Intervalle [x0 − a, x0 + a]. Damit ist y Lösung des An-
Der Satz von Picard-Lindelöf kann angewendet werden, alle fangswertproblems auf ganz R .
Voraussetzungen sind erfüllt.

Aufgabe 2.28 •• Die Funktion u(x) = 1 ist eine spezi- Kommentar: Ein Spezialfall ist der Fall einer linearen
elle Lösung von y  (x) = 2x + (1 − 2x)y(x) − y 2 (x). Mit Differenzialgleichung 1. Ordnung

y(x) = 1 + v(x) , F (x, y(x)) = A(x)y(x) + f (x) .


y  (x) = v  (x) und
Die Lösungen einer linearen Differenzialgleichung existieren
2 2
y (x) = 1 + v (x) + 2v(x) , auf allen Intervallen. Sind A und f auf ganz R definiert und
stetig, so existieren die Lösungen auf ganz R .
folgt
v  (x) = −(1 + 2x)v(x) − v 2 (x) .

Diese Differenzialgleichung kann mittels z(x) = v −1 (x) Aufgabe 2.30 ••• Sei Y (x) eine Fundamentalmatrix,
1
oder v(x) = z(x) in die inhomogene lineare Differenzial- W (x) = det Y (x) die dazugehörige Wronski-Determinante,
gleichung in z x̂ ∈ I und Z(x) eine Hauptfundamentalmatrix.

z (x) = (1 + 2x)z(x) + 1 Die Matrix Ŷ (x) := Z(x)Y (x̂) ist ebenfalls eine Fundamen-
talmatrix mit Ŷ (x̂) = Y (x̂). Daher gilt Ŷ (x) = Y (x) für alle
transformiert werden. Die Lösung der homogenen linearen x ∈ I und es folgt
Differenzialgleichung ist
W (x) = det Y (x) = det Z(x) det Y (x̂) = det Z(x)W (x̂)
x+x 2
zh (x) = ce ,
und
d d  
mit einer Integrationskonstante c ∈ C. Mit dem Ansatz W (x) = W (x̂) det Z(x) .
dx dx
2
zp (x) = c(x)ex+x Da det Z(x) = det(z1 (x), . . . , zn (x)) linear bezüglich jeder
12 Lösungswege zu Kapitel 2

Spalte z1 (x), . . . , zn (x) ist, gilt Somit folgt

d   d d
det Z(x) = det(z1 , . . . , zn ) W (x̂) = Sp A(x̂) W (x̂).
dx dx dx
n
∂ det(z1 , . . . , zn ) dzk
= Da x̂ beliebig war, folgt
∂zk dx
k=1
d
n
dzk W (x) = Sp A(x)W (x) , ∀x ∈ I .
= det(z1 , . . . , , . . . , zn ) dx
dx
k=1

n
Kommentar: Die Wronski-Determinante ist daher entwe-
= det(z1 , . . . , A(x)zk (x), . . . , zn ) .
der identisch gleich 0 oder im gesamten Intervall I von 0
k=1
verschieden. Aus diesem Resultat folgt wieder, dass eine Lö-
Die Auswertung an der Stelle x = x̂ ergibt sungsmatrix genau dann eine Fundamentalmatrix ist, wenn
ihre Spalten an einer Stelle x̂ ∈ I linear unabhängig sind.
d   n
det Z(x̂) = det(e1 , . . . , A(x̂) ek , . . . , en )
dx
k=1

n
= akk (x̂) = Sp A(x̂).
k=1
Kapitel 3 Aufgabe 3.9 •• Berechnen Sie die Lösung (in implizi-
ter Form) der Differenzialgleichung

y 2 − xy
Aufgaben y  (x) = − .
2xy 3 + xy + x 2

Verständnisfragen Aufgabe 3.10 ••• Berechnen Sie die allgemeine Lösung


der Differenzialgleichung
Aufgabe 3.1 • Ist die Differenzialgleichung
(y + xy 2 ) − xy  = 0
e−y + (1 − xe−y )y  (x) = 0
exakt? mittels eines geeigneten integrierenden Faktors der Form
u(y). Bestimmen Sie des Weiteren diejenige Lösung, die
Aufgabe 3.2 •• Ist die Differenzialgleichung durch den Punkt (x, y) = (2, −2) verläuft.

y  = ex−y Aufgabe 3.11 •• Ist die Differenzialgleichung


separabel? (2x + y)dx + (x − y) dy = 0
Berechnen Sie jene Lösung y(x), für die y(0) = 1 gilt. exakt? Begründen Sie Ihre Antwort. Berechnen Sie die all-
gemeine Lösung dieser Differenzialgleichung.
Aufgabe 3.3 •• Bestimmen Sie mithilfe der Definition
der Laplace-Transformation die Bildfunktionen der folgen-
Aufgabe 3.12 •• Finden Sie einen integrierenden Fak-
den Originalfunktionen:
tor für die nicht exakte Differenzialgleichung
a) f (t) = 2 t e−4t
b) f (t) = sinh (at), a ∈ R, a < s (1 − xy) + (xy − x 2 )y  = 0 .
c) f (t) = sin2 t
Aufgabe 3.13 •• Zeigen Sie mittels vollständiger In-
Rechenaufgaben duktion, dass für die Laplace-Transformierte für Monome
fn (t) = t n , n ∈ N0
Aufgabe 3.4 •• Bestimmen Sie den Typ und die Lösung
der Differenzialgleichung n!
L[fn (t)] = n+1 , s>0
s
 2 2 2
y (x) = (x + 1)y + (x − 1)
gilt.
mit der Anfangsbedingung y(0) = 1.
Aufgabe 3.14 •• Lösen Sie die Differenzialgleichung
Aufgabe 3.5 •• Die Bernoulli’sche Differenzialglei- 1. Ordnung
chung  
x + 2y(x) y(x)
y  (x) = −ay(x) + by γ (x) y  (x) = =1+2 .
x x
mit konstanten Koeffizienten a, b ∈ R, a  = 0 und γ  = 1 ist
separabel. Lösen Sie diese Differenzialgleichung mittels der Aufgabe 3.15 •• Berechnen Sie alle möglichen Lösun-
Methode der Separation der Variablen. gen der Differenzialgleichung

Aufgabe 3.6 • Lösen Sie die Differenzialgleichung y  (x) = 2x y
y  (x) = 2x − y(x) mittels Separation der Variablen. Geben Sie auch die kon-
mittels einer geeigneten Substitution. kreten Lösungen zu den Anfangsbedingungen y(0) = 1,
y(0) = 0 und y(0) = −1. Was passiert im letzten Fall?
Aufgabe 3.7 •• Ist die Differenzialgleichung
Aufgabe 3.16 •• Lösen Sie das Randwertproblem
2xy(x) + x 2 y  (x) = 0
y  (x) − y(x) = 1
exakt? Berechnen Sie die Lösungen.
mit den Randbedingungen y(0) = y  (0) = 0.
Aufgabe 3.8 •• Finden Sie einen integrierenden Faktor
u(x, y) , sodass die Differenzialgleichung Aufgabe 3.17 •• Geben Sie alle Lösungen des Rand-
wertproblems
(y 2 − 2x − 2) + 2yy  (x) = 0 y  (x) + y(x) = 0
exakt wird. mit den Randbedingungen y(0) = y(π ) = 0 an.
M. Brokate et al., Arbeitsbuch Grundwissen Mathematikstudium – Höhere Analysis, Numerik und
Stochastik, DOI 10.1007/978-3-642-54946-5_2, © Springer-Verlag Berlin Heidelberg 2016
14 Hinweise zu Kapitel 3

Aufgabe 3.18 •• Bestimmen Sie mithilfe der Lapla- Aufgabe 3.22 ••• Bestimmen Sie die Eigenwerte und
ce–Transformation der Ableitung und der Transformationsta- Eigenfunktionen des Sturm-Liouville-Eigenwertproblems
bellen die Bildfunktionen der ersten Ableitung der folgenden
λ
Funktionen: −(xu ) = u, u(1) = 0 , u(e) = 0 .
x
a) f (t) = sinh(at)
b) f (t) = t 3
Beweisaufgaben
Aufgabe 3.19 ••• Die Funktion f (t) = sin(wt) ist eine
Lösung der Schwingungsgleichung Aufgabe 3.23 ••• Lösen Sie das Anfangswertproblem

f  (t) = −ω2 f (t) . y  (x) = 2 y(x) , x ≥ 0 , y(0) = 0 .

Bestimmen Sie die zugehörige Bildfunktion L[f (t)] = Ist die Lösung eindeutig? Geben Sie eine Begründung!
F (s), indem Sie die Laplace-Transformation auf die Diffe-
renzialgleichung anwenden und dabei die Transformation der
Ableitung verwenden. Aufgabe 3.24 ••• Beweisen Sie die Energieerhaltungs-
gleichung
Aufgabe 3.20 ••• Gegeben ist die Wellengleichung mit
1 
Randbedingungen (ϕ (t))2 − ω2 cos ϕ(t) = const
2
utt = uxx , u(0, t) = u(1, t) = 0 , x ∈ [0, 1] , t ∈ R . für die nichtlineare Pendelgleichung
Führen Sie einen Separationsansatz durch, d. h. bestimmen ϕ  (t) + ω2 sin ϕ(t) = 0 .
Sie alle Lösungen der Form u(x, t) = w(t)v(x) . Dieser An-
satz führt auf ein Sturm-Liouville-Eigenwertproblem für die
Funktion v.
Bestimmen Sie damit die Lösung der Wellengleichung, die
zusätzlich zu den Randbedingungen auch die Anfangsbedin- Hinweise
gungen
Verständnisfragen
u(x, 0) = g(x), ut (x, 0) = h(x) , x ∈ [0, 1]
Aufgabe 3.1 • Überprüfen Sie die Integrabilitätsbe-
mit geeigneten Funktion g , h erfüllt. Entwickeln Sie dazu dingung.
die Lösung in eine Fourierreihe
∞ Aufgabe 3.2 •• –

u(x, t) = (ak cos(kπt) + bk sin(kπt)) sin(kπx).
k=1 Aufgabe 3.3 •• –

Die Konvergenz der auftretenden Reihe muss nicht unter-


sucht werden. Rechenaufgaben

Aufgabe 3.21 •• Zeigen Sie, dass die Differenzialglei- Aufgabe 3.4 •• –


chung
Lu = u + p(x)u + q(x)u = 0 Aufgabe 3.5 •• Finden Sie eine geeignete Substitution
zur Berechnung des y-Integrals.
durch die Substitution
1
Aufgabe 3.6 • –
pdx
v(x) = e 2 u(x)

in die Form Aufgabe 3.7 •• –



v + k(x)v = 0
Aufgabe 3.8 •• Wählen Sie als Ansatz für den integrie-
gebracht werden kann. Geben Sie die Funktion k explizit an. renden Faktor
Wie wirkt sich diese Tranformation auf das Eigenwertpro- u(x, y) = u(x) .
blem Lu = λu aus?
Transformieren Sie die Hermitesche Differenzialgleichung Aufgabe 3.9 •• Ein Ansatz für den integrierenden Fak-
tor ist
u − 2xu + 2nu = 0 , x ∈ R , n ∈ N0 . u(x, y) = x a y b , a, b ∈ R .
Lösungen zu Kapitel 3 15

Aufgabe 3.10 ••• – Lösungen

Aufgabe 3.11 •• – Verständnisfragen


Aufgabe 3.1 • Die Differenzialgleichung ist exakt, da
Aufgabe 3.12 •• –
∂ −y ∂
e = (1 − xe−y ) = −e−y .
∂y ∂x
Aufgabe 3.13 •• Partielle Integration.
Aufgabe 3.2 •• Diese Differenzialgleichung ist exakt,
  •• Diese Differenzialgleichung ist vom
Aufgabe 3.14 ihre Lösung ist
Typ y  = h yx , also homogen.
y  (x) = ln(ex + e + 1) .

Aufgabe 3.15 •• –
Aufgabe 3.3 ••
2
a) F (s) =
Aufgabe 3.16 •• – (s + 4)2
a
b) F (s) = 2
s − a2
Aufgabe 3.17 •• – 2
c) F (s) =
s(s 2 + 4)
Aufgabe 3.18 •• L[f  (t)] = s L[f (t)] − f (0)
Rechenaufgaben
Aufgabe 3.19 ••• L[f  (t)] = s 2 L[f (t)]−sf (0)−f  (0) Aufgabe 3.4 •• Diese Differenzialgleichung ist separa-
3

bel mit Lösung y(x) = tan x3 + x + π4 .
Aufgabe 3.20 ••• Der Separationsansatz führt auf
w (t)v(x) = w(t)v (x), und Division durch wv ergibt Aufgabe 3.5 ••
b c
y(x) = u1−γ (x) mit u(x) = + exp(a(γ −1)x) , c ∈ R
w v  a a
= .
w v
Aufgabe 3.6 • y(x) = ce−x − 2 + 2x , c∈R
Die linke Seite hängt nur noch von t ab und die rechte nur
von x. Die Gleichung kann nur dann für alle x und t erfüllt Aufgabe 3.7 •• Die vorliegende Differenzialgleichung
sein, wenn gilt ist exakt mit Stammfunktion
ϕ(x, y) = x 2 y .
w u
= λ, = λ, mit λ ∈ R. Die Lösungen y(x) dieser Differenzialgleichung sind für
w u
x = 0 durch
c
y(x) = 2 , c ∈ R
x
Aufgabe 3.21 •• –
gegeben.

Aufgabe 3.22 ••• Zeigen Sie zuerst, dass alle Eigen- Aufgabe 3.8 •• Mit dem integrierenden Faktor
werte positiv sind und man daher λ = μ2 , μ ∈ R setzen u(x, y) = ex ist die Differenzialgleichung
kann. Multiplizieren Sie dazu die Gleichung mit u und in- ex (y 2 − 2x − 2) + 2ex y y  (x) = 0
tegrieren Sie über das Intervall [1, e]. Es handelt sich hier
um eine Euler’sche Differenzialgleichung. Machen Sie daher exakt. Die Lösung lautet
einen Ansatz u(x) = x α , α ∈ C . 
y(x) = ± 2x + ce−x , c ∈ R.

Aufgabe 3.9 •• Die Differenzialgleichung


Beweisaufgaben 1 1
2
(y 2 − xy) + 2 (2xy 3 + xy + x 2 )y  (x) = 0
Aufgabe 3.23 ••• Studieren Sie die Voraussetzungen xy xy
des Satzes von Picard-Lindelöf. ist exakt. Die Lösung in impliziter Form lautet:
x
ln(xy) − + y2 = c , c ∈ R.
Aufgabe 3.24 ••• – y
16 Lösungswege zu Kapitel 3

Aufgabe 3.10 ••• Aufgabe 3.21 ••


2x 1 1
y(x) = , c=1 k(x) = − p  − p 2 + q
2c − x 2 2 4

Aufgabe 3.11 •• Die Differenzialgleichung ist exakt, Die Transformation bewirkt beim Eigenwertproblem, dass λ
ihre Lösung lautet ein Eigenwert von L ist, genau dann wenn λ ein Eigenwert
von L̄ mit L̄v = v  + k(x)v .
1
x 2 + xy − y 2 = c , c∈R Die Hermitesche Differenzialgleichung wird zu
2
und in expliziter Form, wenn wir nach y auflösen v  + (1 − x 2 + 2n)v = 0 .

y = x ± 3x 2 − 2c .
Aufgabe 3.22 ••• Die Eigenfunktionen sind uk (x) =
sin(π k ln x) zu den Eigenwerten λk = k 2 π 2 , k ∈ N.
Aufgabe 3.12 ••
1
u(x) = , x = 0
x Beweisaufgaben
ist ein integrierender Faktor. Aufgabe 3.23 ••• Es gibt zwei Lösungen y(x) ≡ 0 und

y(x) = x 2 . Da die Funktion f (y) = 2 y(x) die Bedingung
Aufgabe 3.13 •• –
der Lipschitz-Stetigkeit an der Stelle y = 0 nicht erfüllt, stellt
das keinen Widerspruch zum Satz von Picard-Lindelöf dar.
Aufgabe 3.14 •• y(x) = xz(x) = x(cx − 1) = cx 2 − x,
c∈R
Aufgabe 3.24 ••• –
Aufgabe 3.15 ••
1
y(x) = (x 2 + c)2
4
1 2 Lösungswege
y(0) = 1 ⇒ y(x) = (x + 2)2
4
1
y(0) = 0 ⇒ y(x) = x 4 Verständnisfragen
4
1 Aufgabe 3.1 • –
y(0) = − 1 ⇒ y(x) = (x 2 + 2i)2
4
Für y(0) = −1 existiert keine reelle Lösung. Aufgabe 3.2 •• Diese Differenzialgleichung ist separa-
bel, da die beiden Variablen x und y getrennt werden können.
ex + e−x − 2 Es gilt
Aufgabe 3.16 •• y(x) =
2
dy
= ex e−y ,
Aufgabe 3.17 •• y(x) = c1 sin x , c1 ∈ R beliebig dx
ey dy = ex dx ,
Aufgabe 3.18 ••
und nach Integration erhalten wir
a) L[f  (t)] = L[a cosh(at)] = as
s 2 −a 2
b) L[f  (t)] = L[3t 2 ] = 6
s3 ey = ex + c , c ∈ R ,
ω y(x) = ln(ex + c) .
Aufgabe 3.19 ••• L[f (t)] = L[sin(ωt)] = F (s) = s 2 +ω 2

Wir setzen die gegebene Anfangsbedingung y(0) = 1 ein,


Aufgabe 3.20 •••
woraus
u(x, t) = w(t)v(x)

y(0) = ln(1 + c) = 1 ,
= (ak cos(kπt) + bk sin(kπt)) sin(kπx)
1 + c = e1 ,
k=1
1 c =e − 1
mit ak = 2 g(s) sin(kπs)ds
0 folgt und schließlich
1
2
und bk = h(s) sin(kπs)ds
kπ 0 y(x) = ln(ex + e − 1) .
Lösungswege zu Kapitel 3 17

Aufgabe 3.3 •• Mit partieller Integration folgt: daher gilt



−4t dy 1 yγ 1 1
L[f (t)] = L[2 t e ] = 2 t e−4t e−st dt = du = du
0 by γ − ay 1 − γ by γ − ay 1 − γ b − ay 1−γ

2
=2 t e−(s+4)t dt = ... = = F (s) und
0 (s + 4)2 dy 1 du
= .
by − ay
γ 1−γ b − au

Mit der weiteren Substitution t = b − au, dt = −a du folgt
L[f (t)] = L[sinh (at)] = sinh (at) e−st dt
0
∞  dy 1 dt
1 a = −
= e(a−s)t − e−(a+s)t dt = ... = = F (s) by γ − ay a(1 − γ ) t
2 0 s 2 − a2
ln t ln(b − au)
=− = = x + c1 .
a(1 − γ ) a(γ − 1)

L[f (t)] = L[sin t] dt = 2
sin2 t e−st dt Auflösen nach u ergibt die allgemeine Lösung
0
∞
1  2
= 1 − cos(2t) e−st dt = ... = = F (s) u(x) =
b c
+ exp(a(γ − 1)x) , c ∈ R ,
2 0 s(s + 4)
2
a a
und schließlich
Rechenaufgaben b c
y(x) = u1−γ (x) = ( + exp(a(γ − 1)x))1−γ .
a a
Aufgabe 3.4 •• Die gegebene Differenzialgleichung
In den Aufgaben in Kapitel 3 haben wir die Substitution
dy u(x) = y 1−γ (x) vor der Integration durchgeführt und haben
y  (x) = = (x 2 + 1)(y 2 + 1)
dx eine einfache lineare Differenzialgleichung für u erhalten.
ist separabel. Sie kann daher folgendermaßen gelöst werden: Diese Vorgangsweise funktioniert auch für den allgemeinen
Fall a = a(x) und b = b(x).
dy
= (x 2 + 1) dx ,
y2 +1 Aufgabe 3.6 • Diese Differenzialgleichung ist vom
x3 Typ y  (x) = f (ax + by(x) + c) und wird durch die Substi-
arctan y = + x + c, c ∈ R, tution
3   u(x) = 2x − y(x)
x3
y(x) = tan +x+c . gelöst, daraus folgt u (x) = 2−y  (x) oder y  (x) = 2−u (x)
3
und die entsprechende Differenzialgleichung lautet
Aus der Anfangsbedingung y(0) = tan(0 + 0 + c) = tan c = 1
du
folgt c = π4 und die Lösung ist −u (x) = − = u(x) − 2
dx

  du
x3 π = − dx
y(x) = tan +x+ . u−2
3 4
ln |u − 2| = − x + c1 , c1 ∈ R
u(x) = ec1 e−x + 2
Aufgabe 3.5 •• Die gegebene autonome nichtlineare
Differenzialgleichung mit konstanten Koeffizienten lässt sich y(x) = − ec1 e−x − 2 + 2x = cex − 2 − 2x , c ∈ R.
mittels Separation der Variablen lösen. Aus der äquivalenten
Formulierung Kommentar: Natürlich ist diese Differenzialgleichung
dy
= dx eine lineare skalare inhomogene Differenzialgleichung und
by − ay
γ
kann auch mithilfe der Methoden aus Kapitel 2 gelöst wer-
erhält man den.

dy
= dx = x + c1
by γ − ay
Aufgabe 3.7 •• Die Funktionen 2xy = p(x, y) und
mit einer Integrationskonstante c1 ∈ R. Für die Berechnung x 2 = q(x, y) sind auf ganz R2 definiert und es gilt
dieses Integrals wählen wir die Substitution
∂(2xy) ∂(x 2 )
= 2x = .
u = y 1−γ , du = (1 − γ )y −γ dy , ∂y ∂x
18 Lösungswege zu Kapitel 3

Damit existiert eine Stammfunktion, die durch Integration mit c2 (x) = 2x ex und c1 (y) = 0 ergibt. Die Lösung der
ermittelt werden kann: Differenzialgleichung lautet daher

ϕ(x, y) = 2xy dx = x 2 y + c1 (y) , y(x) = ± 2x + ce−x , c ∈ R.

ϕ(x, y) = x 2 dy = x 2 y + c2 (x). Aufgabe 3.9 •• Durch Umformung erhalten wir die
Differenzialgleichung in gewohnter Form
Die Integrationskonstanten c1 , c2 können jeweils noch von
der Variablen abhängen, nach der nicht integriert wurde. (y 2 − xy) + (2xy 3 + xy + x 2 )y  = 0 .
Durch ϕ(x, y) = x 2 y ist eine Stammfunktion gegeben und
durch Auflösen der Gleichung x 2 y = c für c ∈ R nach y Diese Differenzialgleichung ist nicht exakt, wie man
erhalten wir eine Lösung y(x) der Differenzialgleichung, leicht nachrechnet. Wir versuchen als integrierenden Faktor
u(x, y) = x a y b mit a, b ∈ R . Das führt auf
c
y(x) = für x  = 0 . ∂u(x, y) ∂u(x, y) ∂q ∂p
x2 p −q − u(x, y) + u(x, y) = 0 ,
∂y ∂x ∂x ∂y
(y 2 − xy)bx a y b−1 − (2xy 3 + xy + x 2 )ax a−1 y b
Aufgabe 3.8 •• Diese Differenzialgleichung ist zu-
nächst nicht exakt, da −(2y 3 + y + 2x)x a y b + (2y − x)x a y b = 0 ,

∂ϕ zur Berechnung des entsprechenden integrierenden Faktors.
ϕ(x, y) = dx = p(x, y) dx Nach dem Ausmultiplizieren und der Division durch x a y b
∂x
folgt
= (y 2 − 2x − 2) dx = y 2 x − x 2 − 2x + c1 (y) ,
(−2a − 2)y 3 + (−a + b + 1)y + (−a − b − 3)x = 0 .
∂ϕ
ϕ(x, y) = dy = q(x, y) dy
∂y Alle Koeffizienten müssen verschwinden,

= 2y dy = y 2 + c2 (x) −2a − 2 = 0 ,
−a + b + 1 = 0 ,
und die Konstanten c2 (x) und c1 (y) nicht so gewählt werden
−a − b − 3 = 0 ,
können, dass die Integrabilitätsbedingungen erfüllt sind. Wir
versuchen daher einen integrierenden Faktor u(x, y) zu fin- woraus wir a = −1 und b = −2 erhalten. Für den integrie-
den, der die Differenzialgleichung exakt macht und probieren renden Faktor ergibt sich
den einfachsten Fall u(x, y) = u(x) aus. Wir berechnen die
entsprechende Integrabilitätsbedingung 1
u(x, y) =
xy 2
du(x) du(x) ∂q ∂p
p −q − u(x) + u(x) = 0 und die Differenzialgleichung
dy dx ∂x ∂y
du(x) 1 1
(y 2 − 2x − 2) · 0 − 2y − (0 − 2y) u(x) = 0 (y 2 − xy) + 2 (2xy 3 + xy + x 2 )y  = 0
dx xy 2 xy
du(x)
−2y + 2yu(x) = 0 , ist exakt mit Stammfunktion ϕ(x, y) und Lösung (in impli-
dx
ziter Form)
woraus für den integrierenden Faktor u(x) = ex folgt. Also
x
ist die Differenzialgleichung ϕ(x, y) = ln(xy) − + y2 = c , c ∈ R.
y
ex (y 2 − 2x − 2) + 2ex y y  (x) = 0 Kommentar: Die Entscheidung, nach welcher der beiden
Variablen aufgelöst werden kann, kann im Allgemeinen mit-
exakt mit Stammfunktion hilfe des Satzes über implizite Funktionen getroffen werden.

ϕ(x, y) = ex (y 2 − 2x) ,
Aufgabe 3.10 ••• Wir schreiben die Differenzialglei-
die sich aus der Berechnung der Integrale chung in der Form
y(1 + xy)dx − x dy = 0
ex (y 2 − 2x − 2) dx = ex (y 2 − 2x) + c1 (y) ,
d. h.,
2ex y dy = y 2 ex + c2 (x) ,
p dx+q dy = 0 , p(x, y) = y+xy 2 , q(x, y) = −x .
Lösungswege zu Kapitel 3 19

Es ist py = 1 + 2xy  = qx = −1, also nicht exakt. Wir und daher muss gelten
versuchen u = u(y) so zu finden, dass
x2
c(x) − = c(y)

u(y) (y + xy 2 ) dx −u(y) x dy = 0
        2
hängt nur von y ab
hängt nur von x ab
=: P (x,y) =: Q(x,y)
x2 x2
exakt ist, d. h., wir versuchen die Integrabilitätsbedingungen const = c(y) = c(x) − ⇒ c(x) =
2 2
für P und Q zu erfüllen:
und
 x x2
2
Py = u(y)(1 + 2xy) + u (y)(y + xy ) = Qx = −u(y) . ϕ(x, y) =
+
y 2
Wir wollen ist die gesuchte Stammfunktion.

u (y)(y + xy 2 ) = − u(y)(2 + 2xy) , Lösungen der ursprünglichen Differenzialgleichung sind


charakterisiert durch ϕ(x, y) = const, und daraus folgt durch
u (y)y(1 + xy) = −2 u(y)(1 + xy)
Auflösen nach y
erfüllen und das ist wiederum eine Differenzialgleichung für 2x
y(x) = c ∈ R.
u(y). Nach Kürzen durch den Faktor (1 + xy)  = 0 verbleibt 2c − x 2
als geforderte Gleichung
Für die spezielle Lösung, die durch den Punkt (x, y) =
u (y)y = −2 u(y) . (2, −2) verläuft, ergibt sich

Das ist eine separable Differenzialgleichung für den gesuch- 4


−2 = ⇒ c = 1,
ten integrierenden Faktor u(y), 2c − 22
und daher
1 du dy 2x
=− . y(x) = .
2 u y 2 − x2

Integration mit Integrationskonstante c = 0 gewählt, ergibt Die Lösung ist nicht definiert für x = ± 2.

1 Aufgabe 3.11 •• Wir überprüfen die Integrabilitätsbe-


ln u = − ln y dingung ∂p ∂q
2 ∂y = ∂x
 
1 ∂p ∂
exp ln u = exp(− ln y) = (2x + y) = 1 ,
2 ∂y ∂y
u1/2 = y −1 . ∂q ∂
= (x − y) = 1 .
∂x ∂x
Also ist
1 Die Differenzialgleichung ist also exakt.
u(y) =
y2
Wir suchen nun die Stammfunktion ϕ(x, y), sodass gilt
ein integrierender Faktor und die Differenzialgleichung
∂ϕ(x, y)
  = p(x, y) = 2x + y
1 x ∂x
+ x − 2 dy = 0
y y ∂ϕ(x, y)
= q(x, y) = x − y ,
∂y
ist exakt. Die Lösung dieser exakten Differenzialgleichung
und erhalten, indem wir die erste Gleichung integrieren,
erfordert die Kenntnis einer Stammfunktion ϕ(x, y) des Vek-
torfeldes (P , Q), d. h. ϕ(x, y) mit ∂ϕ
ϕ(x, y) = dx = x 2 + xy + c(y) .
∂x
1 x
ϕx = P = + x, ϕy = Q = − . Um c(y) zu bestimmen, differenzieren wir nach y und ver-
y y2 gleichen
Die Integration dieser beiden Forderungen nach x bzw. y ∂ϕ(x, y)
= x + c (y) = q(x, y) = x − y .
ergibt ∂y

x x2 Also ist c (y) = −y und c(y) = − 21 y 2 + const, d. h., die


ϕ(x, y) = + + c(y) Stammfunktion der exakten Differenzialgleichung ist
y 2
x 1
ϕ(x, y) = + c(x), ϕ(x, y) = x 2 + xy − y 2
y 2
20 Lösungswege zu Kapitel 3

für const = 0 und die allgemeine Lösung erfüllt Aufgabe 3.14 •• Diese homogene Differenzialglei-
chung lässt sich durch die Substitution y(x) = xz(x) fol-
1
x 2 + xy − y 2 = c , c ∈ R . gendermaßen umschreiben:
2
Wir können nach y (oder x) auflösen und erhalten für die y  (x) = z(x) + x z (x) = 1 + 2z(x) ,
Lösung y(x) xz (x) = 1 + z(x) .

y(x) = x ± 3x 2 − 2c , c ∈ R . Das ist eine separable Differenzialgleichung, die durch Tren-
nung der Variablen gelöst werden kann
Aufgabe 3.12 •• Wir versuchen, einen integrierenden dz
Faktor u(x) zu finden, der nur von x abhängt, und überprüfen xz (x) = x = 1 + z(x) ,
dx
die Integrabilitätsbedingung, wobei P (x, y) = p(x, y) u(x) dz dx
und Q(x, y) = q(x, y) u(x): = ,
1+z x

∂P ∂ ∂ dz dx
= (p(x, y) u(x)) = ((1 − xy) u(x)) = −x u(x) , = ,
∂y ∂y ∂y 1+z x
∂Q ∂ ∂ ln(z + 1) = ln(x) + c1 , c1 ∈ R ,
= (q(x, y) u(x)) = ((xy − x 2 ) u(x))
∂x ∂x ∂x z + 1 = ec1 x = cx ,
= (y − 2x) u(x) + (xy − x 2 ) u (x) . z(x) = cx − 1 , c ∈ R.
∂P ∂Q
Aus ∂y = ∂x folgt Rücksubstitution führt auf die allgemeine Lösung

−x u(x) = (y − 2x)u(x) + (xy − x 2 )u (x) , y(x) = xz(x) = x(cx − 1) = cx 2 − x , c ∈ R.



(x − y)u(x) = x(y − x)u (x) , x  = y ,
u(x) = −x u (x) .
Aufgabe 3.15 ••
Wir lösen die Differenzialgleichung für u(x) mittels Tren- dy √
y = = 2x y
nung der Variablen: dx
dy
du dx √ = 2xdx
=− , y
u x
du dx dy
=− , √ = 2xdx
u x y

ln u = − ln x + c , c ∈ R , 2 y = x2 + c
ec
u(x) = . d. h., wir erhalten als allgemeine Lösung
x
Die Differenzialgleichung 1 2
y(x) = (x + c)2 .
4
1 − xy
+ (y − x)y  = 0 Nun lösen wir die drei Anfangswertprobleme:
x
ist exakt. 1 2 1
y(0) = 1 = c ⇒ c = 2, y(x) = (x 2 + 2)2
4 4
Aufgabe 3.13 •• n = 0, 1 2 1
∞ y(0) = 0 = c ⇒ c = 0, y(x) = x 4
e−st ∞ 1 4 4
L[f0 (t)] = F (s) = t 0 e−st dt = −  = 1 2 1
0 s 0 s y(0) = −1 = c ⇒ c = 2i, y(x) = (x 2 + 2i)2
4 4
n → n + 1,
Im dritten Fall existiert keine reelle Lösung.

n+1 −st
L[fn+1 (t)] = F (s) = t e dt
0 Aufgabe 3.16 •• Die allgemeine Lösung einer inhomo-

e−st ∞n + 1 ∞ n −st genen Differenzialgleichung ist y = yh + yp , wobei yh die
= − t n+1  + t e dt allgemeine Lösung der homogenen Gleichung y  − y = 0
s 0 s 0
n + 1 n! (n + 1)! ist und yp irgendeine spezielle Lösung von y  − y = 1 ist.
= = ,
s s n+1 s n+2
Für die homogene Gleichung ist
wobei wir hier partielle Integration verwendet haben und die
Induktionsvoraussetzung eingesetzt haben. y1 (x) = ex , y2 (x) = e−x
Lösungswege zu Kapitel 3 21

ein Fundamentalsystem und die allgemeine homogene Lö- Aufgabe 3.17 •• Die Funktionen
sung hat die Form
y1 (x) = cos x , y2 (x) = sin x
yh (x) = c1 y1 (x) + c2 y2 (x) .
bilden ein Fundamentalsystem für die gegebene homogene
Die Partikulärlösung bestimmen wir durch Variation der Differenzialgleichung. Die allgemeine Lösung ist
Konstanten:
y(x) = c1 y1 (x) + c2 y2 (x) , c1 , c2 ∈ R .
yp (x) = c1 (x)y1 (x) + c2 (x)y2 (x) , c1 , c2 ∈ R .
Wir bestimmen c1 und c2 durch Einsetzen der Lösung in die
Da wir nur an einer speziellen Lösung interessiert sind, kön- Randbedingungen:
nen wir zusätzliche Voraussetzungen an c1 (x) und c2 (x) fest-
legen, etwa 0 = c1 cos 0 + c2 sin 0 = c1 .
0 = c1 cos π + c2 sin π = −c1 ,
c1 (x)y1 (x) + c2 (x)y2 (x) = 0 .
also c1 = 0, c2 ∈ R beliebig und die Lösungen haben die
Wenn wir diesen Ansatz für yp (x) in die Gleichung einsetzen,
Gestalt
erhalten wir
y(x) = c2 sin x, c2 ∈ R .
1 = yp (x) − yp (x) = c1 (x)y1 (x) + c2 (x)y2 (x) .
Aufgabe 3.18 ••
Das ist ein lineares Gleichungssystem für c1 , c2 :
a) Es gilt: f  (t) = a cosh(at) und f (0) = sinh 0 = 0.
ex c1 (x) + e−x c2 (x) = 0 ,
ex c1 (x) − e−x c2 (x) = 1 . L[f  (t)] = L[a cosh(at)] = s L[sinh(at)] − 0
a as
=s = 2
Daraus folgt s 2 − a2 s − a2

c1 (x) =
1 −x
e ,
1
c2 (x) = − ex , b) Es gilt: f  (t) = 3t 2 und f (0) = 03 = 0.
2 2
L[f  (t)] = L[3t 2 ] = s L[t 3 ] − 0
und durch Integration
6 6
1 1 =s = 3
c1 (x) = − e−x , c2 (x) = − ex s4 s
2 2
also ist Aufgabe 3.19 ••• Es gilt: f (0) = sin 0 = 0, f  (t) =
ω cos(ωt) und daher f  (0) = ω. Aus
yp (x) = c1 (x)y1 (x) + c2 (x)y2 (x) ,
L[f  (t)] = L[−ω2 f (t)] = −ω2 L[f (t)] = −ω2 F (s)
1 1
= − e−x ex − ex e−x = −1 ,
2 2 L[f  (t)] = s 2 F (s) − s · 0 − ω
eine Partikulärlösung. (Man könnte auch sofort sehen, dass folgt durch Gleichsetzen
yp (x) = −1 die Gleichung yp − yp = 1 erfüllt.)
s 2 F (s) − ω = −ω2 F (s)
Kommen wir nun zur Lösung des Randwertproblems: oder
ω
x
y(x) = c1 e + c2 e −x
−1 F (s) = = L[sin(ωt)] .
s2 + ω2
ist die allgemeine Lösung. Einsetzen der Randbedingungen
führt auf Aufgabe 3.20 ••• Wir machen einen Separationsansatz
u(x, t) = w(t)v(x) und differenzieren
y(0) = c1 + c2 − 1 = 0 ,
utt (x, t) = w  (t) v(x) ,
y  (0) = c1 − c2 = 0.
uxx (x, t) = w(t) v  (x) ,
Wir lösen dieses Gleichungssystem und erhalten w  (x) v(x) = w(t) v  (x) ,
1 1 w  v 
c1 = , c2 = . = = λ.
2 2 w v
Die Lösung des Randwertproblems ist daher Das führt auf ein Sturm-Liouville Eigenwertproblem
v  = λv mit der Lösung
ex + e−x − 2
y(x) = . √ √
2 v(x) = c1 exp( λ x) + c2 exp(− λ x) .
22 Lösungswege zu Kapitel 3

Wir setzen die Randbedingungen ein: und


1 1
0 = v(0) ⇒ c1 = −c2 = c , c ∈ R Lu = 0 ⇔ v  + (− p  − p 2 + q )v = 0 .
√ √  2 4 
0 = v(1) ⇒ c(exp( λ) − exp(− λ)) = 0 . k(x)
Da hier ein Sturm-Liouville Eigenwertproblem vorliegt, be- Betrachten wir nun das Eigenwertproblem:
trachten wir nur reelle Eigenwerte. Die zweite Bedingung
kann nur dann erfüllt sein wenn λ ≤ 0 oder wenn c = 0 ist. Lu = u + pu + qu = λu ,

Im Fall c = 0 existiert nur die triviale Lösung und im Fall e − 21 pdx 1 1 1
(v  − p  v − p 2 v + qv) = λe− 2 pdx v ,
λ = 0 ist v linear oder konstant, aber durch die Randbedin- 2 4
gungen folgt v ≡ 0. Für λ < 0 gilt v  + k(x)v = λv ,
√ √  0 = v  + (k(x) − λ)v ,
0 = exp( λ) − exp(− λ) = 2i sin( |λ|)
d. h. λ ist ein Eigenwert von L genau dann, wenn λ ein Eigen-
⇒ λk = −k 2 π 2 , k ∈ N
wert von L̄ ist, wobei
mit den Eigenfunktionen
L̄v = v  + k(x)v.
vk (x) = 2i c sin(kπx) .
Wenden wir die Transformation auf die Hermitesche Diffe-
Nun wollen wir noch das Anfangswertproblem lösen. Wir renzialgleichung an
entwickeln dazu die Lösung in eine Fourierreihe u − 2xu + 2nu = 0 ,
u(x, t) = w(t)v(x) p(x) = −2x, q(x) = 2n ,

so folgt
= (ak cos(kπt) + bk sin(kπt)) sin(kπx)
1 1
k=1
⇒ k(x) = − (−2) − (−2x)2 + 2n
2 4
und setzen die Anfangswerte ein:
= 1 − x 2 + 2n ,


u(x, 0) = g(x) = ak sin(kπx) , und wir erhalten
k=0
v  + (1 − x 2 + 2n)v = 0.

ut (x, 0) = h(x) = bk kπ sin(kπx) ,
k=1 Aufgabe 3.22 ••• Zunächst zeigen wir, dass λ > 0 ist.
1 Dazu multiplizieren wir die gegebene Differenzialgleichung
⇒ ak = 2 g(s) sin(kπs)ds , mit u
0 λ
1 (−xu ) = u ,
2 x
⇒ bk = h(s) sin(kπs)ds .
kπ 0   λ
u(−xu ) = u2
x
Aufgabe 3.21 •• Aus und integrieren partiell
e e
1 1 
v(x) = e2 pdx
u(x) oder u(x) = e− 2 pdx
v(x)    e
u(−xu ) dx = (−xu u) 1 + xu2 dx
1    1
=0
  
folgt durch Differenzieren >0
  e
1 1 ! 1 2
u = e− 2 pdx − pv + v  , =λ u dx ,
2 x
   1  
1
1  1  1 1
 − 2 pdx   >0
u =e − pv − p v + v − p − pv + v ,
2 2 2 2 da u(1) = u(e) = 0 und nur positive Funktionen integriert
daher gilt werden. Wir setzen λ = μ2 mit μ ∈ R .
Mit dem Ansatz u(x) = x α , α ∈ C folgt
Lu = u + p(x)u + q(x)u
 u(x) = x α ,
− 21 pdx 1 1
=e − pv  − p  v + v  + p 2 v
2 4 u (x) = αx α−1 ,

1
+ p(v  − pv) + qv u (x) = α(α − 1)x α−2 .
2
  Die gegebene Differenzialgleichung lässt sich umschreiben
1 1 1
= e− 2 pdx v  − p  v − p 2 v + qv zu
2 4
0 = x 2 u + xu + μ2 u .
Lösungswege zu Kapitel 3 23

Einsetzen ergibt und aus der gegebenen Anfangsbedingung folgt y(0) =


(0 + c)2 = c2 = 0, und daher ist y(x) = x 2 eine Lösung
0 = (α(α − 1) + α + μ2 )x α
dieser nichtlinearen Differenzialgleichung. Aber auch die
0 = α 2 + μ2 Funktion y(x) ≡ 0 ist eine Lösung zur gegebenen Anfangs-
± iμ = α. bedingung. Im Satz von Picard-Lindelöf ist die Lipschitz-
Stetigkeit der rechten Seite f (y) die wesentliche Vorausset-
Wir erhalten die Lösungen uμ (x) = c1 x iμ + c2 x −iμ und zung für die Eindeutigkeit der Lösung der Differenzialglei-
Einsetzen der Randbedingungen führt auf √
chung. Die Funktion f (y) = y ist aber an der Stelle y = 0
uμ (1) = 0 ⇒ 0 = c1 + c2 ⇒ c1 = −c2 = c , c ∈ R , nicht Lipschitz-stetig. Die Existenz von zwei Lösungen stellt
daher keinen Widerspruch zum Satz von Picard-Lindelöf
uμ (e) = 0 ⇒ 0 = c(eiμ − e−iμ ) = 2ic sin μ , dar.
⇔ μk = πk ,
⇔ λk = π 2 k 2 , k ∈ N . Aufgabe 3.24 ••• Wir multiplizieren zunächst die Pen-
Es ergeben sich die Eigenfunktionen uk (x) = x iπk − delgleichung mit ϕ  (t)
x −iπ k = 2i sin(πk ln x) zu den Eigenwerten λk = k 2 π 2 .
Bei den Eigenfunktionen kommt es nicht auf die Konstanten ϕ  (t)ϕ  (t) + ω2 ϕ  (t) sin ϕ(t) = 0 .
an, also erhalten wir als (reelle) Eigenfunktionen
Es gilt nach der Kettenregel:
uk (x) = sin(πk ln x)  
1 d
ϕ  (t)ϕ  (t) = ϕ  (t)2 ,
zu den Eigenwerten λk = k 2 π 2 , k ∈ N . 2 dt
 
d
ϕ  (t) sin ϕ(t) = − cos ϕ(t) ,
dt
Beweisaufgaben
d. h., wir erhalten
Aufgabe 3.23 ••• Die Differenzialgleichung ist separa-
 
bel, also gilt d 1 
(ϕ (t))2 − ω2 cos ϕ(t) = 0
dt 2
dy
√ = dx , c ∈ R ,
2 y und daher

y(x) = x + c , 1 
2
(ϕ (t))2 − ω2 cos ϕ(t) = const .
y(x) = (x + c) , 2
Kapitel 4 Aufgabe 4.6
chung
• Betrachtet wird die Differenzialglei-

y  = Ay
Aufgaben mit  
0 −1
A= , a ∈ R.
a−1 a
Verständnisfragen
Geben Sie für a = −1 und a = 3 jeweils ein Fundamental-
Aufgabe 4.1 • Gegeben sei das Anfangswertproblem system an und zeichnen Sie die Phasenporträts.
mit dem Parameter ε ∈ R
Aufgabe 4.7 •• Gegeben ist das System von Differen-
y  = −y + sin(εy) , y(0) = a ∈ R .
zialgleichungen
Es sei y(t, a, ε) die Lösung dieses Anfangswertproblems.
y1 = −y1 y2
Begründen Sie, warum y(t, a, ε) differenzierbar von ε ab-
hängt. y2 = (2y1 − 1) y2 (y1 , y2 ) ∈ R2 .

Aufgabe 4.2 • Bestimmen Sie allgemeine reelle Lö- a) Bestimmen Sie alle Gleichgewichtspunkte.
sungen der folgenden Differenzialgleichungen. Wie verhal- b) Zeigen Sie, dass die y1 - und y2 -Achse invariant sind.
ten sich die Lösungen für t → ∞ ? Welche Lösungen bleiben c) Zeichnen Sie das Phasenporträt.
für t → ∞ beschränkt? Aufgabe 4.8 •• Gegeben ist die Differenzialgleichung
a) y (4) − y 
=0
y  = −y 2 sin t .
b) y (4) − y  = 0 , y(1) = y  (1) = 1 , y  (1) = y  (1) = 0
c) y  + y = 0 a) Skizzieren Sie das Richtungsfeld.
d) y (4) + 4y  + 4y = 0 b) Lösen Sie das Anfangswertproblem y(0) = y0 , y0 ∈ R.
c) Untersuchen Sie in Abhängigkeit von y0 ∈ R für welche
Rechenaufgaben t ≥ 0 die Lösung existiert.
d) Ist die Lösung y(t) = 0 für t ≥ 0 stabil oder asympto-
Aufgabe 4.3 •• Betrachten Sie das Differenzialglei- tisch stabil?
chungssystem
Aufgabe 4.9 • Gegeben ist die Differenzialgleichung
y1 = − y1 − 2y2 + y12 y22 ⎛ ⎞
1 −1 −2 0
y2 = y1 − y2 − y13 y2 . y  = Ay , A = ⎝ 2 −1 0 ⎠ .
2
0 0 3
Konstruieren Sie eine Ljapunov-Funktion V von der Form
V (y1 , y2 ) = ay12 + by22 mit geeigneten a, b ∈ R. Was kön- Untersuchen Sie, ob der Gleichgewichtspunkt ȳ = 0 stabil
nen Sie über die Stabilität des Gleichgewichtspunktes (0, 0) ist.
aussagen?
Aufgabe 4.10 •Gegeben ist die Differenzialgleichung
Aufgabe 4.4 • Gegeben ist das nichtlineare Differen-  
zialgleichungssystem  0 1
y = Ay , A = .
  −2 −1
 −y1 − 3y22
y (t) = , y(t0 = 0) = y 0 . Zeigen Sie, dass die Funktion
y1 y2 − y23
 
1 2 1 7 1
Zeigen Sie, dass V (y) = +
2 (y1 y22 )
eine Ljapunov- V (y) = y P y ,T
P =
4 1 3
Funktion ist und schließen Sie daraus auf die Stabilitätsei-
genschaften des Gleichgewichtspunktes ȳ = 0 ∈ R2 . eine Ljapunov-Funktion ist. Prüfen Sie mit ihrer Hilfe die
Stabilitätseigenschaften der Nulllösung ȳ = 0.
Aufgabe 4.5 • Überprüfen Sie, dass V (y) = 21 (y12+y22 )
eine Ljapunov-Funktion des Differenzialgleichungssystems Aufgabe 4.11 • Skizzieren Sie für die skalare Diffe-
    renzialgleichung
y1 −y1 + y22
y = = f (y) = y  = y − sin t
y2 −y23 − y1 y2
die Isoklinen und das Richtungsfeld. Zeichnen Sie außerdem
ist und zeigen Sie, dass der Ursprung 0 ∈ R2 ein stabiler die Lösungskurven durch ( π2 , 1) bzw. ( π2 , −1). Welche Aus-
Gleichgewichtspunkt des Systems ist. Welche Aussage lässt sagen hinsichtlich Beschränktheit der beiden Lösungskurven
sich mittels Linearisierung treffen? für t ≥ π2 bzw. t ≤ π2 lassen sich analytisch begründen?
M. Brokate et al., Arbeitsbuch Grundwissen Mathematikstudium – Höhere Analysis, Numerik und
Stochastik, DOI 10.1007/978-3-642-54946-5_3, © Springer-Verlag Berlin Heidelberg 2016
Hinweise zu Kapitel 4 25

Aufgabe 4.12 •• Untersuchen Sie die Stabilität bzw. Aufgabe 4.17 ••• Beweisen Sie die folgende allgemeine
asymptotische Stabilität des Gleichgewichtspunktes ȳ = 0 Version des Lemmas von Gronwall:
der folgenden Differenzialgleichungen. Geben Sie jeweils
Seien u und δ , L : I = [t0 , t1 ] → [0, ∞] stetige Funktionen.
auch die an ȳ = 0 linearisierte Differenzialgleichung an.
Falls
a) y = y sin t t
b) y = sin y u(t) ≤ δ(t) + L(x)u(x) dx , ∀t ∈ I ,
t0
c) y = − sin y
d) y = −t sin y dann gilt
t
e) y = −y 2 sin t t
L(v) dv
u(t) ≤ δ(t) + δ(x)L(x)e x dx , ∀t ∈ I .
t0
Aufgabe 4.13 •• Betrachten Sie das ebene System
y1 = −y2 + ay1 (y12 + y22 ) , Aufgabe 4.18 • Betrachten Sie die skalare autonome
Differenzialgleichung y  = f (y), wobei f ∈ C 1 (R, R) mit
y2 = y1 + ay2 (y12 + y22 ) , a ∈ R. f (0) = f (1) = 0 und f (y) > 0 für y ∈ (0, 1) . Geben Sie
Zeigen Sie: den ω-Limes ω(y0 ) für y0 ∈ [0, 1] an.

a) Der Gleichgewichtspunkt (0, 0) ist für die Linearisierung


ein Zentrum.
b) Für die nichtlineare Differenzialgleichung ist (0, 0) für Hinweise
a < 0 stabil und für a > 0 instabil.
c) Zeichnen Sie die Phasenporträts für a < 0, a = 0, und
Verständnisfragen
a > 0.
Aufgabe 4.1 • Falls die Funktion f in y  = f (y, ε)
Aufgabe 4.14 •• Das folgende Beispiel zeigt, dass man stetig differenzierbar vom Parameter ε abhängt, dann hängt
im Fall nichtautonomer linearer Differenzialgleichungen auch die Lösung y dieser Differenzialgleichung stetig diffe-
renzierbar von ε ab.
y  = A(t)y
von den Eigenwerten der Matrix A(t) nicht auf die Stabilität Aufgabe 4.2 • –
des Gleichgewichtspunktes ȳ = 0 schließen kann. Es sei
 
−1 + 23 cos2 t 1 − 23 sin t cos t Rechenaufgaben
A(t) =
−1 − 23 sin t cos t −1 + 23 sin2 t Aufgabe 4.3 •• –
Zeigen Sie
Aufgabe 4.4 • –
a) Die Eigenwerte λ1,2 (t) von A(t), t ∈ R haben negativen
Realteil. Aufgabe 4.5 • –
t
b) y(t) = e 2 (− cos t, sin t)T ist eine Lösung der Differen-
zialgleichung. Aufgabe 4.6 • –
c) Der Gleichgewichtspunkt ȳ = 0 ist instabil.
Aufgabe 4.7 •• Stellen Sie eine Differenzialgleichung
Aufgabe 4.15 •• Das folgende Lotka-Volterra-Modell für y2 als Funktion von y1 auf und lösen Sie diese.
beschreibt das zeitliche Verhalten zweier Spezies y1 und y2
Aufgabe 4.8 •• –
y1 = y1 (3 − y1 − 2y2 )
y2 = y(2 − y1 − y2 ) Aufgabe 4.9 • –
a) Bestimmen Sie alle Gleichgewichtspunkte des Differen-
Aufgabe 4.10 • –
zialgleichungssystems.
b) Untersuchen Sie die Stabilität und den Typ der Gleichge- Aufgabe 4.11 • –
wichtspunkte mittels Linearisierung.
c) Zeichnen Sie ein plausibles Phasenporträt im 1. Quadran- Aufgabe 4.12 •• –
ten (y1 ≥ 0, y2 ≥ 0).
Aufgabe 4.13 •• Verwenden Sie Polarkoordinaten.
Beweise
Aufgabe 4.14 •• –
Aufgabe 4.16 •• Zeigen Sie, das der ω-Limes einer au-
tonomen Differenzialgleichung y  = f (y), y(t0 ) = y 0 ab- Aufgabe 4.15 •• Beachten Sie, dass die Geraden y1 = 0
geschlossen ist. und y2 = 0 invariant sind.
26 Lösungswege zu Kapitel 4

Beweise Aufgabe 4.7 ••


a) Die Gleichgewichtspunkte sind ȳ a = [a, 0]T mit a ∈ R .
Aufgabe 4.16 •• Es ist zu zeigen, dass falls zn ∈ ω(y 0 )
und limn→∞ zn = z , dass dann auch z ∈ ω(y 0 ). Approxi-
Aufgabe 4.8 ••
mieren Sie zn durch Punkte der Bahn y . 1
b) y(t) = 1+y
t − cos t+ y 0
0
Aufgabe 4.17 ••• Setzen Sie y(t) := t0 L(x)u(x) dx  
 c) Für t = arccos 1+y 0
existiert y(t) nicht.
und zeigen

Sie y ≤ Ly + Lδ . Setzen Sie z(t) := y0
− t L(x) dx d) Die Nulllösung ist stabil, aber nicht asymptotisch stabil.
y(t)e t0 und leiten Sie eine Differenzialungleichung
für z(t) her. Aufgabe 4.9 • Der Gleichgewichtspunkt ȳ = 0 ist sta-
bil.
Aufgabe 4.18 • –
Aufgabe 4.10 • –

Aufgabe 4.11 • Für t ≥ π2 sind die Lösungskurven


Lösungen unbeschränkt und für t ≤ π2 beschränkt.

Verständnisfragen Aufgabe 4.12 ••


a) ȳ ist stabil, aber nicht asymptotisch stabil. Die Differen-
Aufgabe 4.1 • – zialgleichung ist bereits linear.
b) ȳ ist instabil.
Aufgabe 4.2 • c) ȳ ist asymptotisch stabil.
a) y(t) = c1 + c2 t + c3 et + c4 e−1 mit c1 , c2 , c3 , c4 ∈ R , d) ȳ ist asymptotisch stabil.
die Lösung ist beschränkt für t → ∞, falls c2 = c3 = 0 . e) ȳ ist stabil, aber nicht asymptotisch stabil.
b) Die Lösung y(t) = t ist unbeschränkt.
√ √
t t
c) y(t) = c1 e−t + c2 e 2 cos( 23 t) + c3 e 2 sin( 23 t) , die Lö- Aufgabe 4.13 •• –
√ für t → ∞, falls
sung ist beschränkt √ c2 = c3 = √ 0.
d) y(t) = √c1 sin( 2t) + c2 t sin( 2t) + c3 cos( 2t) + Aufgabe 4.14 •• –
c4 t cos( 2t) , die Lösung ist beschränkt für t → ∞,
falls c2 = c4 = 0 . Aufgabe 4.15 •• –

Rechenaufgaben Beweise
Aufgabe 4.3 •• Die Ljapunov-Funktion ist V =
1 2 Aufgabe 4.16 •• –
2 (y1 + y2 ) . Der Gleichgewichtspunkt (0, 0) ist asympto-
2

tisch stabil.
Aufgabe 4.17 ••• –
Aufgabe 4.4 • Der Gleichgewichtspunkt ȳ = 0 ist
Aufgabe 4.18 • ω(0) = 0 und ω(y0 ) = 1 für y0 ∈ (0, 1]
asymptotisch stabil.

Aufgabe 4.5 • Der Gleichgewichtspunkt 0 ist asym-


ptotisch stabil. Mittels Linearisierung lässt sich keine Aus-
sage treffen.
Lösungswege
Aufgabe 4.6 •
Für a = −1 bilden die beiden Lösungen Verständnisfragen
   
et e−2t Aufgabe 4.1 • Die Funktion f (y, ε) ist für alle y, ε ∈
y 1 (t) = und y 2 (x) = R stetig differenzierbar. Es folgt, dass die Lösung y von y  =
−et 2 e−2t
f (y, ε) ebenfalls eine stetig differenzierbare Funktion ist und
ein Fundamentalsystem. somit auch stetig differenzierbar von ε abhängt.
Für a = 3 bilden die beiden Lösungen
    Aufgabe 4.2 •
et e2t
y 1 (t) = und y 2 (x) = a) Der Ansatz y(t) = eλt führt auf die charakteristische
−et −2 e2t
Gleichung
ein Fundamentalsystem. p(λ) = λ4 − λ2 = 0
Lösungswege zu Kapitel 4 27

mit den Lösungen λ1,2 = 0 , λ3 = 1 und λ4 = −1 . Die Aufgabe 4.4 • Es gilt V (ȳ) = 0 und V (y) > 0 für
allgemeine Lösung dieser homogenen Differenzialglei- y = ȳ . Des Weiteren gilt
chung ist daher
V  (y(t)) = grad V (y) · y  (t)
   
y(t) = c1 + c2 t + c3 et + c4 e−1 mit c1 , c2 , c3 , c4 ∈ R . y1 −y1 − 3y22
= · = −(y1 + y22 )2 < 0 ,
y2 y1 y2 − y23
Die Lösung ist beschränkt für t → ∞, falls c2 = c3 = 0 .
für y = ȳ. Folglich ist ȳ = 0 asymptotisch stabil.
b) Die allgemeine Lösung dieser Differenzialgleichung ist
Aufgabe 4.5 • Der Ursprung 0 ∈ R2 ist ein Gleichge-
y(t) = c1 + c2 t + c3 et + c4 e−1 mit c1 , c2 , c3 , c4 ∈ R . wichtspunkt des Systems. Die Funktion V ist eine Ljapunov-
Funktion, denn es ergibt sich mit der Kettenregel
Aus den gegebenen Anfangsbedingungen y(1) =
y  (1) = 1 und y  (1) = y  (y) = 0 folgt c1 = c3 = V  (y(t)) = grad V (y) · y  = −y12 − y24 < 0 ,
c4 = 0 , c2 = 1. Die Lösung y(t) = t ist unbeschränkt. für y = 0 . Des Weiteren gilt V (y) > 0 für y ∈ R2 \ {0} und
c) Der Ansatz y(t) = eλt führt auf die charakteristische V (0) = 0. Daher ist 0 ein striktes lokales Minimum von V
Gleichung und der Ursprung ist asymptotisch stabil.
p(λ) = λ3 + 1 = 0 Die Linearisierung liefert in diesem Fall keine Aussage, da

mit den Lösungen λ1 = −1 und λ2,3 = 21 (1 ± i 3) . die Jacobi-Matrix
  
Die allgemeine Lösung dieser homogenen Differenzial-  −1 0
J  =
gleichung ist daher y =0
0 0
√  √  der Funktion f an 0 den Eigenwert λ = 0 hat.
−t t 3 t 3
y(t) = c1 e + c2 e 2 cos t + c3 e 2 sin t
2 2 Aufgabe 4.6 • Wir berechnen jeweils die Eigenwerte
λ1 , λ2 und die Eigenvektoren v 1 ,v 2 von A. Dann bilden
mit c1 , c2 , c3 ∈ R . Die Lösung ist beschränkt für t → ∞,
y 1 (t) = v1 eλ1 t und y 2 (t) = v2 eλ2 t ein Fundamentalsystem.
falls c2 = c3 = 0 .
d) Der Ansatz y(t) = eλt führt auf die charakteristische a = −1
Gleichung Die Eigenwerte sind die Nullstellen des charakteristischen
Polynoms
p(λ) = λ4 + 4λ2 + 4 = 0 p(λ) = det(A − λ I2×2 ) = λ2 + λ − 2 = (λ + 2)(λ − 1)
√ √
mit den Lösungen λ1,2 = i 2 und λ3,4 = −i 2 . Die und wir berechnen λ1 = 1 und λ2 = −2. Für die Eigen-
allgemeine Lösung dieser homogenen Differenzialglei- vektoren gilt
chung ist daher Av i = λi v i i = 1, 2 ,
√ √ √ und wir erhalten v 1 = [1, −1]T und v 2 = [1, 2]T . Daher
y(t) = c1 sin( 2t) + c2 t sin( 2t) + c3 cos( 2t)
√ bilden
 t  
+ c4 t cos( 2t) e e−2t
y 1 (t) = und y 2 (t) =
−et 2 e−2t
mit c1 , c2 , c3 , c4 ∈ R .
ein Fundamentalsystem. Da die beiden Eigenwerte un-
Die Lösung ist beschränkt für t → ∞, falls c2 = c4 = 0 .
terschiedliche Vorzeichen haben, ist der Gleichgewichts-
punkt ȳ = 0 für a = −1 ein Sattelpunkt.
Rechenaufgaben y2

Aufgabe 4.3 •• Für die Ableitung V = dV


dt der
Ljapunov-Funktion V (y1 , y2 ) = ay12 + by22 gilt

V  (y1 , y2 ) =
= grad V · y  = (2ay1 , 2by2 )T · (y1 , y2 )T
   1 
= 2a − y12 − 2y1 y2 + y13 y22 + 2b y1 y2 − y22 − y13 y22
  2 y1
2
1 1 2
= − y1 + y2 + y2 < 0, falls (y1 , y2 )  = (0, 0) .
2 4

Dabei wurde a = b = 21 gewählt. Aus V  < 0 und V > 0


für alle (y1 , y2 ) = (0, 0) folgt, dass der Gleichgewichtspunkt
(0, 0) asymptotisch stabil ist.
28 Lösungswege zu Kapitel 4

a=3 y2
Die Eigenwerte sind die Nullstellen des charakteristischen
Polynoms

p(λ) = det(A − λ I2×2 ) = λ2 − 3λ + 2 = (λ − 2)(λ − 1)

und wir berechnen λ1 = 1 und λ2 = 2. Für die Eigenvek-


toren gilt
Av i = λi v i i = 1, 2.
y1
und wir erhalten v 1 = [1, −1]T und v 2 = [1, −2]T .
Damit bilden
 t   2t 
e e
y 1 (t) = und y 2 (x) =
−et −2 e2t

ein Fundamentalsystem. Da beide Eigenwerte positiv


sind, ist der Gleichgewichtspunkt ȳ = 0 ein instabiler
Knoten.
Aufgabe 4.8 ••
y2 a) Das Richtungsfeld ist durch das Vektorfeld, das jedem
Punkt (t, y)T ∈ R2 den Vektor (1, y  (t))T ∈ R2 zuord-
net, gegeben.
y (t)

y1

Aufgabe 4.7 ••
a) Es gilt y  = 0 genau dann, wenn y2 = 0 und y1
beliebig. Damit erhalten wir die Gleichgewichtspunkte b) Wir lösen die Differenzialgleichung mittels Separation
y a = [a, 0]T mit a ∈ R. der Variablen
b) Aus y1 = 0 folgt y1 = 0 und aus y2 = 0 folgt y2 = 0 , dy
= −y 2 sin t ,
daher sind die beiden Achsen invariant. dt
1
c) Wir betrachten dy = − sin t dt ,
y2
y2 dy2 1 1
= = −2 + − = cos t + c , c ∈ R ,
y1 dy1 y1 y
1
y(t) = ,
und lösen diese Differenzialgleichung durch Separation − cos t − c
der Variablen. 1
Anfangswert: y0 = y(0) = ,
  −1 − c
1
dy2 = −2 + dy1 , 1 + y0
y1 ⇐⇒ c = − ,
y0
y2 = −2y1 + ln |y1 | + c , c ∈ R. 1
Lösung: y(t) = .
− cos t + 1+yy0
0
Das Phasenporträt ist durch diese Kurven bestimmt.
Lösungswege zu Kapitel 4 29

 
c) Für t = arccos 1+y
y0
0
existiert y(t) nicht. Die Lösung der zugehörigen homogenen Differenzialglei-
d) Die Lösung y = 0 ist nicht asymptotisch stabil, denn chung ist y(t) = c et , c ∈ R. Mit dem Ansatz yp (t) = c(t)et
1 erhalten wir eine Partikulärlösung yp (t) = 21 (cos t + sin t)
lim |y(t)| = lim | 1+y0
| = 0 . und die allgemeine Lösung der inhomogenen Differenzial-
t→∞ t→∞ − cos t + y0 gleichung ist
Die Lösung y = 0 ist aber stabil, denn für jedes ε > 0 1
existiert ein δ > 0 sodass y(t) = (cos t + sin t) + c et , c ∈ R.
  2
 
 1  1 In der Abbildung sind die Lösungen zu den Anfangsbedin-
 ≤ = |y0 | < ε .
 1+y0  1 gungen y( π2 ) = 1 und y( π2 ) = −1 , also
 − cos t + y  | − 1 + y0 + 1|
0

Wähle δ = ε . 1 π
y(t) = (cos t + sin t + e− 2 +t ) und
2
Aufgabe 4.9 • Wir berechnen die Eigenwerte der Ma- 1 π
y(t) = (cos t + sin t + −3e− 2 +t )
trix A mithilfe des charakteristischen Polynoms 2
p(λ) = (3 − λ)((−1 − λ)2 + 4) = (3 − λ)(λ2 + 2λ + 5) gezeichnet.
und erhalten
y (t)
λ1 = −3 ,
λ2 = −1 + 2i ,
λ3 = −1 − 2i .
Die Realteile der Eigenwerte sind alle negativ, der Gleichge-
wichtspunkt ȳ = 0 ist daher stabil.

Aufgabe 4.10 • Die Funktion V ist differenzierbar und


0 t
es gilt
V  (y(t)) = grad V (y) · y  (t) ,
1
= (7y1 + y2 , y1 + 3y2 )(y2 , −2y1 − y2 )T ,
2
1
= (7y1 y2 + y22 − 2y12 − 6y1 y2 − y1 y2 − 3y22 ) ,
2
1
= (−2y12 − 3y22 ) < 0 für y ∈ R2 \ {0} .
2 π π
Diese Lösungen sind für t ≥ unbeschränkt und für t ≤
Daher ist V eine Ljapunov-Funktion und ȳ = 0 ist asympto- 2 2
beschränkt.
tisch stabil, da V an der Stelle ȳ ein striktes lokales Maximum
hat. Aufgabe 4.12 •• Für die Stabilität überprüfen wir, ob
für alle ε > 0 ein δ > 0 existiert, sodass
Aufgabe 4.11 • Die Isoklinen, also die Kurven kon-
stanter Steigung dieser skalaren Differenzialgleichung, sind |y(x, y0 ) − ȳ| ≤ ε für alle y0 mit |y0 | ≤ δ
gegeben durch y  = y − sin t = c , c ∈ R und sind in der
folgenden Abbildung dargestellt. gilt und für die asymptotische Stabilität überprüfen wir, ob
limt→∞ y(t, y0 ) = ȳ ist.
y (t)
a) Die Lösung der gegebenen Differenzialgleichung zur An-
fangsbedingung y(0) = y0 ist y(t, y0 ) = y0 e− cos t+1 .
Sei ε > 0, dann gilt
 
 
|y(t, y0 ) − ȳ| = y0 e− cos t+1  ≤ |y0 |e2 ≤ ε ,

falls |y0 | ≤ ε e−2 =: δ , daher ist ȳ stabil.


t Wegen

lim y(t, y0 ) = lim |y0 e− cos t+1 | = 0 = ȳ


t→∞ t→∞

ist ȳ nicht asymptotisch stabil.


Die Differenzialgleichung ist linear und somit identisch
mit ihrer Linearisierung.
30 Lösungswege zu Kapitel 4

b) Bei skalaren autonomen Differenzialgleichungen stabil, aber nicht asymptotisch stabil (siehe Aufgabe 4.6).
y  = f (y) kann die asymptotische Stabilität des Gleich- Die Linearisierung dieser Differenzialgleichung ist
gewichtspunktes ȳ durch das Vorzeichen der Funktion f
y  = −2t sin(t)y|t=0 = 0 .
bestimmt werden.
Der Gleichgewichtspunkt ȳ ist asymptotisch stabil, wenn Aufgabe 4.13 ••
in einem Intervall (ȳ − δ, ȳ + δ) f (y) > 0 für y < ȳ und 

f (y) < 0 für y > ȳ gilt. Der Gleichgewichtspunkt ȳ ist a) Wir berechnen die Linearisierung y  = J  y am Ur-
y =0
instabil, wenn in einem Intervall (ȳ − δ, ȳ + δ) f (y) < 0 sprung (0, 0) , es ist
für y < ȳ oder f (y) > 0 für y > ȳ gilt.  
−y2 + ay1 (y12 + y22 )
f (y1 , y2 ) = ,
Hier ist der Gleichgewichtspunkt ȳ = 0 instabil, da für y1 + ay2 (y12 + y22 )
  
0 < δ < π2 gilt f (−δ) < 0 und f (δ) > 0 .  3ay12 + ay22 −1 + 2ay1 y
Die zugehörige linearisierte Differenzialgleichung ist J  = ,
y =0 1 + 2ay1 y2 ay12 + 3ay22
 
df  0 −1
y = (0) y = y cos 0 = y , y =
1 0
y.
dy

woraus ebenfalls die Instabilität von ȳ folgt. Die Eigenwerte der Jacobi-Matrix J von f an (0, 0) sind
c) Hier gilt f (−δ) > 0 und f (δ) < 0, und es folgt die asym- λ = ±i , daher ist ȳ = 0 ein Zentrum für die Linearisie-
ptotische Stabilität von ȳ. Die am Gleichgewichtspunkt rung.
linearisierte Differenzialgleichung ist b) Wir transformieren die Differenzialgleichung auf Polar-
koordinaten    
y  = − cos(0)y = −y , T:
r

r cos φ
.
φ r sin φ
daher ist ȳ asymptotisch stabil. Es gilt
d) Hier kann die Lösung der Differenzialgleichung mittels
Separation der Variablen berechnet werden y1 = r  cos φ − r sin φ φ  ,
y2 = r  sin φ + r cos φ φ  ,
1
dy = −t dt , und nach Einsetzen in die Differenzialgleichung erhalten
sin y
wir
y 1 t2
mit u = tan du = − +c, c∈R r  cos φ − r sin φ φ  = −r sin φ + ar 3 cos φ ,
2 u 2
y t2 r  sin φ + r cos φ φ  = r cos φ + ar 3 sin φ .
ln tan = − +c,
2 2
Es gilt r  = ar 3 und φ  = 1 , woraus folgt, dass
y t2
tan = c e− 2 , limt→∞ r(t) = 0 für a < 0 und limt→∞ r(t) = ∞
2   für a > 0 . Daher liegt für a < 0 eine stabile und für
t2
y(t) = 2 arctan c e− 2 , c ∈ R. a > 0 eine instabile Spirale vor.
c) In den folgenden Abbildungen sind die Phasenporträts
Der Gleichgewichtspunkt ȳ = 0 ist stabil und asympto- für a < 0, a = 0, und a > 0 gezeichnet.
tisch stabil, denn es gilt für ε > 0 y2
    
 y(0) − t 2 

|y(t)| = 2 arctan tan e
2
2

   
 y(0) − t 2 
≤ 2 tan e 2
2
  
 y(0) 
≤ 2 tan
2 
ε
≤ ε ⇐⇒ y0 < 2 arctan =: δ
2 y1
0
und limt→∞ y(t) = 0.
Die zugehörige linearisierte Differenzialgleichung ist

y  = −ty cos 0 = −ty .

e) Hier kann die Lösung ebenfalls mittels Separation der Va-


riablen berechnet werden. Der Gleichgewichtspunkt ȳ ist
Lösungswege zu Kapitel 4 31

y2 Aufgabe 4.15 ••
a) Bestimmung aller Gleichgewichtspunkte
(y1 = 0 ∧ 3 − y1 − 2y2 = 0) ∨ (y2 = 0 ∧ 2 − y1 − y2 = 0)
ȳ 0 = (0, 0)
ȳ 1 = (1, 1)
ȳ 2 = (0, 2)
ȳ 3 = (3, 0)
0 y1
b) Wir betrachten die Eigenwerte der Jacobi-Matrix an den
Gleichgewichtspunkten,
 
3 − 2y1 − 2y2 −2y1
J = ,
−y2 2 − y1 − 2y2
  
 3 0
J  = y 0 ist ein instabiler Knoten.
y =y 0
0 2
  
 −1 −2 √
y2 J  = Eigenwerte − 1 ± 2 ,
y =y
−1 −1
1

daher ist y 1 ein Zentrum.


  
 −1 0
J  = y 2 ist ein stabiler Knoten.
y =y
−2 −2
 2  
 −3 −6
J  = y 3 ist ein stabiler Knoten.
y =y
0 −1
3

c) Das Phasenporträt ist in der folgenden Abbildung zu


0 y1 sehen.
y2

1
Aufgabe 4.14 ••
a) Wir bestimmen die Eigenwerte der Matrix A(t) über das
charakteristische Polynom
det(A − λI ) = 1 2 3
y1
  
3 2 3 2
−1 + cos t − λ −1 + sin t − λ
2 2
  
3 3 Beweise
− 1 − sin t cos t −1 − sin t cos t
2 2
Aufgabe 4.16 •• Sei zn ∈ ω(y 0 ) und limn→∞ zn = z .
λ 1
= λ2 + + = 0 Es ist zu zeigen, dass auch z ∈ ω(y 0 ). Sei zn1 mit zn1 −z <
2 2 1 1
√ 2 . Da zn1 ∈ ω(y 0 ) existiert ein t1 sodass y(t1 )−zn1  < 2 .
und erhalten Mithilfe der Dreiecksungleichung folgt
−1 ± i 7
λ1,2 = . 1 1
4 y(t1 ) − z < y(t1 ) − zn1  + zn1 − z < + = 1 .
Die Eigenwerte der Matrix A(t) haben negative Realteile. 2 2
b) Wir überprüfen, ob y(t) die Differenzialgleichung erfüllt Sei jetzt zn2 mit zn2 − z < 41 und da zn2 ∈ ω(y 0 ) exi-
 t t

 e 2 sin t − 21 e 2 cos t stiert ein t2 > t1 , sodass y(t2 ) − zn2  < 41 . Mithilfe der
y (t) = t t ,
e 2 cos t + 21 e 2 sin t Dreiecksungleichung folgt wieder
 t t
 1 1 1
− 21 e 2 cos t + e 2 sin t y(t2 ) − z < y(t2 ) − zn2  + zn2 − z < + = .
A(t)y(t) = t t . 4 4 2
e 2 cos t + 21 e 2 sin t 1
Für tn bekommen wir y(tn ) − z < 2n , sodass
c) y(t) beschreibt eine instabile Spirale. limtn →∞ y(tn ) − z = 0 folgt und daher z ∈ ω(y 0 ) .
32 Lösungswege zu Kapitel 4

Aufgabe 4.17 ••• Falls L(t) = 0, ist die Aussage Daraus folgt
des Satzes trivial erfüllt. Daher nehmen wir im Folgenden
t t
L(t) = 0 an. L(x)u(x) dx = y(t) ≤ L(x)δ(x) e−B(x)+B(t) dx
t t0 t0
Wir setzen y(t) := t0 L(x)u(x) dx . Die Funktion y ist stetig
differenzierbar und es gilt und durch Ableitung
 t 
t
− tt L(v)dv
y  (t) = L(t)u(t) ≤ L(t) δ(t) + L(x)u(x) dx L(t)u(t) ≤ L(t)δ(t) + L(x)δ(x)L(t)e 0 dx ,
t0 t0
= Lδ + Ly .
woraus schließlich
t
− L(x) dx t
Wir setzen z(t) := y(t)e t0 und verwenden die Ab- − tt L(v)dv
t u(t) ≤ δ(t) + L(x)δ(x)e 0 dx
kürzung B(t) := t0 L(x) dx . Für die Ableitung gilt t0

z (t) = y  (t)e−B(t) − y(t)L(t)e−B(t) folgt.

= (y  (t) − L(t)y(t))e−B(t) ≤ L(t)δ(t)e−B(t) . Aufgabe 4.18 • Die einzigen Gleichgewichtspunkte


dieser skalaren Differenzialgleichung sind ȳ1 = 0 und
Integration führt auf
ȳ2 = 1 , da f (y) > 0 auf [0, 1] . Es gilt ω(0) = 0 und
t ω(1) = 1 . Für einen Startwert y0 ∈ (0, 1) ist die zugehörige
z(t) − z(t0 ) ≤ L(x)u(x)e−B(x) dx Lösung y(t, y0 ) monoton wachsend auf (0, 1) . Es gilt des
t0
Weiteren, dass y(t, y0 ) < 1 für jedes t und für y0 ∈ (0, 1)
und beliebig, da Orbits einander nicht schneiden können. Da die
  Funktion f auf (0, 1) keine weiteren Nullstellen hat, ist es
t
z(t) − z(t0 ) = y(t)e−B(t) = L(x)u(x) dx e−B(t) . auch nicht möglich, dass y(t, y0 ) gegen einen Wert kleiner
t0 als 1 konvergiert. Daher folgt ω(y0 ) = 1 für y0 ∈ (0, 1] .
Kapitel 5 Aufgabe 5.10 • Sei ein Weg γ gegeben als das Stück
der Parabel y = x 2 , welches die Punkte (0, 0) und (1, 1)
verbindet. Berechnen Sie

Aufgaben
z dz .
γ
Verständnisfragen
Aufgabe 5.1 • In welchen Punkten z ∈ C ist die kom- Aufgabe 5.11 • Finden Sie die Laurentreihe für
plexe Konjugation f (z) = z differenzierbar? f (z) = 1/(z2 − z) in der punktierten Kreisscheibe {0 <
|z| < 1}.
Aufgabe 5.2 •• Gibt es eine biholomorphe Abbildung
von C auf die offene Einheitskreisscheibe E? Aufgabe 5.12 •• Finden Sie die Laurentreihe für

f (z) = (2z + 2)/(z2 + 1)


Rechenaufgaben
in der punktierten Kreisscheibe {0 < |z − i| < 2}.
Aufgabe 5.3 • Für welche Zahlen z ∈ C gilt a) z2 ∈ R,
b) z2 ≥ 0? Aufgabe 5.13 • (a) Berechnen Sie das Residuum in
c = 0 von f (z) = (3z2 − 4z + 5)/z3 .
Aufgabe 5.4 • Berechnen Sie das Bild f (K\{0}) unter (b) Berechnen Sie das Residuum in c = 1 von f (z) =
der Abbildung f (z) = 1/z, wobei K der Kreis um 1 mit (z − 1)−5 log z.
Radius 1 ist.
Aufgabe 5.14 •• (a) Berechnen Sie das Residuum von
Aufgabe 5.5 • Berechnen Sie das Bild f (H) der obe- f (z) = ez / sin z in c = 0.
ren Halbebene H = {z ∈ C : Im z > 0} unter der Abbildung (b) Berechnen Sie das Residuum von f (z) = (1 + z2 )/
f (z) = −z2 . Ist f auf H injektiv? (1 + ez ) in allen Singularitäten von f .

Aufgabe 5.6 •• Sei Aufgabe 5.15 •• Berechnen Sie


  ∞
1 1 1
f (z) = z+ , f : Cx → C . dz .
2 z 0 1 + z6

Berechnen Sie f (Cx ), f (∂ E) und f (Ex ), wobei Ex = E \{0}.


Ist f : Ex → f (Ex ) bijektiv?
Beweisaufgaben
Aufgabe 5.7 ••• Zeigen Sie, dass für die durch die ste- Aufgabe 5.16 • Sei f die Cayley-Transformation
reographische Projektion p : S2 → C ∪ {∞} auf C ∪ {∞}
induzierte Metrik 1+z
f (z) = ·i.
1−z
d(z, z ) = p−1 (z) − p −1 (z )
Zeigen Sie, dass
gilt
2 Im z
2|z − z | f (z) = −
d(z, z ) =  |1 − z|2
(|z|2 + 1)(|z |2 + 1)
gilt falls |z| = 1, z = 1.
für alle z, z ∈ C.
Aufgabe 5.17 • Sei f (z) = cos(1/z). Zeigen Sie, dass
Aufgabe 5.8 ••• Finden Sie alle geraden – das heißt,
f (ix) → ∞ für x → 0 in R.
f (z) = f (−z) für alle z – holomorphen Funktionen f : C →
C mit f (0) = 1, welche f (z2 ) = f (z)2 in C erfüllen.
Aufgabe 5.18 •• Zeigen Sie: Ist K ein Kreis auf S2 , so
ist dessen stereographische Projektion p(K) eine Gerade oder
Aufgabe 5.9 •• Sei u : R2 → R definiert durch
ein Kreis in C, je nachdem, ob der Nordpol N = (0, 0, 1) auf
u(x, y) = x 2 + 2axy + by 2 . K liegt oder nicht.

Stellen Sie fest, für welche a, b ∈ R die Funktion u Realteil Aufgabe 5.19 •• Sei f : C → C holomorph, es gelte
einer holomorphen Funktion f : C → C ist, und bestimmen
Sie alle solchen Funktionen f . max |f (z)| ≤ Cr m
|z|=r

M. Brokate et al., Arbeitsbuch Grundwissen Mathematikstudium – Höhere Analysis, Numerik und


Stochastik, DOI 10.1007/978-3-642-54946-5_4, © Springer-Verlag Berlin Heidelberg 2016
34 Hinweise zu Kapitel 5

für alle r > 0 mit festen C > 0 und m ∈ N. Zeigen Sie: f (a) Zeigen Sie: Ist ψ − ϕ hinreichend klein, so gilt
ist ein Polynom vom Grad kleiner oder gleich m. ∂S f (z) dz = 0.
(b) Zeigen Sie, dass ∂S f (z) dz = 0 auch ohne Einschrän-
Aufgabe 5.20 •• Zeigen Sie, dass für die Laplace- kung an ϕ und ψ gilt, und beweisen Sie damit den Integralsatz
Transformation L auf {s : Re s > α} gilt von Cauchy für Kreisringe auf Seite 132.
(L(f  ))(s) = s(Lf )(s) − f (0) ,
Aufgabe 5.30 •• Sei γ ein geschlossener Weg in C und
falls f : [0, ∞) → C stetig differenzierbar ist und sowohl c ein nicht auf γ liegender Punkt.
|f (t)| als auch |f  (t)| für alle t ≥ 0 durch Ceαt für geeignete (a) Zeigen Sie: Ist
C > 0 und α ∈ R beschränkt sind.
G(t) = (γ (t) − c) e−g(t) ) ,
Aufgabe 5.21 •• Beweisen Sie den Fundamentalsatz wobei
der Algebra:
t γ  (τ )
g(t) = dτ ,
(a) Jedes nichtkonstante Polynom p : C → C hat eine Null- a γ (τ ) − c
stelle. so ist G konstant.
(b) Jedes Polynom p : C → C vom Grad n ≥ 1 lässt (b) Schließen Sie daraus, dass
sich in n Linearfaktoren zerlegen, das heißt, es gilt p(z) =  
a(z − c1 ) · · · (z − cn ) für geeignete komplexe Zahlen 1
exp dz = 1 .
a, c1 , . . . , cn . γ z−c

Aufgabe 5.22 •• Zeigen Sie: Jede holomorphe Funk-


tion f : C → C, deren Realteil auf C beschränkt ist, ist kon-
stant. Hinweise
Aufgabe 5.23 •• Sei f auf einem Gebiet G holomorph
und nicht gleich der Nullfunktion. Zeigen Sie, dass f keinen Verständnisfragen
kompakten Träger in G haben kann. (Holomorphe Funktio- Aufgabe 5.1 • –
nen können also nicht zur Konstruktion von Zerlegungen der
Eins – siehe Band I, Abschnitt 23.4 – herangezogen werden.) Aufgabe 5.2 •• Satz von Liouville.

Aufgabe 5.24 •• Zeigen Sie, dass jede auf einem Gebiet


G holomorphe und injektive Funktion f auf G biholomorph Rechenaufgaben
ist.
Aufgabe 5.3 • –
Aufgabe 5.25 • Sei c eine m-fache Polstelle einer ho-
lomorphen Funktion f : U → C. Zeigen Sie: Ist g eine auf Aufgabe 5.4 • –
U ∪ {c} holomorphe Funktion, so ist c eine m-fache Polstelle
von f + g. Aufgabe 5.5 • –

Aufgabe 5.26 •• Sei c eine wesentliche Singularität Aufgabe 5.6 •• –


einer holomorphen Funktion f . Zeigen Sie, dass c dann auch
eine wesentliche Singularität der Funktion 1/f ist. Aufgabe 5.7 ••• Seien (w, t) = p−1 (z), (w , t  ) =
p −1 (z ). Zeigen Sie durch Rechnen mit den Formeln (5.3) –
Aufgabe 5.27 •• Zeigen Sie: Ist f meromorph auf U (5.6) die Identitäten
und c ∈ U ein Pol von f , so gibt es eine rationale Funktion
r mit P (r) = {c} und P (f − r) = P (f ) \ {c}. d(z, z )2 = |w−w |2 +(t −t  )2 = 2 − ww − w  w − 2tt  ,
2 − 2tt 
Aufgabe 5.28 •• Zeigen Sie: Ist f meromorph auf U = |z|2 + |z |2 ,
(1 − t)(1 − t  )
und P (f ) endlich, so lässt sich f darstellen als Summe einer
ww  + w  w
rationalen Funktion und einer auf U holomorphen Funktion. = |z|2 + |z |2 − |z − z |2 .
(1 − t)(1 − t  )
Aufgabe 5.29 •• Sei A = {z : r < |z − c| < R} ein
Kreisring um c, welcher zusammen mit seinem Rand ∂A Aufgabe 5.8 ••• Potenzreihenentwicklung.
vollständig im offenen Definitionsgebiet U einer holomor-
phen Funktion enthalten ist. Sei Aufgabe 5.9 •• Cauchy-Riemann’sche Differenzial-
S = {z : z ∈ A , ϕ < arg(z − c) < ψ} gleichungen.

der Kreissektor zum Winkelbereich (ϕ, ψ). Aufgabe 5.10 • Definition des Wegintegrals.
Lösungen zu Kapitel 5 35

Aufgabe 5.11 • Partialbruchzerlegung und geometri- Aufgabe 5.29 •• (a) Integralsatz von Cauchy. (b) Zer-
sche Reihe. legung des Sektors in kleine Sektoren und Zusammensetzen
der Integrale.
Aufgabe 5.12 •• Partialbruchzerlegung und geometri-
sche Reihe. Aufgabe 5.30 •• (a) Betrachten Sie die Ableitung von
G. (b) Nutzen Sie aus, dass G(b) = G(a).
Aufgabe 5.13 • Untersuchung der Laurentreihe von f
in c.

Aufgabe 5.14 •• Untersuchung der Laurentreihe Lösungen


von f .
Verständnisfragen
Aufgabe 5.15 •• Anwendung der Formeln, die sich
durch Ergänzung des reellen Integrationsintervalles durch Aufgabe 5.1 • In keinem Punkt.
einen Halbkreis ergeben.
Aufgabe 5.2 •• Nein.

Beweisaufgaben
Rechenaufgaben
Aufgabe 5.16 • Erweitern Sie mit 1 − z.
Aufgabe 5.3 • a) Für reelle oder rein imaginäre z.
Aufgabe 5.17 • Euler’sche Formel. b) Für reelle z.

Aufgabe 5.18 •• Der Kreis K liegt in einer Ebene im Aufgabe 5.4 • f (K) ist die senkrechte Gerade gege-
R3 , also gilt a, k = α für alle k auf K mit geeignetem ben durch Re w = 1/2.
a ∈ R3 und α ∈ R. Gewinnen Sie daraus eine Gleichung
für p(K) und zeigen Sie, dass diese eine Gerade oder einen Aufgabe 5.5 • f (H) = C− = C \ {z : z ≤ 0}, die
Kreis darstellt. längs der negativen reellen Achse aufgeschnittene Ebene. f
ist auf H injektiv.
Aufgabe 5.19 •• Verallgemeinerte Integralformel von
Cauchy. Aufgabe 5.6 •• f (Cx ) = C, f (∂ E) = [−1, 1] und
f (Ex ) = C \ [−1, 1]. Auf Ex ist f bijektiv.
Aufgabe 5.20 •• Partielle Integration über [0, R] und
Grenzübergang R → ∞. Aufgabe 5.7 •••

Aufgabe 5.21 •• (a) Kontraposition (= indirekter Be- Aufgabe 5.8 ••• Die einzige solche Funktion ist die
weis) mit dem Satz von Liouville, angewendet auf 1/p. Konstante f = 1.
(b) Entwicklung von p um die Nullstelle nach (a) und Induk-
tion. Aufgabe 5.9 •• Es muss b = −1 sein, a ∈ R ist belie-
big. Die gesuchten Funktionen haben die Form
Aufgabe 5.22 •• Satz von Liouville, angewendet auf
h = exp ◦f . f (x, y) = x 2 + 2axy − y 2 + i(2xy + ay 2 − ax 2 + d)

mit einem weiteren freien Parameter d ∈ R.


Aufgabe 5.23 •• Identitätssatz.

Aufgabe 5.10 • 1 + 13 i.
Aufgabe 5.24 •• Offenheit und Kriterium für lokale
Biholomorphie.
Aufgabe 5.11 •
Aufgabe 5.25 • Definition einer Polstelle. 1
f (z) = − − 1 − z − z2 − . . .
z
Aufgabe 5.26 •• Satz von Casorati-Weierstraß.
Aufgabe 5.12 ••
Aufgabe 5.27 •• Betrachten Sie die Entwicklung von 1−i 1+i i k

f nahe c. f (z) = + ( ) (z − i)k .
z−i 2i 2
k=0
Aufgabe 5.28 •• Verwenden Sie das Ergebnis der vor-
angehenden Aufgabe. Aufgabe 5.13 • (a) Res (f, 0) = 3. (b) Res (f, 1) = −1/4.
36 Lösungswege zu Kapitel 5

Aufgabe 5.14 •• (a) Res (f, 0) = 1. Variante 3: Für u = Re f und v = Im f gelten u(x, y) = x,
(b) Die Singularitätenmenge von f besteht aus den Punkten v(x, y) = −y, also ∂x u = 1 = −1 = ∂y v. Die
ck = πi+2kπi für k ∈ Z. Es gilt Res (f, ck ) = −1−ck2 )/eck Cauchy-Riemann’schen Differenzialgleichungen sind in kei-
für alle k ∈ Z. nem Punkt erfüllt.

Aufgabe 5.15 •• Das Integral hat den Wert 1/3. Aufgabe 5.2 •• Da E beschränkt ist, ist jedes holomor-
phe f : C → E konstant.
Beweisaufgaben
Aufgabe 5.16 • – Rechenaufgaben
Aufgabe 5.3 • a) Variante 1 (kartesische Koordina-
Aufgabe 5.17 • –
ten): Ist z = x + iy, so ist z2 = x 2 − y 2 + i(xy + yx).
Aufgabe 5.18 •• – Also ist Im z = 0 genau dann, wenn x = 0 oder y = 0.
a) Variante 2 (Polarkoordinaten): Ist z = reiϕ , so ist
Aufgabe 5.19 •• – z2 = r 2 e2iϕ und damit z ∈ R genau dann, wenn ϕ = kπ/2
für ein k ∈ Z.
Aufgabe 5.20 •• –
b) Variante 1: Die Zusatzforderung 0 ≤ z2 = x 2 −y 2 schließt
Aufgabe 5.21 •• – aus, dass y = 0.
b) Variante 2: Es ist z2 ≥ 0 genau dann, wenn ϕ = kπ für
Aufgabe 5.22 •• –
ein k ∈ Z.
Aufgabe 5.23 •• –
Aufgabe 5.4 • Für w ∈ Cx gilt w ∈ f (K) genau dann,
Aufgabe 5.24 •• – wenn 1/w ∈ K gilt, was gleichbedeutend ist mit
1 2  1  1 
Aufgabe 5.25 • – 1 =  − 1 = −1 · −1
w w w
1 − (w + w)
Aufgabe 5.26 •• – =1+ .
ww
Aufgabe 5.27 •• – Dieses wiederum gilt genau dann, wenn 1 = w + w =
2 Re w.
Aufgabe 5.28 •• –
Aufgabe 5.5 • Für gegebenes w = reiϕ hat die Glei-
Aufgabe 5.29 •• –
chung z2 = −w = rei(ϕ+π) die beiden Lösungen z0 =
√ i(ϕ+π)/2
re und z1 = −z0 . Es gilt z0 ∈ H genau dann,
Aufgabe 5.30 •• –
wenn r = 0 und (ϕ + π )/2 ∈ (0, π ), was gleichbedeutend
ist mit r = 0 und ϕ ∈ (−π, π ), und damit mit w ∈ C− . Für
ϕ = −π gilt z0 ∈ R. Es folgt z1 = −z0 ∈ / H, also ist f auf
Lösungswege H injektiv, und f (H) = C− .

Aufgabe 5.6 •• Multiplikation von w = f (z) mit 2z


Verständnisfragen zeigt, dass w = f (z) genau dann gilt, wenn z2 −2wz+1 = 0.
Aufgabe 5.1 • Variante 1: Der Differenzenquotient Diese Gleichung hat für w = ±1 die eindeutige Lösung
z = w und für alle anderen w ∈ C genau zwei Lösungen
f (w) − f (z) w−z w−z z1 , z2 mit z1 ·z2 = 1, es ist also f (Cx ) = C. Für z = eiϕ ∈ ∂ E
= =
w−z w−z w−z gilt
1 
ist gleich 1 für reelles w−z und gleich −1 für rein imaginäres f (z) = eiϕ + e−iϕ = cos ϕ ,
w − z, sein Limes für w → z existiert also nicht. 2
also ist f (∂ E) = [−1, 1]. Wegen |z1 | · |z2 | = 1 gilt für jedes
Variante 2: Aufgefasst als f : R2 → R2 ist f linear mit der w∈ / [−1, 1], dass |z1 | = 1 = |z2 | und dass entweder z1 oder
darstellenden Matrix z2 in E liegt. Also ist
 
1 0
, f : Ex → f (Ex ) = C \ [−1, 1]
0 −1

welche keine Drehstreckung ist. bijektiv, siehe Abbildung 5.1.


Lösungswege zu Kapitel 5 37

Im Im Aufgabe 5.9 •• Ist u = Re f für ein holomorphes f ,


so muss überall 0 = u = ∂x ∂x u + ∂y ∂y u = 2 + 2b gelten,
∂Ex f (∂Ex ) also b = −1. Wir bestimmen nun v : R2 → R so, dass die
Cauchy-Riemann’schen Differenzialgleichungen gelten, also

∂y v = ∂x u = 2x + 2ay , ∂x v = −∂y u = −2ax + 2y .


−1 Ex 1 Re −1 1 Re
Die erste Gleichung liefert v(x, y) = 2xy + ay 2 + c(x) mit
einer zu bestimmenden Funktion c. Die zweite Gleichung
führt auf 2y + c (x) = −2ax + 2y, also c(x) = −ax 2 + d
Abbildung 5.1 Bild von Ex unter f . mit einer Konstanten d. Wir erhalten also

f (x, y) = x 2 + 2axy − y 2 + i(2xy + ay 2 − ax 2 + d)


Aufgabe 5.7 ••• Nach Definition von d gilt
mit frei wählbaren Konstanten a, d ∈ R als allgemeine Form
d(z, z )2 = |w − w  |2 + (t − t  )2 . von f .
Die erste Identität im Hinweis ergibt sich nun aus
Aufgabe 5.10 • Wir definieren γ : [0, 1] → C durch
 2  2
|w − w | + (t − t ) = (w − w )(w 
− w  ) + (t  2
−t ) γ (t) = t + t 2 i. Es folgen γ  (t) = 1 + 2ti und
 2 1 1
= ww + w w 2
+t +t − ww  − w w − 2tt 


z dz = γ (t)γ (t) dt = (t − t 2 i)(1 + 2ti) dt
= 2 − ww  − w  w − 2tt  . γ 0 0
1
1 1 t=1
Die zweite Identität ergibt sich durch Einsetzen aus (5.4) und = t + 2t 3 + t 2 i dt = (t 2 + t 4 ) + t 3 i
0 2 3 t=0
aus
t |z|2 − 1 t |z |2 − 1 1
= , = , = 1+ i.
1−t 2 1−t  2 3
nach Vereinfachung der resultierenden Ausdrücke. Die dritte
Identität folgt unmittelbar aus Aufgabe 5.11 • Partialbruchzerlegung von f ergibt
1 1
ww  + w  w f (z) = − .
= zz + z z z−1 z
(1 − t)(1 − t  )
Im Entwicklungspunkt c = 0 hat f einen einfachen Pol. Es
gemäß (5.3). Subtrahieren wir nun die dritte von der zweiten ist
Identität, so folgt mit Verwendung der ersten Identität 1 1
=− = −(1 + z + z2 + . . . ) ,
z−1 1−z
|w − w  |2 + (t − t  )2
= |z − z |2 für z → 1/z ist nichts zu tun. Addition ergibt
(1 − t)(1 − t  )
1
und hieraus die behauptete Formel. f (z) = − − 1 − z − z2 − . . .
z
Aufgabe 5.8 ••• Als holomorphe  Funktion ist f in eine
Potenzreihe entwickelbar, f (z) = ∞ k=0 ak z k . Da f gerade Aufgabe 5.12 •• Die Faktorisierung des Nenners in
ist, gilt ak = 0 für alle ungeraden k ∈ N. Aus f (0) = 1 folgt z2 + 1 = (z − i)(z + i) ergibt zwei einfache Pole in z = ±i,
a0 = 1. Die Funktionalgleichung f (z2 ) = f (z)2 wird zu der Pol −i liegt auf dem Rand der genannten Kreisscheibe
um i. Der Ansatz

∞ 
k 
ak z2k = aj ak−j zk . 2z + 2 A B
f (z) = = +
k=0 k=0 j =0 z +1
2 z−i z+i
 führt auf A(z + i) + B(z − i) = 2z + 2, also A + B = 2 und
Koeffizientenvergleich führt auf ak = 2k
j =0 aj a2k−j , was A − B = 2/i = −2i und damit
wegen a0 = 1 umgeschrieben werden kann zu
A = 1−i, B = 1+i.

2k−1
2a2k = ak − aj a2k−j . Der erste Term (1 − i)/(z − i) hat bereits die gesuchte Form.
j =1 Im zweiten Term setzen wir w = z − i und entwickeln im
Für k = 1 ergibt sich a2 = 0 aus a1 = 0. Die Rekursionsfor- Bereich |w| < 2
mel zeigt weiterhin, dass a2 = · · · = a2k−2 = 0 impliziert, 1+i 1+i 1+i 1 1+i w k

dass a2k = 0. Also sind alle Koeffizienten bis auf a0 gleich = = w = (− )
z+i w + 2i 2i 1 − (− 2i ) 2i 2i
null. k=0
38 Lösungswege zu Kapitel 5

und erhalten mit −1/(2i) = i/2 Es gilt


1 c
∞ Res (f, c) = =−
1+i 1+i i k 6c 5 6
= ( ) (z − i)k .
z+i 2i 2 gemäß (5.142) und da c6 = −1. Es folgt
k=0

Die gesuchte Laurententwicklung von f ist daher 1 1 ∞ 1
dz = dz
∞ 0 1 + z6 2 −∞ 1 + z6
1−i 1+i i k 1 π i  πi/6  π
f (z) = + ( ) (z − i)k . =− · e + eπi/2 + e5π i/6 = .
z−i 2i 2 2 3 3
k=0

Aufgabe 5.13 • Wir berechnen den Koeffizienten a−1


Beweisaufgaben
in der Laurentreihe von f .
(a) Es ist Aufgabe 5.16 • Es ist wegen zz = |z|2 = 1
5 4 3 (1 + z)(1 − z) 1 + z − z − zz
f (z) = − 2 + , also Res (f, 0) = 3 . f (z) = ·i= ·i
z3 z z (1 − z)(1 − z) (1 − z)(1 − z)
z−z 2 Im z
(b) Wir setzen w = z − 1. Division durch w5 verschiebt die = ·i=− .
|1 − z|2 |1 − z|2
Laurentkoeffizienten von

w2 w3 w4 Aufgabe 5.17 • Die Darstellung


log(1 + w) = w − + − + ···
2 3 4
1 iz
um 5 Stellen, also gilt Res (f, 1) = −1/4. cos z = (e + e−iz )
2
Aufgabe 5.14 •• (a) Es ist folgt aus der Euler’schen Formel. Einsetzen von z = 1/(ix)
ergibt
z
lim zf (z) = lim ez = e0 · 1 = 1 ,
z→0 z→0 sin z 1 1/x
f (ix) = (e + e−1/x ) → ∞ für x → 0 in R.
2
also ist 0 ein einfacher Pol von f und Res (f, 0) =
limz→0 zf (z) = 1.
(b) Nullsetzen von h(z) = ez + 1 = 0 des Nenners liefert Aufgabe 5.18 •• Wir rechnen in kartesischen Koordi-
die Nullstellen ck = πi + 2kπ i für k ∈ Z. Diese sind ein- naten. Es liege k auf K, sei k = p−1 (z) mit z = (x, y). Da
fach, da h (z) = ez = 0 gilt für alle z ∈ C. Da der Zähler K in einer Ebene liegt, gilt mit der im Hinweis angegebenen
g(z) = 1 + z2 holomorph und in C von null verschieden ist, Form der Ebenengleichung
sind alle ck einfache Polstellen von f . Aus (5.142) folgt nun
a, p−1 (z) = α . (5.1)
g(ck ) 1 + ck2
Res (f, ck ) =  = = −1 − ck2 . Gemäß (5.6) gilt
h (ck ) e ck
1  
p−1 (z) = 2z , |z|2 − 1 ,
Aufgabe 5.15 •• Es ist |z|2 +1
∞ ∞
1 1 1 also in kartesischen Koordinaten
dz = dz
0 1 + z6 2 −∞ 1 + z6 1  
p−1 (z) = 2x , 2y , |z|2 − 1 .
wegen Symmetrie. Der Nennergrad ist um 5 größer als der |z|2 + 1
Zählergrad, wir können also die Formel (5.148) anwenden,
Einsetzen in (5.1) mit a = (a1 , a2 , a3 ) führt auf

f (z) dz = 2πi Res (f, c) , 2xa1 + 2ya2 + a3 (|z|2 − 1) = α(|z|2 + 1)
−∞
c∈S +
oder äquivalent
wobei S + die in der oberen Halbebene liegenden Singulari-
täten von f enthält. Die Nullstellen von z  → z6 − 1 sind die (a3 − α)(x 2 + y 2 ) + 2a1 x + 2a2 y = a3 + α . (5.2)
Zahlen c = (1/6)πi + (k/3)πi, 0 ≤ k < 5, es folgt
 Es gilt a, (0, 0, 1) = α genau dann, wenn a3 = α. In
π π 5π
S+ = i, i, i . diesem Fall beschreibt (5.2) in der xy-Ebene eine Gerade,
6 2 6 im Fall a3 = α einen Kreis.
Lösungswege zu Kapitel 5 39

Aufgabe 5.19 •• Als Konsequenz der verallgemeiner- einem Polynom vom Grad n − 1. Nach Induktionsvorausset-
ten Integralformel von Cauchy haben wir die Abschätzung zung lässt sich q schreiben als q(z) = a(z−c1 ) · · · (z−cn−1 ).
Hieraus erhalten wir die Behauptung, indem wir cn = c set-
Mr zen.
|ak | ≤ , Mr = max |f (z)|
rk |z|=r
Aufgabe 5.22 •• Sei h(z) = ef (z) , es gelte |Re f (z)|
für die Koeffizienten ak der Potenzreihenentwicklung
≤ M für alle z ∈ C. Dann gilt
von f im Nullpunkt erhalten. Gemäß Voraussetzung gilt
Mr ≤ Cr m . Es folgt |h(z)| = |eRe f (z)+iIm f (z) | = eRe f (z) ≤ eM ,
|ak | ≤ Cr m−k .
also ist h beschränkt und nach dem Satz von Liouville kon-
stant. Der Wertebereich f (C) von f ist daher in der diskreten
Grenzübergang r → ∞ zeigt, dass ak = 0 gilt für k > m.
Menge {f (0) + 2π ki : k ∈ Z} enthalten. Wäre f nicht kon-
Also ist f ein Polynom, dessen Grad nicht größer als m sein
stant, so wäre f (C) ein Gebiet, ein Widerspruch.
kann.
Aufgabe 5.23 •• Hat f den kompakten Träger K ⊂ G,
Aufgabe 5.20 •• Es gilt
so ist G \ K nichtleer und offen, und es gilt f = 0 auf G \ K.
R t=R R Nach dem Identitätssatz muss f = 0 auf ganz G gelten.

e−st f  (t) dt = e−st f (t) − −se−st f (t) dt
0 t=0 0
R Aufgabe 5.24 •• Da f auf dem Gebiet G nichtkonstant
−sR −st
=e f (R) − f (0) + s e f (t) dt . (5.3) ist, ist f (G) offen (und sogar, was wir nicht benötigen, ein
0 Gebiet). Zu zeigen ist, dass f −1 auf f (G) holomorph ist.
Gemäß Kriterium über lokale Biholomorphie genügt es zu
Aus der Wachstumsbedingung folgt, dass |e−st f (t)| ≤
zeigen, dass f  (z) = 0 gilt für alle z ∈ G. Wie in dessen
Ce−(Re s−α)t und damit für Re s > α
Beweis können wir aus der Injektivität von f schließen, dass

−sR jedes z ∈ G eine endliche f (z)-Stelle ist und dass deren
lim e f (R) = 0 = lim −se−st f (t) dt . Ordnung gleich 1 sein muss. Somit folgt f  (z) = 0 für alle
R→∞ R→∞ R
z ∈ G.
Grenzübergang in (5.3) ergibt
∞ ∞ Aufgabe 5.25 • Wir betrachten die Funktion
e−st f  (t) dt = −f (0) + s e−st f (t) dt ,
0 0 h(z) = (z − c)m (f + g)(z) = (z − c)m f (z)+(z − c)m g(z)
das ist die Behauptung. =: f1 (z) + g1 (z) .

Aufgabe 5.21 •• (a) Hat p keine Nullstelle, so ist f = Nach Voraussetzung an f ist f1 holomorph fortsetzbar auf
1/p auf C holomorph. Da p(z) → ∞ gilt für z → ∞, U ∪ {c} mit f1 (c) = 0. Dasselbe gilt auch für h, da g1
gibt es C, R > 0 mit |f (z)| ≤ C für alle |z| ≥ R. Da holomorph ist auf U ∪ {c} und g1 (c) = 0 gilt. Somit ist c ein
außerdem f auf dem Kompaktum {|z| ≤ R} beschränkt ist, m-facher Pol von f + g gemäß Definition.
ist f auf C beschränkt und nach dem Satz von Liouville
konstant. Folglich ist auch p konstant. Aufgabe 5.26 •• Nach dem Satz von Casorati-
(b) Induktion über n. Für n = 1 ist p(z) = az + b mit Weierstraß liegt f (B x ) dicht in C für jede hinreichend
a = 0, also p(z) = a(z − c1 ) mit c1 = −b/a. Für den kleine punktierte Kreisscheibe B x um c. Da z → 1/z die
Induktionsschritt von n − 1 nach n sei c eine Nullstelle von punktierte Ebene Cx biholomorph auf sich abbildet, liegt
p gemäß (a), sei auch (1/f )(B x ) dicht in C für jedes solche B x . Der Punkt
c kann daher weder eine hebbare Singularität noch ein Pol

n von 1/f sein.
p(z) = ak (z − c)k .
k=0 Aufgabe 5.27 •• Ist c ein m-facher Pol von f , so lässt
Wegen p(c) = 0 gilt a0 = 0 und daher sich f in einer hinreichend kleinen Kreisscheibe B x um c
darstellen in der Form

n
∞ −1
p(z) = ak (z − c)k = (z − c)q(z)
f (z) = r(z) + ak (z − c)k , r(z) = ak (z − c)k ,
k=1
k=0 k=−m
mit

n−1 wie wir in (5.103) gesehen haben. Folglich hat f − r in c
q(z) = ak+1 (z − c)k , eine hebbare Singularität. Da r holomorph ist in C \ {c}, ist
k=0 jeder von c verschiedene Pol von f auch ein Pol von f − r.
40 Lösungswege zu Kapitel 5

Aufgabe 5.28 •• Sei P (f ) = {c1 , . . . , cN } die Menge da sich die Wegintegrale über die radialen Teile der Ränder
der Pole von f . Zu jedem cj wählen wir gemäß der vorange- gegenseitig wegheben, siehe Bild 5.3.
henden Aufgabe eine rationale Funktion rj mit P (f − rj ) =

P (f ) \ {cj } und P (rj ) = {cj }. Wir setzen r = N
j =1 rj . An
der Darstellung
S2
f − r = (f − rj ) − rk , 1 ≤ j ≤ N ,
k =j
S1
erkennen wir, dass jedes cj eine hebbare Singularität von S3
f − r ist. Ist g die holomorphe Fortsetzung von f − r auf U , γR γr
so liefert f = r + g die erwünschte Zerlegung.
c
Aufgabe 5.29 •• (a) Ist ψ − ϕ hinreichend klein, so ist
S ein Sterngebiet mit dem Mittelpunkt

R + r i(ψ+ϕ)/2
S4 S5
p=c+ e
2
von S als Bezugspunkt, siehe Bild 5.2.

Abbildung 5.3 Zerlegung des Kreisrings in Sektoren.

Setzen wir ϕ = −π , ψ = π, so folgt weiter, dass



f (z) dz = f (z) dz = 0 .
∂A ∂S
S
Der äußere Teil γR und der innere Teil γr des Randes von
A werden dabei in entgegengesetzter Richtung durchlaufen.
Damit ergibt sich
c p
f (z) dz = f (z) dz ,
γr γR

was zu beweisen war.

Aufgabe 5.30 •• (a) Wir differenzieren G und erhalten

G (t) = γ  (t) e−g(t) − (γ (t) − c)g  (t) e−g(t) = 0 ,


Abbildung 5.2 Ein Sektor als Sterngebiet. da g  (t) = γ  (t)/(γ (t) − c) gilt. Also ist G konstant.
(b) Es gilt
Die Behauptung folgt nun aus dem Integralsatz von Cauchy.
(b) Ein beliebiger Sektor S lässt sich aus Sektoren Sj , G(a) γ (a) − c g(b)−g(a)
1= = e
1 ≤ j ≤ n mit hinreichend kleinen Winkelbereichen gemäß G(b) γ (b) − c
! "  
(a) zusammensetzen. Es folgt b γ  (t)) 1
= exp dz = exp dz ,
n
a γ (t) − c γ z−c
f (z) dz = f (z) dz = 0 ,
∂S j =1 ∂Sj was zu zeigen war.
Kapitel 6 gilt, wobei dxi , i = 1, . . . , n, die elementaren 1-Formen
sind.

Aufgabe 6.8 •• Sei U ⊆ Rn offen. Für i = 1, . . . , k sei


Aufgaben ωi ∈ pi (U ) eine pi -Differenzialform, wobei ki=1 pi < n
gilt. Bestimmen Sie eine Produktformel für die äußere Ab-
Rechenaufgaben leitung
d(ω1 ∧ . . . ∧ ωk ).
Aufgabe 6.1 • Seien x, y, z ∈ R4 . Prüfen Sie, welche
der folgenden Abbildungen Tensoren auf dem R4 sind. Bei Aufgabe 6.9 •• Sei U ⊆ Rn offen und seien f, g ∈
welchen handelt es sich um alternierende Tensoren? 0 (U ) stetig differenzierbar mit g = ϕ◦f , wobei ϕ : R → R
1. d(x, y) = x1 y3 − x3 y1 stetig differenzierbar ist. Zeigen Sie, dass unter diesen Vor-
2. f (x, y, z) = 2x1 y2 z2 − x2 y3 z1 aussetzungen df ∧ dg = 0 gilt.
3. g = (2,1) − 5(3,1)
4. h(x, y) = (x1 )3 (y2 )3 − (x2 )3 (y1 )3 Aufgabe 6.10 •• Sei ω ∈ 2 (R3 ) durch
ω = xydx ∧ dy + 2xdy ∧ dz + 2ydx ∧ dz
Aufgabe 6.2 • Seien f und g wie in Aufgabe 6.1 ge-
geben. gegeben. Zeigen Sie, dass das Integral von ω auf der oberen
Einheitshalbsphäre
1. Drücken Sie f ⊗ g mithilfe elementarer Tensoren aus.
{(x, y, z) ∈ R3 | x 2 + y 2 + z2 = 1, z ≥ 0}
2. Schreiben Sie f ⊗ g als Funktion von x, y, z, u, v ∈ R4 .
verschwindet.
Aufgabe 6.3 • Sei σ ∈ S5 durch σ : (1, 2, 3, 4, 5) →
(3, 1, 4, 5, 2) definiert. Schreiben Sie σ als Komposition ele- Aufgabe 6.11 •• Zeigen Sie, dass zu jeder orientierten
mentarer Permutationen. d-Mannigfaltigkeit in Rn eine Volumenform dM = ωv
existiert, die jeder Orthonormalbasis (ONB) des Tangenti-
Aufgabe 6.4 •• Sei V ein R-Vektorraum mit Basis alraums Tp (M), p ∈ M, mit natürlicher Orientierung den
{a1 , . . . , an }. Seien J = (j1 , . . . , jd ) ein beliebiges d-Tupel Wert 1 zuordnet, d. h.

und I = (i1 , . . . , id ) ein aufsteigendes d-Tupel mit Ein-
trägen aus {1, . . . , n}, d ≤ n. Weiterhin sei I der zu I ωv = 1 dμ = vol(M).
M M
korrespondierende elementare alternierende d-Tensor bezüg-
lich der Basis {a1 , . . . , an }. Bestimmen Sie den Wert von Aufgabe 6.12 •• Sei M eine orientierte d-
I (aj1 , . . . , ajd ). Mannigfaltigkeit in Rn , die durch eine einzige Karte α
überdeckt werden kann. Zeigen Sie, dass für die Volumen-
Aufgabe 6.5 • Sei α : R3 → R6 eine C ∞ -Abbildung, form ωv (siehe Aufgabe 6.11) dann
d. h. ⎛ ⎞ ⎛ ⎞ 
⎛ ⎞ α1 (x) y1 α ∗ ωv = det(G)dx1 ∧ . . . ∧ dxd
x1 ⎜α2 (x)⎟ ⎜y2 ⎟
⎜ ⎟ ⎜ ⎟
x = ⎝x2 ⎠ → ⎜ . ⎟ = ⎜ . ⎟ = y. gilt, wobei G = (gij )i,j =1,...,d ∈ Rd×d mit
⎝ .. ⎠ ⎝ .. ⎠
x3 ∂α ∂α
α6 (x) y6 gij = ·
∂xi ∂xj
Bestimmen Sie dα1 ∧dα3 ∧dα5 ∈ 3 (R3 ) einmal direkt und
ist.
einmal unter Verwendung des Satzes (Duale Transformation
elementarer Formen), siehe Seite 178. Aufgabe 6.13 •• Sei M eine orientierte und geschlos-
sene d + l + 1-Mannigfaltigkeit in Rn , d. h. kompakt und
Aufgabe 6.6 • Sei A = (0, 1)2 ⊂ R2 und α : A → R3
ohne Rand, und seien ω ∈ d (M) und η ∈ l (M). Zeigen
durch
(u, v) → (u, v, u2 + v 2 + 1) Sie, dass dann

definiert. Bestimmen Sie dω ∧ η = a ω ∧ dη

M M
x2 dx2 ∧ dx3 + x1 x3 dx1 ∧ dx3 ,
M für ein gewisses a ∈ R gilt.
wobei M = α(A) ist.
Aufgabe 6.14 •• Sei M eine kompakte, natürlich orien-
tierte 2-Mannigfaltigkeit in R2 und sei auf ∂M die induzierte
Beweisaufgaben Orientierung gegeben. Beweisen Sie die Green’sche Formel
 
Aufgabe 6.7 •• Auf Rn \{0} sei die 0-Form r(x) := ∂Q ∂P
P dx + Qdy = − dx ∧ dy
x2 definiert. Zeigen Sie, dass ∂M M ∂x ∂y
dr ∧ ∗(dr) = dx1 ∧ . . . ∧ dxn für eine 1-Form P dx + Qdy ∈ 1 (M)
M. Brokate et al., Arbeitsbuch Grundwissen Mathematikstudium – Höhere Analysis, Numerik und
Stochastik, DOI 10.1007/978-3-642-54946-5_5, © Springer-Verlag Berlin Heidelberg 2016
42 Lösungen zu Kapitel 6

Hinweise Lösungen

Rechenaufgaben Rechenaufgaben
Aufgabe 6.1 • Prüfen Sie die definierenden Eigen- Aufgabe 6.1 • d ∈ A2 (R4 ), f ∈ L3 (R4 ), g ∈
schaften. 2 4
L (R ), h ist kein Tensor

Aufgabe 6.2 • –
Aufgabe 6.2 •
Aufgabe 6.3 • – f ⊗ g = 2(1,2,2,2,1) − (2,3,1,2,1) − 10(1,2,2,3,1)
+(2,3,1,3,1)
Aufgabe 6.4 •• Nutzen Sie die Eigenschaft f σ =
sgn(σ )f für alternierende Tensoren (siehe Seite 165).
f ⊗ g(x, y, z, u, v) = 2x1 y2 z2 u2 v1 − x2 y3 z1 u2 v1
Aufgabe 6.5 • –
−10x1 y2 z2 u3 v1 + 5x2 y3 z1 u3 v1 .
Aufgabe 6.6 • –

Aufgabe 6.3 • σ = e4 ◦ e3 ◦ e1 ◦ e2
Beweisaufgaben
Aufgabe 6.7 •• – Aufgabe 6.4 ••
#
(−1)l+1 , |I ∩ J | = d
Aufgabe 6.8 •• Betrachten Sie I (aj1 , . . . , ajd ) =
0 sonst.
 

k j −1
i=1 pi
(−1) ω1 ∧ . . . ∧ dωj ∧ . . . ∧ ωk ,
j =1 Aufgabe 6.5 •
 dα1 ∧ dα3 ∧ dα5
wobei 0i=1 pi := 0, und zeigen Sie, dass dies die gesuchte  
Produktformel ist. ∂(α1 , α3 , α5 )
= det dx1 ∧ dx2 ∧ dx3
∂(x1 , x2 , x3 )
Aufgabe 6.9 •• Zeigen Sie df ∧ dg = dg ∧ df und
argumentieren Sie anschließend mit der Antikommutativität 1
des Dachprodukts. Aufgabe 6.6 • M ω= 2

Aufgabe 6.10 •• Die obere Einheitshalbsphäre lässt


sich durch
# Beweisaufgaben
[0, π2 ) × [0, 2π] → R3 ,
α: Aufgabe 6.7 •• –
(ϑ, ϕ) → (cos ϕ sin ϑ, sin ϕ sin ϑ, cos ϑ)

parametrisieren. Aufgabe 6.8 ••


 

k j −1
Aufgabe 6.11 •• Betrachten Sie für (p; v1 ), . . . , (−1) i=1 pi
ω1 ∧ . . . ∧ dωj ∧ . . . ∧ ωk
(p; vd ) ∈ Tp (M) die Form j =1
⎡ ⎤
v1 · e1 . . . v1 · ed
⎢ .. ⎥ , Aufgabe 6.9 •• –
ωv (p)((p; v1 ), . . . , (p; vd )) := det⎣ ... . ⎦
vd · e1 . . . vd · ed
Aufgabe 6.10 •• –
wobei {(p; e1 ), . . . , (p; ed )} eine beliebige ONB des Tan-
gentialraums Tp (M) ist. Aufgabe 6.11 •• –

Aufgabe 6.12 •• Orientieren Sie sich am Beweis des


Aufgabe 6.12 •• –
Satzes auf Seite 196.

Aufgabe 6.13 •• Benutzen Sie den Satz von Stokes. Aufgabe 6.13 •• –

Aufgabe 6.14 •• Benutzen Sie den Satz von Stokes. Aufgabe 6.14 •• –
Lösungswege zu Kapitel 6 43

Lösungswege Aufgabe 6.2 • 1. Es ist f = 2(1,2,2) − (2,3,1) und


daher gilt
Rechenaufgaben
f ⊗ g = (2(1,2,2) − (2,3,1) ) ⊗ ((2,1) − 5(3,1) )
Aufgabe 6.1 • Seien x, y, z, w ∈ R4 und α ∈ R. = 2(1,2,2) ⊗ (2,1) − (2,3,1) ⊗ (2,1)
1. Es gilt
−10(1,2,2) ⊗ (3,1) + 5(2,3,1) ⊗ (3,1)
d(αx + y, z) = (αx1 + y1 )z3 − (αx3 + y3 )z1 = 2(1,2,2,2,1) − (2,3,1,2,1) − 10(1,2,2,3,1)
= α(x1 z3 − x3 z1 ) + y1 z3 − y3 z1 +(2,3,1,3,1) .
= αd(x, z) + d(y, z),
also ist d linear im ersten Argument. Die Linearität im 2. Mit Teil 1. der Aufgabe folgt
zweiten Argument zeigt man analog. Insgesamt folgt somit
d ∈ L2 (R4 ). Bleibt noch zu prüfen, ob d auch alternierend f ⊗ g(x, y, z, u, v) = 2x1 y2 z2 u2 v1 − x2 y3 z1 u2 v1
ist. Es gilt −10x1 y2 z2 u3 v1 + 5x2 y3 z1 u3 v1 .
d(y, x) = y1 x3 − y3 x1 = −(x1 y3 − x3 y1 ) = −d(x, y),
also ist d alternierend und somit d ∈ A2 (R4 ).
Aufgabe 6.3 • Sukzessives Permutieren liefert
2. Es gilt e2 e1
(1, 2, 3, 4, 5) −
→ (1, 3, 2, 4, 5) −
→ (3, 1, 2, 4, 5)
f (αx + y, z, w) e3 e4

→ (3, 1, 4, 2, 5) −
→ (3, 1, 4, 5, 2)
= 2(αx1 + y1 )z2 w2 − (αx2 + y2 )z3 w1
= α(2x1 z2 w2 − x2 z3 w1 ) + 2y1 z2 w2 − y2 z3 w1 und somit gilt σ = e4 ◦ e3 ◦ e1 ◦ e2 .
= αf (x, z, w) + f (y, z, w),
also ist f linear im ersten Argument. Die Linearität im zwei- Aufgabe 6.4 •• Wir unterscheiden zwei Fälle:
ten und dritten Argument zeigt man analog. Insgesamt folgt (i) Sei |{i1 , . . . , id } ∩ {j1 , . . . , jd }| < d, dann hat J min-
somit f ∈ L3 (R4 ). Weiterhin gilt destens einen Eintrag, der nicht in I enthalten ist, und somit
f (y, x, z) = 2y1 x2 z2 − y2 x3 z1 gilt per definitionem
 = −2x1 y2 z2 + x2 y3 z1
I (aj1 , . . . , ajd ) = 0.
= −f (x, y, z),
d. h., f ist nicht alternierend und daher f ∈
/ A3 (R4 ). (ii) Sei |{i1 , . . . , id } ∩ {j1 , . . . , jd }| = d, dann gibt es eine
Permutation σ ∈ Sd mit σ (I ) = J . Schreibt man σ als
3. g ist als Linearkombination elementarer 2-Tensoren of-
Komposition elementarer Permutationen, also σ = eν ◦ . . . ◦
fensichtlich selbst ein 2-Tensor, d. h. g ∈ L2 (R4 ). Es gilt
eν+l , dann folgt
jedoch
g(y, x) = (2,1) (y, x) − 5(3,1) (y, x) I (aj1 , . . . , ajd ) = I (aσ (i1 ) , . . . , aσ (id ) )
= y2 x1 − 5y3 x1 = Iσ (ai1 , . . . , aid )
e ◦...◦eν+l
= −x2 y1 + 5x3 y1 = I ν (ai1 , . . . , aid )
= −((2,1) (x, y) − 5(3,1) (x, y) = (−1) l+1
I (ai1 , . . . , aid )
= −g(x, y), = (−1) l+1
.
also ist g nicht alternierend und daher g ∈
/ A2 (R4 ).
Insgesamt ergibt sich also
4. Es gilt #
(−1)l+1 , |I ∩ J | = d
h(αx + y, z) I (aj1 , . . . , ajd ) =
0 sonst.
= (αx1 + y1 )3 (z2 )3 − (αx2 + y2 )3 (z1 )3
 
= α (x1 )3 (z2 )3 − (x2 )3 (z1 )3
  Aufgabe 6.5 • Direkte Berechnung:
+ (y1 )3 (z2 )3 − (y2 )3 (z1 )3
Es gilt
= αh(x, z) + h(y, z),

3
also ist h nicht linear und daher auch kein Tensor, d. h. h ∈
/ dαi = Dj αi dxj ,
L2 (R4 ). j =1
44 Lösungswege zu Kapitel 6

für i = 1, 3, 5 und damit folgt folgt



dα1 ∧ dα3 ∧ dα5 ω

3
3
3
M  
= Dj α1 dxj ∧ Dk α3 dxk ∧ Dl α5 dxl
= v(−2u) + u(u2 + v 2 + 1)2v du ∧ dv
j =1 k=1 l=1 A
1 1

3
= Dj α1 Dk α3 Dl α5 dxj ∧ dxk ∧ dxl = 2u3 v + 2uv 3 dλ(u, v)
0 0
j,k,l 1 1
= D1 α1 D2 α3 D3 α5 dx1 ∧ dx2 ∧ dx3 = v + v 3 dλ(v)
0 2
+D1 α1 D3 α3 D2 α5 dx1 ∧ dx3 ∧ dx2 1
+D2 α1 D1 α3 D3 α5 dx2 ∧ dx1 ∧ dx3 = .
2
+D2 α1 D3 α3 D1 α5 dx2 ∧ dx3 ∧ dx1
+D3 α1 D1 α3 D2 α5 dx3 ∧ dx1 ∧ dx2
+D3 α1 D2 α3 D1 α5 dx3 ∧ dx2 ∧ dx1 Beweisaufgaben
= (D1 α1 D2 α3 D3 α5 − D1 α1 D3 α3 D2 α5
Aufgabe 6.7 ••
* Die Funktion r(x) := x2 =
−D2 α1 D1 α3 D3 α5 + D2 α1 D3 α3 D1 α5 n
i=1 xi ist stetig differenzierbar auf R \{0} und wegen
2 n
+D3 α1 D1 α3 D2 α5 − D3 α1 D2 α3 D1 α5 )dx1 ∧ . . .
. . . ∧ dx2 ∧ dx3 ∂r 1 xi
  1
(x) = (x12 + · · · + xn2 )− 2 =
∂(α1 , α3 , α5 ) ∂xi 2 r(x)
= det dx1 ∧ dx2 ∧ dx3 .
∂(x1 , x2 , x3 )
gilt
Berechnung mit dem Satz (Duale Transformation elementa-
n
n
xi
rer Formen): dr = Di rdxi = dxi .
r
i=1 i=1
Es gilt dαi = α ∗ dyi für i = 1, . . . , 6 und da die duale
Transformation α ∗ linear ist, folgt somit Somit folgt

dα1 ∧ dα3 ∧ dα5


n
xi
∗(dr) = ∗ (dxi )
= α ∗ dy1 ∧ α ∗ dy3 ∧ α ∗ dy5 i=1
r
= α ∗ (dy1 ∧ dy3 ∧ dy5 ) n
xi
  = +i ∧ . . . ∧ dxn
sgn(σi )dx1 ∧ . . . ∧ dx
∂(α1 , α3 , α5 ) r
= det dx1 ∧ dx2 ∧ dx3 . i=1
∂(x1 , x2 , x3 )

n
xi
= +i ∧ . . . ∧ dxn ,
(−1)i−1 dx1 ∧ . . . ∧ dx
r
i=1
Aufgabe 6.6 • Setze ω = x2 dx2 ∧ dx3 + x1 x3 dx1 ∧
dx3 ∈ 2 (R3 ), dann gilt
wobei σi ∈ Sn durch

ω = α∗ ω
M A σi : (1, . . . , n) → (i, 1, . . . , i − 1, i + 1, . . . , n)
 
∂(α2 , α3 )
= (x2 ◦ α) det du ∧ dv definiert ist. Damit ergibt sich nun
A ∂(u, v)
 
∂(α1 , α3 )
+(x1 x3 ◦ α) det du ∧ dv. dr ∧ ∗(dr)
∂(u, v)
n
xi
= dr ∧ +i ∧ . . . ∧ dxn
(−1)i−1 dx1 ∧ . . . ∧ dx
Mit r
i=1
   
n
∂(α2 , α3 ) 0 1 xi +i ∧ . . . ∧ dxn
det = det = −2u = (−1)i−1 dr ∧ dx1 ∧ . . . ∧ dx
∂(u, v) 2u 2v r
i=1
⎛ ⎞
und n
xi n
x
(−1)i−1 ⎝ dxj ⎠ ∧ dx1 ∧ . . .
i
=
    r
i=1
r
j =1
∂(α1 , α3 ) 1 0
det = det = 2v +i ∧ . . . ∧ dxn
∂(u, v) 2u 2v . . . ∧ dx
Lösungswege zu Kapitel 6 45


n
x2 Parametrisierung auf A = (0, π2 ) × (0, 2π ) ein, so wird M
= i
(−1)i−1 dxi +i ∧ . . . ∧ dxn
∧ dx1 ∧ . . . ∧ dx
r2 von α bis auf eine Menge vom Maß null überdeckt. Nach
i=1
dem Satz von Seite 195 gilt dann

n
x2
i
= (−1)i−1 (−1)i−1 dx1 ∧ . . . ∧ dxn
r2 ω
i=1
M
1 2
n
= xi dx1 ∧ . . . ∧ dxn = α∗ ω
r2 A
i=1
= dx1 ∧ . . . ∧ dxn . = (xy ◦ α)α ∗ (dx ∧ dy) + (2x ◦ α)α ∗ (dy ∧ dz)
A
+(2y ◦ α)α ∗ (dx ∧ dz)
 
Aufgabe 6.8 •• Wir zeigen per vollständiger Induk- ∂α1,2
tion, dass = sin2 ϑ cos ϕ sin ϕ det dϕ ∧ dϑ
A ∂(ϕ, ϑ)
 
  ∂α2,3

k j −1
+2 sin ϑ cos ϕ det dϕ ∧ dϑ
i=1 pi
d(ω1 ∧. . .∧ωk ) = (−1) ω1 ∧. . .∧dωj ∧. . .∧ωk ∂(ϕ, ϑ)
 
j =1 ∂α1,3
+2 sin ϑ sin ϕ det dϕ ∧ dϑ
ist. Die Formel gilt offensichtlich im Fall k = 1, da ∂(ϕ, ϑ)

  = sin3 ϑ(cos ϕ sin ϕ cos θ − 2 cos2 ϕ

1 0 A
i=1 pi
dω1 = (−1) dωj = dω1 .
+2 sin2 ϕ)dϕ ∧ dϑ
j =1 π 2π
2
Gelte die Formel nun für ein festes k ∈ N, dann gilt Sie auch = sin3 ϑ 2 sin2 ϕ − 2 cos2 ϕ
0 0
für k + 1, denn − cos ϕ sin ϕ cos ϑ dλ(ϕ, ϑ).
d(ω1 ∧ . . . ∧ ωk ∧ ωk+1 )
= d(ω1 ∧ . . . ∧ ωk ) ∧ ωk+1 Für das innere Integral erhält man
 
k
i=1 pi 2π
+(−1) (ω1 ∧ . . . ∧ ωk ) ∧ dωk+1
⎛ ⎞ 2 sin2 ϕ − 2 cos2 ϕ − cos ϕ sin ϕ cos ϑ dλ(ϕ)
 

k j −1
p 0
= ⎝ (−1) i=1 i ω1 ∧ . . . ∧ dωj ∧ . . . ∧ ωk ⎠ . . . = [(ϕ − sin ϕ cos ϕ) − (φ + sin ϕ cos ϕ)
j =1 2π
  1 2
k −(− cos ϕ cos ϑ)
i=1 pi 2
∧ωk+1 + (−1) (ω1 ∧ . . . ∧ ωk ) ∧ dωk+1 0
  = 0

k+1 j −1
i=1 pi
= (−1) ω1 ∧ . . . ∧ dωj ∧ . . . ∧ ωk+1 . und daher ist
j =1
ω = 0.
M

Aufgabe 6.9 •• Sei (x; v) ∈ Tx (Rn ), dann gilt


Aufgabe 6.11 •• Definiere die Volumenform ωv wie
dg(x)(x; v) = Dg(x) · v im Hinweis beschrieben, d. h. für (p; v1 ), . . . , (p; vd ) ∈
Tp (M) sei
= D(ϕ ◦ f )(x) · v
⎡ ⎤
= Dϕ(f (x)) · Df (x) · v v1 · e1 . . . v1 · ed
⎢ .. ⎥ ,
= h(x)df (x)(x; v) ωv (p)((p; v1 ), . . . , (p; vd )) := det ⎣ ... . ⎦
vd · e1 . . . vd · ed
mit h(x) := Dϕ(f (x)) : Rn → R. Also gilt dg = hdf und
somit folgt wobei {(p; e1 ), . . . , (p; ed )} eine beliebige ONB des Tan-
gentialraums Tp (M) ist. Es folgt direkt
df ∧ dg = df ∧ (hdf )
= hdf ∧ df ωv (p)((p; e1 ), . . . , (p; ed )) = det(Ed ) = 1.
= dg ∧ df. Es bleibt noch zu zeigen, dass ωv unabhängig von der ge-
wählten ONB ist. Sei dazu {(p; b1 ), . . . , (p; bd )} eine wei-
Wegen df ∧ dg = −dg ∧ df , muss also df ∧ dg = 0 sein. tere ONB des Tangentialraums Tp (M), dann existiert eine
orthogonale Matrix A ∈ Rd×d mit
Aufgabe 6.10 •• Sei M := {(x, y, z) ∈ R3 | x 2 + y 2 +
z = 1, z ≥ 0}. Schränkt man die im Hinweis gegebene
2 bi = Aei
46 Lösungswege zu Kapitel 6

für i = 1, . . . , d (vgl. Seite 186). Setzt man E = Aufgabe 6.13 •• Betrachtet man die Form ω ∧ η ∈
[e1 , . . . , ed ], B = [b1 , . . . , bd ] und V = [v1 , . . . , vd ], so d+l (M), so gilt
folgt
d(ω ∧ η) = dω ∧ η + (−1)d ω ∧ dη.
ωv (p)((p; v1 ), . . . , (p; vd ))
⎡ ⎤
v1 · b1 . . . v1 · bd Nach Voraussetzung ist ∂M = ∅ und daher folgt mit dem
⎢ .. ⎥
= det ⎣ ... . ⎦
allgemeinen Satz von Stokes
vd · b1 . . . vd · bd
d(ω ∧ η) = dω ∧ η + (−1)d ω ∧ dη
= det(V  B) M
M M
= det(V  AE) = ω∧η
= det(A) det(V  E) ∂M

= 0.
= det(V E)
= ωv (p)((p; v1 ), . . . , (p; vd )), Also ist

also ist ωv unabhängig von der gewählten Basis von Tp (M). dω ∧ η = (−1) d+1
ω ∧ dη
M M
Aufgabe 6.12 •• Sei A ⊆ Rd offen und α : A → Rn
und somit ist die Behauptung mit a = (−1)d+1 gezeigt.
die Karte, die M überdeckt, dann gilt
α ∗ ωv = hdx1 ∧ . . . ∧ dxd ∈ d (U ). Aufgabe 6.14 •• Setzt ω = P dx + Qdy, dann gilt

Für x ∈ U folgt dω = dP ∧ dx + dQ ∧ dy
 
2 ∗ 2 ∂P ∂P
h(x) = α ωv (x)((x; e1 ), . . . , (x; ed )) = dx + dy ∧ dx
∂x ∂y
= ωv (α(x))(α∗ (x; e1 ), . . . , α∗ (x; ed ))2  
   2 ∂Q ∂Q
∂α ∂α + dx + dy ∧ dy
= ωv (p) p; , . . . , p; ∂x ∂y
∂x1 ∂xd
⎡ ∂α ⎤2 ∂P ∂Q
∂α = dy ∧ dx + dx ∧ dy
∂x · e1 . . . ∂x1 · ed ∂y ∂x
⎢ 1. .. ⎥  
= det ⎢
⎣ .. . ⎦
⎥ ∂Q ∂P
= − dx ∧ dy.
∂α ∂α ∂x ∂y
∂x · e1 . . . ∂xd · ed
⎡ ∂αd ∂α ⎤
∂x1 · ∂x1 . . . ∂x1 · ∂xd
∂α ∂α Mit dem allgemeinen Satz von Stokes folgt nun direkt die
⎢ ⎥ Green’sche Formel, denn
= det ⎢

..
.
..
.


∂α ∂α ∂α ∂α
∂xd · ∂x1 . . . ∂xd · ∂xd P dx + Qdy = d(P dx + Qdy)
= det G ∂M M
 
∂Q ∂P
= − dx ∧ dy
und somit ist M ∂x ∂y

α ∗ ωv = det Gdx1 ∧ . . . ∧ dxd .
Kapitel 7 Aufgabe 7.10 • Es seien (, A, μ) ein Maßraum,
( , A ) ein Messraum und f :  →  eine (A, A )-

messbare Abbildung. Zeigen Sie: Ist h :  → R̄ eine nicht-
Aufgaben negative A -messbare Funktion, so gilt

h dμf = h ◦ f dμ, A ∈ A .
Verständnisfragen A f −1 (A )

Aufgabe 7.1 • Zeigen Sie im Falle des Grundraums


 = {1, 2, 3}, dass die Vereinigung von σ -Algebren im All- Aufgabe 7.11 • Es seien (, A, μ) ein Maßraum so-
gemeinen keine σ -Algebra ist. wie p ∈ R mit 0 < p ≤ 1. Zeigen Sie: Für messbare nume-
rische Funktionen f und g auf  gilt
Aufgabe 7.2 • Es seien  eine unendliche Menge und

die Funktion μ∗ : P () → [0, ∞] durch μ∗ (A) := 0, falls A p
endlich, und μ∗ (A) := ∞ sonst definiert. Ist μ∗ ein äußeres |f + g| dμ ≤ |f | dμ + |g|p dμ .
p

Maß?

Aufgabe 7.3 • Es sei G : R → R eine maßdefinie- Aufgabe 7.12 •• Es seien  eine überabzählbare
rende Funktion mit zugehörigem Maß μG . Für x ∈ R Menge und A := {A ⊆  | A abzählbar oder Ac abzählbar}
bezeichne G(x−) := limy↑x,y<x G(y) den linksseitigen die σ -Algebra der abzählbaren oder co-abzählbaren Mengen.
Grenzwert von G an der Stelle x. Wegen der Monotonie von Die Maße ν und μ auf A seien durch ν(A) := 0, falls A ab-
G ist dabei limn→∞ G(yn ) nicht von der speziellen Folge zählbar und ν(A) := ∞ sonst sowie μ(A) := |A|, falls A
(yn ) mit yn ≤ yn+1 , n ∈ N, und yn → x abhängig, was endlich und μ(A) := ∞ sonst definiert. Zeigen Sie:
die verwendete Kurzschreibweise rechtfertigt. Zeigen Sie:
a) ν # μ.
Es gilt
G(x) − G(x−) = μG ({x}) , x ∈ R . b) ν besitzt keine Dichte bezüglich μ.
c) Warum steht dieses Ergebnis nicht im Widerspruch zum
Aufgabe 7.4 • Zeigen Sie: Jede monotone Funktion Satz von Radon-Nikodym?
f : R → R ist Borel-messbar.
Aufgabe 7.13 •• Es seien (, A) ein Messraum und
Aufgabe 7.5 • Es seien (, A) ein Messraum und μ, ν Maße auf A. Weisen Sie in Teil a) – c) ν # μ nach. Ge-
f :  → R̄ eine numerische Funktion. Zeigen Sie, dass aus ben Sie jeweils eine Radon–Nikodym-Dichte f von ν bzgl.
der Messbarkeit von |f | im Allgemeinen nicht die Messbar- μ an.
keit von f folgt.
a) (, A) beliebig, μ ein beliebiges Maß auf A, A0 ∈ A
Aufgabe 7.6 • Zeigen Sie, dass das System Ī := fest, ν(A) := μ(A ∩ A0 ), A ∈ A.
{[−∞, c]| | c ∈ R} einen Erzeuger der σ -Algebra B̄ über b) (, A) := (N, P (N)), P und Q beliebige Wahrschein-
R̄ bildet. lichkeitsmaße auf P (N), μ := P + Q, ν := P .
c) (, A) beliebig, λ ein σ -endliches Maß auf A, P und
Aufgabe 7.7 • Es sei μ ein Inhalt auf einer σ -Algebra Q Wahrscheinlichkeitsmaße auf A mit Dichten f bzw. g
A ⊆ P (). Zeigen Sie: Ist μ stetig von unten, so ist μ σ - bzgl. λ (P = f λ, Q = gλ), μ := P + Q, ν := P .
additiv und somit ein Maß.

Aufgabe 7.8 •• Es seien (, A, μ) ein Maßraum, Rechenaufgaben


( , A ) ein Messraum und f :  →  eine (A, A )- Aufgabe 7.14 • Zeigen Sie: Das im Beweis des Ein-
messbare Abbildung. Prüfen Sie die Gültigkeit folgender Im- deutigkeitssatzes für Maße auf Seite 221 auftretende Men-
plikationen: gensystem DB = {A ∈ A | μ1 (BA) = μ2 (BA)} ist ein
a) μ ist σ -endlich ⇒ μf ist σ -endlich, Dynkin-System.
b) μf ist σ -endlich ⇒ μ ist σ -endlich.
Aufgabe 7.15 • Es sei λk das Borel-Lebesgue-Maß
Aufgabe 7.9 •• Geben Sie Folgen (fn ), (gn ) und (hn ) auf B . Zeigen Sie: λ (Qk ) = 0.
k k

λ1 -integrierbarer reellwertiger Funktionen auf R an, die je-


weils λ1 -f.ü. gegen null konvergieren, und für die Folgendes Aufgabe 7.16 • Betrachten Sie den Messraum
gilt: (N, P (N)) mit dem Zählmaß μ auf N sowie die durch f (1) :=
f (4) := 4.3, f (2) := 1.7, f (3) := f (7) := f (9) := 6.1
limn→∞ fn dλ1 = ∞,
sowie f (n) := 0 sonst definierte Elementarfunktion auf N.
limn→∞ gn dλ1 = 1, Schreiben Sie f in Normaldarstellung und berechnen Sie

lim supn→∞ hn dλ1 = 1, lim inf n→∞ hn dλ1 = −1. f dμ.

M. Brokate et al., Arbeitsbuch Grundwissen Mathematikstudium – Höhere Analysis, Numerik und


Stochastik, DOI 10.1007/978-3-642-54946-5_6, © Springer-Verlag Berlin Heidelberg 2016
48 Aufgaben zu Kapitel 7

Aufgabe 7.17 •• Es seien (, A, μ) := (R>0 , B ∩ Aufgabe 7.26 •• Es seien  := (0, 1] und H der Halb-
R>0 , λ1 |R>0 ) und p ∈ (0, ∞). Zeigen Sie: Es existiert ring aller halboffenen Intervalle der Form (a, b] mit 0 ≤ a ≤
eine Funktion f ∈ Lp (, A, μ) mit der Eigenschaft f ∈/ b ≤ 1. Für (a, b] ∈ H sei μ((a, b]) := b − a gesetzt, falls
Lq (, A, μ) für jedes q ∈ (0, ∞) mit q  = p. 0 < a; weiter ist μ((0, b]) := ∞, 0 < b ≤ 1. Zeigen Sie: μ
ist ein Inhalt, aber kein Prämaß.
Aufgabe 7.18 • Die Funktion f : R2 → R sei durch
⎧ Aufgabe 7.27 •• Zeigen Sie: Die im Lemma von Cara-
⎨ 1, falls x ≥ 0, x ≤ y < x + 1, théodory auf Seite 222 auftretende σ -Algebra
f (x, y) := −1, falls x ≥ 0, x + 1 ≤ y < x + 2,

0 sonst, A(μ∗ )

definiert. Zeigen Sie: = {A ⊆  | μ∗ (A ∩ E) + μ∗ (Ac ∩ E) = μ∗ (E) ∀E ⊆ }


  besitzt folgende Eigenschaft: Ist A ∈ A(μ∗ ) mit μ∗ (A) = 0,
f (x, y)λ1 (dy) λ1 (dx) und ist B ⊆ A, so gilt auch B ∈ A(μ∗ ) (und damit wegen der
  Monotonie und Nichtnegativität von μ∗ auch μ∗ (B) = 0).
= f (x, y)λ1 (dx) λ1 (dy) .
Aufgabe 7.28 ••• Es seien (, A, μ) ein Maßraum und
Warum widerspricht dieses Ergebnis nicht dem Satz von
Aμ := {A ⊆  | ∃E, F ∈ A mit E ⊆ A ⊆ F , μ(F \ E) = 0} .
Fubini?
Die Mengenfunktion μ̄ : Aμ → [0, ∞] sei durch μ̄(A) :=
sup {μ(B) : B ∈ A, B ⊆ A} definiert. Zeigen Sie:
Beweisaufgaben
a) Aμ ist eine σ -Algebra über  mit A ⊆ Aμ .
Aufgabe 7.19 • Es seien R ⊆ P () ein Ring sowie
A := R ∪ {Ac | A ∈ R}. Zeigen Sie: A = α(R). b) μ̄ ist ein Maß auf Aμ mit μ̄|A = μ.
c) Der Maßraum (, Aμ , μ̄) ist vollständig, mit anderen
Aufgabe 7.20 • Es sei (An )n≥1 eine wachsende Folge Worten: Sind A ∈ Aμ mit μ̄(A) = 0 und B ⊆ A, so folgt
von Algebren über , also An ⊆ An+1 für n ≥ 1. Zeigen B ∈ Aμ .
Sie:
Aufgabe 7.29 • Beweisen Sie Teil a) und c) des Lem-
a) ∪∞
n=1 An ist eine Algebra. mas über σ -Algebren und Abbildungen auf Seite 227.
b) Sind An ⊆ P (), n ≥ 1, σ -Algebren mit An ⊂ An+1 ,
n ≥ 1, so ist ∪∞
n=1 An keine σ -Algebra. Aufgabe 7.30 •• Es seien (, A) und ( , A )
Messräume sowie f :  →  eine Abbildung. Ferner seien
Aufgabe 7.21 • Es sei M ⊆ P () ein beliebiges
A1 , A2 , . . . ∈ A paarweise disjunkt mit  = ∞
j =1 Aj . Für
Mengensystem. Wir setzen M0 := M ∪ {∅} sowie induktiv
n ∈ N bezeichne An := A ∩ An die Spur-σ -Algebra von A
Mn := {A \ B, A ∪ B | A, B ∈ Mn−1 }, n ≥ 1. Zeigen Sie:
in An und fn := f |An die Restriktion von f auf An . Zeigen
Der von M erzeugte Ring ist ρ(M) = ∪∞n=0 Mn . Sie:
Aufgabe 7.22 • Es seien Ak und Kk die Systeme der f ist (A, A )-messbar ⇐⇒ fn ist (An , A )-messbar, n ≥ 1.
abgeschlossenen bzw. kompakten Teilmengen des Rk . Zei-
gen Sie: σ (Ak ) = σ (Kk ). Folgern Sie hieraus, dass eine Funktion f : Rk → Rs ,
die höchstens abzählbar viele Unstetigkeitsstellen besitzt,
Aufgabe 7.23 • Es seien I k = {(x, y] | x, y ∈ (Bk , Bs )-messbar ist.
R , x ≤ y} und J := {(−∞, x] | x ∈ Rk }. Zeigen Sie:
k k

σ (I k ) = σ (J k ). Aufgabe 7.31 •• Es seien H ⊆ P () ein Halbring und


A, A1 , . . . , An ∈ H. Zeigen Sie: Es gibt eine natürliche Zahl
Aufgabe 7.24 • a) Es sei   = ∅. Geben Sie eine not- k und disjunkte Mengen C1 , . . . , Ck aus H mit
wendige und hinreichende Bedingung dafür an, dass das
Zähl-Maß μ auf  σ -endlich ist.
k
A \ (A1 ∪ . . . ∪ An ) = A ∩ Ac1 ∩ . . . ∩ Acn = Cj .
b) Auf dem Messraum (R, B) betrachte man das durch j =1
μ(B) := |B ∩ Q|, B ∈ B, definierte Maß. Zeigen Sie, dass
μ σ -endlich ist, obwohl jedes offene Intervall das μ-Maß ∞ Aufgabe 7.32 ••• Es sei μ ein Inhalt auf einem Halbring
besitzt. H ⊆ P (). Zeigen Sie:
n
Aufgabe 7.25 • a) Durch ν(A) := j =1 μ(A ) (A1 , . . . , An ∈ H paar-
Zeigen Sie: Ist μ ein Inhalt auf einem n j
Ring R ⊆ P (), so gilt für A, B ∈ R weise disjunkt, A = j =1 Aj ) entsteht ein auf R :=
ρ(H) wohldefinierter Inhalt, der μ eindeutig fortsetzt.
μ(A ∪ B) + μ(A ∩ B) = μ(A) + μ(B). b) Mit μ ist auch ν ein Prämaß.
Aufgaben zu Kapitel 7 49

Aufgabe 7.33 •• Es sei (, A, μ) ein Maßraum. Aufgabe 7.41 •• Es seien (, A, μ) ein Maßraum und
f , g messbare numerische Funktionen auf . Zeigen Sie:
a) Zeigen Sie: μ ist genau dann σ -endlich, wenn eine Zer-
legung von  in abzählbar viele messbare Teilmengen a) f · g1 ≤ f 1 · g∞ .
endlichen μ-Maßes existiert.
b) Falls μ() < ∞, so gilt
b) Es sei nun μ σ -endlich, und es gelte μ() = ∞. Zeigen
Sie, dass es zu jedem K mit 0 < K < ∞ eine Menge f q ≤ f p · μ()1/q−1/p (1 ≤ q < p ≤ ∞).
A ∈ A mit K < μ(A) < ∞ gibt.
(Konsequenz: Lp ⊆ Lq .)
Aufgabe 7.34 •• Es sei (, A, μ) ein Maßraum. Zei-
gen Sie die Äquivalenz der folgenden Aussagen: Aufgabe 7.42 •• Es seien (, A, μ) ein Maßraum und
a) μ ist σ -endlich, (fn )n≥1 eine Folge nichtnegativer messbarer numerischer
b) Es existiert eine Borel-messbare Abbildung h:  → R Funktionen auf . Zeigen Sie: Für jedes p ∈ [1, ∞] gilt
/∞ /
mit h(ω) > 0 für jedes ω ∈  und h dμ < ∞. / / ∞
/ /
/ fn / ≤ fn p .
Aufgabe 7.35 •• Für eine reelle Zahl κ  = 0 sei / /
n=1 p n=1
Hκ : Rk → Rk die durch Hκ (x) := κ ·x, x ∈ Rk , definierte
zentrische Streckung. Zeigen Sie: Für das Bildmaß von λk Aufgabe 7.43 •• Es seien (, A, μ) ein Maßraum und
unter Hκ gilt p ∈ (0, ∞]. (fn )n≥1 sei eine Funktionenfolge aus Lp mit
1 limn→∞ fn = f μ-f.ü. für eine reelle messbare Funktion f
Hκ (λk ) = · λk .
|κ|k auf . Es existiere eine messbare numerische Funktion g ≥ 0

Aufgabe 7.36 •• Es seien a1 , . . . , ak > 0 und E das auf  mit g p dμ < ∞ und |fn | ≤ g μ-f.ü. für jedes n ≥ 1.
Ellipsoid E := {x ∈ Rk | x12 /a12 + . . . + xk2 /ak2 < 1}. Zeigen Zeigen Sie:

Sie: Es gilt E ∈ Bk , und es ist a) |f |p dμ < ∞.
Lp
λk (E) = a1 · . . . · ak · λk (B), b) limn→∞ |fn − f |p dμ = 0 (d.h. fn → f ).

wobei B := {x ∈ Rk | x < 1} die Einheitskugel im Rk Aufgabe 7.44 •• Es seien (, A, μ) ein Maßraum so-
bezeichnet. wie 0 < p < ∞. Zeigen Sie: Die Menge

Aufgabe 7.37 •• Es seien (, A, μ) ein Maßraum und 0


n
F := u := αk 1{Ak } | n ∈ N, A1 , . . . , An ∈ A,
(An )n≥1 eine Folge von Mengen aus A. Für k ∈ N sei Bk
k=1
die Menge aller ω ∈ , die in mindestens k der Mengen 1
A1 , A2 , . . . liegen. Zeigen Sie: α1 , . . . , αn ∈ R, μ(Aj ) < ∞ für j = 1, . . . , n
a) Bk ∈ A,
∞ liegt dicht in Lp = Lp (, A, μ), d. h. zu jedem f ∈ Lp und
b) kμ(Bk ) ≤ n=1 μ(An ). jedem ε > 0 gibt es ein u ∈ F mit f − up < ε.

Aufgabe 7.38 •• Es seien (, A, μ) ein Maßraum und Aufgabe 7.45 ••• Für A ⊆ N sei dn (A) := n−1 |A ∩
f :  → N0 ∪ {∞} eine messbare Abbildung. Zeigen Sie: {1, . . . , n}| sowie


f dμ = μ (f ≥ n) . C := {A ⊆ N | d(A) := lim dn (A) existiert} .
n→∞
n=1
Die Größe d(A) heißt Dichte von A. Zeigen Sie:
Aufgabe 7.39 •• Es seien (, A, μ) ein Maßraum und
f :  → R̄ eine nichtnegative messbare numerische Funk- a) Die Mengenfunktion d : C → [0, 1] ist endlich-additiv,
tion. Zeigen Sie: aber nicht σ -additiv.
  b) C ist nicht ∩-stabil.
f
lim n log 1 + dμ = f dμ . c) Ist C ein Dynkin-System?
n→∞ n
Aufgabe 7.46 ••• Es seien Ok , Ak und Kk die Systeme
Aufgabe 7.40 •• Es seien (, A, μ) ein endlicher der offenen bzw. abgeschlossenen bzw. kompakten Teilmen-
Maßraum und (fn )n≥1 eine Folge μ-integrierbarer reeller gen des Rk . Beweisen Sie folgende Regularitätseigenschaft
Funktionen auf  mit f := limn→∞ fn gleichmäßig auf . eines endlichen Maßes μ auf Bk :
Zeigen Sie:
a) Zu jedem B ∈ Bk und zu jedem ε > 0 gibt es ein O ∈ Ok

und ein A ∈ Ak mit der Eigenschaft μ(O \ A) < ε.
f dμ = lim fn dμ .
n→∞ b) Es gilt μ(B) = sup{μ(K) | K ⊆ B, K ∈ Kk }.
50 Hinweise zu Kapitel 7

Aufgabe 7.47 ••• Es seien (j , Aj ) Messräume und Aufgabe 7.11 • Für festes a > 0 ist die durch h(x) :=
Mj ⊆ Aj mit σ (Mj ) = Aj (j = 1, . . . , n). In Mj exi- a p +x p −(a+x)p definierte Funktion h : R≥0 → R monoton
stiere eine Folge (Mj k )k≥1 mit Mj k ↑ j bei k → ∞. wachsend.
πj : 1 × · · · × n → j bezeichne die j -te Projektions-
abbildung und Aufgabe 7.12 •• –
2 3
M1×· · ·×Mn := M1 × · · · ×Mn | Mj ∈ Mj , j = 1, . . . , n Aufgabe 7.13 •• –

das System aller „messbaren Rechtecke mit Seiten aus


M1 , . . . , Mn “. Zeigen Sie: Rechenaufgaben
⎛ ⎞
4
n Aufgabe 7.14 • –
a) M1 × · · · × Mn ⊆ σ ⎝ πj−1 (Mj )⎠, ∞
j =1 Aufgabe 7.15 • Es gilt ε = n=1 ε/2
n.

4
n
b) πj−1 (Mj ) ⊆ σ (M1 × · · · × Mn ), Aufgabe 7.16 • –
j =1
5n
c) j =1 Aj = σ (M1 × · · · × Mn ). Aufgabe 7.17 •• Betrachten Sie die Funktion g(x) =
x −1 · (1 + | log(x)|)−2 .
Aufgabe 7.48 ••• Es seien μ und ν Maße auf einer σ -
Algebra A ⊆ P () mit ν() < ∞. Beweisen Sie folgendes Aufgabe 7.18 • –
ε-δ-Kriterium für absolute Stetigkeit:

ν # μ ⇐⇒ ∀ε > 0 ∃δ > 0 ∀A ∈ A : μ(A) ≤ δ ⇒ ν(A) ≤ ε. Beweisaufgaben


Aufgabe 7.19 • –
Aufgabe 7.49 •• Es seien μ und ν Maße auf einer σ -
Algebra A über  mit ν(A) ≤ μ(A), A ∈ A. Weiter sei μ Aufgabe 7.20 • In b) ist echte Inklusion gemeint.
σ -endlich. Zeigen Sie: Es existiert eine A-messbare Funktion
f :  → R mit 0 ≤ f (ω) ≤ 1 für jedes ω ∈ . Aufgabe 7.21 • –

Aufgabe 7.22 • Jede abgeschlossene Menge ist die ab-


zählbare Vereinigung kompakter Mengen.
Hinweise
Aufgabe 7.23 • –
Verständnisfragen Aufgabe 7.24 • Für b) beachte man μ(R \ Q) = 0.
Aufgabe 7.1 • –
Aufgabe 7.25 • –
Aufgabe 7.2 • –
Aufgabe 7.26 •• –
Aufgabe 7.3 • Es ist (−∞, x] = (−∞, x) ∪ {x}.
Aufgabe 7.27 •• –
Aufgabe 7.4 • –
Aufgabe 7.28 ••• –
Aufgabe 7.5 • – Aufgabe 7.29 • –

Aufgabe 7.6 • Bezeichnen σ (M) bzw. σ̄ (M) die von Aufgabe 7.30 •• –
M ⊆ P (R) bzw. M ⊆ P (R̄) über R bzw. über R̄ erzeugte σ -
Algebra, so gilt im Fall M ⊆ P (R) die Inklusionsbeziehung Aufgabe 7.31 •• Vollständige Induktion!
σ (M) ⊆ σ̄ (M).
Aufgabe 7.32 ••• Beachten Sie den Satz über den von
Aufgabe 7.7 • – einem Halbring erzeugten Ring auf Seite 215.

Aufgabe 7.8 •• – Aufgabe 7.33 •• –

Aufgabe 7.9 •• – Aufgabe 7.34 •• Für die Richtung b) ⇒ a) betrachte


man die Mengen {h ≥ 1/n}. Für die andere Richtung hilft
Aufgabe 7.10 • – Teil a) der vorigen Aufgabe.
Lösungen zu Kapitel 7 51

Aufgabe 7.35 •• Wie wirken beide Seiten der obigen Lösungen


Gleichung auf eine Menge (a, b] ∈ I k ?
Verständnisfragen
Aufgabe 7.36 •• – Aufgabe 7.1 • –

Aufgabe 7.2 • –
Aufgabe 7.37 •• –
Aufgabe 7.3 • –
Aufgabe 7.38 •• –
Aufgabe 7.4 • –

Aufgabe 7.39 •• Die durch an := (1 + x/n)n , Aufgabe 7.5 • –


x ∈ [0, ∞], definierte Folge (an )n≥1 ist monoton wachsend.
Aufgabe 7.6 • –

Aufgabe 7.40 •• – Aufgabe 7.7 • –

Aufgabe 7.8 •• –
Aufgabe 7.41 •• –
Aufgabe 7.9 •• –

Aufgabe 7.42 •• – Aufgabe 7.10 • –

Aufgabe 7.11 • –
Aufgabe 7.43 •• Benutzen Sie den Satz von der domi-
nierten Konvergenz.
Aufgabe 7.12 •• –

Aufgabe 7.13 •• –
Aufgabe 7.44 •• Es kann o.B.d.A. f ≥ 0 angenommen
werden.
Rechenaufgaben
Aufgabe 7.45 ••• Um7 b) zu zeigen,
6 2k 6 setzen7 Sie A := G, Aufgabe 7.14 • –
B := ∪∞ k=1 2 , 2 2k+1 ∩ G ∪ 22k−1 , 22k ∩ U , wobei

G die Menge der geraden und U die Menge der ungeraden Aufgabe 7.15 • –
Zahlen bezeichnen.
Aufgabe 7.16 • –

Aufgabe 7.46 ••• Zeigen Sie zunächst, dass das System Aufgabe 7.17 •• –
G aller Borelmengen, die die in a) angegebene Eigenschaft
besitzen, eine σ -Algebra bildet, die das System Ak enthält. Aufgabe 7.18 • –
Eine abgeschlossene Menge lässt sich durch eine absteigende
Folge offener Mengen approximieren. Beachten Sie noch,
dass die Vereinigung von endlich vielen abgeschlossenen Beweisaufgaben
Mengen abgeschlossen ist.
Aufgabe 7.19 • –

Aufgabe 7.47 ••• Für Teil c) ist (7.19) hilfreich. Aufgabe 7.20 • –

Aufgabe 7.21 • –
Aufgabe 7.48 ••• Betrachten Sie zu einer Folge (An )
mit μ(An ) ≤ 2−n und ν(An ) > ε die Menge A := Aufgabe 7.22 • –
∩∞ ∞
n=1 ∪k=n Ak .
Aufgabe 7.23 • –

Aufgabe 7.49 •• Nach dem Satz von Radon-Nikodym Aufgabe 7.24 • a) μ ist σ -endlich ⇐⇒  ist abzähl-
hat ν eine Dichte g bezüglich μ. Zeigen Sie: μ({g > 1}) = 0. bar.
52 Lösungswege zu Kapitel 7

Aufgabe 7.25 • – Lösungswege

Aufgabe 7.26 •• – Verständnisfragen


Aufgabe 7.1 • Es sei
Aufgabe 7.27 •• –
A1 := {∅, , {1}, {2, 3}}
Aufgabe 7.28 ••• –
und
A2 := {∅, , {2}, {1, 3}}.
Aufgabe 7.29 • –
Dann sind A1 und A2 σ -Algebren über . Die Vereinigung
Aufgabe 7.30 •• –
A1 ∪ A2 = {∅, , {1}, {2}, {2, 3}, {1, 3}}

Aufgabe 7.31 •• – ist jedoch keine σ -Algebra, da sie nicht ∪-stabil ist, denn
sie enthält nicht die Vereinigung {1, 2} der einelementigen
Aufgabe 7.32 ••• – Mengen {1} und {2}.

Aufgabe 7.2 • Nein, denn als unendliche Menge ent-


Aufgabe 7.33 •• – hält  eine abzählbar-unendliche Teilmenge 0 :=
{ω1 , ω2 , . . .}. Es gilt
Aufgabe 7.34 •• –


μ∗ (0 ) = μ∗ ( ({ωj })) = ∞,
Aufgabe 7.35 •• – j =1
∞
Aufgabe 7.36 •• – aber μ∗ ({ωj }) = 0 für jedes j und somit j =1 μ
∗ ({ω }) = 0.
j
Folglich ist μ∗ nicht σ -subadditiv.

Aufgabe 7.37 •• – Aufgabe 7.3 • Es sei (yn ) eine Folge mit yn≤ yn+1< x,
n ≥ 1, und limn→∞ yn = x. Dann ist ((−∞, yn ])n≥1 eine
Aufgabe 7.38 •• – aufsteigende Mengenfolge mit (−∞, yn ] ↑ (−∞, x). Da
μG stetig von unten ist, folgt G(yn ) = μG ((−∞, yn ]) ↑
Aufgabe 7.39 •• – μG ((−∞, x)) = G(x−) und somit wegen (−∞, x)+{x} =
(−∞, x] und der Subtraktivität von μG die Behauptung.

Aufgabe 7.40 •• – Aufgabe 7.4 • Es seien f monoton wachsend und


c ∈ R beliebig. Mit a := sup{x ∈ R | f (x) ≤ c} gilt
Aufgabe 7.41 •• – {f ≤ c} ∈ {∅, (−∞, a), (−∞, a]} ⊆ B. Der Fall, dass f
monoton fällt, folgt analog.
Aufgabe 7.42 •• –
Aufgabe 7.5 • Ist A ein echtes Teilsystem der Potenz-
menge von , so gibt es eine Menge A ⊆  mit A ∈ / A. Defi-
Aufgabe 7.43 •• – niert man die Funktion f durch f (ω) := 1, falls ω ∈ A, und
f (ω) := −1 sonst, so ist f nicht messbar, da f −1 ({1}) = A
Aufgabe 7.44 •• – nicht in A liegt. Als konstante Abbildung ist |f | ≡ 1 messbar.

Aufgabe 7.45 ••• – Aufgabe 7.6 • Wir schicken voraus, dass die im Hin-
weis formulierte Behauptung eine Konsequenz der Tatsache
ist, dass für jede σ -Algebra A über R̄ deren Spur A ∩ R eine
Aufgabe 7.46 ••• – σ -Algebra über R ist. Wegen Ī ⊆ B̄ ist nur die Inklusion

Aufgabe 7.47 ••• – B̄ ⊆ σ̄ (Ī ) (7.4)

zu zeigen. Zunächst gilt {−∞} = ∩∞ n=1 [−∞, −n] ∈ σ̄ (Ī )


Aufgabe 7.48 ••• – sowie ∪∞ [−∞, n] = [−∞, ∞) ∈ σ̄ (Ī ), also auch R̄ \
n=1
[−∞, ∞) = {∞} ∈ σ̄ (Ī ). Setzen wir I := {(−∞, c] |
Aufgabe 7.49 •• – c ∈ R}, so ergibt sich hiermit unter anderem die Inklusion
Lösungswege zu Kapitel 7 53

I ⊆ σ̄ (Ī ). Zusammen mit dem Satz von Seite 215 über c) Das Maß μ ist nicht σ -endlich, denn die Existenz einer
Erzeugendensysteme der Borelmengen sowie dem Hinweis aufsteigenden Folge An ↑  von Mengen aus A mit
folgt dann μ(An ) < ∞ zöge die Endlichkeit der Mengen An und so-
B = σ (I ) ⊆ σ̄ (I ) ⊆ σ̄ (Ī ) mit die Abzählbarkeit von  nach sich, im Widerspruch zur
Voraussetzung!
und somit wegen B̄ = {B ∪ E | B ∈ B, E ⊆ {−∞, ∞}} die
Behauptung.
Aufgabe 7.13 •• a) Mit μ(A) = 0 gilt auch ν(A) =
Aufgabe 7.7 • Es seien A1 , A2 , . . . paarweise dis- μ(A ∩ A0 ) = 0. Somit ist ν absolut stetig bzgl. μ. Wegen

junkte Mengen aus A sowie A := j =1 Aj . Weiter sei
n
Bn := j =1 Aj . Wegen Bn ↑ A liefern die Stetigkeit von ν(A) = μ(A ∩ A0 ) = 1A∩A0 dμ = 1A 1A0 dμ
unten und die endliche Additivität von μ wie behauptet


n ∞
gilt ν = 1A0 μ.
μ(A) = lim μ(Bn ) = lim μ(Aj ) = μ(Aj ).
n→∞ n→∞ b) Wegen P ≤ P + Q gilt P # P + Q. Da für eine Dichte
j =1 j =1
f von P bzgl. P + Q

Aufgabe 7.8 •• a) Diese Implikation gilt nicht, wie das P ({n}) = 1{n} dP = 1{n} f (ω) (P + Q)(dω)
Beispiel (, A, μ) = (R, B, λ1 ), ( , A ) = (N, P (N)) und
die durch f (x) := 1, x ∈ R, definierte Abbildung f : R → N = f (n) · (P ({n}) + Q({n}))
zeigt. Es gilt μf (A) = 0 bzw. μf (A) = ∞ je nachdem, ob
1∈ / A oder 1 ∈ A zutrifft. Das Borel-Lebesgue-Maß λ1 ist gelten muss, setzen wir
σ -endlich, sein Bildmaß unter f jedoch nicht. ⎧
⎨ 0 , falls P ({n})+Q({n}) = 0,
b) Istμf σ -endlich, so gibt es eine Folge An ↑  mit f (n) := P ({n})
μ (An ) < ∞, n ≥ 1. Setzen wir An := f −1 (An ), n ≥ 1,
f  ⎩ , sonst.
P ({n}) + Q({n})
so ist (An ) eine Folge von Mengen aus A mit An ↑  und
μ(An ) = μf (An ) < ∞, n ≥ 1. Folglich ist μ σ -endlich. Nach Definition von f gilt dann P ({n}) = f (n)(P ({n}) +
Q({n})) für jedes n ∈ N und somit
Aufgabe 7.9 •• Setzt man kn := 1[n,n+1] , n ≥ 1, so

gilt limn→∞ kn (x) = 0, x ∈ R. Die Funktionen fn := n · kn , P (A) = P ({n}) = f (n)(P ({n}) + Q({n}))
gn := k n und hn := (−1)
· kn , 1n ∈ N, leisten
n
das Verlangte; n∈A n∈A
es gilt f dλ 1 = n, g dλ = 1, h dλ1 = 1 und
n n 2n
h2n+1 dλ1 = −1. = f d(P + Q),
A

Aufgabe 7.10 • Die Behauptung ergibt sich unmittel- was zu zeigen war.
bar aus Teil a) des Transformationssatzes für Integrale auf
Seite 243, wenn man die dort auftretende Funktion h durch c) Wegen P ≤ P + Q folgt P # P + Q. Wir setzen
h1{A } ersetzt, denn es gilt (h1{A }) ◦ f = (h ◦ f ) · ⎧
1{f −1 (A )}. ⎨ 0 , falls f (ω) + g(ω) = 0,
h(ω) := f (ω)
⎩ , sonst.
Aufgabe 7.11 • Die im Hinweis gemachte Aussage f (ω) + g(ω)
bestätigt man durch Differentiation. Es gilt dann für alle
a, b ∈ [0, ∞] die Ungleichung (a + b)p ≤ a p + bp und Wegen P = f λ und Q = gλ folgt dann
somit |f (ω) + g(ω)|p ≤ |f (ω)|p + |g(ω)|p , ω ∈ . Inte-
griert man bezüglich μ, so folgt die Behauptung. P (A) = 1A dP = 1A f dλ

f
Aufgabe 7.12 •• a): Da die leere Menge ∅ die einzige = 1A · (f + g) dλ
μ-Nullmenge ist und ν(∅) = 0 gilt, folgt ν # μ. {f +g>0} f +g

b) Aus der Existenz einer nichtnegativen
(A, B̄1 )-messbaren = 1A hf dλ + 1A hg dλ
Funktion f :  → R̄ mit ν(A) = A f dμ, A ∈ A, würde
für beliebiges ω ∈  = 1A h dP + 1A h dQ

0 = ν({ω}) = f dμ = f (ω) · μ({ω}) = f (ω), = 1A h d(P + Q),
{ω}

also f ≡ 0 und somit ν() = 0 dμ = 0 folgen, was ein A ∈ A. Somit ist (die messbare nichtnegative Funktion) h
Widerspruch zu ν() = ∞ ist. eine Dichte von P bezüglich P + Q.
54 Lösungswege zu Kapitel 7

Rechenaufgaben und somit g ∈ L1 . Im Fall p < 1 gibt es zu jedem k ∈ N ein


xk , sodass 1 + log x ≤ x 1/k und folglich
Aufgabe 7.14 • Offenbar gilt  ∈ DB . Sind D, E ∈
DB mit D ⊆ E, so gilt μ1 (B ∩ D) = μ2 (B ∩ D) und 1 1

μ1 (B ∩ E) = μ2 (B ∩ E). Wegen der Subtraktivität von μ1 x p (1 + log x)2p x p(1+2/k)
und μ2 (Eigenschaft c) auf Seite 220) folgt für j = 1, 2
für jedes x ≥ xk . Ist dann k0 ∈ N mit p(1 + 2/k0 ) ≤ 1, so
μj (B ∩ (E \ D)) = μj (B ∩ E \ B ∩ D) folgt
= μj (B ∩ E) − μj (B ∩ D) ∞ ∞
dx
g(x) dx ≥ =∞
und somit μ1 (B ∩ (E \ D)) = μ2 (B ∩ (E \ D)), also 1 min(1,xk0 ) x
p(1+2/k0 )
E \ D ∈ DB . Sind schließlich D1 , D2 , . . . paarweise dis-
junkte Mengen aus DB , so ergibt sich aufgrund des Distri- und somit g ∈ / Lp . Ist p > 1, so gibt es zu jedem k ∈ N ein
butivgesetzes und der σ -Additivität von μ1 und μ2 xk , sodass 1 − log x ≤ x −1/k und somit
 ∞  ∞
1 1
≥ p(1−2/k)
μ1 B ∩ Dn = μ1 (B ∩ Dn ) x p (1 − log x)2p x
n=1 n=1
∞ für jedes x ≤ xk . Wählt man jetzt k0 ∈ N, sodass
= μ2 (B ∩ Dn ) p(1 − 2/k0 ) ≥ 1, so ergibt sich
n=1 1 max(1,xk )
  ∞
 0 dx
g(x) dx ≥ =∞
p(1−2/k 0)
= μ2 B ∩ Dn 0 0 x
n=1
und somit g ∈
/ Lp .
∞
und folglich n=1 Dn ∈ DB .
Aufgabe 7.18 • Es gilt f (x, y)λ1 (dy) = 0 für je-
Aufgabe 7.15 • Es seien Qk =: {q1 , q2 , . . .} eine Ab- des x, und somit verschwindet das links stehende Integral.
zählung von Q und ε > 0 beliebig. Setzen wir In :=
k
Andererseits gilt
(qn − δn , qn + δn ], wobei δn := (ηn , ηn , . . . , ηn ) und ⎧
ηn := (ε/(2n ))1/k /2, so gilt λk (In ) = (2ηn )k = ε/2n , und ⎪
⎨ y , falls 0 ≤ y ≤ 1,
wegen Qk ⊆ ∪∞ n=1 In liefert die σ -Subadditivität von λ
k 1
f (x, y)λ (dx) = 2 − y, falls 1 ≤ y ≤ 2,



0 , sonst,
ε
λk (Qk ) ≤ = ε.
2n und somit ist das rechts stehende Integral gleich 1. Die-
n=1
ses
Ergebnis widerspricht nicht dem Satz von Fubini, da
Da ε beliebig war, folgt die Behauptung. |f | dλ2 = ∞ gilt und somit f nicht bezüglich des Pro-
duktmaßes λ1 ⊗ λ1 integrierbar ist.
Aufgabe 7.16 • Mit A1 := {1, 4}, A2 := {2}, A3 :=
{3, 7, 9} und A4 := N \ (A1 ∪ A2 ∪ A3 ) ist
Beweisaufgaben
f = 4.3 · 1{A1 } + 1.7 · 1{A2 } + 6.1 · 1{A3 } + 0 · 1{A4 }
Aufgabe 7.19 • Da α(R) eine Algebra ist, die R um-
eine Normaldarstellung von f . Wegen μ(A1 ) = 2, μ(A2 ) =
fasst, muss sie als „komplementstabiles Mengensystem“
3 ) = 3, μ(A4 ) = ∞ und der Konvention 0 · ∞ = 0
1, μ(A
auch A enthalten, was A ⊆ α(R) impliziert. Könnten wir
folgt f dμ = 4.3 · 2 + 1.7 · 1 + 6.1 · 3 = 28.6.
zeigen, dass A eine Algebra ist, wären wir fertig, denn we-
Aufgabe 7.17 •• Wir zeigen gen A ⊇ R wäre dann auch A ⊇ α(R).
Zunächst gilt ∅ ∈ A, und nach Konstruktion enthält A mit
g ∈ Lp ⇐⇒ p = 1. (7.5)
jeder Menge auch deren Komplement. Wir zeigen, dass A ∩-
Hieraus folgt die Behauptung, denn die Funktion f := g 1/p stabil ist. Dann würden mit A, B ∈ A auch A \ B = A ∩ B c
leistet das Verlangte. Um (7.5) nachzuweisen, betrachten wir und A ∪ B = (Ac ∩ B c )c in A liegen, womit A als Algebra
zunächst den Fall p = 1. Wegen d/dx(1 + log x)−1 = nachgewiesen wäre.
−x(1 + log x)−2 folgt
∞ Sind A, B ∈ A, so unterscheiden wir die Fälle, dass beide
1 dx Mengen in R liegen, dass genau eine in R liegt und die an-
g dλ |R>0 =
0 x(1 + | log x|)2 dere das Komplement einer Menge aus R ist oder dass beide
1 ∞ Mengen Komplemente von Mengen aus R sind. Im ersten
dx dx
= + Fall gilt A ∩ B ∈ R ⊆ A, im zweiten sei o.B.d.A. A ∈ R
0 x(1 − log x) 2
1 x(1 + log x)2
1 ∞ und B = D c mit D ∈ R, dann gilt A ∩ B = A \ D ∈ R, und
1  1  im letzten Fall gilt A = E c , B = D c mit D, E ∈ R, woraus
=  +  =2
1 − log x 0 1 + log x 1
 A ∩ B = (E ∪ D)c ∈ A folgt, denn es gilt E ∪ D ∈ R.
Lösungswege zu Kapitel 7 55

Aufgabe 7.20 • a) Sei A := ∪∞ n=1 An . Wegen Aufgabe 7.25 • Wegen


{∅, } ⊆ A1 gilt {∅, } ⊆ A. Ist A ∈ A, so gilt A ∈ An für
ein n und somit auch Ac ∈ An , also Ac ∈ A. Gilt A, B ∈ A, A ∪ B = A + (B \ A)
so gibt es ein m und ein n mit A ∈ Am und B ∈ An . Mit
k := max(m, n) folgt dann A, B ∈ Ak und somit – da Ak und
eine Algebra ist – A \ B ∈ Ak ⊆ A sowie A ∪ B ∈ Ak ⊆ A, B = (A ∩ B) + (B \ A)
was zu zeigen war. liefert die Additivität von μ die beiden Gleichungen
b) Es sei A wie in a). Wegen der echten Inklusion An ⊂ An+1 μ(A ∪ B) = μ(A) + μ(B \ A)
gibt es zu jedem n ≥ 1 eine Menge An ∈ An+1 \ An . Wäre
A eine σ -Algebra, müsste A := ∪∞ n=1 An ∈ A gelten. Dann und
wäre aber Ak = A \ (∪n=k An ) ∈ Ak , im Widerspruch zur μ(B) = μ(A ∩ B) + μ(B \ A).
Annahme Ak ∈ Ak+1 \ Ak .
Durch Addition erhält man
Aufgabe 7.21 • Sei μ(A∪B)+μ(A∩B)+μ(B \A) = μ(A)+μ(B)+μ(B \A).
R := ∪∞
n=0 Mn . Hieraus folgt die Behauptung, wenn μ(B \ A) endlich ist.
Im Fall μ(B \ A) = ∞ folgt aber μ(A ∪ B) = ∞ = μ(B),
Wir zeigen zunächst, dass R ein Ring ist. Wegen R ⊇ M sodass die Behauptung ebenfalls richtig ist.
folgt dann R ⊇ ρ(M). Wegen ∅ ∈ M0 gilt ∅ ∈ R. Sind
A, B ∈ R, so gibt es wegen Mn−1 ⊆ Mn , n ≥ 1, ein n ≥ 1 Aufgabe 7.26 •• Sind A1 := (a1 , b1 ], . . . , An :=
mit A, B ∈ Mn−1 . Nach Definition von Mn liegen dann (an , bn ] paarweise disjunkte Mengen aus H mit
A ∪ B und A \ B in Mn und somit in R, was zu zeigen war.
Für die Inklusion ρ(M) ⊇ R beachte man, dass ρ(M) als A := (a, b] = A1 + . . . + An ∈ H,
Ring, der M enthält, auch M0 und (induktiv!) mit Mn−1
auch Mn enthalten muss. Folglich gilt ρ(M) ⊇ R. so kann nach eventueller Umnummerierung bj = aj +1 für
j = 1, . . . , n − 1 angenommen werden. Gilt a > 0, so folgt
Aufgabe 7.22 • Da jede kompakte Menge abgeschlos-

n
n
sen ist, gilt Kk ⊆ Ak und somit σ (Kk ) ⊆ σ (Ak ). Ist A eine b − a = μ((a, b]) = (bj − aj ) = μ((aj , bj ]).
beliebige abgeschlossene Menge und Bn := [−n, n]k , n ≥ 1, j =1 j =1
so ist A∩Bn eine abgeschlossene und beschränkte, also kom-
pakte Menge, und wegen A = ∪∞ n=1 A ∩ Bn gilt A ∈ σ (K ),
k Im Fall a = 0 gilt
also A ⊆ σ (K ) und somit σ (A ) ⊆ σ (K ).
k k k k

n
μ((a, b]) = ∞ = μ((aj , bj ]),
Aufgabe 7.23 • Es sei en := (−n, . . . , −n) ∈ Rk , j =1
n ≥ 1. Wegen (−∞, x] = ∪∞ n=1 (en , x] gilt J ⊆ σ (I )
k k

und somit σ (J ) ⊆ σ (I ). Zum Nachweis der umgekehrten


k k denn es ist a1 = a = 0. Folglich ist μ endlich-additiv. Wegen
Inklusion seien x = (x1 , . . . , xk ), y = (y1 , . . . , yk ) ∈ Rk μ(∅) = μ((b, b]) = b − b = 0 für ∞b ∈ (0, 1] ist μ ein
mit x < y. Für j = 1, . . . , k bezeichne zj ∈ Rk den Vek- Inhalt. Andererseits gilt (0, 1] = n=1 (1/(n + 1), 1/n],
tor, der bis auf die mit xj besetzte j -te Komponente mit y wobei ∞ = μ((0, 1]) und 1 = ∞ n=1 μ(1/(n + 1), 1/n]).
übereinstimmt, also z1 = (x1 , y2 , . . . , yk ) usw. Wegen Dies bedeutet, dass μ nicht σ -additiv und somit kein Prämaß
  ist.
(x, y] = (−∞, y] \ ∪kj =1 (−∞, zj ]
Aufgabe 7.27 •• Nach Definition von A(μ∗ ) und der
Voraussetzung über A gilt
gilt (x, y] ∈ σ (J k ) und somit I k ⊆ σ (J k ), was zu zeigen
war. μ∗ (A ∩ E) ≤ μ∗ (A) = 0, E ⊆ ,

Aufgabe 7.24 • Ist μ σ -endlich, so gibt es eine aufstei- und damit


gende Folge An ↑  von Teilmengen von  mit μ(An ) <
∞, n ≥ 1. Nach Definition des Zählmaßes ist An eine endli- μ∗ (Ac ∩ E) = μ∗ (E), E ⊆ .
che Menge, und es folgt, dass  abzählbar ist. Ist umgekehrt
 abzählbar, so gibt es eine aufsteigende Folge An ↑  Ist B ⊆ A, so gilt einerseits
endlicher Teilmengen A1 , A2 , . . . von , was zeigt, dass μ μ∗ (B ∩ E) ≤ μ∗ (B) ≤ μ∗ (A) = 0, E ⊆ ,
σ -endlich ist.
b) Es sei Q =: {q1 , q2 , . . .} eine Abzählung von Q und andererseits folgt wegen B c ⊇ Ac die Ungleichung
An := (R \ Q) ∪ {q1 , . . . , qn }, n ≥ 1. Dann ist (An ) ↑ R eine
aufsteigende Mengenfolge mit μ(An ) = n < ∞, n ≥ 1. μ∗ (B c ∩ E) ≥ μ∗ (Ac ∩ E) = μ∗ (E), E ⊆ .
56 Lösungswege zu Kapitel 7

Da μ∗ (B c ∩ E) ≤ μ∗ (E) gilt, folgt zusammen Aufgabe 7.30 •• „⇒“: Für A ∈ A gilt fn−1 (A ) =
{ω ∈ An | fn (ω) ∈ A } = An ∩ f −1 (A ) ∈ An ∩ A = An .
μ∗ (B ∩ E) + μ∗ (B c ∩ E) = μ∗ (E), E ⊆ , „⇐“: Es sei A ∈ A  und A := f −1 (A ). Nun ist A =
 ∞ ∞ 
was zu zeigen war. n=1 An ∩ A = n=1 {ω ∈ An | f (ω) ∈ A } =
∞ −1  −1 
n=1 fn (A ). Wegen fn (A ) ∈ An ∩ A gibt es eine
Aufgabe 7.28 ••• a) Setzt man in der Definition von Aμ Menge Bn ∈ A, sodass fn−1 (A ) = An ∩ Bn ∈ A, n ≥ 1. Da
E := F := , so folgt  ∈ Aμ . Sind A ∈ Aμ und E, F ∈ A A eine σ -Algebra ist, gilt auch A ∈ A, was zu zeigen war.
mit E ⊆ A ⊆ F und μ(F \ E) = 0, so gilt F c ⊆ Ac ⊆ E c Um die Folgerung zu zeigen, sei A1 := {x ∈ Rk | f
mit E c , F c ∈ A und μ(E c \ F c ) = μ(F \ E) = 0. Folglich unstetig an der Stelle x}. Dann ist f1 := f |A1 (A1 ∩ Bk , Bs )-
enthält Aμ mit jeder Menge auch deren Komplement. Sind messbar, denn die Urbilder von Borelmengen unter f1 sind
schließlich A1 , A2 , . . . ∈ Aμ , so gibt es Folgen (En ) und abzählbare Teilmengen von A1 . Es sei A2 := Rk \ A1 sowie
(Fn ) aus A mit En ⊆ An ⊆ Fn und μ(Fn \ En ) = 0, f2 := f |A2 . Ist O ⊆ Bs offen, so ist f2−1 (O) = {x ∈ A2 |
n ≥ 1. Es folgt ∪∞ ∞ ∞
n=1 En ⊆ ∪n=1 An ⊆ ∪n=1 Fn , wobei f (x) ∈ O} eine offene Menge und liegt somit in A2 ∩ Bk .
∞ ∞
∪n=1 En ∈ A, ∪n=1 Fn ∈ A sowie μ(∪n=1 Fn \ ∪∞

n=1 En ) ≤ Nach dem Messbarkeitskriterium ist dann f2 (A2 ∩ Bk , Bs )-
μ(∪n=1 (Fn \ En )) ≤ ∞

n=1 μ(F n \ E n ) = 0. Folglich gilt messbar, und die Behauptung folgt nach dem oben Gezeigten.
∪∞n=1 A n ∈ A μ , womit A μ als σ -Algebra nachgewiesen ist.
Da man zu jedem A ∈ A die Mengen E und F als A wählen Aufgabe 7.31 •• Wir zeigen die Behauptung durch In-
kann, gilt A ⊆ Aμ . duktion über n. Der Fall n = 1 ergibt sich unmittelbar aus
der Definition eines Halbrings. Für den Induktionsschluss
b) Offenbar gilt μ̄(∅) = 0. Um die σ -Additivität von μ̄ zu
n → n + 1 setzen wir die oben angegebene Darstellung vor-
zeigen, stellen wir eine Vorüberlegung an. Sind A ∈ Aμ und
aus. Es folgt
E, F ∈ A mit E ⊆ A ⊆ F und μ(F \ E) = 0 , so gilt
μ̄(A) = μ(E), d.h. es ist μ(E) = sup{μ(B) : B ∈ A, B ⊆
k

A}. Offenbar gilt hier „≤“, und würde „<“ gelten, so gäbe A ∩ Ac1 ∩ . . . ∩ Acn ∩ Acn+1 = (Cj \ An+1 ).
es ein B ∈ A mit B ⊆ A und μ(B) > μ(E). Dann wäre j =1
B ∪ E ∈ A mit B ∪ E ⊆ A und μ(B ∪ E) > μ(E). Da jede der Mengen Cj \An+1 als Vereinigung endlich vieler
Wegen 0 = μ(F \ E) = μ(F \ (B ∪ E)) + μ((B ∪ E) \ E) paarweise disjunkter Mengen aus H dargestellt werden kann,
würde dann μ(B ∪ E) = μ(E) + μ((B ∪ E) \ E) = μ(E) ergibt sich die Behauptung.
folgen, was ein Widerspruch ist. Nach dieser Vorüberlegung n
seien A1 , A2 , . . . ∈ Aμ paarweise disjunkt und (En ), (Fn ) Aufgabe 7.32 ••• a) Sind A = j =1 Aj und A =
m
Folgen aus A mit En ⊆ An ⊆ Fn , n ≥ 1, sowie μ(Fn \ B
i=1 i zwei Darstellungen von A als disjunkte Vereinigung
En ) = 0, n ≥ 1. Nach der Vorüberlegung gilt μ̄(An ) = endlich vieler Mengen aus H, so gilt wegen der Additivität
μ(E ), n ≥ 1, sowiewegen μ(∪∞
n∞

n=1 Fn \ ∪n=1 En ) = 0 auch
von μ

μ̄( n=1 An ) = μ( n=1 En ) (die En sind wegen En ⊆ An   m 
n n m n
paarweise disjunkt). Es folgt μ(Aj )= μ Aj ∩ Bi = μ Aj ∩ B i
∞  ∞  ∞ ∞ j =1 j =1 i=1 j =1 i=1

μ̄ An = μ En = μ(En ) = μ̄(An ). n m

n=1 n=1 n=1 n=1


= μ(Aj ∩ Bi ).
j =1 i=1
Somit ist μ̄ ein Maß, das offenbar μ fortsetzt.
Da der letzte  Ausdruck symmetrisch
 in den Mengen Aj und
c) Es seien A ∈ Aμ mit μ̄(A) = 0 und B ⊆ A. Nach Defini- Bi ist, folgt nj=1 μ(Aj ) = ni=1 μ(Bi ), was zeigt, dass
tion von Aμ gibt es Mengen E, F ∈ A mit E ⊆ A ⊆ F und ν wohldefiniert und eindeutig bestimmt ist. Des Weiteren
μ(F \E) = 0. Aufgrund der in b) angestellten Vorüberlegung ist ν(A) = μ(A), A ∈ H. Sind A = nj=1 Aj und B =
m
gilt μ̄(A) = μ(E). Wegen ∅ ⊆ B ⊆ F mit ∅, F ∈ A und i=1 Bi in R mit A ∩ B = ∅ und paarweise disjunkten
μ(F \ ∅) = μ(F ) = μ(E) + μ(F \ E) = 0 folgt B ∈ Aμ . Mengen Aj , Bi ∈ H, so ist A+B eine disjunkte Vereinigung
aller Aj , Bi , woraus ν(A + B) = ν(A) + ν(B) und damit
Aufgabe 7.29 • a) Zunächst gilt  = f −1 ( ) ∈ die Additivität von ν folgt.
f (A ). Mit A = f −1 (A ) ∈ f −1 (A ) gilt  \ A =
−1 

f −1 ( \A ) ∈ f −1 (A ). Sind schließlich Aj = f −1 (Aj ) ∈ b) Es seien nun μ als σ -additiv vorausgesetzt und A1 , A2 , . . .
paarweise disjunkte Mengen aus R mit A := ∞ j =1 Aj ∈
f −1 (A ), j = 1, 2, . . ., so folgt ∪∞
j =1 Aj = f
−1 (∪∞ A ) ∈
j =1 j R. Wegen A ∈ R gibt es paarweise disjunkte Mengen

f −1 (A ). B1 , . . . , Bm ∈ H mit A = m i=1 Bi , und zu jedem j exi-
c) Wegen f −1 ( ) =  ∈ A liegt  in Af . Gilt A ∈ Af , stieren paarweise disjunkte Mengen Cj,l (l = 1, . . . , nj )
nj
so folgt f −1 ( \ A ) =  \ f −1 (A ) ∈ Af , da A als σ - aus H, sodass Aj = l=1 Cj,l . Da
Algebra das Komplement von f −1 (A ) enthält. Mit Mengen ∞ ∞
nj

A1 , A2 , . . . ∈ Af liegt auch ∪∞ 
j =1 Aj in Af , denn es gilt Bi = Bi ∩ A j = Bi ∩ Cj,l
f −1 (∪∞  ∞
j =1 Aj ) = ∪j =1 f
−1 (A ) ∈ A.
j j =1 j =1 l=1
Lösungswege zu Kapitel 7 57

eine disjunkte Vereinigung von abzählbar vielen Mengen aus Aufgabe 7.35 •• Es sei (a, b] ∈ I k beliebig. Für
H ist, liefern die σ -Additivität von μ auf H und die Definition κ > 0 gilt Hκ−1 ((a, b]) = (a/κ, b/κ], im Fall κ <
von ν 0 ist Hκ−1 ((a, b]) = [−b/|κ|, −a/|κ|). Nach (7.33) gilt
λk (Hκ−1 ((a, b])) = |κ|−k λk ((a, b]), sodass der Eindeu-

nj ∞
tigkeitssatz für Maße die Behauptung liefert. Speziell für
μ(Bi ) = μ(Bi ∩ Cj,l ) = ν(Bi ∩ Aj )
κ = −1 ergibt sich die Spiegelungsinvarianz von λk .
j =1 l=1 j =1

für jedes feste i = 1, . . . , m und somit Aufgabe 7.36 •• Als offene Menge ist E eine Borel-
menge. Es sei A := diag(1/a1 , . . . , 1/ak ) die Diagonalma-

m ∞
m trix mit Einträgen 1/aj , j = 1, . . . , k, und T : Rk → Rk
ν(A) = μ(Bi ) = ν(Bi ∩ Aj ) die durch T (x) := Ax, x = (x1 , . . . , xk ) , definierte bi-
i=1 j =1 i=1 jektive affine Transformation. Nach Definition von E und B
∞ ∞
gilt dann E = T −1 (B) und somit λk (E) = λk (T −1 (B)) =
= ν(A ∩ Aj ) = ν(Aj ), T (λk )(B). Nach Teil a) der Folgerung auf 9 Seite 235 gilt
j =1 j =1 T (λk ) = |det A|−1 λk . Wegen |det A|−1 = kj =1 aj folgt
die Behauptung.
was die σ -Additivität von μ zeigt.
Aufgabe
∞ 7.37 •• a) Es gilt Bk = {ω ∈  |
Aufgabe 7.33 •• a) Ist μ σ -endlich, so gibt es eine −1 ([m, ∞]), wobei f :=
∞ n=1 1{A n } ≥ k}, also Bk = f
Folge (An ) aus A mit An ↑  und μ(An ) < ∞, n ≥ 1.
n=1 1{An }. Da f als Limes messbarer Funktionen messbar
Setzen wir B1 := A1 sowie Bn := An \ (∪n−1 i=1 Ai ), n ≥ 2, ist und [m, ∞] ∈ B̄ gilt, folgt Bk ∈ A.
so sindB1 , B2 , . . . paarweise disjunkte Mengen aus A mit
∞ b) Es gilt (punktweise auf ) k1{Bk } ≤ f mit f wie in a)
 = n=1 Bn  und μ(Bn ) ≤ μ(An ) < ∞, n ≥ 1. Ist
umgekehrt  = ∞ n=1 Bn eine Zerlegung in paarweise dis-
und somit k1{Bk } dμ = kμ(Bk ) ≤ f dμ. Nach  dem Satz
junkte Mengen aus A mit jeweils endlichem Maß, so ist (An ) von der monotonen Konvergenz ist f dμ = ∞ n=1 μ(An ).
mit An := ∪nj=1 Bj eine Folge aus A mit An ↑  und
 Aufgabe 7.38 •• Wir unterscheiden die Fälle
μ(An ) ≤ nj=1 μ(Bj ) < ∞, n ≥ 1.
μ(f = ∞) > 0 und μ(f = ∞) = 0. Im ersten Fall
b) Es sei (An ) wie oben. Wegen limn→∞ μ(An ) = μ() gilt
gibt es zu K ∈ (0, ∞) ein m ∈ N mit K < μ(Am ) < ∞. ∞ ∞

μ(f ≥ n) ≥ μ(f = ∞) = ∞
Aufgabe 7.34 •• „a) ⇒ b)“: Aufgrund
∞ der vorigen Auf-
n=1 n=1

gabe gibt es eine Zerlegung  = n=1 Bn mit Bn ∈ A, sowie f dμ ≥ f 1{f=∞} dμ = ∞. Im Fall μ(f = ∞) = 0
n ≥ 1, und μ(Bn ) < ∞ für jedes n. Außerdem kann (nach setzen wir fk := k1{f = k}, k ∈ N0 . Dann gilt f =
 ∞
eventueller Vereinigung von Mengen) o.B.d.A μ(Bn ) > 0 k=1 fk μ-fast überall. Mit dem Satz von der monotonen
für jedes n angenommen werden. Wir setzen Konvergenz folgt



1
h := · 1{Bn }. f dμ = fk dμ
2n μ(Bn )
n=1 k=1


Dann ist h eine messbare strikt positive Funktion auf , und = k1{f = k} dμ
nach dem Satz von der monotonen Konvergenz gilt k=1
∞ ∞
k

1 = kμ(f = k) = μ(f = k)
h dμ = 1{Bn } dμ
2n μ(Bn ) k=1 k=1 n=1
n=1
∞ ∞

1 = (μ(f = k) + μ(f = ∞))
= = 1.
2n n=1 k=n
n=1

„b) ⇒ a)“: Die Mengen An := {h > 1/n}, n ∈ N, liegen in = μ(f ≥ n).
A, und wegen der strikten Positivität von h gilt  = ∪∞
n=1 An .
n=1
Weiter gilt An ⊆ An+1 , n ∈ N, sowie unter Beachtung der
Markov-Ungleichung (vgl. Seite 245)
Aufgabe 7.39 •• Es sei fn (ω) := n log(1 + f (ω)/n),
ω ∈ . Dabei ist log ∞ := ∞ gesetzt. Als Verkettung
μ(An ) = μ(h ≥ 1/n) ≤ n · h dμ < ∞, n ∈ N. messbarer Funktionen ist fn messbar, und nach dem Hin-
weis ist die Folge (fn ) isoton. Im Fall f (ω) = ∞ gilt
Somit ist μ σ -additiv. fn (ω) = ∞ für jedes n und somit limn→∞ fn (ω) = f (ω).
58 Lösungswege zu Kapitel 7

Letztere Limesbeziehung gilt auch im Fall f (ω) < ∞, da zu zeigen. Mit an := fn ∞ gilt für jedes ε > 0
#∞ : #∞ :
(1 + f (ω)/n)n → exp(f (ω)). Die Behauptung folgt dann ∞
∞ 
ε 
aus dem Satz von der monotonen Konvergenz. fn > an + ε = fn > an + n
2
n=1 n=1 n=1 n=1
∞ 0
Aufgabe 7.40 •• Es sei ε > 0 beliebig. Da (fn ) 4 ε 1
gleichmäßig gegen f konvergiert, gibt es ein n0 ∈ N mit ⊆ fn > an +
2n
n=1
ω∈ |fn (ω) − f (ω)| ≤ ε für jedes n ≥ n0 . Wegen
sup
1 dμ = μ() < ∞ folgt für jedes solche n und somit
∞ 


 ε 
 
  μ fn > an + ε ≤ μ fn > an + n = 0.
 fn dμ − f dμ ≤ |fn − f | dμ ≤ ε · μ() 2
n=1 n=1 n=1
Lässt man ε gegen null streben, so folgt die Behauptung.
und hieraus die Behauptung, da ε beliebig war.
Aufgabe 7.43 •• a) Aus |fn | ≤ g μ-f.ü. folgt |fn |p ≤
Aufgabe 7.41 •• a) Wir können o.B.d.A. g∞ < ∞ g μ-f.ü. Wegen lim fn = f μ-f.ü. erhalten wir |f |p ≤ g p
p
annehmen. Wegen |g| ≤ g∞ μ-f.ü. gilt μ-f.ü. und somit die μ-Integrierbarkeit von |f |p .

b) Aus lim fn = f μ-f.ü. folgt |fn −f |p → 0 μ-f.ü. Für p ≥ 1
fg1 = |fg| dμ ≤ |f |·g∞ dμ ≤ g∞ |f | dμ. gilt |fn − f |p ≤ 2p |fn |p + 2p |f |p und somit |fn − f |p ≤
2p+1 g p μ-f.ü. Im Fall p < 1 gilt |fn − f |p ≤ |fn |p + |f |p
b) Es sei o.B.d.A. f p < ∞. Setzen wir r := p/q > 1 (vgl. Aufgabe 7.11) und somit |fn − f |p ≤ 2g p μ-f.ü. Da
und s := p/(p − q), so gilt 1/r + 1/s = 1. Wendet man g p integrierbar ist, ergibt sich die Behauptung aus dem Satz
die Hölder-Ungleichung auf die Funktionen |f |q und 1 an, von der dominierten Konvergenz.
so folgt
Aufgabe 7.44 •• Die im Hinweis gemachte Aussage
 1/r  1/s
q qr s folgt aus der Zerlegung f = f + − f − von f in Positiv- und
|f | dμ ≤ |f | dμ 1 dμ
Negativteil. Gibt es zu f + und f − Funktionen u, v ∈ F mit
 q/p ε ε
= |f |p dμ · μ()(p−q)/p f + − up < , f − − vp < , (7.6)
2 2
so gilt für p ≥ 1 aufgrund der Minkowski-Ungleichung f −
und somit (u−v)p ≤ f + −up +f − −vp < ε. Dabei gilt u−v ∈
 1/q F . Im Fall p < 1 liefert (7.6) zusammen mit Ungleichung
f q = |f |q dμ (7.43)
 ε p
f − (u − v)p ≤ f + − up + f − − vp < 2 ·
p p p
 1/p
2
≤ |f |p dμ μ()(p−q)/(pq) ,
und somit f − (u − v)p < 21/p−1 ε. Es kann also in der
Tat o.B.d.A. der Fall f ≥ 0 angenommen werden. Aufgrund
was zu zeigen war. des Satzes über die Approximation nichtnegativer messbarer
∞ Funktionen durch Elementarfunktionen auf Seite 239 gibt es
Aufgabe 7.42 •• Wir können o.B.d.A. n=1 fn p eine isotone Folge (un ) aus E + mit 0 ≤ un ↑ f . Wegen
< ∞ annehmen.
 Es sei zunächst
 p < ∞. Wegen fn ≥ 0 f ∈ Lp gilt auch un ∈ Lp und somit un ∈ F (letztere Aus-
gilt dann ( kn=1 fn )p ↑ ( ∞ n=1 f n )p für k → ∞, und der
sage gilt, weil ein in der Darstellung einer Elementarfunktion
Satz von der monotonen Konvergenz liefert eventuell auftretender Summand 0 · 1A mit μ(A) = ∞ weg-
/ k / /∞ / gelassen werden kann). Wegen 0 ≤ f − un ≤ f liefert der
/ / / /
/ / / / Satz von der dominierten Konvergenz limn→∞ f − un p
lim / fn / = / fn / .
k→∞ / / / / = 0. Es gibt also ein u ∈ F mit 0 ≤ u ≤ f und f −up < ε.
n=1 p n=1 p
Aufgabe 7.45 ••• a) Sind A1 , . . . , Ak paarweise dis-
Nach der Minkowski-Ungleichung gilt für jedes k
junkte Mengen aus C , so gilt dn (A1 + . . . + Ak ) = dn (A1 ) +
/ k / . . . + dn (Ak ), n ≥ 1. Da d(Aj ) = limn→∞ dn (Aj ) für
/ /
k ∞
 
/ /
/ fn / ≤ fn p ≤ fn p . jedes j existiert, folgt d( nj=1 Aj )) = kj =1 d(Aj ); also
/ /
n=1 p n=1 n=1 ist d endlich-additiv. Wegen d({j }) = 0, j ∈ N, gilt
1 = d(N) = 0 = ∞ j =1 d({j }), sodass d nicht σ -additiv
Hieraus folgt die Behauptung. Im Fall p = ∞ ist ist.
∞ 

b) Es gilt dn (A) = 1/2 oder dn (A) = (n − 1)/(2n)
μ fn > fn ∞ = 0 je nachdem, ob n gerade oder ungerade ist. Hieraus folgt
n=1 n=1 A ∈ C , wobei d(A) = 1/2. Nach Konstruktion von
Lösungswege zu Kapitel 7 59

B = {3, 4, 6, 8, 9, 11, 13, 15, 16, 18, . . .} unterscheidet sich [−n, n]k =: Kn ↑ A sowie der Endlichkeit von μ gibt es
|B ∩ {1, . . . , n}| von |A ∩ {1, . . . , n}| betragsmäßig um höch- ein n ∈ N mit μ(A) − μ(Kn ) < ε/2. Wegen Kn ∈ Kk
stens 1, und deshalb gelten auch B ∈ C und d(B) = 1/2. existiert also eine kompakte Menge Kn mit Kn ⊆ B und
Nun gilt |G ∩ [22k , 22k+1 ]| = 1 + 22k−1 und folglich μ(B) − μ(Kn ) < ε. Hieraus folgt die Behauptung.

1
k
1 Aufgabe 7.47 ••• a) Es sei Mi ∈ Mi , i = 1, . . . , n.
d22k+1 (A ∩ B) = (1 + 22l−1 ) → bei k → ∞, Wegen M1 × . . . × Mn = ∩ni=1 πi−1 (Mi ) und πi−1 (Mi ) ∈
22k+1 3 ; 
l=1 n −1
σ j =1 πj (Mj ) für jedes i folgt die Behauptung.
andererseits aber auch
b) Wir zeigen π1−1 (M1 ) ⊆ σ (M1 ×· · ·× Mn ). Die Behaup-
1
k−1
1 tung folgt dann aus Symmetriegründen. Wegen π1−1 (M1 ) =
d22k (A ∩ B) = (1 + 22l−1 ) → bei k → ∞. M1 × 2 × . . . × n und M1 × 2 × . . . × n =
22k 6
l=1
∪∞k=1 M1 × M2k × . . . × Mnk ∈ σ (M1 × · · · × Mn ) folgt
Somit kann dn (A ∩ B) nicht konvergieren. Die Menge A ∩ B π1−1 (M1 ) ⊆ σ (M1 × · · · × Mn ), was zu zeigen war.
liegt also nicht in C . 5n −1
j =1 Aj = σ (∪j =1 πj (Aj )) folgt „⊇“ aus
c) Wegen n

c) Wäre C ein Dynkin-System, so müsste es – da jede einele- Aj ⊇ Mj , j = 1, . . . , n, sowie


; a) und b). Für die um-
n −1
mentige Teilmenge von N zu C gehört – jede abzählbare Teil- gekehrte Richtung sei C := σ j =1 πj (Mj ) gesetzt.
menge von N enthalten und somit gleich der Potenzmenge Für festes i gilt πi−1 (Mi ) ⊆ C und somit nach (7.19)
von N sein. Da nach b) A ∩ B ∈
/ C gilt, ist das nicht der Fall.
πi−1 (σ (Mi )) = πi−1 (Ai ) ⊆ C , da C eine σ -Algebra ist.
Aufgabe 7.46 ••• a) Es sei G := {B ∈ Bk | ∃O ∈ Es folgt ∪ni=1 πi−1 (Ai ) ⊆ C und somit die Behauptung.
Ok ∃A ∈ Ak mit μ(O \ A) < ε}. Es gilt Rk ∈ G , da in obiger
Aufgabe 7.48 ••• Die Richtung „⇐“ folgt unmittelbar,
Definition A = O = Rk gesetzt werden kann. Gilt B ∈ G
da zu beliebig vorgegebenem ε > 0 jede μ-Nullmenge A die
und sind A und O wie in der Definition von G gewählt, so
Eigenschaft ν(A) ≤ ε besitzt. Die Implikation „⇒“ beweisen
gilt O c ⊆ B c ⊆ Ac , wobei μ(Ac \ O c ) = μ(O \ A) < ε.
wir durch Kontraposition und nehmen hierzu an, es gäbe ein
Da Ac und O c als Komplemente einer abgeschlossenen bzw.
ε > 0, sodass zu jedem δ > 0 eine Menge A in A mit
einer offenen Menge offen bzw. abgeschlossen sind, folgt
μ(A) ≤ δ und ν(A) > ε existierte. Dann gibt es eine Folge
B c ∈ G . Wir zeigen jetzt, dass G mit Mengen B1 , B2 , . . .
(An ) von Mengen aus A mit μ(An ) ≤ 2−n und ν(An ) > ε,
auch deren Vereinigung B := ∪∞ n=1 Bn enthält (damit wäre G n ≥ 1. Setzen wir A := ∩∞ ∞ ∞
n=1 ∪k=n Ak und Bn := ∪k=n Ak ,
eine σ -Algebra). Es sei ε > 0 gegeben. Zu jedem n ≥ 1 exi-
so gilt A ⊆ Bn , n ≥ 1, und somit
stieren ein On ∈ Ok und ein An ∈ Ak mit An ⊆ Bn ⊆ On
∞ ∞
und μ(On \ An ) < ε/2n+1 . Die Menge O := ∪∞ n=1 On
1 1
ist offen. Setzen wir A := ∪∞ A , so ist A nicht unbe- μ(A) ≤ μ(Bn ) ≤ μ(Ak ) ≤ k
= n−1 ,
n=1 n 2 2
k=n k=n
dingt abgeschlossen, aber für die durch Cn := ∪nj=1 Aj
definierte Folge (Cn ) gilt Cn ↑ A, und jede der Men- n ≥ 1, also μ(A) = 0. Da ν stetig von oben ist und Bn ↓ A
gen Cn ist abgeschlossen. Da μ stetig von unten ist, gilt gilt, liefern die Bedingung ν() < ∞ sowie Bn ⊇ An
∞ 
μ(Cn ) → μ(A). Wegen μ(A) < ∞ gibt es somit ein m ∈ N <
mit μ(A) − μ(Cm ) < ε/2. Es folgt Cm ⊆ B ⊆ O, wobei ν(A) = ν Bn = lim ν(Bn ) ≥ lim sup ν(An ) ≥ ε,
n→∞ n→∞
n=1
μ(O \ Cm ) = μ(O \ A) + μ(A \ Cm )
was ein Widerspruch zu μ(A) = 0 und ν # μ ist.


ε ε ε
< μ(On \ An ) + ≤ + Aufgabe 7.49 •• Wegen ν ≤ μ gilt ν # μ, und somit
2 2n+1 2
n=1 n=1 gibt es nach dem Satz von Radon-Nikodym eine nichtnega-
= ε.
tive A-messbare Funktion g :  → R mit ν = gμ. Es sei
Somit gilt auch ∪∞ A := {g > 1}. Wir behaupten, dass μ(A) = 0 gilt. Dann
n=1 Bn ∈ G , sodass G eine σ -Algebra ist.
würde die Funktion f := 1{Ac } · g das Verlangte leisten,
Wir zeigen jetzt, dass jede abgeschlossene Menge A zu G denn es wäre 0 ≤ f ≤ 1 und f = g μ-fast überall und damit
gehört. Es sei On := {x ∈ Rk | ∃y ∈ A mit x − y < 1/n}. ν = f μ. Nun gilt
Dann ist On offen, und es gilt A ⊆ On , n ≥ 1. Außerdem
gilt On ↓ A bei n → ∞, denn x ∈ ∩∞ n=1 On bedeutet, dass μ(A) = 1A dμ ≤ 1A g dμ = 1A dν = ν(A)
x als Grenzwert einer konvergenten Folge (yn ) aus A in A
liegt, da A abgeschlossen ist. Wegen der Endlichkeit von μ und damit μ(A) = ν(A). Es ergibt sich
gibt es zu vorgegebenem ε > 0 ein n mit μ(On )−μ(A) < ε.
Somit gilt A ∈ G , also Ak ⊆ G . Da G eine σ -Algebra ist, 0 = ν(A) − μ(A) = 1A (g − 1) dμ.
folgt Bk ⊆ G , was zu zeigen war.
Wegen 1A (g − 1) ≥ 0 folgt 1A (g − 1) = 0 μ-fast überall,
b) Es sei B ∈ Bk . Zu ε > 0 gibt es eine abgeschlossene also μ({1A (g − 1) > 0}) = 0 und somit μ({1A > 0}) = 0.
Menge A mit A ⊆ B und μ(B \ A) < ε/2. Wegen A ∩ Die letzte Aussage ist zu μ(A) = 0 äquivalent.
0
Kapitel 8 C 0,α () = f ∈ C() : f beschränkt und ∃ c > 0 mit
1
|f (x) − f (y)| ≤ c|x − y|α für x, y ∈ 
Aufgaben mit
|(f (x)) − f (y)|
Verständnisfragen f 0,α = sup |f (x)| + sup
x ∈ x  =y |x − y|α
1
Aufgabe 8.1 •• Es seien f, g ∈ C([0, 1]) und auch g ∈
ein normierter Raum gegeben ist. Der Raum wird Raum
C([0, 1]) und p, q > 1 mit p1 + q1 = 1. Welche der folgenden
der hölderstetigen Funktionen genannt.
Ungleichungen ist falsch?
Beweisen Sie, dass C 0,α () im Fall einer kompakten
  1  1 Menge  ⊆ Rd ein Banachraum ist.
1 p+q 1 2 1 2
p q
1. |f (t)| 2 dt ≤ |f (t)| dt |f (t)| dt
0 0 0 Aufgabe 8.7 • Berechnen Sie die Lösung der Integral-
 q  p gleichung
1 1 1 1 1
2. |f (t)g(t)| dt ≤ |f (t)| p dt |g(t)| q dt t
0 0 0
x(t) − (t − s) x(s) ds = 1, 0≤t ≤1
 p  1−p 0
1 1 1 1 −1
3. |f (t)g(t)| dt ≥ |f (t)| dt
p |g(t)| p−1 dt (a) durch Differenzieren (siehe auch Aufgabe 8.3),
0 0 0
(b) mit der Neumann’schen Reihe in der Form

t
Aufgabe 8.2 • Zeigen Sie, dass der Folgenraum lp mit
x(t) = y(t) + km (t, s) y(s) ds
1 < p < ∞ ein Banachraum ist.
m=1 0

Aufgabe 8.3 •• Formulieren Sie das Anfangswertpro- und dem iterierten Kernen
blem t
(t − s)2n+1
kn+1 (t, s) = k(t, τ ) kn (τ, s) dτ = ,
x  (t) + g(t)x(t) = h(t), x(0) = a, x  (0) = b s (2n + 1)!
mit g, h ∈ C([0, ∞)) und a, b ∈ R als Volterra-Integral- n ∈ N, wobei k1 (t, s) = k(t, s) gesetzt ist.
gleichung und zeigen Sie, dass das Lösen der Volterra-Inte-
gralgleichung in den stetigen Funktionen äquivalent ist zum Aufgabe 8.8 • Beschreiben Sie das Randwertproblem
Lösen des Anfangswertproblems für zweimal stetig differen-
zierbare Funktionen. x  (t) = x 2 (t) + 1, x(0) = x(1) = 0

durch einen Integraloperator.


Aufgabe 8.4 • Gegeben sind die beiden Operatoren
R, L : lp → lp mit
Aufgabe 8.9 •• Zeigen Sie:
Rx = (0, x1 , x2 , . . . ) und Lx = (x2 , x3 , x4 , . . . ) . Der Dualraum (l1 ) ist normisomorph zu l∞ .
Zeigen Sie, dass R, L linear und beschränkt sind mit R = Der Dualraum zu
0 1
L = 1 und c0 = (xn )n∈N | (xn ) ist Nullfolge in R
R injektiv, aber nicht surjektiv,
L surjektiv, aber nicht injektiv. mit der Supremumsnorm (xn ) = supn∈N |xn | ist norm-
isomorph zu l1 .
Aufgabe 8.5 • Es sei L ∈ L(X, Y ) eine Isometrie in Der Raum l1 ist nicht reflexiv.
normierten Räumen X, Y , d. h., Lx = x für jedes x ∈
X. Zeigen Sie:
Beweisaufgaben
(a) Die Abbildung L: X → Y ist injektiv und
L−1: L(X) → X ist auch isometrisch. Aufgabe 8.10 • Sind A, B : X → X lineare Operato-
(b) Ist X Banachraum, so ist L(X) ⊆ Y abgeschlossen. ren in einem Vektorraum X, die kommutieren, d. h. AB =
BA. Sei weiter AB : X → X invertierbar. Dann sind auch
A und B invertierbar und es gilt A−1 = B(AB)−1 und
Rechenaufgaben B −1 = A(AB)−1 .
Aufgabe 8.6 ••
Aufgabe 8.11 • Zeigen Sie, dass für lineare be-
Zeigen Sie, dass zu reell- oder komplexwertigen Funktio- schränkte Operatoren A, B ∈ L(X, X) auf einem normierten
nen über einer Menge  ⊆ Rd und α ∈ (0, 1] durch Raum X stets AB − BA = I gilt.
M. Brokate et al., Arbeitsbuch Grundwissen Mathematikstudium – Höhere Analysis, Numerik und
Stochastik, DOI 10.1007/978-3-642-54946-5_7, © Springer-Verlag Berlin Heidelberg 2016
Hinweise zu Kapitel 8 61

Aufgabe 8.12 •• Seien X, Y normierte Räume, A ∈ Konvergenz der Folgenglieder und zeige dann, dass diese
L(X, Y ) und X̃, Ỹ zugehörige Vervollständigungen von X Folge in lp ist.
bzw. Y . Zeigen Sie, dass es genau einen linearen, beschränk-
ten Operator à : X̃ → Ỹ gibt mit Ãx ∼ = Ax für x ∈ X. Aufgabe 8.3 •• Die Integralgleichung ergibt sich durch
Beweisen Sie weiterhin à = A. zweimaliges Integrieren der Differenzialgleichung und Ver-
tauschen der Integrationsreihenfolge.
Aufgabe 8.13 •••
(a) Zeigen Sie, dass für ein lineares Funktional ϕ : X → R Aufgabe 8.4 • Linearität und auch Beschränktheit
mit ϕ = 0 auf einem normierten Raum X folgende Be- sind relativ offensichtlich. Für den Wert der Operator-Norm
dingungen äquivalent sind. lassen sich passende Beispiele finden. Gegenbeispiele bele-
(i) ϕ ist stetig. gen, dass R nicht surjektiv und L nicht injektiv ist.
(ii) Kern(ϕ) = {x ∈ X : ϕ(x) = 0} ⊆ X ist abgeschlos-
sen. Aufgabe 8.5 • Für Teilaufgabe (b) zeige man, dass
(iii) Kern(ϕ) ist nicht dicht in X. (xn ) Cauchy-Folge ist, wenn die Folge (Lxn ) in Y konver-
(b) Sei X = {x ∈ C([−1, 1]) : x ist in 0 diff’bar} mit der giert.
Maximumsnorm ausgestattet. Zeigen Sie, dass {x ∈
X : x  (0) = 0} dicht liegt in X. Rechenaufgaben
Aufgabe 8.14 •• Mit dem Satz über die stetige Inverse Aufgabe 8.6 •• Um die Normeigenschaften zu prüfen,
lässt sich die Stetigkeit eines linearen Operators auf Ba- zeige man, dass durch sup |(f (|xx))−f
−y |α
(y )|
eine Halbnorm ge-
nachräumen auch anders beschreiben. Zeigen Sie den Satz x  =y

vom abgeschlossenen Graphen: geben ist, d. h., es gelten alle Normeigenschaften bis auf die
Definitheit.
Ein linearer Operator A : X → Y auf Banachräumen
X, Y ist genau dann beschränkt, wenn der Graph der Für die Vollständigkeit in der zweiten Teilaufgabe betrachte
Abbildung man eine Cauchy-Folge und nutze, dass die stetigen Funktio-
nen vollständig sind und die Funktionen gleichmäßig stetig
G = {(x, Ax) ∈ X × Y | x ∈ X} ⊆ X × Y sind.
eine abgeschlossene Teilmenge ist.
Aufgabe 8.7 • Im zweiten Teil führt jeweils eine In-
Aufgabe 8.15 •• Ist X normierter Raum und M ⊆ X. duktion auf die Rekursionsgleichung und auf die explizite
Zeigen Sie, dass M genau dann beschränkt ist, wenn l(M) Darstellung der iterierten Kerne. Ein Einsetzen der expliziten
beschränkt ist für jedes l ∈ X . Darstellung in die Neumann’sche Reihe liefert die Lösung.

Aufgabe 8.16 • Beweisen Sie den folgenden strikten Aufgabe 8.8 • Zweimaliges Integrieren der Differen-
Trennungssatz: Ist X ein normierter Raum, A ⊆ X eine zialgleichung und Vertauschen der Integrationsreihenfolge
konvexe, abgeschlossenen Teilmenge und x ∈ X mit x  ∈ A, führt auf eine Integralgleichung.
dann gibt es ein Funktional l ∈ X  und γ ∈ R mit
Aufgabe 8.9 •• Zu (yn ) ∈ l∞ betrachte das lineare
l(y) ≤ γ < l(x) , für jedes y ∈ A .
Funktional ly : l1 → C mit

Aufgabe 8.17 • Beweisen Sie: Ist A ⊆ X eine abge- ∞



schlossene, konvexe Teilmenge eines normierten Raums X ly (x) = xn yn .
und (xn ) eine schwach konvergente Folge in A mit xn  x ∈ n=1
X, dann folgt x ∈ A. Analog untersuche man in der zweiten Teilaufgabe diese
Funktionale mit (yn ) ∈ l1 und (xn ) ∈ co . Bei der dritten
Aufgabe 8.18 ••• Zeigen Sie, dass ein Banachraum X Teilaufgabe betrachte man die Eigenschaft der Separabilität
genau dann reflexiv ist, wenn X  reflexiv ist. der involvierten Räume.

Beweisaufgaben
Hinweise
Aufgabe 8.10 • Zeigen Sie zu den angegebene Dar-
stellungen die Identitäten AA−1 = I , BB −1 = I , A−1 A = I
Verständnisfragen und B −1 B = I .
Aufgabe 8.1 •• Hölder’sche Ungleichung!
Aufgabe 8.11 • Zeigen Sie induktiv, dass aus AB −
Aufgabe 8.2 • Man betrachte eine Cauchy-Folge in lp , BA = I die Gleichung AB n − B n A = nB n−1 folgt und
definiere sich einen Kandidaten für die Grenzfolge aus der folgern Sie daraus einen Widerspruch.
62 Lösungswege zu Kapitel 8

Aufgabe 8.12 •• Mit einer Cauchy-Folge (xn ) in X ist Aufgabe 8.8 •


(Axn ) Cauchy-Folge in Y , da A beschränkt ist. Damit lässt 1 1
sich der Operator A : X → Y zu einem linearen beschränkten x(t) − k(t, s) x 2 (s) ds = t (t − 1).
0 2
Operator auf X̃ erweitern.

Aufgabe 8.13 ••• Für die Implikation „(i) ⇒ (ii)“ be- Aufgabe 8.9 •• –
trachte man den Quotienten X̃/Kern(ϕ) und zeige, dass durch
x̃ = inf x∈X̃ xX eine Norm auf diesem eindimensiona-
len Raum gegeben ist. Beweisaufgaben
Aufgabe 8.10 • –
Aufgabe 8.14 •• Für eine der beiden zu zeigenden Im-
plikationen wende man den Satz über die stetige Inverse auf
Aufgabe 8.11 • –
die lineare Abbildung B : G → X mit B(x, Ax) = x an.
Aufgabe 8.12 •• –
Aufgabe 8.15 •• Für die nicht offensichtliche Richtung
des Beweises nutze man das Prinzip der gleichmäßigen Be-
Aufgabe 8.13 ••• –
schränktheit.
Aufgabe 8.14 •• –
Aufgabe 8.16 • Anwenden des Trennungssatzes von
Eidelheit.
Aufgabe 8.15 •• –
Aufgabe 8.17 • Verwenden Sie den Trennungssatz aus
Aufgabe 8.16 • –
Aufgabe 8.16.
Aufgabe 8.17 • –
Aufgabe 8.18 ••• Nutzen Sie die Notationen Jx ∈ X
und Ll ∈ X für Funktionale, die x ∈ X bzw. l ∈ X zuge-
ordnet werden. Für eine Richtung des Beweises zeige man, Aufgabe 8.18 ••• –
dass, wenn X nicht reflexiv ist, die Menge {Jx | x ∈ X} ⊆
X  ein abgeschlossener echter Unterraum ist und verwende
die Folgerung von Seite 300.
Lösungswege

Verständnisfragen
Lösungen
Aufgabe 8.1 •• Die erste Ungleichung ist korrekt; denn
mit der Hölder’schen Ungleichung im Spezialfall, dass beide
Verständnisfragen Potenzen 21 sind, gilt
Aufgabe 8.1 •• 1. richtig, 2. falsch, 3. richtig.  1  1
1 1 2 1 2
p+q p q
2 2
Aufgabe 8.2 • – |f (t)| 2 dt ≤ (|f (t)| 2 ) dt (|f (t)| 2 ) dt
0 0 0
  1  1
Aufgabe 8.3 •• Die äquivalente Formulierung ist die 1
p
2 1
q
2

Integralgleichung = |f (t)| dt |f (t)| dt .


0 0
(I + A)x = f
t t Die zweite Ungleichung ist falsch; denn setzen wir etwa
mit Ax(t) = 0 (t − s)g(s)x(s) ds und f (t) = 0 (t −
s)h(s) ds + bt + a in den stetigen Funktionen. p = 3, f (t) = 2 und g(t) = 1, so ist die linke Seite
1
Aufgabe 8.4 • – |f (t)g(t)| dt = 2 ,
0
Aufgabe 8.5 • – aber rechts gilt
 q  p
1 1 1 1 √
Rechenaufgaben |f (t)| p dt |g(t)| q dt = 2 < 2.
0 0
Aufgabe 8.6 •• –
Die dritte Ungleichung ist richtig. Sie wird auch als die um-
Aufgabe 8.7 • – gekehrte Hölder’sche Ungleichung bezeichnet. Denn es gilt
Lösungswege zu Kapitel 8 63

mit der Hölder’schen Ungleichung Da g und x stetig sind, lassen sich mit dem Satz von Fubini
die Integrationsreihenfolgen vertauschen und es folgt
1 1 1 1
− p1 t τ t t
|f (t)| p dt = |f (t)g(t)| p |g(t)| dt
0 0 g(s)x(s) ds dτ = dτ g(s)x(s) ds
  1  1 0 0 0 s
1 p 1
− pq
q t
≤ |f (t)g(t)| dt |g(t)| dt . = (t − s)g(s)x(s) ds .
0 0 0

p
Mit q = p−1 , d. h. q
= 1
und 1
= p−1 Analog vertauschen wir die Integrationsreihenfolge auf der
p p−1 q p , ergibt sich die
Ungleichung. rechten Seite und erhalten, dass x Lösung der Volterra-
Integralgleichung
Aufgabe 8.2 • Zunächst überlegen wir uns generell
x(t) + Ax(t) = f (t)
noch einmal, dass Cauchy-Folgen in einem normierten Raum
beschränkt sind; denn ist (x n ) Cauchy-Folge, so gibt es n ∈ N ist mit dem Integraloperator
mit x n − x m  ≤ 1 für alle n, m > N und somit ist t
x n  ≤ x n − x N  + x N  ≤ 1 + x N  Ax(t) = (t − s)g(s)x(s) ds
0

für jedes n ≥ N . und der stetigen Funktion f ∈ C([0, ∞)) gegeben durch
Sei nun (x n ) eine Cauchy-Folge in l p . Dann ist für die Fol- t
genglieder zu festen k ∈ N die Folge (xkn )n∈N eine Cauchy- f (t) = (t − s)h(s) ds + bt + a .
0
Folge in R bzw. C und somit konvergent. Wir definieren
xk = limn→∞ xkn und erhalten mit der Folge (xk )k∈N einen „⇐“ : Sei x ∈ C([0, ∞)) Lösung der Integralgleichung
Kandidaten für den Grenzwert.
Es bleibt zu zeigen, dass diese Folge in lp liegt. Dies sehen wir x(t) + Ax(t) = f (t)
mithilfe der Dreiecksungleichung in l p , d. h. der Minkowski- mit dem oben definierte Volterra-Operator A und der stetigen
Ungleichung. Es gilt Funktion f . Offensichtlich folgt aus der Gleichung x(0) = a.
  p1 K  p1 Mit dem Hauptsatz der Differenzial- und Integralrechnung

K
p
|xk | = |xk − xkn + xkn |p sind die Funktionen Ax und f differenzierbar. Da x die In-
k=1 k=1 tegralgleichung erfüllt, ist deswegen auch x ∈ C 1 ([0, ∞))
K  p1   p1 und wir erhalten durch Differenzieren

K
t t
≤ |xk − xkn |p + |xkn |p .
k=1 k=1
x  (t) + g(s)x(s) ds = h(s) ds + b .
0 0

Insbesondere gilt x  (0) = b. Wiederum mit dem Hauptsatz


1
Wählen wir nun zu K ∈ N ein n > N, sodass |xk − xkn | ≤ K
für alle k = 1, . . . , K gilt, so folgt folgt, dass x  ∈ C 1 ([0, ∞)) ist. Eine weitere Ableitung führt
auf die Differenzialgleichung
  p1

K
|xk | p
≤ 2 + x N p . x  (t) + g(t) x(t) = h(t) .
k=1
Wir haben gezeigt, dass x ∈ C 2 ([0, ∞)) ist und das An-
Die Abschätzung gilt für jedes K ∈ N, d. h. x ∈ lp . fangswertproblem löst.

Aufgabe 8.3 •• „⇒“ : Ist x ∈ C 2 ([0, ∞)) eine Lösung Aufgabe 8.4 • Mit Folgen (xn ) und (yn ) in lp ist Li-
des Anfangswertproblems, so folgt durch Integration nearität, d. h. R((λxn + μyn )) = λR((xn )) + μR((yn )), ana-
t t log für L, gewährleistet. Weiter folgt aus
x  (t) + g(s)x(s) ds = h(s) ds + b , ∞

0 0
Rxp = |xn−1 |p = xp ,
wobei sich die Integrationskonstante aus der Anfangsbedin- n=2
gung x  (0) = b ergibt.
dass R beschränkt ist mit der Operatornorm R = 1.
Eine weitere Integration führt zusammen mit der Bedingung
Für den Operator L gilt
x(0) = a auf
t τ t τ ∞


x(t)+ g(s)x(s) ds dτ = h(s) ds dτ +bt +a . Lxp = |xn+1 |p ≤ |xn |p = xp .
0 0 0 0 n=1 n=1
64 Lösungswege zu Kapitel 8

Somit ist auch L beschränkt mit L ≤ 1. Wählen wir eine Wir wissen bereits, dass durch f ∞ = supx ∈ |f (x)| eine
Folge x̂ ∈ lp \{0} mit x̂1 = 0, so ist Lx̂ = x̂ und es folgt Norm zu den beschränkten Funktionen auf  gegeben ist.
L = supx∈X\{0} Lx Lx̂
x ≥ x = 1. Wir haben gezeigt,
Außerdem ist durch
dass L = 1 ist. |(f (x)) − f (y)|
|f |α = sup
Aus Rx = 0 ergibt sich direkt xn = 0 für jedes n ∈ N. Also x  =y |x − y|α
ist R injektiv. Da etwa die Folge y = (1, 0, 0, . . . ) ∈ lp sich
nicht durch Rx für eine Folge x = (xn ) beschreiben lässt, ist eine Halbnorm gegeben, d. h., | . |α erfüllt alle Normeigen-
R nicht surjektiv. schaften mit Ausnahme der Definitheit. Wir zeigen dazu:

Für den Operator L lässt sich zur Gleichung Lx = y durch Es gilt für f ∈ C 0,α () stets |f |α ≥ 0, d. h., die Abbil-
die Folge (xn ) = (1, y1 , y2 , . . . ) ∈ lp zu jeder Folge y ∈ lp dung ist positiv,
ein Urbild finden. Somit ist L surjektiv. Da aber alle Folgen weiterhin ist | · |α homogen wegen |λf |α = |λ| |f |α für
der Form x = (λ, 0, 0, . . . ) durch Lx = 0 auf die Nullfolge λ ∈ K,
abgebildet werden, ist L nicht injektiv. und es gilt die Dreiecksungleichung

Bemerkung: Man beachte, dass LR = I gilt, aber die einzel- |((f + g)(x)) − (f + g)(y)|
|f + g|α = sup
nen Operatoren nicht invertierbar sind. Insbesondere sehen x  =y |x − y|α
wir an dem Beispiel, dass im Gegensatz zum endlich dimen- |f (x)) − f (y)| + |g(x)) − g(y)|
≤ sup
sionalen Fall die Injektivität/Surjektivität im Allgemeinen x  =y |x − y|α
nicht Surjektivität/Injektivität impliziert.
≤ |f |α + |g|α .
Aufgabe 8.5 • zu (a): Aus Lx = 0 folgt x = Betrachten wir nun die Summe aus Norm und Halbnorm, so
Lx = 0, d. h., L ist injektiv. Für den inversen Operator folgt auch die Definitheit und wir erhalten, dass durch
L−1 : L(X) → X gilt, wenn y = Lx ∈ L(X) ist, dass
f 0,α = f ∞ + |f |α
L−1 y = L−1 Lx = x = Lx = y .
eine Norm gegeben ist.
Somit ist auch L−1 : L(X) → X isometrisch. Nun nehmen wir zusätzlich an, dass  kompakt ist. Sei (fn )
zu (b): Sei (xn ) eine Folge in X für die eine Cauchy-Folge in C 0,α (). Dann gilt

Lxn → y ∈ Y , n→∞ fn − fm ∞ ≤ fn − fm 0,α → 0

für n, m → ∞. Somit ist (fn ) eine Cauchy-Folge in den


gilt. Es folgt für n, m ∈ N
stetigen Funktionen und es gibt eine stetige Funktion f ∈
C() mit
xn − xm  = L(xn − xm )
fn − f ∞ → 0, n → ∞ .
≤ Lxn − y + y − Lxm 
−→ 0 , n, m → ∞ . Sei nun weiterhin ε > 0 vorgegeben, so gibt es N ∈ N mit
fn − fm 0,α ≤ ε für alle n, m ≥ N . Insbesondere gilt
Damit ist (xn ) eine Cauchy-Folge in X, und da X vollständig
ist, gibt es x ∈ X mit xn → x, n → ∞. |(fn (x) − fm (x)) − (fn (y) − fm (y))| ≤ ε|x − y|α

Aus für x, y ∈  und n, m ≥ N. Da |fm (x) − f (x)| → 0,


m → ∞, gleichmäßig für alle x ∈  konvergiert, folgt
Lx − y ≤ L(x − xn ) + Lxn − y aus der letzten Ungleichung mit Übergang zum Grenzwert
= x − xn  + Lxn − y m→∞
−→ 0 , n→∞ |(fn (x) − f (x)) − (fn (y) − f (y))| ≤ ε|x − y|α

folgt y = L(x) ∈ L(X). Also ist die Menge L(X) abge- für x, y ∈  und n ≥ N. Also ist fn − f ∈ C 0,α (). Damit
schlossen. ist auch f ∈ C 0,α () und es gilt die Konvergenz

fn − f 0,α → 0, n → ∞.
Rechenaufgaben
Bemerkungen: Im Fall α = 1 werden die Funktionen lip-
Aufgabe 8.6 •• Die Vektorraum-Eigenschaften der
schitzstetig genannt (siehe Band 1, Abschnitt 9.3).
Menge C 0,α () sind aufgrund der Dreiecksungleichung
für den Betrag offensichtlich. Es bleiben die Eigenschaften Der Raum C 0,α () ist zwischen den stetigen und den
einer Norm zu prüfen. stetig differenzierbaren Funktionen angesiedelt. So ist
Lösungswege zu Kapitel 8 65

g : (− 21 , 21 ) → R mit g(t) = 1/ ln(|t|) für t  = 0 und Aufgabe 8.8 • Zweimaliges Integrieren der Differen-
g(0) = 0 stetig, aber nicht hölderstetig.
√ Andererseits ist etwa zialgleichung und vertauschen der Integrationsreihenfolge
f : (− 21 , 21 ) → R mit f (t) = |t| auf (−1/2, 1/2) hölder- führt auf
stetig mit α = 1/2, aber f ist nicht differenzierbar. t τ t
x(t) = x 2 (s) ds dτ + (τ + α) dτ + β
Aufgabe 8.7 • (a) Es handelt sich um eine Volterra- 0 0 0
t
Integralgleichung 2. Art mit stetigem Kern, d. h., die IGL ist 1
= (t − s)x 2 (s) ds + t 2 + αt + β.
eindeutig lösbar. Ist x ∈ C[0, 1] Lösung der Integralglei- 0 2
chung, so ist
t
x(t) = (t − s) x(s) ds + 1 ∈ C 1 [0, a] Aus x(0) = 0 folgt β = 0 und aus x(1) = 0 ergibt sich
0 1
1
und Differenzieren führt auf α=− − (1 − s)x 2 (s) ds .
t 2 0
x  (t) = 0 + x(s) ds ∈ C 1 [0, a] .
0 Also erhalten wir durch Einsetzen von α
t
Eine weitere Ableitung ergibt die Differenzialgleichung
x(t) = s(t − 1)x 2 (s) ds
x  (t) = x(t) . 0
1 1
+ t (s − 1)x 2 (s) ds + t (t − 1)
Die allgemeine Lösung dieser gewöhnlichen Differenzial- t 2
gleichung ist x(t) = Aet +Be−t (siehe Beispiel auf Seite 28).
Aus den Anfangsbedingungen x(0) = 1, x  (0) = 0 folgt Mit der Definition

1 t (t − 1)s, 0≤s≤t ≤1
x(t) = (e + e−t ) = cosh t. k(t, s) =
2 (s − 1)t, 0 ≤ t ≤ s ≤ 1.

(b) Zunächst erhalten wir induktiv aus ergibt sich eine nichtlineare Integralgleichung zweiter Art
t 1
An y(t) = k(t, s)An−1 y(s) ds x(t) − k(t, s) x 2 (s) ds =
1
t (t − 1) ,
0 2
t s 0
= k(t, s) kn−1 (s, σ )y(σ ) dσ ds d. h., dieses Randwertproblem ist beschrieben durch eine
0 0
t t Operatorgleichung F (x) = y in den stetigen Funktionen
= k(t, s)kn−1 (s, σ ) ds y(σ ) dσ mit y(t) = 21 t (t − 1) und dem nichtlinearen Operator
0 σ
   F : C([0, 1]) → C([0, 1]), der durch
= kn (t, σ ) 1
die angegebene Darstellung der Neumann’schen Reihe mit- F (x)(t) = x(t) − k(t, s) x 2 (s) ds
0
hilfe der iterierten Kerne.
definiert ist.
Eine weitere Induktion beweist aus
t
kn+1 (t, s) = k(t, τ ) kn (τ, s) ds Aufgabe 8.9 ••
s
Zu y = (yn ) ∈ l∞ definieren wir ly : l1 → C durch
t(τ − s)2n−1
= (t − τ ) dτ ∞
s (2n − 1)!
t ly (x) = xn yn , für x = (xn ) ∈ l1 ,
(τ − s)2n
= 0+ dτ n=1
s (2n)!
(t − s)2n+1 Wegen
=
(2n + 1)! ∞


für den Kern k(t, s) = t − s die angegebene Formel. |ly (x)| ≤ |xn yn | ≤ sup |yn | |xn | = y∞ x1
n=1 n∈N n=1
Damit erhalten wir
t ist ly beschränkt durch ly  ≤ y∞ .
t 2n
kn (t, s) 1 ds = . Wir können die Abbildung J : l∞ → (l1 ) mit J y =
0 (2n)! ly definieren. Nach Konstruktion ist J linear und es gilt
Folglich ist J y ≤ y∞ . Andererseits gilt

t 2n |ly (x)|
x(t) = 1 + = cosh t. J y = sup ≥ sup |ly (ek )| = sup |yk )| = y∞
(2n)!
n=1 x∈l \{0}
1 x 1 k∈N k∈N
66 Lösungswege zu Kapitel 8

für ek = (0, . . . , 0, 1, 0, . . . ) ∈ l1 mit ekk = 1 an der k-ten (l1 ) separabel ist. Mit dem Lemma auf Seite 306 ist l∞
Stelle. Also ist Jy = y∞ und insbesondere injektiv. separabel im Widerspruch zum Beispiel auf Seite 306.
Es bleibt zu zeigen, dass J surjektiv ist. Also ist l1 nicht reflexiv.
Dazu sei l ∈ (l1 ) und wir definieren yk = l(ek ) mit den
oben angegebenen Folgen ek . Aus |yk | ≤ lek 1 = l
ergibt sich für die so definierte Folge y= (yk ) ∈ l∞ . Beweisaufgaben
Weiter konvergiert zu x ∈ l1 die Folge N k
k=1 xk e in l
1
Aufgabe 8.10 • Definieren wir die Operatoren A−1 =
gegen x und mit der Stetigkeit von l ist B(AB) und B = A(AB)−1 wie im Text angegeben, so
−1 −1

N  gilt offensichtlich

k
l(x) = lim l xk e AA−1 = A B(AB)−1 = I
N→∞
k=1

N und mit der Vertauschbarkeit der Operatoren auch
= lim xk l(ek )
N→∞
k=1 BB −1 = B A(AB)−1 = (AB) (AB)−1 = I .
N
= lim xk yk = ly (x) . Weiter erhalten wir aus der letzten Gleichung
N→∞
k=1
A−1 AB = B(AB)−1 AB = B
Damit ist J surjektiv und wir haben insgesamt gezeigt,
dass J einen Normisomorphismus zwischen l∞ und dem und es folgt
Dualraum (l1 ) liefert.
A−1 A = A−1 A (BB −1 ) = (A−1 AB)B −1 = BB −1 = I .
Für die zweite Aussage betrachten wir wiederum, diesmal
zu y ∈ l1 , das lineare Funktional Entsprechend ergibt sich mit der ersten Identität

B −1 AB = A(AB)−1 AB = A
ly (x) = xn yn , für x = (xn ) ∈ c0 .
n=1
und es ist
Offensichtlich gilt |ly (x)| ≤ x∞ y1 , d. h., das Funk-
B −1 B = B −1 B(AA−1 ) = (B −1 AB)A−1 = AA−1 = I .
tional ist beschränkt. Wir erhalten durch Jy = ly eine
lineare Abbildung J : l1 → (c0 ) mit Jy ≤ y1 . Mit den vier Gleichungen ist die Invertierbarkeit sowohl von
Weiter gilt A als auch von B gezeigt.
|ly (x)|
Jy = sup ≥ sup |ly (bk )| Aufgabe 8.11 • Angenommen es gilt AB − BA = I .
x∈c0 \{0} x∞ k∈N
Dies impliziert induktiv AB n − B n A = nB n−1 ; denn mit

k AB − BA = I haben wir einen Induktionsanfang und aus
= sup |yk | = y1
k∈N j =1 AB n+1 − B n+1 A = (AB n − B n A)B + B n (AB − BA)
= nB n−1 B + bn = (n + 1)B n
für bk = (1, . . . , 1, 1, 0, . . . ) ∈ c0 mit bjk = 1 für j ≤ k
und bjk = 0 für j > k. Also ist Jy = y1 und insbe- ergibt sich der Induktionsschritt.
sondere ist J injektiv.
Da die beiden Operatoren beschränkt sind, folgt aus dieser
Es bleibt auch in diesem Beispiel noch zu zeigen, dass
Identität mithilfe der Dreiecksungleichung für hinreichend
J surjektiv ist. Dazu sei l ∈ (c0 ) und wir definieren die
große n ∈ N der Widerspruch
Folge (yk ) durch yk = l(ek ), wobei die Folgen ek wie im
ersten Teil definiert sind, d. h., es gilt Jy = l, wenn wir nB n−1  ≤ 2ABB n−1 
noch beweisen, dass y ∈ l1 gilt. Dazu betrachte man
bzw. n ≤ 2AB.

N
N
k N N
|yk | = l(sgn(yk )e ) = l(b ) ≤ l b ∞ = l
Aufgabe 8.12 •• Ohne Einschränkung interpretieren
k=1 k=1
wir X ⊆ X̃ als eine dichte Teilmenge von X̃ und entspre-
für die Nullfolge bN ∈ c0 mit bkN = sgn(yk ) für k ≤ N chend Y ⊆ Ỹ .
und bkN = 0 für k > N . Diese Abschätzung gilt für jedes Da X dicht liegt in X̃, existiert zu einem x ∈ X̃ eine Folge
N ∈ N und es folgt y ∈ l1 . Die Abbildung J liefert somit (xn ) ⊆ X mit xn → x für n → ∞. Insbesondere ist (xn )
auch in diesem Fall einen Normisomorphismus. eine Cauchy-Folge in X ⊆ X̃. Da A auf X beschränkt ist,
Angenommen l1 ist reflexiv, d. h. (l1 ) ∼
= l1 . Da der l1 se- folgt (Axn ) ⊆ Y ist eine Cauchy-Folge in Y . Also existiert
parabel ist, folgt mit der ersten Teilaufgabe, dass (l∞ ) ∼= y ∈ Ỹ mit Axn → y, n → ∞.
Lösungswege zu Kapitel 8 67

Sei (zn ) ⊆ X eine weitere Folge mit zn → x, n → ∞. Dann Auf dem eindimensionalen normierten Raum (X̃, .) defi-
folgt nieren wir die lineare Abbildung

Axn − Azn  ≤ Axn − zn  ϕ̃ : x̃ → ϕ(x  ), x  ∈ x̃.


≤ A(xn − x + x − zn ) → 0 Dann ist ϕ̃ beschränkt, da es eine lineare Abbildung auf einem
für n → 0. Also gilt lim Axn = lim Azn . endlich dimensionalen Raum ist, und es folgt
n→∞ n→∞
|ϕ(x)| = |ϕ̃(x̃)| ≤ ϕ̃ x̃ ≤ ϕ̃ xX ,
Somit lässt sich der Operator à : X̃ → Ỹ durch
d. h., ϕ ist beschränkt und damit stetig.
Ãx = lim Axn
n→∞
(ii)⇒(iii): Annahme: Sei Kern(ϕ) dicht in X und abgeschlos-
definieren, da die Definition von der Wahl der Folge (xn ) sen. Dann ist X = Kern(ϕ) = Kern(ϕ), also ϕ = 0 im
unabhängig ist. Widerspruch zur Voraussetzung.

Weiter gilt für x ∈ X̃ mit der Definition von à (iii)⇒(ii): Es gilt

Ãx ≤ Ãx − Ãxn  + Ãxn  Kern(ϕ) ⊆ Kernϕ  X.


= Ãx − Axn  + Axn  und für die Quotientenräume ergibt sich
≤ Ãx − Axn  + A xn  → A x
=
für n → ∞, da die Norm stetig ist. Also ist à beschränkt mit {0} = Kern(ϕ) Kern(ϕ)
à ≤ A. =
⊆ Kernϕ Kern(ϕ)
Für x ∈ X folgt =
 X Kern(ϕ) ) Bild(ϕ) = R.
Ax = Ãx ≤ Ã x,
Vergleichen wir die Dimensionen
 dieser
> Räume,
 folgt we-
d. h. A ≤ Ã, und insgesamt erhalten wir A = Ã. gen der echten Inklusion dim Kernϕ Kernϕ = 0, also
Es bleibt die Eindeutigkeit zu zeigen. Sind Ã, B̃ zwei stetige Kernϕ = Kernϕ.
Operatoren auf X̃ mit Ãx = Ax = B̃x für x ∈ X. Für x ∈ X̃ (b) Setze ϕ(x) = x  (0). Dann ist Kern(ϕ) = {x ∈
und (xn ) ⊆ X mit xn → x für n → ∞ folgt X : x  (0) = 0}. Wir zeigen nun, dass ϕ unstetig ist, was
Ãx = lim Axn = lim B̃xn = B̃x . nach Teil a) impliziert, dass der Kern dicht liegt in X.
n→∞ n→∞
Definiere xn (x) = n1 sin nt. So ist xn (t) = − cos nt, d. h.
Insgesamt haben wir gezeigt, dass es genau eine Vervollstän- xn (0) = −1, aber xn ∞ → 0, n → ∞. Also gilt
digung von A auf den gesamten Raum X̃ gibt.
−1 = lim ϕ(xn ) = ϕ( lim xn ) = 0,
n→∞ n→∞
Aufgabe 8.13 ••• (i)⇒(ii): Kern(ϕ) = ϕ −1 ({0})
ist ab-
geschlossen, denn aus xn ∈ Kern(ϕ) mit xn → x ∈ X, ϕ ist unstetig.
n → ∞ folgt ϕ(x) = lim ϕ(xn ) = 0. Also x ∈ Kern(ϕ).
n→∞ Aufgabe 8.14 •• Ist A beschränkt, so ist A stetig und
Allgemein gilt bei stetigen Funktionen, dass das Urbild ab-
es ergibt sich für eine Folge (xn , yn ) in G mit xn → x ∈ X
geschlossener Mengen abgeschlossen ist.
und yn = Axn
(ii)⇒(i): Sei X̃ = X/Kern(ϕ) = {x̃ ∈ P (X) : x, y ∈ x̃ ⇔
ϕ(x) = ϕ(y)} der Quotient. Dann gilt X̃ ) Bild(ϕ) = R; lim yn = lim Axn = Ax ,
n→∞ n→∞
denn ϕ(X̃) ⊆ R ist ein Unterraum, und ϕ  = 0.
d. h. lim(xn , yn ) = (x, Ax) ∈ G. Also ist G abgeschlossen.
Beh.: Auf X̃ ist durch
Sei nun andererseits der Graph G abgeschlossen. Dann ist G
x̃ = inf xX als abgeschlossener Unterraum von X × Y insbesondere ein
x∈X̃ Banachraum. Wir definieren B : G → X durch
eine Norm definiert. B(x, Ax) = x .
Bew.: Offensichtlich ist . positiv, homogen und es gilt die
B : G → X ist ein bijektiver linearer beschränkter Operator.
Dreiecksungleichung.
Nach dem Satz von der stetigen Inversen ist B −1 stetig. Somit
Sei x̃ = 0. Dann existiert eine Folge (xn )n ⊆ x̃ mit folgt aus xn → x, n → ∞, die Konvergenz (xn , Axn ) →
limn→∞ xn X = 0, d. h. xn → 0 für n → ∞. Da (x, Ax) und insbesondere ergibt sich Axn → Ax für n →
x̃ = x  + Kern(ϕ) mit Repräsentant x  ∈ x̃ nach Voraus- ∞. Dies bedeutet, dass A : X → Y ein beschränkter Operator
setzung abgeschlossen ist, folgt 0 ∈ x̃ bzw. x̃ = Kern(ϕ). ist.
68 Lösungswege zu Kapitel 8

Aufgabe 8.15 •• Ist M beschränkt, so ist auch die im Widerspruch zu limn→∞ l(xn ) = l(x). Also ist der
Menge l(M) beschränkt, da l ∈ X ein beschränktes lineares schwache Grenzwert x ∈ A. Diese Aussage wird in der Li-
Funktional ist und somit teratur auch als Satz von Mazur bezeichnet.

|l(x)| ≤ l x ≤ cl


Aufgabe 8.18 ••• Angenommen X ist reflexiv, d. h., die
für x ∈ X gilt, wenn c > 0 Schranke zu M ist. Abbildung x → Jx ∈ X  mit Jx (l) = l(x) für l ∈ X  ist
normisomorph und wir können X  = {Jx | x ∈ X} setzen.
„⇐“ Es sei die Bildmenge l(M) ∈ K beschränkt für jedes Weiterhin betrachten wir l → Ll ∈ X mit
l ∈ X  , d. h., zu l ∈ X  gibt es eine Konstante cl > 0 mit
|l(x)| ≤ c für jedes x ∈ M. Definieren wir weiter zu x ∈ M Ll (Jx ) = Jx (l) = l(x) .
den linearen Operator Jx : X  → K durch Jx (l) = l(x),
Es gilt
so ist Jx punktweise beschränkt durch |Jx (l)| = |l(x)| ≤
cl für jedes x ∈ M. Nach dem Prinzip der gleichmäßigen Ll  = sup |Jx (l)| = sup |l(x)| = l ,
Beschränktheit folgt, dass eine Konstante c ≥ 0 existiert mit Jx =1 x=1
Jx  ≤ c für alle x ∈ M. Mit dem Resultat der Selbstfrage
d. h., die Abbildung l → Ll ∈ X  ist isometrisch und des-
auf Seite 300 ergibt sich für x ∈ M
wegen insbesondere injektiv.
|l(x)| |Jx (l)| Die Abbildung ist darüber hinaus surjektiv; denn ist L̃ ∈ X
x = sup = sup ≤ c,
l∈X  \{0} l l∈X  \{0} l so lässt sich durch

d. h., M ist beschränkt. l(x) = L̃(Jx )


Beachten Sie: aus diesem Resultat folgt insbesondere, dass ein lineares Funktional l ∈ X definieren und mit
schwach konvergente Folgen in einem normierten Raum X
stets beschränkt sind; denn ist (xn ) schwach konvergent, so |l(x)| = |L̃(Jx )| ≤ L̃Jx  = L̃x
ist insbesondere {l(xn ) | n ∈ N} ⊆ K zu einem l ∈ X eine
ist l beschränkt. Also ist X reflexiv.
beschränkte Menge.
Für die zweite Implikation nehmen wir an, X sei nicht refle-
Aufgabe 8.16 • Da A abgeschlossen ist, ist X\A offen xiv, dann ist U = {Jx | x ∈ X} ⊆ X ein echter Unterraum.
und es gibt eine offene konvexe Umgebung U ⊆ X\A um Außerdem ist U abgeschlossen: denn sei Jxn → J˜ ∈ X 
x ∈ U . Da U ◦ = ∅ und U ◦ ∩ A = ∅ gilt, folgt aus dem konvergent, so ist Jxn insbesondere eine Cauchy-Folge. Mit
Trennungssatz von Eidelheit die Existenz von l ∈ X  mit
xn − xm  = Jxn − Jxm  → 0, für n, m → ∞
l(y) ≤ γ < l(x)
ist auch (xn ) Cauchy-Folge in X. Mit der Voraussetzung,
für y ∈ A . dass X Banachraum ist, gilt xn → x ∈ X und wir erhalten
J˜ = Jx ∈ U .
Bemerkung: Diesen Trennungssatz haben wir im Kapi-
tel 24.3 des Band 1 in der Lupe zum starken Dualitätssatz an- Insgesamt ist U ein abgeschlossener Unterraum, der nicht
gesprochen, um eine zweite, allgemeinere Beweisidee zum dicht in X ist. Nach der Folgerung auf Seite 300 gibt es ein
Satz vorzustellen. Der Zugang über Trennungssätze liefert Funktional L ∈ X \{0} mit
die entscheidende Idee zum Beweis eines starken Dualitäts- L̃(Jx ) = 0 für jedes x ∈ X .
satzes in der konvexen Optimierung.
Wäre X nun reflexiv, so gäbe es ein l ∈ X mit L̃ = Ll , d. h.
Aufgabe 8.17 • Angenommen es gilt x  ∈ A. Dann
L̃(J ) = Ll (J ) = J (l) für J ∈ X , und es folgt
existiert nach der vorherigen Aufgabe ein trennendes Funk-
tional l ∈ X und γ ∈ R mit 0 = Ll (Jx ) = Jx (l) = l(x)

l(x) > γ ≥ l(xn ) für alle x ∈ X, d. h. l = 0 im Widerspruch zu Ll = 0 .


Kapitel 9 Aufgabe 9.6 • Es seien X, Y Banachräume und
T : X → Y ein Operator, der sich darstellen lässt durch


Aufgaben Tx = λj lj (x) yj für x ∈ X
j =1

Verständnisfragen mit einer Folge (λj ) ∈ l1 , Funktionalen lj ∈ X mit lj  = 1


Aufgabe 9.1 • Ist der Operator T : C([0, 1]) → und Elementen yj ∈ Y mit yj  = 1. Zeigen Sie, dass T
C([0, 1]) mit kompakt ist.

T x(t) = tx(t) , t ∈ [0, 1] Aufgabe 9.7 ••• Bestimmen Sie in Abhängigkeit von
λ die Riesz’sche Zahl des Integraloperators λ1 A in
kompakt? L = (λI − A) : C([−1, 1]) → C([−1, 1]) mit
1
Aufgabe 9.2 •• Zeigen Sie mithilfe des Satzes von Lx(t) = λx(t) − (1 − |t − s|)x(s) ds .
Arzela-Ascoli, dass durch −1

b
Ax(t) = k(t, s)x(s) ds Aufgabe 9.8 •• Finden Sie zu dem homogenen Rand-
a wertproblem
mit stetigem Kern k ∈ C([a, b] × [a, b]) ein kompakter Ope- (px  ) − qx = f mit x(0) = x(1) = 0
rator A : C([a, b]) → C([a, b]) gegeben ist.
mit p ∈ C 1 ([0, 1]), p > 0 und q, f ∈ C([0, 1]) eine äqui-
Aufgabe 9.3 • Durch valente Fredholm’sche
t 1 Integralgleichung, wobei vorausge-
setzt ist, dass 0 p(s) ds existiert. Formulieren Sie die Fred-
2π holm’sche Alternative für das Randwertproblem.
f, g = g(0) f (t) dt
0

ist eine Bilinearform ·, · : C([0, 2π]) × C([0, 2π]) → R Beweisaufgaben


definiert. Aufgabe 9.9 • Zeigen Sie: Wenn X, Y Banachräume
(a) Bestimmen Sie ein f ∈ C([0, 2π]) mit f(0)=1 und sind und A ∈ K(X, Y ) ein kompakter Operator, der offen ist,
f, g = 0 für alle g ∈ C([0, 2π ]). so hat Y endliche Dimension.
(b) Zeigen
 Sie, dass die linearen
 Operatoren A, B ∈
L C([0, 2π]), C([0, 2π]) mit Aufgabe 9.10 • Sei X normierter Raum, A : X → X
kompakt und r die Riesz’sche Zahl von L = I − A. Dann ist
Aϕ(t) = ϕ(0)f (t) und Bϕ(t) = 0 durch die direkte Summe
X = N (Lr ) ⊕ Lr (X)
kompakt und bzgl. ·, · adjungiert sind.
(c) Warum gilt der 1. Fredholm’sche Satz nicht? ein Projektionsoperator P : X → N (Lr ) definiert. Zeigen
Sie, dass L − P bijektiv ist.
Aufgabe 9.4 • Beweisen Sie, dass der adjungierte
Operator A∗ : Y  → X  eines invertierbaren Operators A ∈ Aufgabe 9.11 •• Sind X, Y Banachräume, T ∈ L(Y,X),
L(X, Y ) auf normierten Räumen X, Y auch invertierbar ist. A11 ∈ K(X, X), A12 ∈ K(Y, X) und A22 ∈ K(Y, Y ). Weiter
besitze A11 die Riesz-Zahl r = 1 und A22 die Riesz-Zahl
r = 0. Zeigen Sie, dass die Riesztheorie auf den Operator
Rechenaufgaben E − A : X × Y → X × Y mit
   
I T A11 A12
Aufgabe 9.5 •• In Aufgabe 8.13 sind die normierten E= und A =
0 I 0 A22
Räume C 0,α (G) der Hölder-stetigen Funktionen eingeführt.
Man zeige, dass, wenn 0 < α < β ≤ 1 und eine kompakte angewendet werden kann, und bestimmen Sie die Riesz-Zahl
Menge G ⊆ Rd gegeben sind, die Einbettungsoperatoren des Operators A ∈ K(X × Y, X × Y ).
Iβ : C 0,β (G) → C(G)
Aufgabe 9.12 •• Sei (X, X, ·, ·) ein Dualsystem und
A ∈ K(X,X) mit adjungiertem Operator A∗ ∈ K(X, X). Sei
und
weiter N (I −A)  = {0}. Zeigen Sie, dass die Operatoren I −A
Iα,β : C 0,β (G) → C 0,α (G)
und I − A∗ genau dann die Riesz’sche Zahl r = 1 haben,
kompakt sind. wenn für je zwei Basen {ϕ1 , . . . ϕm } bzw. {ψ1 , . . . ψm } von
M. Brokate et al., Arbeitsbuch Grundwissen Mathematikstudium – Höhere Analysis, Numerik und
Stochastik, DOI 10.1007/978-3-642-54946-5_8, © Springer-Verlag Berlin Heidelberg 2016
70 Lösungen zu Kapitel 9

N (I −A) bzw. N (I −A∗ ) die Gram’sche Matrix T ∈ Cm×m Aufgabe 9.7 ••• Beachten Sie, dass A = A∗ bzgl. des
mit Tij = ϕi , ψj  für i, j = 1, . . . , m regulär ist. L2 -Skalarprodukts symmetrisch ist, und unterscheiden Sie
die Fälle Im(λ) = 0 und λ ∈ R. Überlegen Sie sich im
Aufgabe 9.13 • Sind X, Y reflexive normierte Räume zweiten Fall zunächst, dass r ≤ 1 gilt, und betrachten Sie
und ist A ∈ L(X, Y ), dann gilt (A∗ )∗ = A, wenn wir die dann die beiden Situationen λ = 0 und λ = 0.
Bidualräume mit den Räumen identifizieren.
Aufgabe 9.8 •• Zweimaliges Integrieren und eine Ver-
Aufgabe 9.14 ••• Man nennt einen Operator R ∈ tauschung der Integrationsreihenfolge führt auf die gesuchte
L(Y, X) einen Regularisierer zum Operator L ∈ L(X, Y ) Integralgleichung, die mit der Fredholm-Theorie analysiert
auf normierten Räumen X, Y , wenn es kompakte Operato- werden kann.
ren A1 : X → X bzw. A2 : Y → Y gibt, sodass
RL = I − A1 und LR = I − A2 Beweisaufgaben
gilt und somit die Fredholm-Theorie genutzt werden kann. Aufgabe 9.9 • Beachten Sie, dass Y genau dann end-
Zeigen Sie mit y ∈ Y die folgenden beiden Aussagen. liche Dimension hat, wenn es ε > 0 gibt, sodass
– Ist R injektiv, so gilt: x ∈ X ist genau dann Lösung zu
Lx = y, wenn x Lösung zu (I − A1 )x = Ry ist. B(0, ε) = {y ∈ Y | y ≤ ε} ⊆ Y
– Ist R surjektiv, so gilt: x ∈ X ist genau dann Lösung zu
kompakt ist.
Lx = y, wenn z mit x = Rz Lösung zu (I − A2 )z = y
ist. Aufgabe 9.10 • Man beachte, dass P nach dem
Finden Sie für den Volterra-Operator L : C([0, 1]) → Lemma auf Seite 330 ein kompakter Operator ist.
C,1 ([0, 1]) = {y ∈ C 1 ([0, 1]) | y(0) = 0} mit
t Aufgabe 9.11 •• Zeigen Sie zunächst, dass E invertier-
Lx(t) = k(t, s) x(s) ds bar ist.
0

und differenzierbarer Kernfunktion k ∈ C 1 ([0, 1]×[0, 1]) Aufgabe 9.12 •• Für die eine Richtung des Beweises
mit k(t, t) = 1 für t ∈ [0, 1] einen Regularisierer. zeige man, dass T injektiv ist und für die Rückrichtung führe
man die Annahme r > 1 zum Widerspruch.

Aufgabe 9.13 • Man betrachte direkt die Definition


Hinweise des Bidualraums und zugehörige adjungierte Operatoren.

Verständnisfragen Aufgabe 9.14 ••• Im zweiten Teil betrachte man den


Ableitungsoperator D ∈ L(C,1 ([0, 1]), C([0, 1])) mit
Aufgabe 9.1 • Man konstruiere eine beschränkte Dy = y  . Beachten Sie, dass gezeigt werden muss, dass
Folge stetiger Funktionen, die punktweise, auch nach Multi- die auftretenden Integraloperatoren auf den entsprechenden
plikation mit t, gegen eine unstetige Funktion konvergiert. Räumen kompakt sind.
Aufgabe 9.2 •• Der Satz von Arzela-Ascoli lässt sich
auf eine Teilmenge A(M) ⊆ C([a, b]) zu einer beschränkten
Menge M ⊆ C([a, b]) anwenden. Lösungen
Aufgabe 9.3 • In Teil (b) betrachte man die Dimensio-
nen der Bildräume und zum Beantworten von Teil (c) ist das Verständnisfragen
Beispiel aus Teil (a) nützlich.
Aufgabe 9.1 • –
Aufgabe 9.4 • Man verwende die Folgerung (b) auf
Seite 319. Aufgabe 9.2 •• –

Aufgabe 9.3 • –
Rechenaufgaben
Aufgabe 9.5 •• Verwenden Sie den Satz von Arzela- Aufgabe 9.4 • –
Ascoli.
Rechenaufgaben
Aufgabe 9.6 • Überlegen Sie sich, dass die Partial-
summen der Reihe eine Folge kompakter Operatoren liefern, Aufgabe 9.5 •• –
die gegen den Operator T in der Operatornorm konvergiert.
Warum sind Banachräume vorausgesetzt? Aufgabe 9.6 • –
Lösungswege zu Kapitel 9 71

Aufgabe 9.7 ••• – für t, t˜, s ∈ [a, b] und |t − t˜| ≤ δ. Mit dieser Abschätzung
ergibt sich
Aufgabe 9.8 •• –  
   b  
Ax(t) − Ax(t˜) =  
k(t, s) − k(t˜, s) x(s) ds 
 a 
Beweisaufgaben
≤ c|b − a| max |k(t, s) − k(t˜, s)| ≤ ε
Aufgabe 9.9 • – t,s∈[a,b]

für alle x ∈ M und t, t˜ ∈ [a, b] mit |t − t˜| ≤ δ . Wir haben


Aufgabe 9.10 • –
gezeigt, dass A(M) gleichgradig stetig ist. Nach dem Satz
von Arzela-Ascoli ist A(M) ⊆ C([a, b]) kompakt, womit
Aufgabe 9.11 •• Der Operator hat die Riesz-Zahl
bewiesen ist, dass der Operator A : C([a, b]) → C([a, b])
r = 1.
kompakt ist.
Aufgabe 9.12 •• – Bemerkung: Man vergleiche diesen Beweis mit dem explizit
gegebenen ohne Anwendung des Satzes von Arzela-Ascoli
Aufgabe 9.13 • – im Beispiel auf Seite 321.

Aufgabe 9.14 ••• – Aufgabe 9.3 • zu (a): Man wähle etwa f (x) = cos x,
x ∈ [0, 1].
zu (b): Die Operatoren A, B besitzen endlich dimensionale
Bildräume und sind bzgl. || · ||∞ beschränkt. Also sind die
Lösungswege Operatoren kompakt. Aus

Verständnisfragen Aϕ, ψ = ϕ(0)f, ψ = ϕ(0)f, ψ


= 0 = ϕ, 0 = ϕ, Bψ
Aufgabe 9.1 • Wir definieren eine beschränkte Folge
stetiger Funktionen durch ergibt sich, dass die Operatoren adjungiert zueinander sind.

⎪ 1 zu (c): Ist ϕ ∈ N (I − A), so gilt ϕ(t) = ϕ(0)f (t), d. h.

⎪ 0, 0≤t ≤1−

⎪ n − 1 ϕ ∈ span{f }. Ist umgekehrt ϕ ∈ span{f }, also ϕ = λf , so

⎨ 1 1 1 1 folgt ϕ(0) = λf (0) = λ und somit ϕ = ϕ(0)f = Aϕ. Also
xn (t) = t +1− 2 , 1− <t ≤1− ist N (I − A) = span{f }.

⎪ n(n − 1) n n−1 n



⎪ 1
⎩ 1, 1 − ≤ t ≤ 1. Somit ist
n
dim(N (I −A)) = dim(span{f }) = 1 = 0 = dim(N (I −B)) .
Dann gilt
# Der 1. Fredholm’sche Satz gilt in diesem Fall nicht, da die
1 , falls t = 1 Bilinearform degeneriert ist, wie wir mit dem Beispiel aus
lim t xn (t) =
n→∞ 0 , falls t ∈ [0, 1) . Teilaufgabe (a) sehen.

Somit besitzt die Bildfolge T xn keine konvergente Teilfolge Aufgabe 9.4 • Ist A : X → Y invertierbar, so gilt
in den stetigen Funktionen und wir haben gezeigt, dass der IX = A−1 A und IY = AA−1 . Außerdem erhalten wir auf
Multiplikationsoperator nicht kompakt ist. den Dualräumen aus (IX l)(x) = l(IX x) die Darstellung
IX = (IX )∗ = (A−1 A)∗ . Mit der Folgerung auf Seite 329
Aufgabe 9.2 •• Sei M ⊆ C([a, b]) eine beschränkte ergibt sich
Teilmenge, d. h., es gibt c > 0 mit x∞ ≤ c für alle x ∈ M. IX = A∗ (A−1 )∗ .
Damit folgt
Analog folgt IY  = (A−1 )∗ A∗ und wir haben gezeigt, dass
|Ax(t)| ≤ c|b − a| max |k(t, s)| A∗ invertierbar ist mit (A∗ )−1 = (A−1 )∗ .
t,s∈[a,b]

für jedes t ∈ [a, b] und jede Funktion x ∈ C([a, b]). Also Rechenaufgaben
ist die Menge A(M) ⊆ C([a, b]) beschränkt.
Weiter ist k gleichmäßig stetig auf der kompakten Menge Aufgabe 9.5 •• zu (a) Sei U ⊆ C 0,β (G) eine be-
[a, b] × [a, b], d. h., insbesondere zu ε > 0 gibt es δ > 0 mit schränkte Teilmenge, d. h., es existiert C > 0 mit ϕ0,β ≤ C
für alle ϕ ∈ U . Also gelten insbesondere folgende Abschät-
ε zungen
|k(t, s) − k(t˜, s)| ≤
c|b − a| |ϕ(t) − ϕ(s)| ≤ C |t − s|β
72 Lösungswege zu Kapitel 9

und gilt. Es folgt


|ϕ(t)| ≤ C Tn x − Tm xY ≤ εx
für alle t, s ∈ G und für alle ϕ ∈ U . Damit sind die steti- für alle n, m ≥ M, d. h. Tn − Tm  ≤ ε. Somit ist (Tn ) eine
gen Funktionen in U gleichmäßig beschränkt und gleichgra- Cauchy-Folge in K(X, Y ) und es folgt Konvergenz Tn →
dig stetig. Der Satz von Arzela-Ascoli besagt, dass U relativ T ∈ L(X, Y ) mit der in der Aufgabe angegebenen Reihen-
kompakt in C(G) ist. Dies bedeutet, dass der Operator Iβ Darstellung.
beschränkte Mengen auf relativ kompakte Mengen abbildet
–also kompakt ist. Da X, Y Banachräume sind, ist K(X, Y ) ⊆ L(X, Y ) ein ab-
geschlossener Unterraum, und wir folgern, dass der Grenz-
zu (b) Es bleibt zu zeigen, dass U auch relativ kompakt ist wert T ein kompakter Operator ist.
bezüglich der C 0,α Norm. Das erhalten wir aus folgender
Abschätzung: Für s, t ∈ G und ϕ, ψ ∈ U gilt
Aufgabe 9.7 ••• Da k(t, s) = 1 − |t − s| = k(s, t) gilt
  A = A∗ .
(ϕ(t) − ϕ(s)) − (ψ(t) − ψ(s))
 α 1. Beh.: Für λ ∈ C mit Im(λ) = 0 ist r = 0
= (ϕ(t) − ϕ(s)) − (ψ(t) − ψ(s)) β
 1− α Bew.: Sei x ∈ N (λ I − A)\{0}, so folgt
· (ϕ(t) − ϕ(s)) − (ψ(t) − ψ(s)) β
 α
≤ |ϕ(t) − ϕ(s)| + |ψ(t) − ψ(s)| β λx2 = Ax, x = x, Ax = λx2 .
 1− α
· |ϕ(t) − ψ(t)| + |ψ(s) − ϕ(s)| β
Also ist λ = λ oder x = 0, d. h. N (λI − A) = {0} für
α
1− βα Im(λ) = 0.
≤ (2C) β |t − s|α (2ϕ − ψ∞ ) .
2. Beh: Es gilt r ≤ 1 für die Riesz’sche Zahl zu λI − A.
Also folgt
Bew.: Sei L = λI − A nicht injektiv (r = 0). Da in diesem
α
1− βα 1− α
ϕ − ψα ≤ ϕ − ψ∞ + (2C) β 2 ϕ − ψ∞ β , Fall λ ∈ R ist, ist auch L = λI − A selbstadjungiert. Sei nun
x ∈ N (L2 ). Es folgt
d. h., jede bzgl. .∞ konvergierende Folge in U konvergiert
auch bzgl. .α . Mit Teilaufgabe (a) ist U auch als Teilmenge 0 = L2 x, x = Lx, Lx = Lx2 .
von C 0,α (G) relativ kompakt.
Also ist x ∈ N (L) und wir erhalten N (L) = N (L2 ) bzw.
Aufgabe 9.6 • Man definiere die linearen und be- r = 1.
schränkten Operatoren Tn : X → Y durch
3. Sei λ ∈ R und x ∈ C([−1, 1]) Lösung der homogenen

n IGL
Tn x = λj lj (x)yj . t
j =1 λx(t) − Ax(t) = λx(t) − (1 − (t − s))x(s) ds
−1
Da Tn (X) ⊆ span{y1 , . . . , yn } ist, haben diese Operatoren 1
jeweils ein endlich dimensionales Bild. Mit der Selbstfrage − (1 + (t − s))x(s) ds
auf Seite 319 sind die Operatoren somit kompakt. t
= 0.
Weiter können wir für m, n ∈ N, m ≤ n, mit der Dreiecks-
ungleichung abschätzen Dann ist x ∈ C 2 ([−1, 1]) und wir erhalten durch differen-
zieren

n
Tn x − Tm xY =  λj lj (x)yj  t 1
j =m+1 λx  (t) − x(s) ds + x(s) ds = 0 (9.7)
−1 t
n
≤ |λj ||lj (x)|yj  und
j =m+1
⎛ ⎞ λx  (t) − 2x(t) = 0. (9.8)

n
≤⎝ |λj |⎠ x . 1. Fall: Für λ = 0 ist x = 0, d. h., A ist injektiv. Aber ein
j =m+1 Operator λ1 A existiert nicht und die gesuchte Riesz’sche Zahl
ist nicht definiert.
Da die Folge (λj ) in l1 ist, gibt es zu ε > 0 ein M ∈ N, 2. Fall: Sei λ = 0. Aus der IGL folgt
sodass

1
|λj | ≤ ε λx(1) = sx(s) ds = −λx(−1),
j =M+1 −1
Lösungswege zu Kapitel 9 73

t
d. h., es gilt die Randbedingung x(−1) + x(1) = 0. Analog Wir setzen P (t) = 0 1/p(s) ds. Dann folgt aus der Rand-
erhalten wir aus der Gleichung (9.7) die Bedingung x  (−1)+ bedingung für t = 1
x  (1) = 0. Daraus ergibt sich 1 
1 1 − P (1) − P (τ ) r(τ ) dτ = αP (1) .
0 < |x  (s)|2 ds = x  (s)x  (s) ds 0
−1 −1
Einsetzen dieser Darstellung für α in die IGL ergibt
1
= [x(s)x  (s)]1−1 − x(s)x  (s) ds t
 
−1 x(t) − P (t) − P (τ ) r(τ ) dτ
1 0
2 2
= − |x(s)|2 ds = − x2L2 P (t) 1 
λ −1 λ + P (1) − P (τ ) r(τ ) dτ = 0.
P (1) 0
Also ist N (λI − A) = {0} für λ > 0.
Also erhalten wir die Fredholm’sche Integralgleichung
Sei nun λ < 0. Aus der DGL (9.8) folgt
1 1
? x(t) − k(t, s)q(s)x(s) ds = k(t, s)f (s) ds
2 0 0
x(t) = A cos μt + B sin μt mit μ =
|λ|
mit dem stetigen Kern
und die Randbedingungen liefern das LGS 
1 P (t)(P (1) − P (s)), t ≤ s
k(t, s) =
P (1) P (s)(P (1) − P (t)), s ≤ t.
0 = 2A cos μ, 0 = 2Bμ cos μ .

Ist cos μ = 0, so folgt A = B = 0, also auch x = 0, und wir Lesen wir die Herleitung von unten nach oben, so folgt ins-
erhalten r = 0. gesamt die Äquivalenz der IGL mit dem RWP, wobei wir
berücksichtigen müssen, dass aus der allgemeinen Theorie
Im anderen Fall, d. h. μk = π
2 +kπ mit k ∈ N0 (man beachte: parameterabhängiger Integrale aus der Fredholmgleichung
μ > 0), erhalten wir mit zunächst die Regularität x ∈ C 1 ([0, 1]) folgt. Die Identität
nach der Integration der Differenzialgleichung am Anfang
2 8
λk = = , k ∈ N0 , des Beweises liefert dann x ∈ C 2 ([0, 1]).
μ2k (2k + 1)2 π 2
(b) Der adjungierte Integraloperator ist
nicht-triviale Lösungen der DGL und somit auch der IGL. 1
Also gilt in diesem Fall r = 1. A∗ x(t) = q(t) k(t, s)x(s) ds.
0
Bem.: (a) Die Lösbarkeit der IGL im zweiten Fall wird durch
die Fredholm-Theorie geklärt. Die speziellen Werte λk hei- Aus der Fredholm’schen Alternative erhalten wir, dass die
ßen Eigenwerte zu A (siehe Kapitel 10). Integralgleichung genau dann lösbar ist, wenn
(b) A ist ein „positiver Operator“, denn alle Eigenwerte sind 1
positiv. g(t) = k(t, s)f (s) ds ∈ (N (I − A∗ ))⊥
0

Aufgabe 9.8 •• zu (a) Sei x ∈ C 2 ([0, 1]) eine Lösung gilt. Analog zum Beispiel auf Seite 335 folgt, dass das RWP
der Randwertaufgabe. Integration liefert genau dann lösbar ist, wenn
1
t
p(t)x  (t) − [q(s)x(s) + f (s)] ds = α f (t)z(t) dt = 0
0 0

mit Integrationskonstanten α. Sei r(t) = q(t)x(t) + f (t). ist für alle z ∈ C 2 ([0, 1]), die das homogene RWP
Eine weitere Integration ergibt
(pz ) − qz = 0, z(0) = z(1) = 0
t s t
1 1
x(t) − r(τ ) dτ ds = α ds, lösen.
0 p(s) 0 0 p(s)

wobei die zweite Integrationskonstante verschwindet wegen


Beweisaufgaben
der Randbedingung x(0) = 0. Wir vertauschen die Integra-
tionsreihenfolge und erhalten Aufgabe 9.9 • Bezeichnen wir mit  = {x ∈ X |
x < 1} ⊆ X die offene Einheitskugel in X. Da der Ope-
t t 1 t 1
x(t) − r(τ ) ds dτ = α ds. rator A offen ist, ist auch A() offen. Also gibt es ε > 0,
0 τ p(s) 0 p(s) sodass B(0, ε) = {y ∈ Y | y < ε} ⊆ A() ist.
74 Lösungswege zu Kapitel 9

Weiter wissen wir, da A kompakt ist, dass A() ⊆ Y kom- wobei das erste Gleichheitszeichen aus der Fredholm’schen
pakt ist. Es folgt, B(0, ε) ⊆ A() ist kompakt. Strecken wir Alternative folgt 
und das zweite aus dem dritten Riesz’schen
die Kugel mit dem Faktor 1/ε, so folgt, dass die Einheitsku- Satz. Folglich ist ak ψk = 0 und wegen der linearen Unab-
gel in Y relativ kompakt ist, und nach dem Satz auf Seite 315 hängigkeit der ψk folgt a = 0. Also ist T injektiv und somit
ist Y endlich dimensional. regulär.

Aufgabe 9.10 • Mit der Riesz-Theorie genügt es zu „⇐“: Für die Rückrichtung des Beweises müssen wir zeigen,
zeigen, dass L − P injektiv ist, da L − P = I − (A + P ) dass I − A die Riesz’sche Zahl r = 1 hat.
und A + P kompakt ist.
Angenommen N (I − A)  N ((I − A)2 ), d. h., es gibt ein
Sei x ∈ X mit (L − P )x = 0. Es folgt Lx = P x ∈ N (Lr ) x ∈ N ((I − A)2 ) \ N (I − A). Dann ist ϕ1 := (I − A)x = 0
bzw. Lr+1 x = Lr P x = 0. Also ist x ∈ N (Lr+1 ) = N (Lr ) und (I − A)ϕ1 = 0, d. h. ϕ1 ∈ N (I − A). Nun ergänzen wir
und wir erhalten Lx = P x = x. Damit ergibt sich x = Lx = ϕ1 zu einer Basis von N (I − A). Ist weiter ψ1 , . . . , ψm eine
L(Lx) = . . . = Lr x und schließlich x ∈ N (Lr )∩Lr (X) = {0}. Basis von N (I − A∗ ), so folgt

Aufgabe 9.11 •• Mit ϕ1 , ψk  = (I − A)x, ψk  = x, (I − A∗ )ψk  = 0,


 
I −T
E −1 = : X×Y →X×Y für k = 1, . . . , m. Die erste Zeile der Matrix
0 I
erhalten wir den inversen Operator zu E, wie leicht nachzu-
T = ϕi , ψj i,j =1,...,m
rechnen ist. Also ergibt sich E − A = E(I − E −1 A) und
wir können die Riesz-Theorie des Kapitels auf I − E −1 A
anwenden. enthält nur Nullen. Damit ist T singulär.

Für die Riesz-Zahl zeigen wir N ((E − A)2 ) = N (E − A). Also muss unter der Voraussetzung, dass dieser Fall nicht
Betrachten wir (x, y) ∈ N ((E − A)2 ), d. h., es gilt eintritt, N (I −A) = N ((I −A)2 ) sein und für die Riesz’sche
 2     Zahl gilt r ≤ 1. Nach Voraussetzung ist {0} = N (I − A),
I − A11 T − A12 x 0 und daher ist die Riesz-Zahl von I − A gleich 1.
=
0 I − A22 y 0
bzw. Aufgabe 9.13 • Wir betrachten die Abbildungen
A : X → Y , A∗ : Y  → X  und A∗∗ = (A∗ )∗ : X  → Y  .
(I − A11 )2 x Außerdem nutzen wir die Bezeichnung Jx ∈ X  bzw.
+ [(I − A11 )(T − A12 ) + (T − A12 )(I − A22 )] y = 0 Jy ∈ Y  für Elemente in den Bidualräumen, die wir auf-
grund der Reflexivität eindeutig den Elementen x ∈ X bzw.
und
y ∈ Y durch Jx l = l(x) bzw. Jy l˜ = l(y)
˜ zuordnen können.
(I − A22 )2 y = 0 . ˜
Damit erhalten wir für Elemente l ∈ Y  die Gleichungen
Da wegen der vorausgesetzten Riesz-Zahl I − A22 injektiv
ist, folgt aus der zweiten Gleichung y = 0. Damit liefert die (A∗∗ Jx )(l)
˜ = Jx (A∗ l)
˜ = (A∗ l)(x)
˜
erste Zeile (I − A11 )2 x = 0. Mit der Riesz-Zahl r = 1 von
˜
= l(Ax) = JAx l˜ .
A11 ergibt sich x ∈ N (I − A11 )2 = N (I − A11 ). Somit
ist (x, y) ∈ N (E − A) und wir haben bewiesen, dass die
Riesz’sche Zahl von E − A durch r = 1 gegeben ist. Damit ist gezeigt, dass (A∗∗ Jx ) = JAx gilt und mit der Iden-
tifizierung der Bidualräume können wir diese Identität auch
Aufgabe 9.12 •• Man beachte, dass nach dem ersten durch A∗∗ = A beschreiben.
Fredholm’schen Satz gilt

dim(N (I − A)) = dim(N (I − A∗ )). Aufgabe 9.14 •••

„⇒“: Sei r = 1 die Riezsche Zahl von (I − A) und (I − A∗ ) Ist x ∈ X Lösung zu Lx = y, so folgt direkt
und T := ϕi , ψj i,j =1,...,m mit Basen zu den Nullräumen.
Wir wollen zeigen, dass T regulär ist. Ry = RLx = (I − A1 )x .
Dazu sei a ∈
 Cm
mT a = 0. Also gilt 0 =
mit
m
k=1 ϕ i , ψk a k = ϕi , k=1 ak ψk , i = 1, . . . , m. Also
Andererseits, wenn x Lösung zu (I − A1 )x = Ry ist, so
ist gilt

m R(Lx − y) = 0 .
ak ψk ∈ N (I − A∗ ) ∩ N (I − A)⊥
k=1 Da R injektiv vorausgesetzt ist, ergibt sich Lx = y, d. h.,
= N (I − A∗ ) ∩ (I − A∗ )(Y ) = {0} , X ist eine Lösung der ursprünglichen Gleichung.
Lösungswege zu Kapitel 9 75

Ist z Lösung zu (I − A2 )z = y, so gilt LRz = y, d. h., Andererseits ist x = z = Dz mit z ∈ C,1 ([0, 1]), so ergibt
x = Rz ist Lösung der ursprünglichen Gleichung. sich durch partielle Integration
Andererseits gibt es zu einer Lösung x der Gleichung t
Lx = y, da R surjektiv vorausgesetzt ist, ein z ∈ Y mit LRz(t) = k(t, s) z (s) ds
x = Rz und es folgt 0
t
∂k(t, s)
= z(t) − z(s) ds ,
y = Lx = LRz = (I − A2 )z . 0 ∂s
d. h., wir erhalten
Abschließend betrachten wir den Volterra-Operator
LR = I − A2
t
Lx(t) = k(t, s) x(s) ds . mit dem Integraloperator
0 t
∂k(t, s)
Differenzieren wir diese Funktion, so erhalten wir mit der A2 z(t) = z(s) ds .
0 ∂s
Produktregel
Es bleibt zu zeigen, dass A2 : C,1 ([0, 1]) → C,1 ([0, 1])
t ∂k(t, s) kompakt ist. Da A2 als Integraloperator mit stück-
DLx(t) = x(t) − x(s) ds .
0 ∂t weise stetigem Kern gegeben ist, gilt wie oben, dass
A2 : C,1 ([0, 1]) → C([0, 1]) kompakt ist. Für eine be-
Wie im Lemma auf Seite 284 gezeigt, lässt sich der In- schränkte Folge zn ∈ C 1 ([0, 1]) existiert somit eine in
tegraloperator durch Operatoren mit stetigem Kern, also C([0, 1]) konvergierende Teilfolge. Wir wählen eine sol-
kompakte Operatoren approximieren. Da der Unterraum che Teilfolge (zn ) ohne eine neue Notation einzuführen.
der kompakten Operatoren abgeschlossen ist, ist der Inte- Weiter ist
graloperator A1 : C([0, 1]) → C([0, 1]) mit
DA2 = D(I − (I − A2 )) = D(I − LD)
t ∂k(t, s) = (I − DL)D = A1 D .
A1 x(t) = x(s) ds
0 ∂t
Da die Verkettung A1 D : C, ([0, 1]) → C([0, 1]) kom-
kompakt. Definieren wir den beschränkten Operator pakt ist, gilt dies entsprechend für DA2 . Damit existiert
R : C,1 ([0, 1]) → C([0, 1]) durch Ry(t) = Dy(t), so auch für die Ableitung (A2 zn ) eine konvergente Teil-
folgt die gewünschte Darstellung folge bzgl. .∞ und wir haben gezeigt, dass (A2 zn )
in C,1 ([0, 1]) eine konvergente Teilfolge besitzt, d. h.,
RLy = I − A1 . A2 : C,1 ([0, 1]) → C,1 ([0, 1]) ist kompakt.
Kapitel 10 (b) Beweisen Sie, dass der Integraloperator A : X → X mit
dem Kern

(t − 1)s, 0 ≤ s ≤ t ≤ 1
Aufgaben k(t, s) =
(s − 1)t, 0 ≤ t ≤ s ≤ 1
bzgl. der vom Skalarprodukt induzierten Norm kompakt ist.
Verständnisfragen
(j )
Aufgabe 10.1 • Sei X Prä-Hilbertraum über C und Aufgabe 10.7 •• Wir bezeichnen mit (en ) die Folgen
(j ) (j )
sind x, y ∈ X mit x  = 0. Zeigen Sie, dass genau dann mit ej = 1 und en = 0 für n = j und betrachten den
Gleichheit in der Cauchy-Schwarz’schen Ungleichung gilt, Prä-Hilbertraum
wenn ein μ ∈ C existiert mit y = μx.
lend = {(xn )n∈N ∈ R | xn = 0 für endlich viele n ∈ N}
Aufgabe 10.2 •• Es seien X, Y Hilberträume und {xn |   
mit dem Skalarprodukt (xn ), (yn ) = ∞ n=1 xn yn . Weiter-
n ∈ N} ⊆ X und {yn | n ∈ N} ⊆ Y Orthonormalsysteme. hin definieren wir zu einer reellen Folge (λj ) mit λj < 1 und
Weiter ist eine monotone Nullfolge (an ) in C gegeben. Zeigen limj →∞ λj = 1 den Operator A : lend → lend durch
Sie, dass durch


(j ) (j )
Kx = (x, xn )an yn A(xn ) = λj ((xn ), (en ))en .
n=0 j =1
ein kompakter Operator K : X → Y definiert ist.
Zeigen Sie:
Aufgabe 10.3 • Wir betrachten eine beschränkte, (j )
Die Menge {(en ) | j ∈ N} ist ein vollständiges Ortho-
koerzive Sesquilinearform a : X × X → C in einem Hilbert- normalsystem in lend .
raum X, d. h., es gilt |a(u, v)| ≤ Cu v und |a(u, u)| ≥ A : lend → lend ist beschränkt mit A = 1
Ku2 für u, v ∈ X mit Konstanten C, K > 0. Ist l ∈ X und Der Operator I − A ist bijektiv, aber λ = 1 ∈ ρ(A) liegt
bezeichnen wir mit Uh ⊆ X einen abgeschlossenen Unter- im Spektrum des Operators.
raum, so gibt es nach dem Satz von Lax-Milgram Lösungen
u ∈ X und uh ∈ Uh zu Aufgabe 10.8 •• Bestimmen Sie die Eigenwerte und
a(v, u) = l(v) , für alle v ∈ X Eigenfunktionen der Operatoren A1 , A2 , A1 + A2 : L2 (0, 1)
und → L2 (0, 1) mit
a(v, uh ) = l(v) , für alle v ∈ Uh . 1
Man zeige das Cea-Lemma: A1 x(t) = min{t, s} x(s) ds,
C 0
u − uh  ≤ inf u − v . 1
K v∈Uh A2 x(t) = max{t, s} x(s) ds.
0
Aufgabe 10.4 •• Zeigen Sie, dass es zu einem kom-
pakten, selbstadjungierten, positiv definiten Operator A ∈
K(X, X) in einem Hilbertraum X genau eine k-te Wurzel Aufgabe 10.9 •• Sei T ein kompakter, selbstadjungier-
gibt, d. h., zu k ∈ N gibt es genau einen kompakten, selbst- ter Operator im Hilbertraum X mit den Eigenwerten λn = 0,
adjungierten, positiv definiten Operator B : X → X mit n ∈ N, und zugehörigen orthonormierten Eigenvektoren
A = Bk. xn ∈ X.
(a) Zeigen Sie, dass für λ = 0 und λ = λn , n ∈ N, die
Rechenaufgaben eindeutige Lösung von
Aufgabe 10.5 • Zeigen Sie, dass zu quadratischen Ma- (λI − T )x = y, y∈X
trizen A, B ∈ Cn×n durch (A, B) = Spur(AB ∗ ) ein Ska-
larprodukt definiert ist. gegeben ist durch
√  ∞

Aufgabe 10.6 •• Seien h : [0, 1] → R mit h(t) = t 1 λn
−1
(λI − T ) y= y+ (y, xn ) xn .
und λ λ − λn
n=1
X = {x ∈ (0, 1) : hx ∈ L2 (0, 1)}
gegeben. Warum konvergiert diese Reihe?
(a) Zeigen Sie, dass X mit (b) Wenden Sie Teil (a) an auf die Sturm’sche Randwertauf-
gabe
1
(x, y) = t x(t) y(t) dt x  − λx = f in [0, 1], x(0) = x(1) = 0
0
ein Hilbertraum ist. mit f ∈ C([0, 1]).
M. Brokate et al., Arbeitsbuch Grundwissen Mathematikstudium – Höhere Analysis, Numerik und
Stochastik, DOI 10.1007/978-3-642-54946-5_9, © Springer-Verlag Berlin Heidelberg 2016
Hinweise zu Kapitel 10 77

Beweisaufgaben Hinweise
Aufgabe 10.10 • Zeigen Sie in Hilberträume X, Y ,
dass ein Operator A ∈ L(X, Y ) genau dann eine Isometrie Verständnisfragen
ist, wenn
Aufgabe 10.1 • Man verwende den Projektionssatz.
(Ax, Az)Y = (x, z)X
für alle x, z ∈ X gilt. Aufgabe 10.2 •• Betrachten Sie die Partialsummen der
Reihe von Operatoren und verwenden Sie die Abgeschlos-
Aufgabe 10.11 •• Es sei X Hilbertraum und A ∈ senheit der Menge kompakter Operatoren.
L(X, X). Zeigen Sie:
 ⊥ Aufgabe 10.3 • Schätzen Sie a(u−uh , u−uh ) sowohl
(A(X))⊥ = N (A∗ ) und N (A∗ ) = A(X).
nach oben als auch nach unten ab.

Aufgabe 10.12 •• Zeigen Sie, dass die Dimension Aufgabe 10.4 •• Mit dem Spektralsatz lässt sich
des Nullraums N (I − A) zu einem Integraloperator
∞ 

A : C([0, 1]) → C([0, 1]) mit
Bx = k
λn (x, xn ) xn
1
n=1
Ax(t) = k(t, s)x(s) ds
0 mit den Eigenwerten λn und Eigenvektoren xn , n ∈ N von A
mit stetigem Kern k ∈ C([0, 1] × [0, 1]) durch definieren. Man zeige die angegebenen Eigenschaften.
dim (N (I − A)) ≤ k2∞
Rechenaufgaben
abgeschätzt werden kann.
Aufgabe 10.5 • Es müssen alle Eigenschaften eines
Aufgabe 10.13 • Es sei A ∈ L(X, X) ein linearer be- Skalarproduktes explizit gezeigt werden.
schränkter Operator in einem Hilbertraum X. Zeigen Sie,
dass A genau dann kompakt ist, wenn A∗ A kompakt ist. Aufgabe 10.6 •• Für den zweiten Teil verwende man,
dass Integraloperatoren über kompakten Intervallen mit ste-
Aufgabe 10.14 •• Ist X Hilbertraum über dem Grund- tigem Kern kompakt sind (siehe Seite 321).
körper C, dann ist A ∈ L(X, X) genau dann selbstadjungiert,
wenn (Ax, x) ∈ R für alle x ∈ X gilt. Aufgabe 10.7 •• Für die Vollständigkeit des Orthonor-
malsystems betrachte man den Fourier-Entwicklungssatz.
Aufgabe 10.15 •• Die Hermite-Polynome sind defi-
niert durch Aufgabe 10.8 •• Die auftretenden Operatoren sind
dn −t 2
n t2 selbstadjungiert, somit sind nur die Fälle λ = 0, λ > 0 und
Hn (x) = (−1) e e , t ∈ R , n ∈ N0 .
dt n λ < 0 zu untersuchen. Dazu leite man aus der jeweiligen In-
Zeigen Sie die Rekursionen tegralgleichung eine Differenzialgleichung zweiter Ordnung
Hn+1 (t) = 2tHn (t) − 2nHn−1 (t) her.
und
Hn (t) = 2nHn−1 (t) , Aufgabe 10.9 •• Für den ersten Teil wende man den
für n ∈ N, und folgern Sie daraus, dass Hn Lösung der Spektralsatz an. Umformulieren des Randwertproblems zu
Differenzialgleichung einer Integralgleichung ermöglicht dann die Anwendung des
ersten Teils der Aufgabe.
u (t) − 2tu (t) + 2nu(t) = 0 , t ∈ R,
ist.
Beweisen Sie, dass mit den Funktionen
Beweisaufgaben
1 −t 2 Aufgabe 10.10 • Für eine der beiden Implikationen
hn (t) = √ e 2 Hn (t) , betrachte man Elemente x + λy ∈ X mit x, y ∈ X und
π 2n n!
λ ∈ C.
n ∈ N0 , ein Orthonormalsystem in L2 (−∞, ∞) gegeben
ist. Aufgabe 10.11 •• Mit der Definition des adjungierten
Operators in Hilberträumen ergeben sich entsprechende In-
Aufgabe 10.16 ••• Sei a ∈ C([0, 1]). Zeigen Sie zum klusionen relativ direkt, wobei nur eine aufwendiger ist. Hier
Multiplikationsoperator Ma : L2 (0, 1) → L2 (0, 1) mit hilft der Projektionssatz weiter.
Ma x = ax die Identität
Aufgabe 10.12 •• Die Abschätzung lässt sich mithilfe
σ (Ma ) = {z ∈ C : ∃t ∈ [0, 1] mit z = a(t)}
einer Orthonormalbasis von N (I − A) bezüglich des L2 -
für das Spektrum des Operators. Skalarprodukts und der Bessel’schen Ungleichung zeigen.
78 Lösungswege zu Kapitel 10

Aufgabe 10.13 • Aus der Konvergenz einer Folge Lösungswege


(A∗ Axn ) kann man zeigen, dass Axn Cauchy-Folge ist.
Verständnisfragen
Aufgabe 10.14 •• Für eine der beiden zu zeigenden Im-
plikationen betrachte man x + αy ∈ X für spezielle Wahlen Aufgabe 10.1 • Ist y = μx, so folgt
von α ∈ C.
(x, y) = μ(x, x) = μx2 = x μx = x y ,
Aufgabe 10.15 •• Für die erste Rekursionsgleichung d. h., in der Cauchy-Schwarz’schen Ungleichung gilt das
verwende man die Leibniz’sche Formel (siehe Aufgabe 15.16 Gleichheitszeichen.
in Band 1).
Es bleibt die andere Richtung in der Äquivalenz zu zeigen.
Aufgabe 10.16 ••• Überlegen Sie sich, unter welcher Nehmen wir an, es sind x, y ∈ X mit (x, y) = xy und
Bedingung der Multiplikationsoperator beschränkt invertier- definieren den abgeschlossenen Unterraum U = span{x}.
bar ist. Mit dem Projektionssatz gibt es zu y ein μ ∈ C und ein
Element x ⊥ ∈ U ⊥ mit

y = μx + x ⊥ .
Lösungen Setzen wir die Darstellung in die Voraussetzung ein, so folgt

x y = (x, y) = (x, μx + x ⊥ ) = μx2


Verständnisfragen
bzw. y = μx. Also ist μ ∈ R≥0 . Weiter erhalten wir
Aufgabe 10.1 • – aus der Darstellung

Aufgabe 10.2 •• – y2 = (μx + x ⊥ , μx + x ⊥ ) = μ2 x2 + x ⊥ 2 ,

und die vorherige Identität führt auf


Aufgabe 10.3 • –
μ2 x2 = y2 = μ2 x2 + x ⊥ 2 .
Aufgabe 10.4 •• –
Also ist x ⊥ = 0 und wir haben gezeigt, dass y = μx gilt.

Rechenaufgaben Aufgabe 10.2 •• Wir betrachten die Operatoren


KN : X → Y , die sich aus den Partialsummen ergeben, d. h.
Aufgabe 10.5 • –

N
Aufgabe 10.6 •• – KN = (x, xn )an yn .
n=1
Aufgabe 10.7 •• – Die Operatoren sind beschränkt, denn mit der Cauchy-
Schwarz’schen Ungleichung ist
Aufgabe 10.8 •• –

N
N

Aufgabe 10.9 •• – KN x ≤ |(x, xn )| |an | ≤ x |an | .


n=1 n=1

Außerdem ist das Bild K(X) ⊆ span{y1 , . . . yN } endlich


Beweisaufgaben dimensional. Also ist KN kompakt (siehe Seite 318). Weiter
Aufgabe 10.10 • – gilt mit der Orthonormalität des Systems {yn | n ∈ N} und
der Bessel’schen Ungleichung, dass
Aufgabe 10.11 •• – / /2
/ M /
/ /
KM x − KN x2 = / / (x, xn ) an yn
/
/
Aufgabe 10.12 •• – /n=N+1 /

Aufgabe 10.13 • –
M
= |(x, xn )|2 |an |2 yn 2
n=N+1
Aufgabe 10.14 •• –

M
≤ |aN +1 |2 |(x, xn )|2
Aufgabe 10.15 •• –
n=N+1
2 2
Aufgabe 10.16 ••• – ≤ |aN +1 | x
Lösungswege zu Kapitel 10 79

abgeschätzt werden kann. Somit gilt Weiterhin gilt


KM − KN  ≤ |aN +1 | → 0, M ≥ N → ∞, ∞ 

(Bx, y) = k
λn (x, xn )(xn , y)
d. h., (KN ) ist Cauchy-Folge. Da die Menge der kompakten
n=1
Operatoren abgeschlossener Unterraum des Banachraums
L(X, Y ) ist (siehe Seite 319), folgt, dass K kompakt ist. = (x, By) ,

Aufgabe 10.3 • Wir subtrahieren die beiden Gleichun- d. h., B ist selbstadjungiert, und
gen und erhalten ∞ 

(Bx, x) = k
λn |(x, xn )|2 ≥ 0 ,
a(u − uh , v) = 0 für v ∈ Uh .
n=1
Setzen wir unter anderem v = uh in diese Identität ein, so
ergibt sich mit der Koerzitivität und der Beschränktheit der d. h., der Operator ist auch positiv definit.
Sesquilinearform Es bleibt noch zu zeigen, dass der Operator eindeutig be-
1 stimmt ist. Nehmen wir an, es gibt einen weiteren kompakten,
u − uh 2 ≤ a(u − uh , u − uh ) selbstadjungierten, positiv definiten Operator C ∈ K(X, X).
k
= a(u − uh , u) − a(u − uh , v) Wegen des Spektralsatzes angewendet auf C gibt es ein voll-
ständiges System von Eigenwerten σn und Eigenwerten yn .
= a(u − uh , u − v)
Weiter folgt aus Ayn = C k yn = σnk yn , dass σ k Eigenwert
c
≤ u − uh u − v und yn Eigenvektor zu A ist. Da die Systeme von Eigen-
k vektoren maximal sind, folgt {yn : n ∈ N} = {xn : n ∈ N}
für jedes v ∈ Uh und wir haben die Behauptung gezeigt. und A(X) = C k (X). Sind die Eigenwerte monoton fallend
Kommentar: Das Lemma spielt eine wichtige Rolle in geordnet, so gilt σnk = λn und wir erhalten
der numerischen Mathematik, speziell bei sogenannten ∞ 

Galerkin-Verfahren; denn ist Uh ein endlich dimensionaler Cx = C (x, xn )xn
Unterraum, so liefert das Lemma eine Fehlerabschätzung n=1
zwischen der Lösung u des kontinuierlichen Problems und ∞

der endlich dimensionalen Näherung uh . = (x, xn )σn xn
n=1
Aufgabe 10.4 •• Da A kompakt und selbstadjungiert ∞ 
ist, lässt sich der Spektralsatz anwenden. Wir bezeichnen mit = (x, xn ) k λn xn = Bx
λn und xn die Eigenwerte und Eigenvektoren zu A und defi- n=1
nieren die linearen beschränkten Operatoren
und wir haben auch die Eindeutigkeit gezeigt.

N 
BN x = k
λn (x, xn )xn .
n=1 Rechenaufgaben
Da das Bild von BN endlich dimensional ist, ist Bn kompakt. Aufgabe 10.5 • Aus
Außerdem gilt

n
n

N  Spur(AB ∗ ) = aij bi,j
BN x − BM x =  k
λn (x, xn )xn 
i=1 j =1
n=M+1

≤ sup{ k λn : M + 1 ≤ n ≤ N}x → 0 ergeben sich direkt die Eigenschaften eines Skalarprodukts;
denn offensichtlich gilt
für M, N → ∞. Es handelt sich also um eine Cauchy-Folge
und wir erhalten den kompakten Operator B = limN →∞ BN
n
n
(siehe auch Aufgabe 10.2) mit (λA, B) = λaij bi,j = λ(A, B)
∞  i=1 j =1

BN x = k
λn (x, xn )xn . und
n=1

n
n
Mit der Konvergenz der Reihe ergibt sich (λA+C, B) = λ(aij +cij )bi,j = λ(A, B)+(C, B)

∞  i=1 j =1

B2x = k
λn k λj (x, xj )(xj , xn )xn
für A, B, C ∈ Cn×n . Weiterhin erhalten wir
n=1 j =1


n
n
n
n
= λn (x, xn )xn = Ax . (A, B) = aij bi,j = aij bi,j = (B, A)
n=1 i=1 j =1 i=1 j =1
80 Lösungswege zu Kapitel 10

und k = max{j ∈ N | xj = 0} die Identität



n
n
(A, A) = |aij |2 ≥ 0 . ∞

k
(j )  (j ) (j )
i=1 j =1 (xn ), (en (en ) = xj (en )
j =1 j =1
Schließlich
n n ist (A, A) = 0 äquivalent zu
= (xn ) .
j =1 |aij | = 0 bzw. A = 0.
2
i=1

Somit haben wir die Eigenschaften eines inneren Pro- Dies ist die Fourier-Entwicklung von (xn ). Da die Entwick-
dukts überprüft. Insbesondere erhalten wir die Cauchy- lung für jedes (xn ) ∈ lend erfüllt ist, ergibt sich mit dem
Schwarz’sche Ungleichung Entwicklungssatz die Vollständigkeit des ONS.
Als Nächstes zeigen wir, dass A beschränkt ist. Dazu sei
SpurAB ∗ ≤ Spur(AA∗ )1/2 Spur(BB ∗ )1/2 .
wiederum (xn ) ∈ lend und k = max{n ∈ N | xn = 0}.
Wir erhalten mit ONS und der Parzeval’schen Gleichung die
Abschätzung
Aufgabe 10.6 •• (a) Die Sesquilinearform
1
A(xn )2
⎛ ⎞
(x, y) = t x(t) y(t) dt k
  (j )
k
  (j )
=⎝ λj (xn ), (en ) (en )⎠
0 (j ) (j )
λj (xn ), (en (en ) ,
ist offensichtlich positiv, definit und hermitesch. Außerdem j =1 j =1
ist X vollständig. Denn mit (xn ) in X konvergiert auch

k
 (j ) 2
t xn (t) in L2 . Also ist X ein Hilbertraum. = λ2j (xn ), (en )
√ j =1
(b) Setze J : X → L2 (0, 1) mit J x(t) = t x(t). Die Abbil-
dung ist nach Definition von X wohldefiniert. Weiter ist J of-
k
 (j ) 2
≤ (xn ), (en )
fensichtlich injektiv und mit x(t) = √1 y(t) für y ∈ L2 (0, 1)
t j =1
auch surjektiv. J ist sogar ein Normisomorphismus wegen ∞
 (j ) 2
1√ = (xn ), (en ) = (xn )2 .

x2X = (x, x) = t x(t) t x(t) dt = J x2L2 . j =1
0
Also folgt A ≤ 1.
Insbesondere folgt J ∈ L(X, Y ) und J −1 ∈ L(Y, X) bzgl.
Betrachten wir weiterhin
der durch die Skalarprodukte induzierten Normen. Setze
A(en(k) ) = |λk |(en(k) ) → (en(k) ) , k → ∞,
1
Ax(t) = k(t, s)x(s) ds . so folgt A ≥ 1. Also gilt insgesamt, dass A beschränkt ist
0
mit A = 1.
Es ist √
√ 1 t √ Nun zeigen wir Injektivität von I − A. Nehmen wir an, für
tAx(t) = √ k(t, s) s x(s) ds . (xn ) ∈ lend gilt
0 s
(I − A)(xn ) = 0 .
Mit
Dann folgt mit der Fourier-Entwicklung
⎧√
√ ⎨ ts(t − 1), 0≤s≤t ≤1 
t (j )   (j ) 
k̃(t, s) = √ k(t, s) = √ xj = (xn ), (en ) = λj (xn ), (en ) λj xj
s ⎩ t (s − 1) √ t , 0≤t ≤s≤1
s für jedes j ∈ N. Also ist λj = 1 oder xj = 0. Da λj < 1
vorausgesetzt ist, bleibt nur xj = 0 für alle j ∈ N, d. h.,
ist k̃ stetig und aus A = J −1 ÃJ folgt, dass A ∈ K(X, X), da für die Folge (xn ) = 0, und wir haben gezeigt, dass I − A
der Integraloperator à : L2 (0, 1) → L2 (0, 1) mit dem Kern injektiv ist.
k̃ kompakt ist (siehe Seite 320).
Surjektivität des Operators zeigen wir durch explizite Kon-
Aufgabe 10.7 •• Direkt aus struktion: Zu (yn ) ∈ lend mit k = max{j ∈ N | yj = 0}
definieren wir
∞  #
 (j )  1, für j = k 1
(en ), (en(k) ) =
(j )
en en(k) = yj , für j ≤ k
0, sonst xj = 1−λj
n=1 0, für j > k .

folgt, dass es sich um ein Orthonormalsystem handelt. Dann ist (xn ) ∈ lend und wir haben die Fourierkoeffizienten
Vollständigkeit erhalten wir aus folgender Überlegung: Ist  (j )  1  (j ) 
 (j )  (xn ), (en ) = (yn ), (en ) .
(xn ) ∈ lend , so gilt mit xj = (xn ), (en für j ∈ N und mit 1 − λj
Lösungswege zu Kapitel 10 81

Damit erhalten wir (ii) Analog erhalten wir für einen Eigenwert λ mit EF x zu

A2 , d. h. A2 x = λx, dass x ∈ C 2 (0, 1) ist mit
 (j )  (j ) t
(I − A)(xn ) = (1 − λj ) (xn ), (en ) (en )
j =1 λx  (t) = x(s) ds und λx  (t) = x(t)
0

k
1 − λj  (j )  (j )
= (yn ), (en ) (en ) Außerdem gelten die Randbedingungen x  (0) = 0 und
1 − λj
j =1
x(1) = x  (1).
∞ Wiederum ist λ = 0 offensichtlich kein Eigenwert.
 (j )  (j )
= (yn ), (en ) (en ) = (yn ) . √
j =1
1. Fall λ > 0: Mit ω = 1/ λ ist die allgemeine Lösung der
Differenzialgleichung
Es bleibt noch zu zeigen, dass λ = 1 nicht in der Resolven-
x(t) = αeωt + βe−ωt .
tenmenge liegt. Dazu zeigen wir, dass der inverse Operator
(I − A)−1 unbeschränkt ist. Dies folgt mit der eben angege- Aus den Randbedingungen folgt α = β und f (ω) :=
benen Konstruktion aus ω tanh ω = 1. Da f (0) = 0, limω→∞ f (ω) = ∞ und
1 f  (ω) > 0 für ω > 0 ist, folgt aus dem Zwischenwertsatz
(I − A)−1 (en ) =
j
→ ∞, j → ∞, und der Monotonie, dass es genau einen positiven Eigenwerte
1 − λj
zu A2 gibt.
da limj →∞ λj = 1 gilt. √
2. Fall λ < 0: In diesem Fall ist mit ω = 1/ |λ| durch
x(t) = α cos(ωt) + β sin(ωt) die allgemeine Lösung gege-
Aufgabe 10.8 •• Offensichtlich sind die Operatoren ben. Aus den Randbedingungen erhalten wir β = 0 und die
selbstadjungiert und es gilt Fixpunktgleichung ω = − cot ω. Diese Gleichung besitzt ab-
t 1
zählbar unendlich viele Lösungen mit ωn ∈ [(n − 1)π, nπ],
A1 x(t) = sx(s) ds + tx(s) ds n ∈ N. Also ergeben sich abzählbar unendlich viele negative
0 t EW λ = −ωn2 mit Eigenfunktionen xn (t) = α cos ωn t.
t 1
A2 x(t) = tx(s) ds + sx(s) ds (c) Für die Summe L = A1 + A2 erhalten wir, wenn λ Ei-
0 t
genwert mit Eigenfunktion x zu L ist, dass x ∈ C 2 (0, 1) gilt
1
mit
Bx(t) = A1 x(t) + A2 x(t) = (t + s)x(s) ds 1
0 λx  (t) = x(s) ds und λx  (t) = 0 .
0
(i) Sei λ Eigenwert mit Eigenfunktion x zu A1 , d. h., es gilt Außerdem gelten die Anfangsbedingungen λx(0) =
1 
1
A1 x = λx. Dann ist x ∈ C 2 (0, 1) mit 0 sx(s) ds und λx (0) = 0 x(s) ds.
1 Offensichtlich ist wieder λ = 0 kein Eigenwert. Die allge-
λx  (t) = x(s) ds und λx  (t) = −x(t) . meine Lösung der Differenzialgleichung ist x(t) = αt + β
t
und aus den Randbedingungen ergibt sich die quadratische
Außerdem gilt x(0) = x  (1) = 0. Gleichung
 2
1 1
Aus λx  = −x folgt sofort, dass λ = 0 kein Eigenwert ist. −λ =
√ 2 3
1. Fall λ < 0: Setze ω = 1/ |λ|. Dann ist die allgemeine mit den beiden Lösungen
Lösung der Differenzialgleichung durch
1 1
ωt −ωt λ1,2 = ±√ .
x(t) = αe + βe 2 3
Also erhalten wir genau einen positiven und einen negativen
gegeben. Aus den Randbedingungen folgt aber α = β = 0.
Eigenwert.
Also ist λ < 0 kein EW.
2. Fall λ > 0: In diesem Fall ist x(t) = √
α cos(ωt)+β sin(ωt) Aufgabe 10.9 •• Zu (a): Nach dem Spektralsatz gibt es
die allgemeine Lösung, wenn ω = 1/ λ ist. Aus x(0) = 0 zu x, y ∈ X Elemente x0 , y0 ∈ N(T ) mit
folgt α = 0 und x  (1) = 0 impliziert β = 0 oder cos ω = 0. ∞ ∞

Also ist λ > 0 genau dann EW, wenn ω = (n − 21 )π, n ∈ N x = x0 + (x, xn )xn und y = y0 + (y, xn )xn .
gilt. Wir erhalten alle EW n=1 n=1

1 Also ist (λI − T )x = y äquivalent zu


λn = 1 2 2
, n∈N
(n − 2) π ∞
6 7
√ λx0 − y0 + (λ − λn )(x, xn ) − (y, xn ) xn = 0 .
mit den normierten Eigenfunktionen xn (t) = 2 sin ωn t. n=1
82 Lösungswege zu Kapitel 10

Projektion auf xm bzw. auf N (T ) ergibt durch Koeffizienten- Andererseits gehen wir davon aus, dass A eine Isometrie ist,
vergleich λx0 − y0 = 0 und (λ − λn )(x, xn ) − (y, xn ) = 0 dann gilt etwa
für alle n ∈ N. Also ist x0 = yλ0 und (x, xn ) = (y,x n)
λ−λn und
A(x + λy)2 = x + λy2 .


1 (y, xn )
x = y0 + xn Mit dem Skalarprodukt folgt
λ λ − λn
n=1
 ∞
 ∞ x + λy2 = (x + λy, x + λy)
1 (y, xn )
= y− (y, xn )xn + xn = x2 + 2Re(λ(x, y) + y2
λ λ − λn
n=1 n=1
 ∞

1 λn und
= y+ (y, xn )xn .
λ λ − λn A(x + λy)2 = (Ax + λAy, Ax + λAy)
n=1

Wegen der Orthonormalität der xn und der Vollständigkeit = Ax2 + 2Re(λ(Ax, Ay) + Ay2
∞X konvergiert
von
λn
die Reihe genau dann, wenn die Reihe = x2 + 2Re(λ(Ax, Ay) + y2 .
|
n=1 λ−λn (y, xn )| 2 konvergiert.
Somit ergibt sich die Identität
Die Folge (λn ) ist durch T  beschränkt. Ferner gibt es ein
δ > 0 mit |λ−λn | ≥ δ, denn sonst wäre λ ein Häufungspunkt Re(λ(x, y) = Re(λ(Ax, Ay) .
der Folge (λn ), was wegen λ  = 0 nicht sein kann. Mit der
Bessel’sche Ungleichung folgt Dies gilt für jedes λ ∈ C. Wählen wir λ = 1 und λ = i so
∞  2 ∞
folgt die gesuchte Gleichung
 λn  2
 (y, x )  ≤ T  |(y, xn )|2
λ − λ n  δ2 (Ax, Az)Y = (x, z)X .
n
n=1 n=1
T 2
≤ y2 . Aufgabe 10.11 •• Sei v ∈ (A(X))⊥ , d. h. (Ax, v) = 0
δ2
für alle x ∈ X bzw.
(b) Das Randwertproblem ist äquivalent zur Integralglei-
chung (x, A∗ v) = 0
1 1 für alle x ∈ X. Dies ist äquivalent zu A∗ v = 0, also ist v ∈
x(t) − λ k(t, s)x(s) ds = k(t, s)f (s) ds N (A∗ ). Andererseits folgt offensichtlich mit v ∈ N (A∗ ),
0 0
= y(t), 0≤t ≤1 für jedes x ∈ X die Identität (Ax, v) = (x, A∗ v) = 0, d. h.
v ∈ (A(X))⊥ .
mit k(t, s) = ts − min(t, s). Dies lässt sich für λ  = 0 in
Nun zeigen wir die zweite Identität: Nach dem ersten Teil ist
der Form λ1 I − A x = λ1 y schreiben, wobei Ax(t) =  ⊥
1 (N (A∗ ))⊥ = (A(X))⊥ und somit
0 k(t, s)x(s) ds. Man beachte, dass im Fall λ = 0 die Inte-
gralgleichung eine explizite Lösungsformel liefert. A(X) ⊆ (N (A∗ ))⊥ .
Die Eigenwerte und Eigenfunktionen von A sind λn = n−1
√ Also bleibt noch A(X) ⊇ (N (A∗ ))⊥ zu zeigen. Da A(X)
2π 2

mit xn (t) = 2 sin(nπt) für n ∈ N . Für λ1  = λn erhalten ein abgeschlossener Unterraum ist, existiert mit dem Projek-
wir tionssatz zu x ∈ X eine Zerlegung x = u + v mit u ∈ A(X)
 −1  
1 −1 und v ∈ (A(X))⊥ . Sei nun x = u + v ∈ (N (A∗ ))⊥ =
x = I −A y  ⊥
λ λ (A(X))⊥ , so gilt
 ∞ −1

1 1
= λ y+ n2 π 2
( y, xn )xn 0 = (x, v) = (u, v) + (v, v) = v2 .
1 1
λ + 2 2
λ λ
n=1 n π
Also folgt x = u ∈ A(X), d. h.

1
= y−λ (y, xn )xn (N (A∗ ))⊥ ⊆ A(X),
n2 π 2 + λ
n=1

und die Reihe konvergiert absolut und gleichmäßig. und wir haben insgesamt auch die zweite Identität gezeigt.
Man vergleiche das Resultat mit dem Lemma auf Seite 329
Beweisaufgaben in allgemeinen Dualsystemen.

Aufgabe 10.10 • Die eine Richtung der Äquivalenz ist Aufgabe 10.12 •• Nach dem ersten Riesz’schen Satz
offensichtlich, denn gilt (Ax, Az)Y = (x, z)X für alle x, z ∈ X, ist die Dimension des Nullraums endlich. Wir setzen
so folgt mit x = z die Isometrie-Eigenschaft Ax = x. n = dim(N (I − A)) und wählen eine Orthonormalbasis
Lösungswege zu Kapitel 10 83

{x1 , . . . , xn } zu N (I − A) bezüglich des L2 -Skalarprodukts. folgt deswegen


Mit xj = Axj , j = 1, . . . , n können wir mit der Bessel’schen    
0 = α (Ay, x) − (y, Ax) + α (Ax, y) − (x, Ay)
Ungleichung abschätzen
für jedes α ∈ C. Mit α = 1 und α = i erhalten wir die beiden

n
n
n= xj 2L2 = Axj 2L2 Gleichungen
j =1 j =1 (Ay, x) − (y, Ax) = −(Ax, y) + (x, Ay)
n 1  1
 2
 (Ay, x) − (y, Ax) = (Ax, y) − (x, Ay)
 
=  k(t, s)xj (s) ds  dt
  Aus der Differenz der Gleichungen folgt (Ax, y) = (x, Ay),
j =1 0 0
1 n d. h., der Operator ist selbstadjungiert.
 
= (k(t, ·), xj ) 2 2 dt
L
0 j =1 Aufgabe 10.15 ••
1 Mit der Leibnizformel berechnen wir
≤ k(t, ·)2L2 dt
0 dn+1 −t 2
2
1 1
Hn+1 (t) = (−1)n+1 et e
dt n+1
= |k(t, s)|2 ds dt ≤ k2∞ . 2 d
n
(−2te−t )
2
0 0 = (−1)n+1 et n
dt
Dabei haben wir mit der häufig genutzten Notation n   k
2 n d dn−k −t 2
k(t, ·) : [0, 1] → R die Funktion bezüglich des zweiten Ar- n t
= 2(−1) e (t) e
k dt k dt n−k
guments bei gegebene t ∈ [0, 1] bezeichnet. k=0
 
n t2 dn −t 2 dn−1 −t 2
Aufgabe 10.13 • Die eine Richtung der Äquivalenz ist = 2(−1) e t ne + n n−1 e
dt dt
leicht zu sehen, denn mit A ∈ L(X, X) im Hilbertraum ist
auch A∗ ∈ L(X, X). Damit ist die Verkettung A∗ A kompakt, = 2tHn (t) − 2nHn−1 (t) .
wenn einer der Operatoren, A oder A∗ kompakt ist.
Insbesondere sehen wir mit H0 (t) = 1 und H1 (t) = 2t
Nehmen wir nun andererseits an, dass A∗ A kompakt ist und induktiv aus der ersten Rekursionsformel, dass Hn ein
betrachten eine beschränkte Folge (xn ) in X mit xn  ≤ c Polynom ist.
für alle n ∈ N. Dann gibt es eine konvergente Teilfolge zu Weiter erhalten wir durch Differenzieren
A∗ Axn . Ohne die Notation zu modifizieren, nehmen wir an,  n 
 n d t2 d −t 2
dass (A∗ Axn ) diese Teilfolge ist. Für die Differenz Axn − Hn (t) = (−1) e e
dt dt n
Axk  mit n, k ∈ N erhalten wir mit der Cauchy-Schwarz’sche  
n n+1
Ungleichung n t2 d −t 2 t2 d −t 2
= (−1) 2te e +e e
dt n dt n+1
Axn − Axk 2 = (xn − xk , A∗ A(xn − xk ))
= 2tHn (t) − Hn+1 (t) .
≤ xn − xk  A∗ A(xn − xk )
≤ 2cA∗ A(xn − xk ) → 0, n, k → ∞ . Aus der ersten Rekursion ergibt sich somit Hn (t) =
2nHn−1 (t).
Somit ist (Axn ) eine Cauchy-Folge in X und deswegen Mit den Rekursionen erhalten wir die Differenzialglei-
konvergent. Insgesamt haben wir gezeigt, dass zu jeder be- chung durch
schränkten Folge die Bildfolge (Axn ) eine konvergente Teil-
Hn (t) = 2nHn−1

(t)
folge besitzt, d. h., der Operator A ist kompakt.  
= 2tHn (t) − Hn+1 (t)
Aufgabe 10.14 •• Ist A selbstadjungiert, so folgt = 2Hn (t) + 2tHn (t) − 2(n + 1)Hn (t)
(Ax, x) = (x, Ax) = (Ax, x) . = 2tHn (t) − 2nHn (t) .
Wir betrachten das Produkt
Also ist (Ax, x) ∈ R. ∞
Andererseits mit der Voraussetzung (Ax, x) ∈ R für jedes hn (t) hm (t) dt
−∞
x ∈ X gilt
(−1)n+m ∞ n e−t 2 dm e−t
2
t2 d
= √ e dt
h = (A(x + αy), x + αy) π 2n+m n! m! −∞ dt n dt m

dm e−t
2 ∞
= (Ax, x) + α(Ax, y) + α(Ay, x) + |α| (Ay, y) ∈ R ,
2
(−1)m
= √ Hn (t) dt .
π 2n+m n! m! −∞ dt m
für alle x, y ∈ X und α ∈ C. Insbesondere ist h = h. Aus
Da der Ausdruck symmetrisch in n und m ist, können wir
h = (Ax, x) + α(y, Ax) + α(x, Ay) + |α|2 (Ay, y) ohne Einschränkung annehmen, dass m ≥ n gilt. Im Fall
84 Lösungswege zu Kapitel 10

m = n ergibt sich mit der Rekursionsgleichung für die Aufgabe 10.16 ••• Sei λ ∈ ρ(Ma ) in der Resolventen-
Ableitung durch m-malige partielle Integration menge, d. h., λI − Ma = Mλ−a ist beschränkt invertierbar.

Wir zeigen die Behauptung: Mψ : L2 (0, 1) → L2 (0, 1) mit
hn (t) hm (t) dt
−∞ ψ ∈ C([0, 1]) ist beschränkt invertierbar genau dann, wenn
(−1)n ψ(t) = 0 ist für alle t ∈ [0, 1]. Denn mit ψ = λ − a folgt
= √ n die gesuchte Aussage der Aufgabe.
π 2 n!
 
2 ∞ ∞ 
dn−1 e−t  dn−1 e−t
2 Bew.: „⇐“ Aus ψ(t) = 0 folgt (Mψ )−1 = M 1 ist ein

· Hn (t)  − Hn (t) dt ψ
dt n−1  −∞ dt n−1 beschränkter linearer Operator in L2 .
−∞

dn−1 e−t
2
−(−1)n „⇒“ Sei Mψ beschränkt invertierbar. Dann existiert insbe-
= √ n 2nHn−1 (t) dt sondere die Lösung x ∈ L2 (0, 1) zu Mψ x = 1. Also ist
π 2 n! −∞ dt n−1 1
ψ = x ∈ L (0, 1). Weiter gilt Mψ (xy) = Mψ (x)y =
2
= ...
∞ y ∈ L2 (0, 1), sodass aus der beschränkten Invertierbarkeit
(−1)n (−1)n −t 2
= √ H0 (t) e dt = 1 . xy ∈ L2 (0, 1) für alle y ∈ L2 (0, 1) folgt.
π −∞
Dabei wird genutzt, dass die Randterme wegen des Faktors Angenommen es sei ψ(t0 ) = 0 für ein t0 ∈ [0, 1]. Dann ist
durch
e−t stets verschwinden.
2


Im Fall m > n führen wir m + 1 partielle Integrationen 1 1
aus und erhalten, da Hn Polynom vom Grad n ist, In = t ∈ [0, 1] : < ψ(t) ≤ , n∈N
n+1 n

hn (t) hm (t) dt eine disjunkte Zerlegung der Menge {t ∈ [0, 1] : ψ(t) ≤ 1}
−∞ ⊆ [0, 1] gegeben und es gilt μ(In ) = 0. Setze
 
2 ∞
(−1)m dm−1 e−t  #
= √ Hn (t)  √1 t ∈ In
π 2n+m n! m! dt m−1  fn (t) = n μ(In )
,
−∞
∞  0, sonst.
d m−1 e −t 2
− Hn (t) dt
−∞ dt m−1 Es folgt
= ... ∞ ∞ ∞
1 1
| fn |2 dt = |fn |2 dt = < ∞,
∞ dm−n−1 e−t
2
(−1)n+1 (−1)m n2
= √ Hn(n+1) (t) dt 0 n=1 n=1 In n=1
π 2n+m n! m! −∞ dt m−n−1
∞
= 0. d. h. f = ∈ L2 (0, 1). Weiter ist aber
n=1 fn
Insgesamt haben wir gezeigt, dass es sich bei den Funk-

N N

N
tionen {hn ∈ L2 (−∞, ∞) | n ∈ N0 } um ein Orthonor- 1
2
malsystem handelt. |x fn | dt = |xfn |2 dt ≥ 1→∞
0 n=1 n=1 In n=1
Bemerkung: Die Vollständigkeit dieses Orthonormalsys-
tems über dem unbeschränkten Intervall (−∞, ∞) erfor-
für N → ∞ im Widerspruch zu xf ∈ L2 (0, 1).
dert einige weitere Überlegungen und kann mithilfe der
sogenannten generierenden Funktion oder mithilfe der Bemerkung: Die Aussage lässt sich wie folgt verallgemei-
Fouriertransformation gezeigt werden (siehe etwa Cou- nern: Mψ , Mψ−1 ∈ L(L2 (I ), L2 (I )) genau dann, wenn
rant, Hilbert; Methoden der Mathematischen Physik). ψ, 1/ψ ∈ L∞ (I ) gilt.
Kapitel 11 Beweisaufgaben
Aufgabe 11.5 •• Seien @ x2 , . . . , @
x1 , @ xn Approximatio-
nen an die reellen Zahlen x1 , x2 , . . . , xn und der maximale
Aufgaben Fehler sei in jedem Fall e. Zeigen Sie, dass die Summe

n
Verständnisfragen @
xi
i=1
Aufgabe 11.1 • Auf einer Maschine, die im Dezimal-
einen maximalen Fehler von ne aufweist.
system mit 4 Stellen und maximal 2 Stellen im Exponenten
rechnet, soll die Zahl 0.012345·10−99 in normalisierter Form
Aufgabe 11.6 •• Für die Summe der ersten n Quadrat-
dargestellt werden. Ist das auf dieser Maschine möglich?
zahlen, n ∈ N, gilt
Aufgabe 11.2 •• Gegeben sei die für alle x ∈ R defi-
n  
1 1
nierte Funktion Sn := i2 = n n+ (n + 1).
3 2
i=1
1 3 1 2 1
Sx := x + x + x. Zeigen Sie
3 2 6
 
Gilt Sx = O(x 3 ) oder Sx = O(x)? Ist eine solche Frage (a) Sn = O n3 ,
 
ohne die Angabe x → x0 überhaupt sinnvoll? (b) Sn = 13 n3 + O n2 ,
 42 
(c) Sn = O n ,
Aufgabe 11.3 •• Warum eignet sich die Potenzreihe
und diskutieren Sie, welche „Güte“ diese drei Abschätzungen
x2 x4 x6 relativ zueinander haben.
cos x = 1 − + − ± ...
2! 4! 6!
Aufgabe 11.7 • Zeigen Sie, dass der Diskretisierungs-
ganz hervorragend zur numerischen Berechnung von cos 0.5, fehler der zentralen Differenz
aber überhaupt nicht zur Berechnung von cos 2?
u(x + h) − 2u(x) + u(x − h)
Du :=
Aufgabe 11.4 ••• Wir wollen eine Differenzialglei- h2
chung von zweiter Ordnung ist, wenn man mit Du die zweite Ablei-
y  (x) = f (x, y) tung u = d2 u/dx 2 einer glatten Funktion u approximiert.
auf dem Intervall [0, 1] numerisch lösen, wobei eine An-
fangsbedingung y(0) = y0 gegeben sei und die Lösung bei
Rechenaufgaben
x = 1 gesucht ist. Die Schrittweite des Gitters G sei h = 1/n,
wobei n ∈ N frei wählbar ist. Das Gitter ist damit gegeben Aufgabe 11.8 •• Die Zahl π wird durch die rationalen
als Zahlen
G := {kh | k = 0, 1, . . . , n} . 22 355
x1 = und x2 =
7 113
Die numerische Methode soll das einfache Euler’sche Poly-
angenähert.
gonzugverfahren sein, das durch
Y (kh) − Y ((k − 1)h) (a) Wie groß sind die absoluten Fehler?
= f (Y ((k − 1)h)) (b) Welcher Fehler hat der mit x1 bzw. x2 berechnete Kreis
h
vom Durchmesser 10 m?
gegeben ist. Der Anfangswert für das Verfahren ist Y0 , die
Projektion von y0 auf das Gitter (Ein Anfangswert y0 = π Runden Sie jeweils auf 6 Nachkommastellen.
ist numerisch auf einer Maschine nicht realisierbar, daher ist
Y0 eine Approximation an π im Rahmen der Darstellbarkeit Aufgabe 11.9 •• Berechnen Sie auf einem Taschen-
der Maschinenzahlen). rechner oder mithilfe eines Computers die Summe

100 √
Im Hinblick auf Abbildung 11.3 seien die Räume E und F k,
mit den jeweiligen Normen als k=1

E = C 1 ([0, 1]),yE := max |y(x)|, in dem Sie jede Wurzel mit nur zwei Nachkommastellen be-
x∈[0,1] rechnen. Welchen Gesamtfehler erwarten Sie im Hinblick auf
/ /
/ d0 / Aufgabe 11.5?
F = R × C([0, 1]), // / := |d0 | + max |d(x)|
d /F x∈[0,1]
Aufgabe 11.10 •• Gegeben sei das Gleichungssystem
definiert. Geben Sie die Operatoren L1 , L2 , φh , T und Th an, Ax = b,     
sodass das Diagramm in Abbildung 11.3 die Diskretisierung 1 1 x 22
2 1 = 43 .
der Euler’schen Polygonzugmethode zeigt. 21 9 y 20

M. Brokate et al., Arbeitsbuch Grundwissen Mathematikstudium – Höhere Analysis, Numerik und


Stochastik, DOI 10.1007/978-3-642-54946-5_10, © Springer-Verlag Berlin Heidelberg 2016
86 Lösungswege zu Kapitel 11

(a) Berechnen Sie die Kondition der Koeffizientenmatrix Lösungen


mit der Frobenius-Norm.
(b) Betrachten Sie die zwei Zeilen des Gleichungssystems
Verständnisfragen
als Geradengleichungen. Was bedeutet die Kondition
geometrisch für die beiden Geraden? Aufgabe 11.1 • Die auf das Maschinenformat gerun-
dete Zahl lautet 0.1235 · 10−100 und ist damit keine Maschi-
Aufgabe 11.11 •• Sei A ∈ R2×2 die Matrix nenzahl.
 
A=
1 1
. Aufgabe 11.2 •• –
1 0
Aufgabe 11.3 •• –
Berechnen Sie den Spektralradius und die Spektralnorm
von A. Aufgabe 11.4 ••• –

Beweisaufgaben
Hinweise Aufgabe 11.5 •• –

Aufgabe 11.6 •• –
Verständnisfragen
Aufgabe 11.1 • Runden Sie die Zahl auf das Format Aufgabe 11.7 • –
der Maschine.
Rechenaufgaben
Aufgabe 11.2 •• Bedenken Sie, für welches x0 in
x → x0 Sie eine asymptotische Aussage treffen möchten. Aufgabe 11.8 •• –

Aufgabe 11.3 •• Die Fehlerabschätzung im Leibniz- Aufgabe 11.9 •• –


Kriterium für alternierende Reihen ist hilfreich.
Aufgabe 11.10 •• –

 11.4 ••• Definieren


Aufgabe  Sie
Aufgabe 11.11 •• –
y(0)−y0
Ty =  .
y (x)−f (x, y)

Beweisaufgaben Lösungswege
Aufgabe 11.5 •• –
Verständnisfragen
Aufgabe 11.6 •• Multiplizieren Sie die Faktoren in der Aufgabe 11.1 • –
Definition von Sn aus und schätzen dann ab.
Aufgabe 11.2 •• Natürlich muss immer die Angabe
Aufgabe 11.7 • Taylor-Entwicklung. x → x0 Aussagen mit den Landau-Symbolen begleiten.
Reflektieren Sie die Definition: Sx = O(x 3 ) für x → x0
bedeutet die Existenz einer Konstanten c und eines ε > 0,
Rechenaufgaben sodass
|Sx | ≤ c|x 3 |
Aufgabe 11.8 •• –  
 
für alle x mit |x − x0 | < ε gilt. Es muss also  Sxx3  eine
Aufgabe 11.9 •• Wenn jede Wurzel nur auf zwei Stel- Konstante sein. Wegen
len genau berechnet wird, dann ist der maximale Fehler jedes
Summanden 0.005. Sx 1 1 1
= + + 2
x3 3 2x 6x
Aufgabe 11.10 •• (b). Berechnen Sie den Schnittwin- ist aber Sx /x 3 unbeschränkt für x → 0. Andererseits gilt
kel der beiden Geraden. limx→∞ Sx /x 3 = 13 . Es gilt also

Aufgabe 11.11 •• – Sx = O(x 3 ), x → ∞,


Lösungswege zu Kapitel 11 87

aber sicher nicht für x → 0. Ganz analog sehen wir, dass Beweisaufgaben

Sx = O(x), x → 0, Aufgabe 11.5 •• Wir können für i = 1, 2, . . . , n

xi − e ≤ xi ≤ @
@ xi + e
aber das gilt sicher nicht für x → ∞, da in diesem Fall Sx /x
unbeschränkt ist.
schreiben. Summation über i liefert

Aufgabe 11.3 •• Das Leibniz-Kriterium für alternie-


n
n
n
@
xi − ne ≤ xi ≤ @
xi + ne,
rende Reihen lehrt, dass der Fehler bei Abbruch der Sum-
i=1 i=1 i=1
mation nicht größer ist als der erste fortgelassene Summand.
Im Fall von x = 0.5 erhalten wir also

n
n
cos 0.5 = 1 − 0.125 + 0.0026041 − 0.000217 ± . . . −ne ≤ xi − @
xi ≤ ne,
i=1 i=1
≈ 0.877582.
was zu beweisen war.
Der erste fortgelassene Summand ist x 8 /8! ≈ 0.0000001
und der Fehler der obigen Formel ist damit kleiner als dieser Aufgabe 11.6 •• Es ist Sn = 13 n3 + 21 n2 + 16 n.
Wert! Alle Nachkommastellen sind demnach korrekt.
(a) Eine Abschätzung mit der Dreiecksungleichung liefert
Im Fall x = 2 ist ≈ 0.00635 und die obige Summe
x 8 /8!  
 1 3 1 2 1  1  3 1  2 1  
mit nur vier Summanden wäre höchstens in der ersten Nach-  n + n + n ≤ n  + n  + n
3 2 6  3 2 6
kommastelle korrekt. Um die gleiche Genauigkeit wie für
x = 0.5 zu erreichen, muss bis x 13 /13! summiert werden, 1  3  1  3  1  3 
≤ n + n + n
denn x 14 /14! ≈ 0.0000002. 3
 3
2 6

= n . 
Aufgabe 11.4 ••• Ist y ∈ E, dann ist
Natürlich kann der Betrag entfallen, da n ∈ N und alle
  Ausdrücke positiv sind.
y(0) − y0
Ty = ∈ F. (b) Folgt aus (a), denn |Sn | ≤ 13 |n3 | + 21 |n2 | + 16 |n| und
y  (x) − f (x, y)
1 2 1
2 |n | + 6 |n| = O (n ).
2

Für die Operatoren L1 und L2 gilt (c) Da n ∈ N, gilt für alle k ≥ 3 natürlich Sn = O(nk ), also
auch für k = 42.
L1 y(kh) = y(kh),
 Die Angabe (b) ist spezifischer als (a). Abschätzung (c) gilt,
d0 ; k = 0, aber ist sehr grob.
L2 d = ,
d((k − 1)h) ; k = 1, 2, . . . , n
Aufgabe 11.7 • Die Taylorentwicklungen
wobei
 
d0 h2  h3 
d := . u(x +h) = u(x) + hu (x) + u (x) + u (x) + O(h4 ),
d(x) 2 3!
h2 h3 
Schließlich ist der Operator φh gegeben durch u(x −h) = u(x) − hu (x) + u (x) − u (x) + O(h4 )
2 3!
φh (T )(kh) = liefern

⎨Y0 − y0 ; k =0
Y (kh) − Y ((k − 1)h) u(x + h) + u(x − h) = 2u(x) + h2 u (x) + O(h4 ),
⎩ −f (Y ((k−1)h)) ; k = 1, 2, . . . , n
h
also
Die diskreten Räume Eh und Fh sind beides Räume von
u(x + h) − 2u(x) + u(x − h)
Funktionen vom Gitter nach R, jeweils mit den konkordanten Du = = u (x) + O(h2 ).
Normen h2
Benutzen wir den Operator Lu(x) := uh (x) := u(x)|G ,
Y Eh := max |Y (kh)|, um unsere kontinuierlichen Größen auf ein Gitter G mit Ma-
k=0,1,...,n
schenweite h zu transferieren, dann folgt für den Approxi-
δFh := |δ0 | + max |δ(kh)|
k=0,1,...,n mationsfehler

versehen. Duh − (d2 u/dx 2 )h Fh = O(h2 ).


88 Lösungswege zu Kapitel 11

Rechenaufgaben α = arctan(m1 ) − arctan(m2 )


 
Aufgabe 11.8 •• 18
= arctan(−1) − arctan −
21
(a) Für die absoluten Fehler ergeben sich auf sechs Stellen
gerundet = −45◦ + 40.6013◦ = −4.3987◦ .

22
φ1 = x1 − π = − 3.141593 = 0.001264,
7 15

355
φ2 = x2 − π = − 3.141593 = 0. 10

113
5

(b) Für den Umfang (2πr) des Kreises mit Radius r = 5 m


gilt U (π) = 10π m, also U (xi ) = 10xi . Nach dem 0 -5 0 5 10 15 20 25 30 35 40 45 5

Fortpflanzungsgesetz für absolute Fehler gilt -5

• 
Ui = U (xi )φi , -10

also für i = 1 -15


U1 = 10 m · 0.001264 = 0.01264 m.
Dieser harmlos erscheinende Wert des Schnittwinkels
Wir erhalten damit im Fall von x1 die Abschätzung
erweist sich erst bei einer Veranschaulichung der bei-
10π m − 1.26 cm ≤ U ≤ 10π m − 1.26cm. den Geraden als das eigentliche Problem. Die schlechte
Kondition des Gleichungssystems bedeutet geometrisch
Im Fall von x2 erhalten wir (im Rahmen der gewählten einen sehr kleinen Schnittwinkel der beiden Geraden.
Genauigkeit) das exakte Ergebnis.
100 √ Aufgabe 11.11 •• Die Eigenwerte von A sind die Null-
Aufgabe 11.9 •• Sie müssten k=1 k ≈ 671.38 stellen des charakteristischen Polynoms
erhalten haben. Da Sie nur mit zwei Nachkommastellen  
gerechnet haben, ist der maximale Fehler je Summand  1−λ 1 
pA (λ) =   = λ2 − λ − 1.
e = 0.005. Nach Aufgabe 11.5 erhalten wir damit den Ma- 1 −λ 
ximalfehler 100 · 0.005 = 0.5, sodass unsere Summe nicht
Mit quadratischer Ergänzung folgt für die Nullstellen
einmal in der ersten Nachkommastelle richtig ist!
 
1 2 5
Aufgabe 11.10 •• λ− − = 0,
    2 4
1 1 7 −63
(a) Die Inverse von A = ist A−1 =
. also
2 1 −6 63 1 1√ 1 √
21 9
Dementsprechend lauten die Frobenius-Normen ±
λ1,2 = 5 = (1 ± 5).
2 2 2
A
B Der Spektralradius ist der betragsgrößte Eigenwert von A,
B 2 2
B 4 1 hier also

AF = C |aij |2 = 1+1+ + = 2.021416 1
441 81 ρ(A) = (1 + 5).
i=1 j =1 2
Zur Berechnung der Spektralnorm benötigen wir das Produkt
und entsprechend A−1 F = 49 + 26569 + 36 +     
26569 = 53223. Damit ergibt sich für die Kondition A∗ A =
1 1 1 1
=
2 1
.
1 0 1 0 1 1
κ(A) = AF A−1 F = 107585.824.
Wieder benötigen wir die Eigenwerte aus den Nullstellen des
Man würde das Gleichungssystem also als schlecht kon- charakteristischen Polynoms
ditioniert bezeichnen.  
(b) Das Gleichungssystem lösen ist äquivalent zur Berech- 3 2 5
pA∗ A (λ) = (2−λ)(1−λ)−1 = λ2 −3λ+1 = λ − − ,
nung des Schnittpunktes der beiden Geraden 2 4

x + y = 22, also √
3 1√ 3± 5
2 1 43 λ1,2 = ± 5= .
x+ y= . 2 2 2
21 9 20 √
Die Steigung der ersten Geraden ist m1 = −1 wegen Damit ist ρ(A∗ A) = 3+ 5
und wir erhalten
2
y = −x + 22, die Steigung der zweiten Geraden ist ? √
 3+ 5
m2 = −18/21 wegen y = − 18 387
21 x + 20 . Ihr Schnittwin- ∗
A2 = ρ(A A) = .
kel ist mithin 2
Kapitel 12 Aufgabe 12.7 ••• Beweisen Sie die Hermite’sche Dar-
stellung III. der dividierten Differenzen aus der Hinter-
grund-und-Ausblick-Box auf Seite 411: Ist die Abbildung
Aufgaben u : Rn → R definiert durch

un := u(t1 , . . . , tn ) :=(1 − t1 )x0 + (t1 − t2 )x1 + . . .


Verständnisfragen + (tn−1 − tn )xn−1 + tn xn ,
Aufgabe 12.1 • Welche der folgenden Polynome sind
dann gilt
Monome?
1 t1
(a) x 3 − 2x + 1 tn−1 dn f
f [x0 , . . . , xn ] = ··· (un ) dtn . . . dt1 .
(b) −42x 7 0 0 0 dun
(c) x 12
(d) 4x − 1
Aufgabe 12.8 • Gegeben sind die n + 1 Daten
Aufgabe 12.2 • Wie heißt das Interpolationspolynom
zu den Daten (0, 3), (1, 3), (1.25, 3), (4.2, 3)? (x0 , f (x0 )), (x0 , f  (x0 )), (x0 , f  (x0 )), . . . , (x0 , f (n) (x0 )).

Zeigen Sie, dass die Lösung des Interpolationsproblems zu


Aufgabe 12.3 • Die auf [0, 1] definierte Funktion
diesen Daten das Taylor-Polynom vom Grad nicht größer als
f (x) = 4x 4 −3x ist stetig. Wie lautet die Bestapproximation
n ist.
p∗ ∈ 4 ([0, 1])?

Aufgabe 12.4 •• In der Definition der Bestapproxima- Rechenaufgaben


tion tauchen Polynome aus dem Raum n ([a, b]) auf, also
solche mit Grad nicht kleiner als n. Im Weierstraß’schen Ap- Aufgabe 12.9 •• Sie wollen äquidistante Funktions-

proximationssatz wird jedoch die Existenz eines Polynoms werte von f (x) = x ab x0 = 1 tabellieren. Welche Schritt-
aus ([a, b]), dem Raum aller Polynome, postuliert. Erklä- weite h dürfen Sie maximal wählen, damit ein kubisches Po-
ren Sie diesen Unterschied. lynom noch auf fünf Nachkommastellen genau interpoliert?

Aufgabe 12.10 • Berechnen Sie das Langrange’sche


Beweisaufgaben
Interpolationspolynom zu den Daten
Aufgabe 12.5 ••• Zeigen Sie die Darstellung I. der di-
k 0 1 2
vidierten Differenzen und die Folgerung daraus aus der
Hintergrund-und-Ausblick-Box auf Seite 411: xk 0 1 3
fk 1 3 2
Darstellung I.
(a) über die Vandermonde’sche Matrix,

i+k
f (x ) (b) mithilfe der Lagrange’schen Basispolynome.
f [xi , . . . , xi+k ] = , k ≥ 1.
9
i+k
=i (x − xm ) Aufgabe 12.11 • Werten Sie das Langrange’sche In-
m=i
m=
terpolationspolynom aus Aufgabe 12.10 an der Stelle x = 2
9i+k mithilfe des Neville-Tableaus aus.
Folgerung: Ist ωi+k (x) := m=1 (x − xm ), dann gilt
Aufgabe 12.12 • Berechnen Sie das Newton’sche In-

i+k
f (x ) terpolationspolynom zu der Wertetabelle
f [xi , . . . , xi+k ] = d
.
ω (x )
=i dx i+k  k 0 1 2 3 4
xk 2 4 6 8 10
Aufgabe 12.6 ••• Beweisen Sie die Darstellung II. der fk 0 0 1 0 0
dividierten Differenzen aus der Hintergrund-und-Ausblick-
Box auf Seite 411: Ist V (x0 , . . . , xn ) die Vandermonde’sche Aufgabe 12.13 ••• Berechnen Sie den kubischen Spline,
Matrix, dann gilt der f (x) = sin x im Intervall [0, π] interpoliert. Benutzen
  Sie dazu nur die beiden inneren Punkte x2 = π/3 und x3 =
 1 1 ... 1 
  2π/3. Die Randpunkte sind demnach x1 = 0 und x4 = π .
 x0 x1 . . . xn 

 .. .. .. ..  Damit ergibt sich die folgende Datentabelle.
 . . . . 

 x n−1 x n−1 . . . x n−1  k 1 2 3 4
 0 n 
 f (x ) f 1(x ) . . . f (x )  xk 0 π/3 2π/3 π
f [x0 , . . . , xn ] =
0 1 n
. √ √
det V (x0 , . . . , xn ) fk 0 3/2 3/2 0

M. Brokate et al., Arbeitsbuch Grundwissen Mathematikstudium – Höhere Analysis, Numerik und


Stochastik, DOI 10.1007/978-3-642-54946-5_11, © Springer-Verlag Berlin Heidelberg 2016
90 Lösungen zu Kapitel 12

Überlegen Sie sich zuerst, welchen Spline Sie auf Anhieb nach der (n+1)-ten Zeile und beweisen
verwenden würden, den natürlichen oder den vollständigen?
Berechnen Sie auf jeden Fall beide Arten von Splines und
n 
D= (−1)n++2 f (x ) (xj − xi ).
vergleichen Sie die Ergebnisse. 0≤i<j ≤n
=0
i,j =

Aufgabe 12.14 •• Gegeben seien die äquidistanten Da-


ten Aufgabe 12.7 ••• Vollständige Induktion über n.
i 0 1 2 3 4
Aufgabe 12.8 •
 Verwenden Sie den Ansatz p(x) =
xi 0 2π /5 4π/5 6π/5 8π/5 n
i=0 ai (x − x0 ) .
i
fi 0 2 1 2 1
einer auf [0, 2π] definierten periodischen Funktion f . Be- Rechenaufgaben
rechnen Sie das trigonometrische Interpolationspolynom.
Stellen Sie die gegebenen Daten und das trigonometrische Aufgabe 12.9 •• Verwenden Sie die Fehlerdarstellung
Polynom dar, indem Sie das berechnete Polynom an 2000 (12.22).
Stellen zwischen 0 und 2π auswerten.
Aufgabe 12.10 • –

Aufgabe 12.11 • –
Hinweise
Aufgabe 12.12 • –
Verständnisfragen Aufgabe 12.13 ••• Die zweite Ableitung von sin x ver-
Aufgabe 12.1 • – schwindet an den Rändern. Daher läge es nahe, zuerst den
natürlichen Spline zu berechnen.
Aufgabe 12.2 • –
Aufgabe 12.14 •• Verwenden Sie (12.57) und (12.58).
Aufgabe 12.3 • –

Aufgabe 12.4 •• –

Lösungen
Beweisaufgaben
Aufgabe 12.5 ••• Verwenden Sie vollständige Induk- Verständnisfragen
tion über k.
Aufgabe 12.1 • Die Polynome (b) und (c) sind Mo-
9 nome. Die Polynome (a) und (d) bestehen aus Summen von
Aufgabe 12.6 ••• Beginnen Sie mit P := 0≤i<j ≤n
Monomen, sind also selbst keine Monome.
(xj − xi ) und zeigen Sie zuerst, dass für 0 ≤  ≤ n
Aufgabe 12.2 • Da alle Daten auf einer horizontalen
P =(x − x0 )(x − x1 ) · . . . · (x − x−1 )
Geraden liegen, ist das eindeutig bestimmte Interpolations-
· (−1)n− [(x − x+1 ) · . . . · (x − xn )] polynom p(x) = 3.

· (xj − xi )
0≤i<j ≤n Aufgabe 12.3 • Da f ein Polynom vom Grad 4 ist, ist
i,j = p∗ = f .

n 
=(−1)n− (x − xm ) · (xj − xi ) Aufgabe 12.4 •• –
m=0 0≤i<j ≤n
m= i,j =

gilt. Setzen Sie dieses Resultat in Darstellung I. ein. Dann Beweisaufgaben


entwickeln Sie Aufgabe 12.5 ••• –
 
 1 1 ··· 1 
  ••• –
 x0 x ··· xn  Aufgabe 12.6
 1
 .. .. .. .. 
D :=  . . . .  ••• –
 Aufgabe 12.7
 x n−1 x n−1 ··· n−1 
 0 xn 
 f (x ) f 1(x ) ··· f (xn )  Aufgabe 12.8 • –
0 1
Lösungswege zu Kapitel 12 91

Rechenaufgaben Für k = 2 finden wir


Aufgabe 12.9 •• –
f [xi+1 , xi+2 ] − f [xi , xi+1 ]
f [xi , xi+1 , xi+2 ] =
Aufgabe 12.10 • – xi+2 − xi

Aufgabe 12.11 • – f [xi+2 ] − f [xi+1 ]


=
(xi+2 − xi+1 )(xi+2 − xi )
Aufgabe 12.12 • –
f [xi+1 ] − f [xi ]
− .
Aufgabe 12.13 ••• – (xi+1 − xi )(xi+2 − xi )

Aufgabe 12.14 •• –
Durch Auseinanderziehen und Erweitern in der Form

f (xi+2 )
Lösungswege f [xi , xi+1 , xi+2 ] = −
(xi+2 − xi+1 )(xi+2 − xi )
f (xi+1 )(xi+1 − xi )
Verständnisfragen
(xi+2 − xi+1 )(xi+2 − xi )(xi+1 − xi )
Aufgabe 12.1 • –
f (xi+1 )(xi+2 − xi+1 )

Aufgabe 12.2 • – (xi+1 − xi )(xi+2 − xi )(xi+2 − xi+1 )

Aufgabe 12.3 • – f (xi )


+
(xi+1 − xi )(xi+2 − xi )
Aufgabe 12.4 •• Wir hätten den Weierstraß’schen Ap-
proximationssatz auch so formulieren können: Zu jeder auf
erhalten wir schließlich
[a, b] stetigen Funktion f und für alle ε > 0 gibt es ein
n = n(ε) ∈ N und ein Polynom pn vom Grad nicht höher als
n, sodass f (xi )
f [xi , xi+1 , xi+2 ] =
|f (x) − pn (x)| < ε (xi − xi+1 )(xi − xi+2 )
für alle x ∈ [a, b] gilt, also f (xi+1 )
+
f − pn ∞ < ε. (xi+1 − xi )(xi+1 − xi+2 )
f (xi+2 )
In dieser Formulierung sieht man, dass der Grad des Poly- +
(xi+2 − xi )(xi+2 − xi+1 )
noms natürlich nicht fest ist, sondern mit der Vorgabe des
ε > 0 variiert. Im Gegensatz dazu sagt die Existenz der Be-
i+2
f (x )
stapproximation, dass man unter allen p ∈ n ([a, b]) stets = .
ein Polynom p ∗ findet, sodass 9
i+2
=i (x − xm )
m=i

f − p ∞ = min f − p∞ m=
p∈n ([a,b])

gilt. Ist f selbst kein Polynom, dann kann man die rechte Jetzt zum Induktionsschluss. Die Behauptung sei richtig
Seite nicht kleiner als ein beliebig wählbares ε > 0 machen. für

Beweisaufgaben
i+k−1
f (x )
Aufgabe 12.5 ••• Wir beweisen durch Induktion über k. f [xi , . . . , xi+k−1 ] =
9
i+k−1
Für k = 1 gilt =i (x − xm )
m=i
f [xi+1 ] − f [xi ] m=
f [xi , xi+1 ] =
xi+1 − xi
f (xi ) f (xi+1 ) und
= +
xi − xi+1 xi+1 − xi

i+1
f (x )
i+k
f (x )
= . f [xi+1 , . . . , xi+k ] = .
9
i+1 9
i+k
=i (x − xk ) =i+1 (x − xm )
k=i m=i+1
k= m=
92 Lösungswege zu Kapitel 12

Nach Definition der dividierten Differenzen gilt f (xi ) f (xi+k )


f [xi , . . . , xi+k ] = +
9
i+k 9
i+k
(xi − xm ) (xi+k − xm )
m=i m=i
f [xi+1 , . . . , xi+k ] − f [xi , . . . , xi+k−1 ] m=i m=i+k
f [xi , . . . , xi+k ] =
xi+k − xi
i+k−1
f (x )
f [xi+1 , . . . , xi+k ] f [xi , . . . , xi+k−1 ] +
= − 9
i+k
xi+k − xi xi+k − xi
=i+1 (x − xm )
m=i
m=

i+k
f (x )
= ⎛ ⎞
i+k
f (x )
9
i+k = .
=i+1
⎝ (x − xm )⎠ (xi+k − xi ) 9
i+k
=i (x − xm )
m=i+1 m=i
m= m=


i+k−1
f (x )
− ⎛ ⎞ . 9
Aufgabe 12.6 ••• Sei P := 0≤i<j ≤n (xj − xi ). Für
=i 9
i+k−1
⎝ (x − xm )⎠ (xi+k − xi ) 0 ≤  ≤ n bemerken wir
m=i
m=
P =(x1 − x0 )(x2 − x0 ) · . . . · (x − x0 ) · . . . · · · (xn − x0 )·
· (x2 − x1 )(x3 − x1 ) · . . . · (x − x1 ) · . . . · (xn − x1 )·
Lösen wir die beiden Summen auf in der Form · ···
· (x − x−1 )(x+1 − x−1 ) · . . . · (xn − x−1 )·
f (xi )
f [xi , . . . , xi+k ] = ⎛ ⎞ · (x+1 − x ) (x+2 − x ) · . . . · (xn − x )·
9
i+k−1
⎝ (xi − xm )⎠ (xi − xi+k ) · (x+2 − x+1 )(x+3 − x+1 ) · . . . · (xn − x+1 )·
m=i
m=i · ···
f (xi+k ) · (xn − xn−1 )
+⎛ ⎞
= (x − x0 )(x − x1 ) · . . . · (x − x−1 )
9
i+k   
⎝ (xi+k − xm )⎠ (xi+k − xi ) Alle unterstrichenen Terme der ersten  Zeilen
m=i+1 6 7
m=i+k · (−1)n− (x − x+1 ) · . . . · (x − xn )
⎛   
Alle unterstrichenen Terme in Zeile
⎜  + 1 mit jeweils reversem Vorzeichen



i+k−1
⎜ f (x )(x − xi ) · (xj − xi )
+ ⎜⎛ ⎞ 0≤i<j ≤n
⎜ i+k
=i+1 ⎜ 9 
i,j =
 
⎝⎝ (x − xm )⎠ (xi+k − xi )(x − xi )
m=i+1 Alle nichtunterstrichenen Terme
m=
⎞ Schreiben wir unsere Umformung in etwas abgekürzter Form
auf, dann ergibt sich

⎟  
n
f (x )(x − xi+k ) ⎟
⎟ (xj − xi ) = (−1)n− (x − xm )
− ⎛ ⎞ ⎟,
⎟ 0≤i<j ≤n
9
i+k−1 ⎟ m=0
⎝ (x − xm )⎠ (xi+k − xi )(x − xi+k ) ⎠ 
m=

m=i
m= · (xj − xi ).
0≤i<j ≤n
i,j =

dann ergibt sich die letzte große Klammer in der Summe Setzen wir die eben gewonnene Darstellung in Darstellung I
gerade zu ein, dann folgt

n
f (x )
f (x ) f [x0 , . . . , xn ] = (12.9)
9
n
9
i+k =0 (x − xm )
(x − xm ) m=0
m=i m=
m= 9
(−1)n− f (x ) (xj − xi )
0≤i<j ≤n

n
i,j =
und eine Indexverschiebung in den beiden ersten Summan- = 9 .
(xj − xi )
=0
den führt dann auf 0≤i<j ≤n
Lösungswege zu Kapitel 12 93

Wir untersuchen nun Darstellungen einer Determinante. Ent- Nun sei die behauptete Darstellung richtig für n. Es gilt
wicklung nach den Elementen der (n + 1)-ten Zeile von tn
f (n+1) (un+1 ) dtn+1
 
 1 1 ··· 1 
0
tn
  f (n) ((1−t1 )x0 +(t1 −t2 )x1 +. . .+(tn −tn+1 )xn +tn+1 xn+1 ) 
 x0 x1 · · · xn  = 
 xn+1 −xn 
 . .. .. ..  tn+1 =0
D :=  .. . . . 
 f (n) ((1−t1 )x0 +(t1 −t2 )x1 +. . .+(tn−1 −tn )xn−1 +tn xn+1 )
 x n−1 x n−1 · · · x n−1  =
 0 n  xn+1 −xn
 f (x ) f 1(x ) · · · f (x ) 
0 1 n f (n) ((1 − t1 )x0 + (t1 − t2 )x1 + . . . + (tn−1 − tn )xn−1 + tn xn )

xn+1 − xn
liefert
  f (n) (un )
 1  =
··· 1 xn − xn+1
 
 x1 · · · xn  f (n) ((1 − t1 )x0 + (t1 − t2 )x1 + . . . + (tn−1 − tn )xn−1 + tn xn+1 )
  −
D = (−1)n+2 f (x0 )  .. .. .. 
 xn − xn+1
 . . . 
 ..  und damit folgt
 x n−1 . xnn−1  1 t1
1 tn
  ··· f (n+1) (un+1 ) dtn+1
 1 1 ··· 1 
  0 0 0
 .. 
 x0 x2 . xn  f [x0 , . . . , xn−1 , xn ] − f [x0 , . . . , xn−1 , xn+1 ]
+ (−1) f (x1 )  .
n+3 
 = ,
 ..
.. .. ..
 xn − xn+1
 . . . 
 x n−1 x2n−1 · · · xn
n−1  was nach dem Satz über die Symmetrie der dividierten Dif-
0
ferenzen auf Seite 410 dasselbe ist wie
+ ... f [xn , x1 , x2 , . . . ,xn−1 , x0 ]−f [x1 , x2 , . . . ,xn−1 , x0 , xn+1 ]
  .
 1 ··· 1  xn − xn+1
 
 x0 · · · xn−1 
2n+2   Nun gilt
+ (−1) f (xn )  .. .. .. .
 . . . 
  f [xn , x1 , . . . , xn−1 , x0 , xn+1 ]
 x n−1 n−1
· · · xn−1 
0 f [x1 , . . . , xn−1 , x0 , xn+1 ] − f [xn , x1 , . . . , xn−1 , x0 ]
=
Wir haben damit die Darstellung xn+1 − xn
und wiederum wegen des Satzes über die Symmetrie der di-

n 
D= (−1)n++2 f (x ) (xj − xi ) vidierten Differenzen
=0 0≤i<j ≤n
i,j =
f [xn , x1 , . . . , xn−1 , x0 , xn+1 ] = f [x0 , . . . , xn+1 ].
Damit folgt
gewonnen. Ist n +  ungerade, dann auch n − , und ist n +  1 t1 tn
gerade, dann auch n − . Wir haben daher
··· f (n+1) (un+1 ) dtn+1 . . . dt1 = f [x0 ,. . . ,xn+1 ].
0 0 0

n 
D= (−1)n− f (x ) (xj − xi )
=0 0≤i<j ≤n
Aufgabe 12.8 • Mit dem Ansatz
i,j =

n

und setzen wir dies nun in (12.9) ein, dann ergibt sich die p(x) = ai (x − x0 )i
gesuchte Darstellung i=0
= a0 + a1 (x − x0 ) + a2 (x − x0 )2 + . . .
D
f [x0 , . . . , xn ] = . + an (x − x0 )n
det V (x0 , · · · , xn )
folgt
p (x) = a1 + 2a2 (x − x0 ) + 3a3 (x − x0 )2 + . . .
Aufgabe 12.7 ••• Der Beweis der Hermite’schen Dar-
stellung wird mit vollständiger Induktion geführt. Für n = 1 + nan (x − x0 )n−1
erhalten wir p (x) = 2a2 + 2 · 3a3 (x − x0 ) + . . .
1 1
+ n(n − 1)(x − x0 )n−2 ,
f  (u1 ) dt1 = f  ((1 − t1 )x0 + t1 x1 ) dt1
0 0 p (x) = 2 · 3a3 + 2 · 3 · 4a4 (x − x0 ) + . . .

f ((1 − t1 )x0 + t1 x1 ) 1 + n(n − 1)(n − 2)(x − x0 )n−3 ,
= 
x −x 1 0 t1 =0 ..
f (x1 ) − f (x0 ) .
= .
x1 − x 0 p (n) (x) =n!an .
94 Lösungswege zu Kapitel 12

!
Aus den Bedingungen p (k) (x0 ) = f (k) (x0 ) für k = 0, Wenn die Interpolation noch bis auf die fünfte Nachkomma-
1, . . . , n erhalten wir sukzessive stelle genau sein soll, dann muss

p(x0 ) = a0 = f (x0 ), |f (x) − p(x)| < 0.000005


p (x0 ) = a1 = f  (x0 ),
gelten, also
p  (x0 ) = 2a2 = f  (x0 ),
p  (x0 ) = 2 · 3a3 = f  (x0 ), 9 4 15 1
h · < 0.000005,
.. 16 16 24
.
woraus sich für h die Abschätzung h4 < 0.000228, also
p(n) (x0 ) = n!an = f (n) (x0 )
h < 0.1228 ergibt.
und damit das Taylor-Polynom
Aufgabe 12.10 • Bei drei gegebenen Daten ist ein

n
f (i) (x0 ) Polynom vom Grad höchstens 2 gesucht.
p(x) = (x − x0 )i .
i! (a) Der Ansatz
i=0

p(x) = a0 + a1 x + a2 x 2
Rechenaufgaben
führt mit den gegebenen Daten auf das Gleichungs-
Aufgabe 12.9 •• Wir betrachten die ersten vier Daten- system
punkte x0 = 1, x1 = 1 + h, x2 = 1 + 2h, x3 = 1 + 3h,
!
durch die ein Interpolationspolynom vom Grad drei eindeutig p(0) = a0 = f0 = 1,
bestimmt ist. Aus (12.22) erhalten wir !
  p(1) = a0 + a1 + a2 = f1 = 3,
 f (4) (ξ )  !
 p(3) = a0 + 3a1 + 9a2 = f2 = 2,
|f (x) − p(x)| = (x −x0 )(x −x1 )(x −x2 )(x −x3 ) 
 4! 
also ⎛ ⎞⎛ ⎞ ⎛ ⎞
für ein min{x, x0 , x1 , x2 , x3 } < ξ < max{x, x0 , x1 , x2 , x3 }. 1 0 0 a0 1
Um abschätzen zu können, wählen wir auf der rechten Seite ⎝1 1 1⎠ ⎝a1 ⎠ = ⎝3⎠ .
immer die maximalen Terme. Zur Berechnung des Maxi- 1 3 9 a2 2
mums von ω4 (x) = (x −1)(x −1−h)(x −1−2h)(x −1−3h)
leiten wir einmal ab und erhalten die drei reellen Nullstellen Als Lösung ergibt sich
√ √ ⎛ ⎞ ⎛ ⎞
h( 5 + 3) + 2 h( 5 − 3) − 2 a0 1
xN,1 = , xN,2 = − , ⎝a1 ⎠ = ⎝ 17 ⎠
2 2 6
3h + 2 a2 − 56
xN,3 = ,
2
und damit lautet das Interpolationspolynom
von denen xN,1 und xN,2 außerhalb des Interpolationsinter-
valls [1, 1 + 3h] liegen. Der Wert 17 5
p(x) = 1 + x − x2.
9 4 6 6
ω4 (xN,3 ) = h
16 (b) Die drei Lagrange’schen Basispolynome lauten
liefert daher das Maximum. Für die vierte Ableitung von x − x 1 x − x2 x−1 x−3

f (x) = x erhalten wir L0 (x) = · = ·
x0 − x1 x0 − x2 −1 −3
15 1 1 2
f (4) (x) = − √ , = (x − 4x + 3),
16 x 7 3
x − x0 x − x 2 x x−3
sodass das Maximum dieser vierten Ableitung bei x = x0 = 1 L1 (x) = · = ·
x1 − x0 x1 − x2 1 −2
angenommen wird. Mit 4! = 24 folgt damit 1
  = (3x − x 2 ),
 2
 f (4) (ξ )  x − x0 x − x1 x x−1
|f (x)−p(x)| = (x −x0 )(x −x1 )(x −x2 )(x −x3 )  L2 (x) = · = ·
 4!  x2 − x0 x2 − x1 3 2
9 4 15 1 1 2
≤ h · . = (x − x).
16 16 24 6
Lösungswege zu Kapitel 12 95

Damit ergibt sich das Lagrange’sche Interpolationspoly- Aufgabe 12.12 • Wir berechnen die dividierten Diffe-
nom zu renzen

p(x) = f (x0 )L0 (x) + f (x1 )L1 (x) + f (x2 )L2 (x) f [x0 ] = f0 = f1 = f [x1 ] = 0,
1 3 2 f [x2 ] = f2 = 1,
= (x 2 − 4x + 3) + (3x − x 2 ) + (x 2 − x)
3 2 6 f [x3 ] = f3 = f4 = f [x4 ] = 0,
5 2 17
= − x + x + 1. f [x1 ] − f [x0 ] 0−0
f [x0 , x1 ] = = = 0,
6 6 x1 − x0 4−2
f [x2 ] − f [x1 ] 1−0 1
f [x1 , x2 ] = = = ,
Aufgabe 12.11 • Die erste Spalte des Neville-Tab- x2 − x1 6−4 2
leaus f [x3 ] − f [x2 ] 0−1 1
f [x2 , x3 ] = = =− ,
x3 − x2 8−6 2
x0 F0,0 f [x4 ] − f [x3 ] 0−0
f [x3 , x4 ] = = = 0,
x4 − x3 10 − 8
x1 F1,0 f [x1 , x2 ] − f [x0 , x1 ]
f [x0 , x1 , x2 ] =
x2 − x0
x2 F2,0 1/2 − 0 1
= = ,
enthält die Werte Fj,0 = fj , j = 0, 1, 2, also 6−2 8
f [x2 , x3 ] − f [x1 , x2 ]
f [x1 , x2 , x3 ] =
x3 − x1
x0 = 0 1 −1/2 − 1/2 1
= =− ,
8−4 4
x1 = 1 3 f [x3 , x4 ] − f [x2 , x3 ]
f [x2 , x3 , x4 ] =
x4 − x2
x2 = 3 2
0 + 1/2 1
= = ,
Für die zweite Spalte sind die Größen 10 − 6 8
f [x1 , x2 , x3 ] − f [x0 , x1 , x2 ]
Fj,0 − Fj −1,0 f [x0 , x1 , x2 , x3 ] =
Fj,1 = Fj,0 + , j = 1, 2 x3 − x 0
x−xj −1
−1 −1/4 − 1/8 1
x−xj = =− ,
8−2 16
mit x = 2 zu berechnen. Das geht einfach durch Blick auf f [x2 , x3 , x4 ] − f [x1 , x2 , x3 ]
f [x1 , x2 , x3 , x4 ] =
das Anfangstableau: x4 − x 1
1/8 + 1/4 1
3−1 = = ,
F1,1 = 3 + = 5, 10 − 4 16
2−0
2−1 −1 f [x1 , x2 , x3 , x4 ] − f [x0 , x1 , x2 , x3 ]
f [x0 , x1 , x2 , x3 , x4 ] =
2−3 x4 − x0
F2,1 = 2 + = 2.5,
2−1
−1 1/16 + 1/16 1
2−3 = = .
10 − 2 64
womit wir Bei Handrechnung kann die Berechnung auch vorteilhaft im
m=1 Tableau der dividierten Differenzen erfolgen. Damit lautet
0 1 das Newton’sche Interpolationspolynom
5
1 3 p(x) =f [x0 ] + f [x0 , x1 ](x − x0 )
2.5 + f [x0 , x1 , x2 ](x − x0 )(x − x1 )
3 2 + f [x0 , x1 , x2 , x3 ](x − x0 )(x − x1 )(x − x2 )
erhalten. Nun bleibt uns nur noch, + f [x0 , x1 , x2 , x3 , x4 ](x − x0 )(x − x1 )(x − x2 )
· (x − x3 )
F2,1 − F1,1
F2,2 = F2,1 + 1 1
x−x0
x−x2 −1 = (x − 2)(x − 4) − (x − 2)(x − 4)(x − 6)
8 16
2.5 − 5 10 1
= 2.5 + 2−0
= = p(2). + (x − 2)(x − 4)(x − 6)(x − 8)
2−3 −1 3 64
1 3 49 39
= x 4 − x 3 + x 2 − x + 10.
zu berechnen. 64 8 16 4
96 Lösungswege zu Kapitel 12

1 lösen, also
p(x)
⎛ ⎞⎛ ⎞ ⎛ ⎞
0.8 2 1 S1 3
⎜π 4π π ⎟ ⎜S2 ⎟ 3 √ ⎜π ⎟
⎜3 ⎟⎜ ⎟ 3⎜ ⎟
π ⎠ ⎝S ⎠ =
0.6 3 3
⎝ π 4π
3 2π ⎝−π ⎠ .
3 3 3
0.4
2 1 S4 −3
p(x)

0.2
Als Lösungsvektor ergibt sich
0 ⎛ √ ⎞
⎛ ⎞ 15 3
–0.2
S1 ⎜ 38π√ ⎟
⎜S2 ⎟ ⎜ 3 ⎟
⎜ ⎟ = ⎜ 4π√ ⎟ .
–0.4 ⎝ S3 ⎠ ⎜ − 3 3 ⎟
⎝ 8π √ ⎠
S4 15 3
–0.6
2 3 4 5 6 7 8 9 10 − 4π
x
Nach (12.33) ist auf jedem [xi , xi+1 ], i = 1, 2, 3, ein kubi-
Abbildung 12.1 Das Newton’sche Interpolationspolynom zu den gegebenen sches Polynom der Form
Daten.
si (x) = c1,i + c2,i (x − xi ) + c3,i (x − xi )2 + c4,i (x − xi )3
Aufgabe 12.13 ••• Die zweite Ableitung der Sinusfunk- für i = 1, 2, 3 definiert. Aus (12.34) entnehmen wir
tion an den Rändern des Intervalls ist
c1,i = fi , c2,i = Si ,
− sin(0) = 0, − sin(π) = 0,
also
also bietet sich hier der natürliche Spline an. Mit den Be- √ √
3 3
zeichnungen aus (12.39)–(12.41) berechnen wir c1,1 = f1 = 0, c1,2 = f2 = , c1,3 = ,
2 2
π
x1 = x2 − x1 = √
3 15 3
π c2,1 = S1 = ,
l1 = x2 = x3 − x2 = 8π
3 √
π 3 3
l2 = x3 = x4 − x3 = c2,2 = S2 = ,
3 4π

4π 3 3
d1 = 2(x2 + x1 ) = c2,3 = S3 = − .
3 8π

d2 = 2(x3 + x2 ) = Die zwei verbleibenden Sätze von Koeffizienten berechnen
3 wir aus (12.35) und (12.36):
π
r1 = x1 =
3 3f2 − 3f1 − 2S1 x1 − S2 x1
π c3,1 =
r2 = x2 = (x1 )2
3 3
√ 30
√ π 3
√ π
2 3 − 8π 3 3 − 4π 3 3
=
und π2
⎛ ⎞ 9
3 fx22 −f 1 ⎛ f −f ⎞ = 0,
⎜  (f −f )x −x1 ⎟ 3 2 1
⎜3 3 2 1
+ (f2 −f1 )x 2 ⎟ ⎜ x2 −x1 ⎟
⎜ ⎟ = ⎜ 3(f3 − f1 )⎟
R + = ⎜  (f −f 2)x
x x1
⎜3 (f3 −f2 )x3 ⎟⎟

⎝3(f

4 − f2 )⎠
3f3 − 3f2 − 2S2 x2 − S3 x2

4 3
x3
2
+ x2 ⎠ c3,2 =
f4 −f3 (x2 )2
3 x4 −x3
3 fx44 −f
−x3
3
6
√ π 3
√ π
− 4π 3 3 + 8π 33
⎛ 9 √ ⎞ ⎛ ⎞ =
2π√ 3 3 π2
⎜ 3 3 ⎟ 3 √ ⎜π ⎟ √
9
=⎜ 2 √ ⎟
⎝ − 3 3 ⎠ = 2π 3 ⎝−π ⎠ .
⎜ ⎟
27 3
2 √ =− ,
− 2π9
3 −3 8π 2

3f4 − 3f3 − 2S3 x3 − S4 x3


Nach (12.44) müssen wir nun das lineare Gleichungssystem c3,3 =
(x3 )2
⎛ ⎞⎛ ⎞ √ √ π √ π
2 1 S1 − 23 3 + 8π
6 15
3 3 + 4π 33
⎜l1 d1 r1 ⎟ ⎜S2 ⎟ =
⎜ ⎟⎜ ⎟ + π2
⎝ l2 d2 r2 ⎠ ⎝S3 ⎠ = R 9
2 1 S4 = 0,
Lösungswege zu Kapitel 12 97

2f1 − 2f2 + S1 x1 + S2 x1 1


c4,1 =
(x1 )3 0.9
√ 15
√ 3
√ π
− 3 + 8π 3 π3 + 4π 33 0.8
=
π3 0.7
27
√ 0.6
27 3
=− , 0.5
8π 3
0.4
2f2 − 2f3 + S2 x2 + S3 x2
c4,2 = 0.3
(x2 )3
3
√ π 3
√ π 0.2
4π 3 3 − 8π 3 3
= 3
0.1
π
27 0

27 3 –0.1
= , 0 0.5 1 1.5 2 2.5 3
8π 3
2f3 − 2f4 + S3 x3 + S4 x3 Abbildung 12.2 Der natürliche Spline (rot) zu den Daten der Sinus-Funktion
c4,3 = (grün).
(x3 )3
√ 3
√ π 15
√ π
3 − 8π 3 3 − 4π 33
= 3 π
27 die Steigungen

81 3
=− .
8π 3 1 3 √
S1 = 1, S2 = − + 3, S3 = −S2 , S4 = −1.
Damit haben wir unsere drei kubischen Polynome gefunden. 3 2π
Für das Intervall [0, π/3]
√ √  
In analogen Rechnungen zum natürlichen Spline erhalten wir
15 √ 27 3 3 3 hier:
s1 (x) = 3x − 3
x = 3
15π 2 x − 27x 3 ,
8π 8π 8π
√ √
für das Intervall [π/3, 2π/3] 3 3
√ √  √  c1,1 = 0, c1,2 = , c1,3 = ,
  2 2
3 3 3 π 27 3 π 2 √ 3 √
s2 (x) = + x− − x − c2,1 = 1, c2,2 = − +
1 3
3, c2,3 = −
1
3,
2 4π 3 8π 2 3 3 2π 3 2π
√  3 √ √ √
+
27 3
x−
π 9 3 − 5π π − 29 3 π − 29 3
8π 3 3 c3,1 = , c3,2 = , c3,3 = ,
√  π2 π2 π2
    √ √
3 3 2 π π 2 6π − 272 3
27
2 3 − 6π
= 3 4π + 6π x − − 27π x − c4,1 = , c4,2 = 0, c4,3 = .
8π 3 3 π 3 π3
 3 
π
+ 27 x − , Der dadurch definierte vollständige Spline hat überlegene
3
Approximationseigenschaften gegenüber dem natürlichen
und für das Intervall [2π/3, π] Spline, wie wir in Abbildung 12.3 sehen können. In der Ab-
√     bildung korrespondiert der Spline zum durchgezogenen Gra-
3 3 √ 2π 81 √ 2π 3
s3 (x) = − 3 x− − 3 x− phen. Die Kreise liegen mit ihren Mittelpunkten auf sin x;
2 8π 3 8π 3 3
√   ohne diese Art der Darstellung könnte man mit dem bloßen
3 3 2 2π 2π 3 Auge keinerlei Unterschied zwischen dem Spline und der
= 3 4π − 3π (x − ) − 81(x − ) .
8π 3 3 Funktion y = sin x erkennen.

Allerdings können wir mit dem Spline nicht zufrieden sein, Ein natürlicher Spline muss also nicht zwangsweise „natür-
wenn wir auf die Abbildung 12.2 blicken. Der Spline berück- licher“ sein als ein vollständiger Spline!
sichtigt nicht die Symmetrie der Sinus-Funktion auf [0, π].
Geben wir die Steigungen der Sinus-Funktion bei x = 0 und Aufgabe 12.14 •• Wir haben n = 5 Daten gegeben und
x = π mit σ1 = 1 und σ4 = −1 vor und berechnen den damit ist n = 2N + 1 ungerade mit N = 2. Wir erwarten
vollständigen Spline nach (12.45), dann folgen aus nach (12.58) daher ein Polynom der Form
⎛ ⎞⎛ ⎞ ⎛ ⎞
1 S1 √ 1
⎜ π 4π π ⎟ ⎜ S2 ⎟ ⎜ 3 3 ⎟
a0
N
⎜ 3 3 3 ⎟⎜ ⎟ ⎜ 2√ ⎟
⎝ π 4π π ⎠ ⎝ S ⎠ = ⎝
3 3 3 3 − 23 3 ⎠ p(x) = + (ak cos kx + bk sin kx).
2
1 S4 −1 k=1
98 Lösungswege zu Kapitel 12

1
0.9
2
0.8 a2 = (2 cos 4π/5 + cos 8π/5 + 2 cos 12π/5 + cos 16π/5)
5
0.7
= −0.6
0.6
0.5 und
0.4
2
0.3 b1 = (2 sin 2π/5 + sin 4π/5 + 2 sin 6π/5 + sin 8π/5)
5
0.2
= 0.145309,
0.1
2
0 b2 = (2 sin 4π/5 + sin 8π/5 + 2 sin 12π/5 + sin 16π/5)
5
–0.1
0 0.5 1 1.5 2 2.5 3 = 0.615537.

Abbildung 12.3 Der vollständige Spline korrespondiert zu dem als Linie darge-
stellten Graphen. Die Kreise liegen mit ihrem Mittelpunkt auf dem Graphen der
2.5
Funktion y = sin x.

2
Nach (12.57) folgen die Koeffizienten zu
1.5
2
aj = (f0 cos 0 + f1 cos xj + f2 cos 2xj
n
1
+ f3 cos 3xj + f4 cos 4xj ),
2
bj = (f0 sin 0 + f1 sin xj + f2 sin 2xj 0.5
n
+ f3 sin 3xj + f4 sin 4xj ). 0

Damit können wir berechnen:


–0.5
1 2 3 4 5 6 7
2 12
a0 = (2 cos 0 + cos 0 + 2 cos 0 + cos 0) = = 2.4, Abbildung 12.4 Das trigonometrische Interpolationspolynom zu den gegebe-
5 5
nen Daten
2
a1 = (2 cos 2π/5 + cos 4π/5 + 2 cos 6π/5 + cos 8π/5)
5
= −0.6,
Kapitel 13 Aufgabe 13.8 ••
Quadraturregel
Ermitteln Sie den Fehlerterm für die

b−a
Aufgaben Q[f ] =
2
(f (a) + f (b))

b
Verständnisfragen für a f (x) dx durch Integration des Interpolationsfehlers
f (x) − p(x) bei Interpolation von f durch ein lineares Poly-
Aufgabe 13.1 •• Die Gewichte αi der geschlossenen nom p(x) = f (a) + f (b)−f (a)
(x − a).
b−a
Newton-Cotes-Formeln bzw. die σi := sαi mit dem Haupt-
nenner s der αi werden in der Tabelle auf Seite 446 für wach- Aufgabe 13.9 •• Die zusammengesetzte Trapezregel
sendes n immer größer. Gilt limi→∞ σi = ∞? lautet
 
Aufgabe 13.2 • Warum ist es keine gute Idee, Poly- 1 1
QzTr
2,m = h f (a) + f (x 1 ) + . . . + f (x m−1 ) + f (b) .
nome möglichst hohen Grades zu verwenden, um auf äqui- 2 2
distanten Stützstellen interpolatorische Quadraturregeln zu
konstruieren? Die Daten f (xk ) seien nicht exakt bekannt, sondern es ste-
hen nur Näherungen yk zur Verfügung, deren Fehler ek :=
Aufgabe 13.3 • Gegeben seien äquidistante Daten auf f (xk ) − yk jeweils im Betrag durch eine obere Schranke E
einer sehr großen Anzahl von Datenpunkten. Sie wollen beschränkt sind. Welchen Effekt haben diese Fehler auf die
keine zusammengesetzten Newton-Cotes-Formeln verwen- zusammengesetzte Trapezformel
den. Welche Möglichkeit zur Konstruktion einer interpola-  
torischen Quadraturregel auf äquidistanten Gittern sehen Sie @zTr = h 1 y0 + y1 + . . . + ym−1 + 1 ym ?
Q 2,m
noch? 2 2

Aufgabe 13.4 •• Wie lautet der Höchstgrad der Poly-


Aufgabe 13.10 •• Zeigen Sie mithilfe des Hauptsatzes
nome, die von einer Quadraturregel mit n + 1 frei wählbaren
über Peano-Kerne, dass für eine s-mal stetig differenzierbare
Knoten noch exakt integriert werden? Welche Quadraturre-
Funktion f : [a, b] → R das Fehlerfunktional durch
geln erreichen diese Ordnung und wie ist die Knotenvertei-
lung? Rn+1 (x s ) (s)
Rn+1 [f ] = f (ξ ), ξ ∈ (a, b)
s!
Aufgabe 13.5 •• Welche Nachteile haben Gauß-Qua-
draturen bei Handrechnung? abgeschätzt werden kann, wenn der s-te Peano-Kern auf
[a, b] sein Vorzeichen nicht ändert.
Beweisaufgaben Aufgabe 13.11 ••• Die Simpson-Regel
Aufgabe 13.6 ••• Ist die Funktion f : [a, b] → R stetig,    
b−a a+b
dann ist der Stetigkeitsmodul von f definiert als Q3 [f ] = f (a) + 4f + f (b)
6 2
w(δ) := max |f (x) − f (y)|, a ≤ x, y ≤ b.
|x−y|≤δ integriert kubische Polynome p ∈ 3 exakt. Berechnen Sie
Zeigen Sie für eine in [a, b] stetige Funktion f die Abschät- den Peano-Kern K4 und bestimmen Sie damit das Fehler-
zung funktional R3 [f ] nach dem Hauptsatz über Peano-Kerne für
  Funktionen f : [−1, 1] → R, d. h. für a = −1, b = 1.
 b
n−1   
  b−a
 f (x) dx −h f (a + (k + 1)h) ≤ (b−a)w .
 a  n
k=0 Rechenaufgaben
Dabei ist h = (b−a)/n.
 Interpretieren Sie diese Ungleichung Aufgabe 13.12 •• Berechnen Sie mithilfe eines Com-
und den Term h n−1k=0 f (a + (k + 1)h). puterprogramms die Werte der Riemann’schen Summe
n−1  
Aufgabe 13.7 •• Betrachten Sie die Riemann’sche 1 k
Summe R= f
D n n
1
n−1 k=0
k √
n n für die Funktion f (x) = x auf [a, b] = [0, 1] und die
k=0
1√ Stützstellenanzahl n = 2 und n = 212 = 4096. Rechnen Sie
als Quadraturregel für das Integral 0 x dx. Berechnen Sie auf 8 Nachkommastellen. Geben Sie die absoluten Fehler an.
den Stetigkeitsmodul aus Aufgabe und geben Sie eine Schät-
zung des maximal auftretenden Fehlers in Abhängigkeit von Aufgabe 13.13 ••• Bestimmen Sie m in der zusammen-
n an. gesetzten Trapezregel
M. Brokate et al., Arbeitsbuch Grundwissen Mathematikstudium – Höhere Analysis, Numerik und
Stochastik, DOI 10.1007/978-3-642-54946-5_12, © Springer-Verlag Berlin Heidelberg 2016
100 Lösungen zu Kapitel 13

 
1 1 Aufgabe 13.7 •• Aufgabe 13.6.
QzTr
2,m = h f (a) + f (x1 ) + . . . + f (xm−1 ) + f (b)
2 2
!m−1 " Aufgabe 13.8 •• Kapitel 12, (12.22).
1
=h f (xk ) + (f (a) + f (b))
2 Aufgabe 13.9 •• –
k=1
für das Integral
1 Aufgabe 13.10 •• Kapitel 13.3.
e−x dx
2
I [exp(−x 2 )] =
0 Aufgabe 13.11 ••• Kapitel 13.3 und Aufgabe 13.10.
so, dass das Resultat sicher auf 6 Nachkommastellen genau
ist.
Rechenaufgaben
Aufgabe 13.14 •• Die Funktion
Aufgabe 13.12 •• –

1 1
f (x) = cos(7k πx) Aufgabe 13.13 ••• Schätzen Sie den Fehlerterm der zu-
π 2k
k=1 sammengesetzten Trapezformel auf [0, 1] ab.
ist stetig, aber nirgends differenzierbar. Berechnen Sie das
b Aufgabe 13.14 •• –
Integral a f (x) dx für das Intervall [a, b] = [0, 0.1] bzw.
für [a, b] = [0.4, 0.5] jeweils mit der Trapezformel und der Aufgabe 13.15 ••• Vergessen Sie nicht die affine Trans-
Simpson-Regel. Die exakten Vergleichswerte sind: formation von [−1, 1] auf [0, 1]!
für [a, b] = [0, 0.1]: 0.0189929,
für [a, b] = [0.4, 0.5]: −0.0329802.
Verwenden Sie eine Schrittweite von h = 0.001 und brechen Lösungen
Sie die Reihe an einer Stelle ab, an der die weiteren Sum-
manden keinen Einfluss mehr haben (für k = 200 ist bereits
2k = 1.6 · 1060 !). Berechnen Sie die Quadraturfehler und
Verständnisfragen
vergleichen Sie diese. Rechnen Sie unbedingt mit doppelter Aufgabe 13.1 •• Das ist in der Tat der Fall und ein gu-
Genauigkeit. ter Grund, Newton-Cotes-Formeln nicht für ngroßes n zu ver-
wenden! Wie
 wir gezeigt haben, gilt stets i=0 αi = n, also
Aufgabe 13.15 ••• Schreiben Sie ein Programm zur limn→∞ ni=0 αi = ∞.
Gauß-Quadratur von Funktionen f : [0, 1] → R, wobei
Sie 2 und 4 Integrationspunkte zulassen. Berechnen Sie Aufgabe 13.2 • Polynome höheren Grades zeigen auf
1 dx
0 1+x 4 bis auf acht Nachkommastellen. Der Vergleichswert
äquidistanten Knoten das Runge-Phänomen, d. h., sie oszil-
ist 0.86697299. lieren zwischen den Knoten.

Aufgabe 13.3 • Sie können selbstverständlich kubi-


sche Splines verwenden und in der Praxis wird diese Technik
Hinweise auch angewendet.

Aufgabe 13.4 •• Nach dem Satz über die maximale


Verständnisfragen Ordnung einer Quadraturregel ist der Höchstgrad 2n + 1.
Aufgabe 13.1 •• – Die Klasse der Gauß-Quadraturen erreicht diese Ordnung
tatsächlich. Die Knotenverteilung ist dabei gegeben durch
Aufgabe 13.2 • Kapitel 12, Abschnitt 12.4. die Nullstellen von Orthogonalpolynomen.

Aufgabe 13.3 • Suchen Sie nach alternativen Interpo- Aufgabe 13.5 •• Die Knoten sind Nullstellen von Or-
lationen in Kapitel 12. thogonalpolynomen und das sind in der Regel irrationale
Zahlen. Man hat also in der Regel mit zahlreichen Nach-
Aufgabe 13.4 •• – kommastellen bei den Knoten zu rechnen, was die Gauß-
Quadraturen für Handrechnung unattraktiv macht.
Aufgabe 13.5 •• –

Beweisaufgaben Beweisaufgaben
Aufgabe 13.6 ••• Die Definition des Stetigkeitsmoduls Aufgabe 13.6 ••• –
bedeutet, dass für |x −y| ≤ δ die Abschätzung |f (x)−f (y)|
≤ w(δ) folgt. Nutzen Sie diese Abschätzung mit δ := h. Aufgabe 13.7 •• –
Lösungswege zu Kapitel 13 101

Aufgabe 13.8 •• – Lösungswege

Aufgabe 13.9 •• – Verständnisfragen

Aufgabe 13.10 •• – Aufgabe 13.1 •• –

Aufgabe 13.2 • –
Aufgabe 13.11 ••• –
Aufgabe 13.3 • –

Rechenaufgaben
Aufgabe 13.4 •• –
Aufgabe 13.12 •• –
Aufgabe 13.5 •• –
Aufgabe 13.13 ••• –
Beweisaufgaben
Aufgabe 13.14 •• Für das Intervall [0, 0.1] erhält man
mit der Trapezformel den Wert 0.0189876, also einen Feh- Aufgabe 13.6 ••• Wir schreiben
ler im Betrag von 0.000053, während die Simpson-Regel
b
n−1
den Wert 0.0190144 und damit den Fehler von 0.0000215
f (x) dx − h f (a + (k + 1)h)
liefert. Im Intervall [0.4, 0.5] liefert die Trapezregel den a k=0
Wert −0.0330067 und einen Fehler von 0.0000265, und
die Simpson-Regel −0.0328827 mit einem Fehler von a+(k+1)h
n−1
= (f (x) − f (a + (k + 1)h)) dx.
0.0000975.
k=0 a+kh
Der Fehler der (eigentlich genaueren!) Simpson-Regel ist
damit höher als der der Trapezregel. Unsere Fehlerabschät- Gilt nun a + kh ≤ x ≤ a + (k + 1)h, dann ist
zungen basieren stets auf der Differenzierbarkeit von f . Ist
|f (x) − f (a + (k + 1)h)| ≤ w(h),
diese Differenzierbarkeit wie in diesem Fall nicht gegeben,
macht die Verwendung eines aufwendigeren Verfahrens kei-
denn das folgt sofort aus der Definition des Stetigkeitsmo-
nen Sinn.
duls. Damit gilt aber
 
Aufgabe 13.15 ••• Für die Gauß-Quadratur mit zwei  a+(k+1)h 
 
Stützstellen ergibt sich  (f (x) − f (a + (k + 1)h)) dx  ≤ hw(h),
 a+kh 
 
1 1 1
QG =  4 +  4 und wenn wir n von diesen Ungleichungen addieren, folgt
1 + x4 −1
1+ √ 1 + √1 n−1
3 3
 a+(k+1)h
√ 
mit x0 = −x1 = −1/ 3 und α0 = α1 = 1. Die affine  f (x) dx
 a+kh
k=0
Transformation auf [0, 1] liefert 

n−1 a+(k+1)h 
xi + 1 
αi − f (a + (k + 1)h) dx  ≤ nhw(h),
yi := , αi :=
@ , i = 0, 1, a+kh 
2 2 k=0

und damit also


⎛ ⎞
 
 b
n−1 
  ⎜ ⎟  
 f (x) dx − h f (a + (k + 1)h)) dx  ≤ (b − a)w(h)
@G 1 1⎜
⎜ 1 1 ⎟
⎟  a 
Q = ⎜  4 +  4 ⎟ k=0
2
1 + x4 2⎜ 1− √1 1 ⎟
⎝ 1+ √ ⎠
1+ 3
1+ 3
mit h = b−a
n .
2 2

= 0.8595 2249. Der Term h n−1 k=0 f (a + (k + 1)h) ist eine spezielle Rie-
mann’sche Summe. Die bewiesene Ungleichung sagt aus,
Die Gauß-Quadratur mit vier Knoten liefert den Wert dass für eine stetige Funktion f diese Riemann’sche Summe
0.8669 5566. Der Fehler liegt also für die Formel mit zwei gegen das Integral konvergiert, und zwar schlimmstenfalls
Knoten bei 0.0075 und für die Formel mit vier Knoten bei mit der Konvergenzgeschwindigkeit, mit der w((b − a)/n)
0.0000 1733. gegen null geht.
102 Lösungswege zu Kapitel 13

Aufgabe 13.7 •• Der Stetigkeitsmodul ist nach Defini- und der zu erwartende Fehlerterm ist die Differenz
 @zTr 
tion Q2,m − QzTr 2,m . Gilt |ek | ≤ E für alle k = 0, 1, . . . , m,
√ √
w(1/n) = max | x − y|. dann folgt
|x−y|≤1/n
 
√√ √ √ √  1 1 
Wegen
√ | x − y| ≤ |x − y| folgt | x − x + 1/n| ≤ 
h  e0 + e1 + . . . + em−1 + em 
2 2
1/n und damit gilt  
1 1
≤ h |e0 | + |e1 | + . . . + |em−1 | + |em |
1 2 2
w(1/n) ≤ √ .
n ≤ hmE = (b − a)E.

Der schlimmstenfalls auftretende Fehler ist also 1/ n.
Aufgabe 13.10 •• Da Ks nach Voraussetzung auf [a, b]
b
Aufgabe 13.8 •• Für p(x) dx erhalten wir sein Vorzeichen nicht ändert, ist der Mittelwertsatz der Inte-
a
gralrechnung auf
b f (b) − f (a)

b
f (a) + (x − a) dx
a b−a Rn+1 [f ] = f (s) (x)Ks (x) dx
a
1 f (b)−f (a) 2 2
= f (a)(b−a) − af (b)+af (a)+ (b −a ) anwendbar, d. h., es existiert ein ξ ∈ (a, b), sodass wegen
2 b−a
1 des obigen Hauptsatzes über Peano-Kerne die Gleichung
= bf (a) − af (b) + (bf (b) − bf (a) + af (b) − af (a)) b
2
b−a b−a b−a Rn+1 [f ] = f (s) (ξ ) Ks (x) dx
= f (a) + f (b) = (f (a) + f (b)) a
2 2 2
= Q[f ]. gilt. Das Integral über den Peano-Kern Ks hängt nicht von f
ab, daher folgt
Es gilt nach (12.22)
Rn+1 [x s ] (s)
Rn+1 [f ] = f (ξ ).
1 s!
f (x) − p(x) = (x − a)(x − b)f  (ξ )
2
Aufgabe 13.11 ••• Nach Definition der Peano-Kerne ist
für ein ξ ∈ (a, b) und damit
in unserem Fall s = 4 und n = 2. Damit ist
b b ! "
f (x) dx − p(x) dx (· − x)3+ 1
K4 (x) = R3 = R3 [(· − x)3+ ]
a a 3! 6
b b−a
= f (x) dx − (f (a) + f (b)) wegen der Linearität des Fehlerfunktionals. Damit folgt
a 2
b 
1
= (x − a)(x − b)f  (ξ ) dx K4 (x) =
1 1 4 1
(−1 − x)3+ + (0 − x)3+ + (1 − x)3+
2
a 6 3 3 3
b 
1 1
= f  (ξ ) (x − a)(x − b) dx
2 a − (z − x)3+ dz .
1 1 h −1
= − f  (ξ ) (b − a) = − f  (ξ )
2 6 12
Aus der Definition der abgeschnittenen Funktion (z − x)3+
für ein ξ ∈ (a, b) und h := b − a. auf [−1, 1] folgt sofort
1 1
1
Aufgabe 13.9 •• Wir schreiben yk = f (xk ) − ek und (z − x)3+ dz = (z − x)3 dz = (1 − x)4 .
@zTr ein, um −1 x 4
setzen dies in Q 2,m
 Weiterhin folgt
@zTr 1 
Q2,m = h (f (a) − e0 ) + (f (x1 ) − e1 ) + . . . 0 ; x≥0
2 (−1 − x)3+ = 0, (0 − x)3+ = ,
 −x 3 ; x < 0
1
. . . + (f (xm−1 ) − em−1 ) + (f (b) − em )
2 (1 − x)3+ = (1 − x)3
zu erhalten. Damit ergibt sich und damit ist K4 gegeben durch
   1
(1 − x)3 (1 + 3x) ; 0≤x≤1
@zTr = QzTr − h 1 e0 + e1 + . . . + em−1 + 1 em
Q K4 (x) = 722 3 1 .
2,m 2,m
2 2 − 9 x + 72 (1 − x)3 (1 + 3x) ; −1 ≤ x < 0
Lösungswege zu Kapitel 13 103

Der Peano-Kern K4 ändert auf [−1, 1] sein Vorzeichen nicht. wobei wir aber über die Lage von ξ nichts wissen. Bestenfalls
Daher ist das Resultat aus Aufgabe 13.10 anwendbar und es können wir verlangen, dass der Fehler im Betrag nicht größer
folgt mit ist als die obere Schranke
 1 
R3 [x 4 ] 1 |f  (τ )|m−2
= Q3 [x 4 ] − x 4 dx max .
4! 4! −1 0≤τ ≤1 12
 1 
1 1 4 1 4 Wegen f  (x) = −2xe−x folgt
2
= ·1+ ·0+ ·1− x dx
24 3 3 3 −1
 
f  (x) = −2e−x + 4x 2 e−x = e−x (4x 2 − 2).
2 2 2
1 2 2 1
= − = ,
24 3 5 90
Gesucht ist nun das Maximum dieser zweiten Ableitung
dass auf [0, 1]. Dazu setzen wir die dritte Ableitung von f zu
Rn+1 [x 4 ] (4) 1 (4) null,
R3 [f ] = f (ξ ) = f (ξ ) !
f  (x) = 4xe−x (3 − 2x 2 ) = 0,
2
4! 90
gilt. √
und bemerken, dass f  bei x1 = 0 und bei x2,3 = ± 3/2
verschwindet. In der Tat ist f  eine auf [0, 1] monoton fal-
Rechenaufgaben lende Funktion, sodass das Maximum bei x = 0 auftritt,
Aufgabe 13.12 •• Für n = 2 ergibt sich R = 21 (f (0) + max |f  (τ )| = |f  (0)| = 2.
√ 0≤τ ≤1
f (1/2)) = 21 1/2 = 0.35355339, für n = 4096 erhält man
R = 0.666533537.
Genauigkeit bis zur sechsten Nachkommastelle bedeutet
2 1 √ 1
Der wahre Wert des Integrals ist 0 x 2 dx = 23 x 3  =
0 2m−2
2
= 0.66666667. Für n = 2 ergibt sich daher ein Fehler von < 5 · 10−7 ,
3 12
0.313113277, für n = 4096 folgt ein Fehler von 0.00013313.
Die Konvergenz ist also ausgesprochen langsam. vergl. Kapitel 11. Damit folgt m2 > 106 /3 und es ergibt sich

Aufgabe 13.13 ••• Den Fehler der zusammengesetzten 103


m > √ ≈ 578.
Trapezformel hatten wir in (13.8) zu 3
b − a 2 
h f (ξ )
12 Aufgabe 13.14 •• –
mit f (x) = exp(−x 2 ) und mit einem ξ ∈ (a, b) bestimmt.
Wegen [a, b] = [0, 1] gilt h = 1/m und wir erhalten für den Aufgabe 13.15 ••• –
Fehler b − a −2 
m f (ξ ) ,
12
Kapitel 14 Aufgabe 14.8 •• Gegeben sei die Matrix
⎛ ⎞
1 4 7
A = ⎝2 α β ⎠ .
Aufgaben 0 1 1
Unter welchen Voraussetzungen an die Werte α, β ∈ R ist
die Matrix regulär und besitzt zudem eine LR-Zerlegung?
Verständnisfragen Geben Sie zudem ein Parameterpaar (α, β) derart an, dass
Aufgabe 14.1 •• Geben Sie ein Beispiel an, bei dem die die Matrix A regulär ist und keine LR-Zerlegung besitzt.
linearen Iterationsverfahren ψ und φ nicht konvergieren und
die Produktiteration ψ ◦ φ konvergiert. Aufgabe 14.9 •• Gegeben sei die Matrix
 π 
1 − 78 cos π4 7
8 sin 4
A =
Aufgabe 14.2 •• Wir betrachten Matrizen A = − 78 sin π
1− 7
cos π4
4 8
(aij )i,j =1,...,n ∈ Rn×n (n ≥ 2) mit aii = 1, i = 1, . . . , n und der Vektor  
und aij = a für i = j . 0
b = .
0
(a) Wie sieht die Iterationsmatrix des Jacobi-Verfahrens zu Zeigen Sie, dass das lineare Iterationsverfahren
A aus? Berechnen Sie ihre Eigenwerte und die Eigen- x n+1 = (I − A)x n + b, n = 0, 1, 2, . . .
werte von A.
für jeden Startvektor x 0 ∈ R2
gegen die eindeutig bestimmte
(b) Für welche a konvergiert das Jacobi-Verfahren? Für wel-
Lösung x = (0, 0)T konvergiert, obwohl eine induzierte Ma-
che a ist A positiv definit?
trixnorm mit
(c) Gibt es positiv definite Matrizen A ∈ Rn×n , für die das I − A > 1
Jacobi-Verfahren nicht konvergiert? existiert. Veranschaulichen Sie beide Sachverhalte zudem
grafisch.

Beweisaufgaben
Rechenaufgaben
Aufgabe 14.3 • Zeigen Sie: Bei einer positiv definiten
Matrix A ∈ Rn×n sind alle Hauptabschnittsmatrizen eben- Aufgabe 14.10 • Bestimmen Sie für das System
falls positiv definit. ⎛ ⎞⎛ ⎞ ⎛ ⎞
4 0 2 x1 4
⎝0 5 2 ⎠ ⎝x2 ⎠ = ⎝−3⎠
Aufgabe 14.4 • Beweisen Sie das auf Seite 493 aufge- 5 4 10 x3 2
führte Korollar.
die Spektralradien der Iterationsmatrizen für das Gesamt-
Aufgabe 14.5 •• Zeigen Sie, dass für zwei lineare Itera- und Einzelschrittverfahren und schreiben Sie beide Verfah-
tionsverfahren φ, ψ mit den Iterationsmatrizen M φ und M ψ ren in Komponenten. Zeigen Sie, dass die Matrix konsistent
die beiden Produktiterationen φ ◦ ψ und ψ ◦ φ die gleichen geordnet ist und bestimmen Sie den optimalen Relaxations-
Konvergenzeigenschaften im Sinne von parameter für das Einzelschrittverfahren.

ρ(M φ◦ψ ) = ρ(M ψ◦φ ) Aufgabe 14.11 •• Das Gleichungssystem


    
3 −1 x1 1
besitzen. =
−1 3 x2 −1
soll mit dem Jacobi- und Gauß-Seidel-Verfahren gelöst wer-
Aufgabe 14.6 • Zeigen Sie, dass jede unitäre Matrix
den. Wie viele Iterationen sind jeweils ungefähr erforderlich,
Q ∈ Cn×n längenerhaltend bezüglich der euklidischen
um den Fehler x n −x2 um den Faktor 10−6 zu reduzieren?
Norm ist und Q2 = 1 gilt.
Aufgabe 14.12 •• Zeigen Sie: Sei A ∈ Rn×n eine sym-
Aufgabe 14.7 •• Gegeben sei die Matrix metrische, positiv definite Matrix. Dann ist B = P AP T für
⎛ ⎞ jede invertierbare Matrix P ∈ Rn×n ebenfalls symmetrisch
1 3 4 1
⎜2 7 a 4⎟ und positiv definit.
A = ⎜
⎝1 4 6 1 ⎠ .

3 4 9 0 Aufgabe 14.13 • Berechnen Sie die LR-Zerlegung der


Matrix ⎛ ⎞
(a) Für welchen Wert von a besitzt A keine LR-Zerlegung? 1 4 5
(b) Berechnen Sie eine LR-Zerlegung von A im Existenzfall. A = ⎝1 6 11⎠
(c) Für den Fall, dass A keine LR-Zerlegung besitzt, geben 2 14 31
Sie eine Permutationsmatrix P derart an, dass P A eine und lösen Sie hiermit das lineare Gleichungssystem
LR-Zerlegung besitzt. Ax = (17, 31, 82)T .
M. Brokate et al., Arbeitsbuch Grundwissen Mathematikstudium – Höhere Analysis, Numerik und
Stochastik, DOI 10.1007/978-3-642-54946-5_13, © Springer-Verlag Berlin Heidelberg 2016
Lösungen zu Kapitel 14 105

Aufgabe 14.14 •
Berechnen Sie die QR-Zerlegung der Aufgabe 14.9 •• Beachten Sie den Einheitskreis be-
Matrix   züglich der sogenannten Betragssummennorm .1 im Ab-
1 3
A= schnitt zur Funktionalanalysis auf Seite 277.
−1 1
und lösen Sie hiermit das lineare Gleichungssystem
Ax = (16, 0)T . Rechenaufgaben
Aufgabe 14.15 • Berechnen Sie eine Cholesky-Zerle- Aufgabe 14.10 • Nutzen Sie den Satz zur Konvergenz
gung der Matrix des SOR-Verfahrens.
⎛ ⎞
9 3 9 Aufgabe 14.11 •• Die Konvergenzgeschwindigkeit
A = ⎝3 9 11⎠ hängt vom Spektralradius der Iterationsmatrix ab. Hiermit
9 11 17 steht somit auch die benötigte Iterationszahl in Verbindung
und lösen Sie hiermit das lineare Gleichungssystem mit dem Spektralradius.
⎛ ⎞
24 Aufgabe 14.12 •• Nutzen Sie das Skalarprodukt beim
Ax = ⎝16⎠ . Nachweis der positiven Definitheit.
32
Aufgabe 14.13 • Betrachten Sie das Beispiel auf
Aufgabe 14.16 •• Sei Seite 494.
⎛ ⎞
2 −1 −1 0 Aufgabe 14.14 • Betrachten Sie das Beispiel auf
⎜−1 2.5 0 −1⎟
A=⎜ ⎝−1 0 2.5 −1⎠
⎟ Seite 493.

0 −1 −1 2 Aufgabe 14.15 • Nutzen Sie den vorgestellten Algo-


die Matrix eines linearen Gleichungssystems. rithmus.

Zeigen Sie, dass A irreduzibel ist und dass das Jacobi- Aufgabe 14.16 •• Nutzen Sie die graphentheoretische
Verfahren sowie das Gauß-Seidel-Verfahren konvergent sind. Betrachtung zum Nachweis der Irreduzibilität.

Lösungen
Hinweise
Verständnisfragen
Verständnisfragen
Aufgabe 14.1 •• –
Aufgabe 14.1 •• Setzen Sie geeignete Diagonalmatri-
zen für die Iterationsmatrizen an. Aufgabe 14.2 •• Es gibt positiv definite Matrizen, für
die das Jacobi-Verfahren nicht konvergiert.
Aufgabe 14.2 •• Betrachten Sie im Teil (c) die Schnitt-
menge der Mengen aus Werten für a, die Konvergenz nach
sich ziehen und die, die positive Definitheit liefern. Beweisaufgaben
Aufgabe 14.3 • –
Beweisaufgaben
Aufgabe 14.4 • –
Aufgabe 14.3 • Betrachten Sie xT A[k]x.
Aufgabe 14.5 •• –
Aufgabe 14.4 • Nutzen Sie komplexe Zahlen der
Form ei Aufgabe 14.6 • –

Aufgabe 14.5 •• Zeigen Sie, dass jeder Eigenwert λ = Aufgabe 14.7 •• –


0 der Iterationsmatrix M φ M ψ auch Eigenwert der Matrix Aufgabe 14.8 •• –
M ψ M φ ist und umgekehrt.
Aufgabe 14.9 •• –
Aufgabe 14.6 • Nutzen Sie den Zusammenhang zwi-
schen der euklidischen Norm und dem Skalarprodukt.
Rechenaufgaben
Aufgabe 14.7 •• Nutzen Sie den Satz zur Existenz einer Aufgabe 14.10* • Die Spektralradien lauten
LR-Zerlegung, der sich auf Seite 488 befindet. 41 41
ρ(M J ) = 100 , ρ(M GS ) = 100 . Die Matrix ist
Aufgabe 14.8 •• Nutzen Sie den Satz zur Existenz einer konsistent geordnet und der optimale Relaxationsparameter
LR-Zerlegung, der sich auf Seite 488 befindet. hat den Wert ω ≈ 1.131.
106 Lösungswege zu Kapitel 14

Aufgabe 14.11 •• Es sind 7 Iterationen. Aufgabe 14.2 ••


(a) Wir erhalten
Aufgabe 14.12 •• –
⎛ ⎞
−a . . . . . . a
0
Aufgabe 14.13 • Wir erhalten ⎜ . .. ⎟
⎛ ⎞ ⎛ ⎞ ⎜−a . . . ⎟
⎜ ⎟
1 0 0 1 4 5 ⎜ .. . . . . .. ⎟
.
MJ = ⎜ . ⎟ ∈ Rn×n .
A = ⎝1 1 0⎠ ⎝0 2 6⎠ . ⎜ . . ⎟
⎜ . .. .. ⎟
2 3 1 0 0 3 ⎝ .. . . −a ⎠
     
=L =R −a . . . . . . −a 0

Zur Berechnung der Eigenwerte überführen wir A in


Aufgabe 14.14 • Es gilt eine Dreiecksgestalt. Hierzu ziehen wir im ersten Schritt
⎛ √ √ ⎞ für j = n − 1, . . . , 1 sukzessive die j -te Zeile von der
2 2 √ √ 
2 √2 (j + 1) -ten Zeile ab. Folglich ergibt sich
A = ⎝ 2√ √2 ⎠ .
− 22 2 0 2 2 ⎛ ⎞
 
2
    1 a ... ... a
=Q
=R ⎜a − 1 1 − a 0 ... 0 ⎟
⎜ ⎟
⎜ . . . .. ⎟
(1)
A =⎜ 0 ⎜ . . . . . . . ⎟⎟.
• ⎜ . . . . ⎟
Aufgabe 14.15 Es gilt ⎝ . . . . . . . . 0 ⎠
⎛ ⎞ ⎛ ⎞
3 0 0 3 1 3 0 ... 0 a−1 1−a
A = ⎝1 4 0⎠ ⎝0 4 2⎠ .
Im zweiten Schritt multiplizieren wir sukzessive für
3 2 2 0 0 2 a
      j = n, . . . , 2 die j -te Zeile mit (n + 1 − j ) 1−a und
=L =LT subtrahieren diese Zeile von der ersten Zeile. Dieser Vor-
gang resultiert in der Matrix
•• ⎛ ⎞
Aufgabe 14.16 – 1 + (n − 1)a 0 ... ... 0
⎜ . .. ⎟

⎜ a−1 1 − a .. . ⎟⎟
(2) ⎜ . . . . ⎟
A =⎜ 0 . . . . . . . ⎟
. .
Lösungswege ⎜ ⎟
⎜ . .. .. .. ⎟
⎝ .. . . . 0 ⎠
Verständnisfragen 0 ... 0 a−1 1−a

Aufgabe 14.1 •• Mit Da die Eigenwerte von A und A(2) übereinstimmen,


  gilt
2 0 σ (A) = {1 + (n − 1)a , 1 − a} .
φ(x, b) = x + Nφ b
0 1 Mit M J = I − A ergibt sich hiermit
 4 
=M φ
σ (M J ) = {(1 − n)a , a} .
und 1  (b) Das Jacobi-Verfahren konvergiert genau dann, wenn
0
ψ(x, b) =
4
x + Nψ b ρ(M J ) < 1 gilt und somit genau für alle a ∈ R mit
1
0 2 |a| < n−1 .
  
=M ψ Als symmetrische Matrix ist A genau dann positiv defi-
gilt nit, wenn alle Eigenwerte positiv sind. Diese Eigenschaft
ρ(M φ ) = ρ(M ψ ) = 2 , liegt genau dann vor, wenn

womit beide Verfahren divergent sind, während 1


1>a>−
n−1
(ψ ◦ φ)(x, b) = M ψ M φ x + N ψ◦φ b
gilt.
wegen (c) Aus obigen Überlegungen ersehen wir, dass für n > 2
1  mit
0 1 1
Mψ Mφ =
2
, ρ(M ψ M φ ) = ≤a<1
1 2 n−1
0 2
eine positive definite Matrix A vorliegt, bei der das
ein konvergentes Verfahren darstellt. Jacobi-Verfahren divergiert.
Lösungswege zu Kapitel 14 107

Beweisaufgaben (c) Unter Nutzung der Permutationsmatrix


⎛ ⎞
Aufgabe 14.3 • Sei x ∈ Rk \ {0}, k < n, dann definie- 1 0 0 0
ren wir y = (x, 0, . . . , 0)T ∈ Rn \ {0}. Somit folgt ⎜0 1 0 0⎟
P =⎜ ⎝0 0 0 1⎠

x T A[k]x = y T Ay > 0, 0 0 1 0

sodass alle A[k] für k = 1, . . . , n positiv definit sind. ergibt sich für P A ein LR-Zerlegung für a = 10 .

Aufgabe 14.8 •• Subtraktion des 2-Fachen der ersten


Aufgabe 14.4 • Gelte A = QR, dann schreibe
von der zweiten Zeile liefert
rjj = rj · eij 2für j = 1, . . . , n3mit rj ∈ R \ {0} und ⎛ ⎞
@R
wähle D = diag e−i1 , . . . , e−in . Damit folgt A = Q @ 1 4 7
⎜ ⎟
@ −1 @
mit Q = QD und R = DR. Hierbei stellt Q eine or- @ ⎜0 α − 8 β − 14⎟ .
⎝ ⎠
@
thogonale Matrix dar und die rechte obere Dreiecksmatrix R
0 1 1
besitzt die Diagonalelemente r1 , . . . , rn .
Folglich besitzt A für alle α = 8 bei beliebigem β ∈ R
Aufgabe 14.5 •• Sei v Eigenvektor der Matrix M φ M ψ eine LR-Zerlegung. Mit
zum Eigenwert λ = 0, so ergibt sich aus M φ M ψ v = λv  = 0 det A = α − 8 − (β − 14) = α − β + 6
die Eigenschaft M ψ v = 0, wodurch
 aufgrund
 der Gleichung
 
M ψ M φ M ψ v = λM ψ v stets ist A für alle Paare (α, β) ∈ R2 mit β = α + 6 regulär.
 ρ M φ ψ ≤ ρ M
M  ψ Mφ
gilt. Analog erhalten wir ρ M ψ M φ ≤ ρ M φ M ψ . Damit Für α = 8 und β = 0 = 14 ist A invertierbar ohne eine
stimmen die Spektralradien der Iterationsmatrizen und somit LR-Zerlegung zu besitzen.
auch das Konvergenzverhalten der Produktiterationen φ ◦ ψ
und ψ ◦ φ überein. Aufgabe 14.9 ••
Zunächst erhalten wir
⎛ ⎞
cos π4 − sin π4
Aufgabe 14.6 • Bei einer unitären Matrix Q ∈ Cn×n 7⎝ ⎠.
M =I −A=
ergibt sich für jeden Vektor x ∈ Cn wegen Q∗ Q = I die 8 π
sin 4 cos 4 π
Eigenschaft
  Die Iterationsmatrix stellt folglich eine Drehung in Kombi-
Qx2 = Qx, Qx = (Qx)∗ Qx nation mit einer Stauchung dar. Die beiden komplexwertigen
 √ Eigenwerte lauten
= x ∗ Q∗ Qx = x ∗ x = x2 .
7 π π
λ= cos ± i sin ,
Hiermit folgt unmittelbar 8 4 4
womit ρ(M) = 78 und somit Konvergenz der Methode vor-
Q2 = sup Qx2 = sup x2 = 1.
x ∈C n x ∈C n liegt. Dennoch gilt
x 2 =1 x 2 =1
7 π π 7√
M1 = cos + sin = 2 > 1.
8 4 4 8
Aufgabe 14.7 •• Die hiermit einhergehende Wirkung können wir der Abbil-
dung 14.1 entnehmen. Bei einem Startvektor x 0 = (1, 0)T
(a) Durch elementare Umformung kann die Matrix A ohne
ergibt sich eine „spiralförmige“ Konvergenz gegen den Lö-
Zeilen- resp. Spaltentausch auf die Gestalt
sungsvektor x ∗ = (0, 0)T . Anhand des eingezeichneten
⎛ ⎞
1 3 4 1 Einheitskreises bezüglich der Betragssummennorm erken-
⎜0 1 a − 8 2⎟ nen wir, dass x 0 auf dem Rand des Einheitskreises liegt,
⎜ ⎟
⎝0 0 10 − a −2⎠ während sich x 1 außerhalb des Einheitskreises befindet, da
 π 

0 0 5a − 43 7 7  cos 4  7√
x 1 1 = Mx 0 1 =   = 2>1
gebracht werden. Hieraus ersehen wir, dass eine LR- 8  sin π  8
4 1
Zerlegung von A genau dann existiert, wenn a  = 10 gilt.
gilt, obwohl im Fall a = 10 mit det A = 14 eine
reguläre Matrix vorliegt.
(b) Für a = 10 erhalten wir Rechenaufgaben
⎛ ⎞ ⎛ ⎞ Aufgabe 14.10 • Für das Jacobi-Verfahren erhalten
1 0 0 0 1 3 4 1
wir
⎜2 1 0⎟ ⎜ ⎟
A=⎜
0 ⎟ ⎜0 1 a − 8 2 ⎟. ⎛
− 21

⎝1 1 1 0⎠ ⎝0 0 10 − a −2 ⎠ 0 0
⎜ ⎟
3 −5 5a−43 1 0 0 0 7 + 2 5a−43 M J = D −1 (D − A) = ⎝ 0 0 − 25 ⎠ .
 10−a    10−a

=L =R − 25 − 21 0
108 Lösungswege zu Kapitel 14

sodass
41
1 ρ(M GS ) = <1
100
x2 folgt. Wir erhalten
x1
x3 C(α) = −(αD −1 L + α −1 D −1 R)
⎛ 1 ⎞
0 0 2α
x0 ⎜
= ⎝ 0 0 5α 2 ⎟,

1
α 2α
2 5 0
womit
   
2α 2 α 1
det C(α) − λ I = λ λ2 − · + ·λ·
5 5α 2 2α
 
4 1
= λ λ2 − + λ
Abbildung 14.1 Konvergenzverlauf. 25 4
gilt und daher die Eigenwerte von C(α) keine Abhängigkeit
Damit gilt von α aufweisen. Die Matrix ist folglich konsistent geordnet
und erfüllt alle im Satz auf Seite 510 geforderten Eigenschaf-
1 ten. Für den optimalen Relaxationsparameter gilt somit
xm+1,1 = − xm,3 + 1
2
2
2
xm+1,2 = − xm,3 −
3 wopt = * ≈ 1.131 .
41
5 5 1+ 1− 100
2 1 1
xm+1,3 = − xm,1 − xm,2 + Aufgabe 14.11 •• Mit q := M2 und
5 2 5
sowie wegen x m − x2 ≤ q m x 0 − x2
  erhalten wir die Forderung
41
det(M J − λ I ) = λ λ2 − q m ≤ 10−6
100
respektive
zudem ln 10−6
D D m ≥ .
ln q
41 41 Für das Jacobi-Verfahren gilt
λ1 = 0 , λ2,3 =± , ρ(M J ) = < 1.  
100 100 0 13 * 1
Für das Gauß-Seidel-Verfahren ermittelt man unter Verwen- MJ = 1 sowie M J 2 = ρ(M ∗J M J ) = .
3 0
3
dung von
Somit ergibt sich mit
⎛ 1 ⎞
4 0 0 ln 10−6
⎜ ⎟ m≥ ≈ 12.575
(D + L)−1 = ⎝ 0 1
5 0⎠ ln 13
− 18 − 25
2 1
10
ein Bedarf von 13 Iterationen.

die Iterationsmatrix Für das Gauß-Seidel-Verfahren


  erhalten wirD
⎛ 1⎞ 0 13 1 1
0 0 −2 M GS = sowie M GS 2 = + .
−1 ⎜ ⎟ 0 19 9 81
M GS = −(D + L) R = ⎝0 0 − 25 ⎠ .
Folglich gilt
41
0 0 100 ln 10−6
m ≥ * ≈ 13.21 .
Für die Komponentenschreibweise gilt ln 19 + 811

1 Dieser scheinbar höhere Iterationsbedarf des Gauß-Seidel-


xm+1,1 = − xm,3 + 1
2 Verfahrens gegenüber dem Jacobi-Verfahren beruht auf der
2 3 expliziten Betrachtung der euklidischen Norm.
xm+1,2 = − xm,3 −
5 5
41 3 Asymptotisch gibt der Spektralradius der Iterationsmatrix die
xm+1,3 = xm,3 − . Konvergenzrate bezüglich jeder Norm wieder. Nutzen wir
100 50
in diesem Sinne q = ρ(M GS ) = 19 , so erhalten wir die
Aufgrund der einfachen Gestalt der Iterationsmatrix ermit- realistische Aussage
teln wir die Eigenwerte von M GS zu ln 10−6
m ≥ ≈ 6.288 ,
λ1,2 = 0 , λ3 =
41
, ln 19
100 sodass üblicherweise 7 Iterationen ausreichend sind.
Lösungswege zu Kapitel 14 109

Aufgabe 14.12 •• Die Symmetrie erhalten wir unmit- durch Rückwärtselimination zu


telbar aus der Gleichung  
4
x= .
B T = (P T )T 
AT P T = P AP T = B. 4
  
=P =A

Für jeden Vektor x ∈ Rn \ {0} erhalten wir aufgrund der Aufgabe 14.15 • Wir erhalten durch die bekannte Eli-
Invertierbarkeit der Matrix P direkt y = P T x ∈ Rn \{0} und minationstechnik
⎛ ⎞ ⎛ ⎞ ⎛ ⎞
folglich ergibt sich die behauptete Eigenschaft der positiven 24 24 1
Definitheit gemäß x = A−1 ⎝16⎠ = L−T L−1 ⎝16⎠ = ⎝−1⎠ .
32 32 2
Bx, x = P AP T x, x = AP T x, P T x = Ay, y > 0.

Aufgabe 14.16 •• Mit (1, 2)(2, 4)(4, 3)(3, 1) haben


Aufgabe 14.13 • Zur Lösung des Gleichungssystems wir einen geschlossenen gerichteten Weg, der über alle Zei-
⎛ ⎞ lenindizes verläuft, womit die Irreduzibilität nachgewiesen
17 ist. Für
⎝31⎠ = Ax = L Rx n
|aij |
 pi =
82 =y |aii |
j =1
j =i
berechnen wir zunächst durch eine Vorwärtselimination aus
Ly = (17, 31, 82)T den Hilfsvektor ergibt sich durch einfaches Nachrechnen
⎛ ⎞ 4
17 p1 = p4 = 1 und p2 = p3 = ,
5
y = ⎝14⎠ .
6 sodass auf der Grundlage des auf Seite 505 aufgeführten
zweiten Satzes zur Konvergenz des Jacobi-Verfahrens die Be-
Analog ergibt sich aus einer Rückwärtselimination hauptung nachgewiesen ist. Für das Gauß-Seidel-Verfahren
⎛ ⎞ nutzen wir den entsprechenden Satz gemäß Seite 506. Für
3 die rekursiv definierten Größen
−1
x = R y = ⎝1⎠ .
2
i−1
|aij |
n
|aij |
pi = pj +
|aii | |aii |
j =1 j =i+1
Aufgabe 14.14 • Wir erhalten die Lösung des Glei- erhalten wir
chungssystems mit
4
    p1 = 1 und p2 = p3 = p4 =
16 √8 5
QRx = ⇔ Rx = QT x = 2
0 √8 und folglich die Konvergenz der Gauß-Seidel-Methode.
2
Kapitel 15 Aufgabe 15.6 • Zeigen Sie: Gilt für die Eigenwerte
λ1 , . . . , λn ∈ C der Matrix A ∈ Rn×n die Bedingung
|λi |  = |λj | für alle Indizes i = j , so sind alle Eigenwerte
Aufgaben reell.

Aufgabe 15.7 • Zeigen Sie die Gültigkeit folgender


Verständnisfragen Aussage: Das Produkt unitärer respektive orthogonaler Ma-
Aufgabe 15.1 •• Gegeben sei die zirkulante Shiftma- trizen ist wiederum unitär beziehungsweise orthogonal.
trix ⎛ ⎞
0 1 0 0
Aufgabe 15.8 • Zeigen Sie, dass jede Matrix A ∈
⎜ . ⎟
⎜ 0 .. ⎟ Rn×n mit paarweise disjunkten Gerschgorin-Kreisen aus-

S=⎜ ⎟ ∈ Rn×n .
.. ⎟ schließlich reelle Eigenwerte besitzt.
⎝ . 1⎠
1 0
Aufgabe 15.9 •• Zeigen Sie: Gegeben sei eine Matrix
(a) Zeigen Sie mittels der Gerschgorin-Kreise, dass alle A ∈ Rn×n , die sich mit α ∈ R und einer schiefsymmetri-
Eigenwerte von S im abgeschlossenen Einheitskreis lie- schen Matrix S in der Form A = αI + S schreiben lässt.
gen. Dann besteht der reelle Wertebereich
(b) Wie lautet die k-te Iterierte z(k) der Potenzmethode bei #  :
Nutzung des Startvektors z(0) = (1, 0, . . . , 0)T ? x T Ax  n
WR (A) := ξ = T  x ∈ R \ {0}
(c) Konvergiert die Potenzmethode bei obigem Startvektor x x
und steht diese Aussage im Widerspruch zum Konver-
genzsatz laut Seite 556? aus genau einem Punkt.

Beweisaufgaben Rechenaufgaben
Aufgabe 15.2 • Zeigen Sie: Jede strikt diagonaldomi- Aufgabe 15.10 • Berechnen Sie die Gerschgorin-
nante Matrix A ∈ Cn×n ist regulär. Kreise für die gegebene Matrix
⎛ ⎞
3 3 2
Aufgabe 15.3 ••• Sei H ∈ Rn×n spaltenstochastisch
A = ⎝2 4 1 ⎠ ∈ R3×3
und M := αH + (1 − α) n1 E mit E = (1)i,j =1,...,n und
1 0 −4
α ∈ [0, 1[.
und nehmen Sie hiermit eine Eigenwerteinschließung vor.
Zeigen Sie:
Können Sie unter Verwendung der Gerschgorin-Kreise ge-
(a) M ist spaltenstochastisch und positiv, d. h., mij > 0 für nauere Aussagen über die Lage einzelner Eigenwerte ma-
alle i, j = 1, ..., n. chen?
(b) Der Eigenvektor x zum Eigenwert λ = 1 der Matrix M
besitzt entweder ausschließlich positive oder ausschließ- Aufgabe 15.11 • Nehmen Sie eine Eigenwertein-
lich negative Einträge. schließung auf der Grundlage geeigneter Wertebereiche be-
(c) Der Eigenraum von M zum Eigenwert λ = 1 ist ein- zogen auf die Matrix
dimensional. ⎛ ⎞
1 2 1
Aufgabe 15.4 ••• Zeigen Sie: Zu gegebener Matrix A ∈ A = ⎝1 1 1⎠ ∈ R3×3
Cn×n sei λ ∈ σ (A) ein Punkt auf dem Rand des Wertebe- 0 2 2
reichs W (A). Zudem sei M die Menge aller Eigenvektoren
w zu Eigenwerten μ ∈ σ (A) \ {λ}, dann gilt v ⊥ M für alle vor.
Eigenvektoren v zum Eigenwert λ. Aufgabe 15.12 ••
Gegeben sei die Matrix
⎛ ⎞
Aufgabe 15.5 •• Weisen Sie nach: Erfüllen die Kom- 1 2 1
ponenten der gegebenen Matrix L ∈ Cn×n die Bedingung A = ⎝1 1 1⎠ ∈ R3×3 .
⎧ 0 2 2

⎨ 0 , für i < j,
ij = 1 , für i = j, Berechnen Sie mit der Potenzmethode den betragsgrößten


O(q ), k → ∞ , für i > j
k Eigenwert der obigen Matrix nebst zugehörigem Eigenvek-
tor. Reduzieren Sie anschließend mit der Deflation die Di-
mit 0 < q < 1, dann gilt für die Matrix W = V L bei mension des Problems und verfahren Sie in dieser Kombina-
beliebig gewähltem V ∈ Cn×n die Darstellung tion weiter, bis alle Eigenwerte der Matrix A ermittelt wur-
W = V + E k mit E k 2 = O(q k ), k → ∞ . den. Überprüfen Sie hiermit auch das Ergebnis der Aufgabe .
M. Brokate et al., Arbeitsbuch Grundwissen Mathematikstudium – Höhere Analysis, Numerik und
Stochastik, DOI 10.1007/978-3-642-54946-5_14, © Springer-Verlag Berlin Heidelberg 2016
Lösungen zu Kapitel 15 111

Aufgabe 15.13 •• Geben Sie die Gerschgorin-Kreise Aufgabe 15.11 • Verwenden Sie den Satz von Bendix-
für die Matrix son und die Gerschgorin-Kreise für die Mengenabschätzung
⎛ ⎞ der eingehenden Wertebereiche.
1 0.3 0 2.1
⎜0.3 2.7 −0.3 0.9⎟ Aufgabe 15.12 •• –
A=⎜
⎝ 0 −0.3 5 0.1⎠

2.1 0.9 0.1 −1 Aufgabe 15.13 •• –

an und leiten Sie aus diesen eine Konvergenzaussage für die Aufgabe 15.14 • –
Potenzmethode sowie eine bestmögliche Abschätzung für
dessen Konvergenzgeschwindigkeit ab.

Aufgabe 15.14 • Stellen Sie sich ein Netz bestehend


Lösungen
aus vier Seiten vor, bei dem zwei Paare vorliegen, deren Sei-
ten auf den jeweiligen Partner verweisen und sonst keine Verständnisfragen
weiteren Links beinhalten. Ist die resultierende Matrix H @
spaltenstochastisch und gibt es in diesem Fall linear unab- Aufgabe 15.1 •• –
hängige Eigenvektoren zum Eigenwert λ = 1?
Beweisaufgaben
Aufgabe 15.2 • –
Hinweise Aufgabe 15.3 ••• –

Verständnisfragen Aufgabe 15.4 ••• –

Aufgabe 15.1 •• – Aufgabe 15.5 •• –

Aufgabe 15.6 • –
Beweisaufgaben
Aufgabe 15.2 • Eine Matrix ist genau dann regulär, Aufgabe 15.7 • –
wenn λ = 0 kein Eigenwert der Matrix ist.
Aufgabe 15.8 • –
Aufgabe 15.3 ••• – Aufgabe 15.9 •• –

Aufgabe 15.4 ••• Weisen Sie unter der Annahme v ⊥


M die Eigenschaft λ ∈
/ ∂W (A) nach. Rechenaufgaben
;
Aufgabe 15.10 • σ (A) ⊆ 3i=1 Ki mit K1 = K(3, 5),
Aufgabe 15.5 •• – K2 = K(4, 3) und K3 = K(−4, 1).

Aufgabe 15.6 • Führen Sie einen Widerspruchs- Aufgabe 15.11 • σ (A) ⊂ [−4, 8] + [−2i, 2i] .
beweis, bei dem Sie die Existenz eines nicht reellwertigen
Eigenwertes annehmen. Aufgabe 15.12 •• Die Eigenwerte lauten λ1 = 23 +
1
√ 3 1

2 17 ≈ 3.56, λ1 = 1 und λ3 = 2 − 2 17 ≈ −0.56.
Aufgabe 15.7 • Führen sie den Nachweis durch ein- Dabei liegen wie zu erwarten alle Eigenwerte in der Menge
faches Nachrechnen. [−2, 4] + [−2i, 2i].

Aufgabe 15.8 • Nutzen Sie aus, dass in disjunkten Aufgabe 15.13 •• Die Gerschgorin-Kreise lauten
Gerschgorin-Kreisen stets nur ein Eigenwert liegt.
K1 = K(1, 2.4), K2 = K(2.7, 1.5)
Aufgabe 15.9 •• Es gilt WR (A) = {α}. K3 = K(5, 0.4), K4 = K(−1, 3.1) .

Die Konvergenzrate q der Potenzmethode erfüllt q < 0.914.


Rechenaufgaben
Aufgabe 15.14 • Die Matrix ist spaltenstochastisch
Aufgabe 15.10 • Betrachten Sie für die obige Frage und besitzt zwei linear unabhängige Eigenvektoren zum Ei-
die Schnittmengen der Gerschgorin-Kreise. genwert λ = 1.
112 Lösungswege zu Kapitel 15

Lösungswege für i, j = 1, ..., N und


n
n n
1
Verständnisfragen mij = α hij +(1 − α) = α + (1 − α) = 1
n
i=1 i=1 i=1
Aufgabe 15.1 ••      
=1 =1
(a) Offensichtlich sind alle Gerschgorin-Kreise durch
K(0, 1) gegeben, sodass hiermit die getroffene Behaup- für j = 1, ..., N.
tung nachgewiesen ist. (b) Als spaltenstochastische Matrix besitzt M laut dem auf
(b) Die Multiplikation eines Vektors mit der Matrix S be- der Seite 549 aufgeführten Beispiel den Eigenwert λ =
wirkt lediglich eine Permutation der Vektoreinträge und 1. Sei x ∈ RN \{0} der zum Eigenwert λ = 1 gehörige
z(k) stellt stets den Koordinateneinheitsvektor ej mit Eigenvektor, dann gilt Mx = x respektive
j = 1 + (k mod n) dar.
(c) Die Folge der Iterierten z(k) zeigt, dass die Potenz-
N
xi = mij xj für i = 1, ..., N.
methode bei dem obigen Startvektor nicht konvergiert.
j =1
Elementares Nachrechnen liefert das charakteristische
Polynom in der Form pA (λ) = (−1)n (λn − 1). So- Aufgrund der in (a) nachgewiesenen Positivität aller Ko-
mit repräsentieren die n-ten komplexen Einheitswurzeln effizienten mij ergibt sich somit
2πk
λk = ei n für k = 1, . . . , n die Eigenwerte der Ma-
trix. Wie leicht zu sehen ist, weisen alle Eigenwerte
N
N
den gleichen Betrag auf, wodurch die für den Konver- |xi | = | mij xj | ≤ mij |xj |,
genzsatz grundlegende Bedingung der Existenz eines j =1 j =1
betragsgrößten Eigenwertes nicht erfüllt ist. Die Auf-
gabe belegt demzufolge noch einmal nachdrücklich die wobei Gleichheit genau dann gilt, wenn alle Komponen-
Notwendigkeit dieser Forderung und steht folglich auch ten des Vektors x nicht negativ oder nicht positiv sind.
nicht im Widerspruch zum Konvergenzsatz. Nehmen wir an, dass x = 0 sowohl positive als auch
negative Komponenten aufweist. Dann liefert

Beweisaufgaben
N
N
N
N
N
|xi | < mij |xj | = |xj | mij = |xj |
Aufgabe 15.2 • Wir nutzen den Satz von Gerschgorin i=1 i,j =1 j =1 i=1 j =1
  
und müssen lediglich zeigen, dass die Null nicht in der Ver- =1
einigungsmenge der Gerschgorin-Kreise liegt. Aufgrund der
strikten Diagonaldominanz der Matrix gilt einen Widerspruch. Folglich sind alle Komponenten von
x entweder nicht negativ oder nicht positiv.

n
Sei xi = 0 für mindestens ein i ∈ {1, ..., N}, dann ergibt
|aii | > |aij | für i = 1, . . . , n,
sich wegen
j =1,j  =i

womit sich direkt die benötigte Eigenschaft 0 ∈


/ Ki , i = 1,
N
0 = |xi | = mij |xj |
. . . , n ergibt. 
j =1
>0
Aufgabe 15.3 ••• Da die Matrix H laut Voraussetzung
sogleich x1 = x2 = ... = xN = 0, womit wiederum
spaltenstochastisch ist, erfüllen ihre Matrixelemente hij die
ein Widerspruch zu x = 0 vorliegt. Zusammenfassend
Bedingungen
besitzt x ausschließlich positive oder negative Kompo-
nenten.
hij ≥ 0 für alle i, j ∈ {1, ..., N}
(c) Wir führen einen Widerspruchsbeweis und nehmen an,
dass die Dimension des Eigenraumes zum Eigenwert
und

n λ = 1 größer als eins ist. Folglich existieren zwei li-
hij = 1 für j ∈ {1, ..., N}. near unabhängige Eigenvektoren v, w zum Eigenwert
i=1 λ = 1. Somit ist auch z = αv + βw für alle α, β ∈ R
mit |α| + |β| = 0 Eigenvektor zu λ = 1.
(a) Für die Elemente der Matrix M = αH + (1 − α) n1 E Mit Teil (b) besitzen die Vektoren v, w ausschließlich
erhalten wir mit α ∈ [0, 1[ die Aussagen positive oder negative Komponenten. Setzen wir
1
mij = αhij + (1 − α) > 0
N
   N α= wi = 0 ,
≥0
>0 i=1
Lösungswege zu Kapitel 15 113


N
gilt und M einen Kreis mit positivem Radius darstellt, kann
so erhalten wir mit der Wahl β = − vi die
i=1 λ im Widerspruch zur Voraussetzung kein Randpunkt von
Schlussfolgerung W (A) sein. Damit folgt die Behauptung aus v ∗ w = 0.


N
N
N Aufgabe 15.5 •• Zunächst lässt sich die Matrix L in
zj = αvj + βwj = (wi vj − vi wj ) = 0. der Form L = I + @ L mit E E
ij = ij für i  = j und  ii = 0
j =1 j =1 i,j =1 schreiben. Folglich ergibt sich

Da z = 0 gilt, muss z sowohl positive als auch negative W = V L = V +  V@L ,


Komponenten besitzen, wodurch ein Widerspruch zum =: E k
Resultat laut Teil (b) vorliegt.  
(k)
wobei alle Komponenten von E k = eij die
Aufgabe 15.4 ••• Wir betrachten zunächst einige allge- i,j =1,...,n
Eigenschaft
meine Eigenschaften. Seien v, w ∈ Cn Eigenvektoren zu λ (k)
eij = O(q k ), k → ∞
respektive μ mit v2 = w2 = 1. Unter der Voraussetzung
λ = μ sind v und w linear unabhängig. Somit gilt erfüllen. Für jeden Vektor x mit x2 = 1 erhalten wir
|xj | ≤ 1 für j = 1, . . . , n und somit
x := v + aw  = 0 für alle a ∈ C .

n
(k)

n
(k)
Der Rayleigh-Quotient zu x lässt sich wegen |(E k x)i | ≤ |eij ||xj | ≤ |eij | = O(q k ), k → ∞ .
j =1 j =1   
=O(q k )
Ax = λv + aμw
Hiermit folgt direkt
in der Form
E k 2 = sup E k x2 = O(q k ), k → ∞ .
x ∗ Ax (v + aw)∗ (λv + aμw) x 2 =1

=
x x (v + aw)∗ (v + aw)
λ + aμv ∗ w + aλw ∗ v + |a|2 μ Aufgabe 15.6 • Wir führen einen Widerspruchsbe-
=
1 + aw∗ v + av ∗ w + |a|2 weis und nehmen an, dass ein Eigenwert λ von A existiert, der
av ∗ w + |a|2 nicht reellwertig ist. Dann existiert ein Vektor x ∈ Cn \ {0}
= λ + (μ − λ) mit Ax = λx. Für x ∈ Cn \ {0} erhalten wir
1 + aw∗ v + av ∗ w + |a|2
  
=: δ(a) Ax = Ax = λx = λx .

schreiben. Somit gilt Somit stellt auch λ = λ einen Eigenwert von A dar und wir
erhalten wegen |λ| = |λ| den gewünschten Widerspruch zur
λ + (μ − λ)δ(a) ∈ W (A) Voraussetzung der Aussage.

für alle a ∈ C. Aufgabe 15.7 • Wir führen den Nachweis nur für den
Fall eines Produktes unitärer Matrizen, da sich die Vorge-
Wir beginnen nun den Widerspruchsbeweis und nehmen an,
hensweise direkt auf orthogonale Matrizen übertragen lässt.
dass v und w nicht senkrecht aufeinanderstehen.
Seien A, B ∈ Cn×n unitär, dann folgt die Behauptung un-
Unter Verwendung der Abkürzung c := v ∗ w  = 0 ergibt sich mittelbar aus

0 = (v + aw)∗ (v + aw) = 1 + ac + ac + |a|2 ∈ R . (AB)∗ AB = B ∗ A



A B = B ∗ B = I .
=I
Wählen wir a = −ceiθ mit θ ∈ [0, 2π[, so erhalten wir mit
b := (1 + ac + ac + |a|2 )−1 die Darstellung Aufgabe 15.8 • Da die Gerschgorin-Kreise paarweise
    disjunkt sind, liegt in jedem Kreis genau ein Eigenwert. Zu-
δ(a) = δ(−ceiθ ) = b −|c|2 eiθ + |c|2 = b|c|2 1 − eiθ . dem liegt der Mittelpunkt jedes Kreises auf der reellen Achse.
Würde ein komplexwertiger Eigenwert λ ∈ Ki existieren,
Die Menge so wäre auch λ Eigenwert von A mit λ ∈ Ki , was im
0  1 Widerspruch zur Existenz genau je eines Eigenwertes pro

M := δ(a) a = −ceiθ mit θ ∈ [0, 2π[ Gerschgorin-Kreis ist.

beschreibt folglich einen Kreis um b|c|2 ∈ R mit Radius Aufgabe 15.9 •• Da der allgemeine Wertebereich einer
r = b|c|2 > 0. Da schiefsymmetrischen Matrix stets ein Intervall auf der imagi-
näre Achse darstellt und andererseits WR (S) ⊂ R gilt, erhal-
λ + (μ − λ)ξ ∈ W (A) für alle ξ ∈ M ten wir WR (S) = {0}. Folglich ergibt sich für alle x ∈ Rn \{0}
114 Lösungswege zu Kapitel 15

aus Wegen K3 ∩ (K1 ∪ K2 ∪ K4 ) = ∅ liegt in K3 genau ein


x T Ax xT I x x T Sx Eigenwert und es gilt
T
=α T + T =α
x x x x xx 
=0 |ξ | > |ν| für alle ξ ∈ K3 und ν ∈ K1 ∪ K2 ∪ K4 .
die Schlussfolgerung WR (A) = {α}. Damit kann die Konvergenzrate q der Potenzmethode durch

maxν∈K1 ∪K2 ∪K4 |ν| 4.2


Rechenaufgaben q≤ = < 0.914
minξ ∈K3 |ξ | 4.6
Aufgabe 15.10 • Aufgrund der resultierenden Men-
nach oben abgeschätzt werden, wodurch wegen q < 1 auch
gen erhalten wir K3 ∩ (K1 ∪ K2 ) = ∅, womit wir schlussfol-
die Konvergenz nachgewiesen ist.
gern können, dass sich in K3 ein Eigenwert und in K1 ∪ K2
zwei Eigenwerte befinden.
Aufgabe 15.14 • Schematisch ergibt sich der Zusam-
Aufgabe 15.11 • Mit menhang
⎛ ⎞
1 3/2 1/2
A + A∗
H = = ⎝3/2 1 3/2⎠
2
1/2 3/2 2 1 2
sowie ⎛ ⎞
0 1 1
A − A∗
S= = ⎝−1 0 −1⎠
2
−1 1 0
erhalten wir 3 4
σ (H ) ⊂ (K(1, 2) ∪ K(1, 3) ∪ K(2, 2)) ∩ R = [−2, 4]
Abbildung 15.1 Netzstruktur
und

σ (S) ⊂ K(0, 2) ∩ {z ∈ C | Re(z) = 0} = [−2i, 2i]. und die spaltenstochastische Matrix weist somit die Form
⎛ ⎞
0 1 0 0
Damit ergibt sich unter Berücksichtigung von W (H ) ⊆ ⎜ ⎟
[−2, 4] und W (S) ⊆ [−2i, 2i] die gesuchte Eigenwertein- @ = ⎜1 0 0 0⎟ ∈ R4×4
H ⎝0 0 0 1⎠
schließung in der Form
0 0 1 0
σ (A) ⊂ W (A) ⊆ R
auf. Damit ergeben sich zum Eigenwert λ = 1 mit
= W (H ) + W (S) ⊆ [−2, 4] + [−2i, 2i]. ⎛1⎞ ⎛ ⎞
2
0
Aufgabe 15.12 •• – ⎜1⎟ ⎜0⎟
⎜ ⎟ ⎜ ⎟
w 1 = ⎜ 2 ⎟ und w2 = ⎜ 1 ⎟
⎝0⎠ ⎝2⎠
Aufgabe 15.13 •• Die Gerschgorin-Kreise lauten 0 1
2
K1 = K(1, 2.4), K2 = K(2.7, 1.5), zwei linear unabhängige Eigenvektoren, die jeweils den Be-
K3 = K(5, 0.4), K4 = K(−1, 3.1). dingungen wi ≥ 0 und w1 = 1 genügen.
Kapitel 16 k
tk
0
−2
1
−1
2
1
3
2
bk 1 3 2 3
Aufgaben Ist das resultierende Polynom von der Variation der Koeffizi-
enten innerhalb des Lösungsraums der Normalgleichungen
Verständnisfragen abhängig?

Aufgabe 16.1 •• Untersuchen Sie die Existenz und Ein- Aufgabe 16.9 • Bestimmen Sie eine Ausgleichskurve
deutigkeit der Lösung des bereits auf Seite 584 vorgestellten der Form y(t) = a1 sin t + a2 cos t derart, dass für die durch
Problems    
1 1 k 0 1 2 3 4
α0 = ,
1 2 tk −π −π /2 0 π /2 π
 
=A =b yk 2 4 1 2 0
wobei anstelle von Aα0 − b2 die Minimierung von 4
gegebenen Messdaten der Ausdruck k=0 (yk − y(tk ))2 mi-
Aα0 − b1 respektive Aα0 − b∞ betrachtet wird. Veran- nimal wird.
schaulichen Sie Ihr Ergebnis in beiden Fällen auch geome-
trisch. Aufgabe 16.10 • Für gegebene Daten

Aufgabe 16.2 • Warum kann zu einer Matrix A ∈ k 0 1 2 3 4


Rm×n mit m > n keine Matrix B existieren, die der Glei- tk −1 0 1 2 3
chung BA = AB genügt? yk 2.2 1.1 1.9 4.5 10.2
zk −4.8 −3.2 −0.8 1.5 2.8
Aufgabe 16.3 • Überprüfen Sie folgende Aussage:
Die Inverse B ∈ Rn×n einer regulären Matrix A ∈ Rn×n bestimme man
erfüllt die Eigenschaften einer Moore-Penrose-Inversen. jeweils
 eine Ausgleichsgerade g(t) = a1 + a2 t derart,
dass 4k=0 (yk − g(tk ))2 respektive 4k=0 (zk − g(tk ))2
Aufgabe 16.4 • Gilt für die Moore-Penrose-Inversen minimal wird.
die Rechenregel jeweils = b1 + b2 t 2 derart,
4eine Ausgleichsparabel p(t) 
(AB)/ = B / A/ ? dass k=0 (yk − p(tk )) respektive 4k=0 (zk − p(tk ))2
2

minimal wird.
Berechnen Sie für alle vier obigen Fälle den minimalen
Beweisaufgaben Wert und begründen Sie ausschließlich durch Betrachtung
der Werte yk , zk , k = 0, . . . , 4 die erzielten Ergebnisse
Aufgabe 16.5 • Beweisen Sie die Behauptung: Jede
Permutationsmatrix ist orthogonal.
4
4
(yk − g(tk ))2 0 (yk − p(tk ))2
Aufgabe 16.6 • Zeigen Sie: Die Matrix A ∈ Rm×n k=0 k=0
mit m ≥ n besitzt genau dann maximalen Rang, wenn die
und
Matrix A A ∈ Rn×n invertierbar ist.
4
4
(zk − g(tk ))2 # (zk − p(tk ))2 .
Aufgabe 16.7 • Sei eine Matrix A mit Moore- k=0 k=0
Penrose-Inverse A/ gegeben. Zeigen Sie, dass für B = cA
mit c ∈ R \ {0} die Eigenschaft
Aufgabe 16.11 •• Bestimmen Sie die Moore-Penrose-
1 Inverse A/ der Matrix
B/ = A/  
c 3 4
A= .
gilt. 6 8

Rechenaufgaben Aufgabe 16.12 •• Berechnen Sie mittels einer QR-


Aufgabe 16.8 •• Wir betrachten die bereits auf Zerlegung oder unter Verwendung der Normalgleichungen
Seite 587 vorgestellten linear abhängigen Funktionen eine Ausgleichsgerade der Form p(t) = α0 + α1 t, die bei
den gegebenen Daten
0 (t) = t, 1 (t) = (t − 1)2 − 1, 2 (t) = t 2 .
k 0 1 2 3
Berechnen Sie die auf der Grundlage dieser Funktionen er- tk 1 2 3 4
zielte Optimallösung zu den folgenden Daten: bk 3 2 5 4
M. Brokate et al., Arbeitsbuch Grundwissen Mathematikstudium – Höhere Analysis, Numerik und
Stochastik, DOI 10.1007/978-3-642-54946-5_15, © Springer-Verlag Berlin Heidelberg 2016
116 Lösungen zu Kapitel 16


den Ausdruck 3k=0 (bk − p(tk ))2 über die Menge aller af- Aufgabe 16.6 • –
fin linearen Funktionen minimiert. Tragen Sie zunächst die
Punkte in ein Koordinatensystem ein und stellen Sie eine Aufgabe 16.7 • Nutzen Sie die explizite Darstellung
grafische Vorabvermutung für die Lösung auf. der Matrix A innerhalb der Singulärwertzerlegung.

Aufgabe 16.13 •• Wir wenden uns durch diese Aufga-


ben nochmals der bereits im Beispiel auf Seite 588 betrachte-
Rechenaufgaben
ten Problemstellung zur Ermittlung von Prognosewerten für Aufgabe 16.8 •• Die Matrix der zugehörigen Normal-
den Bremsweg eines Autos zu. Entgegen den im angegebe- gleichungen kann der Seite 587 entnommen werden.
nen Beispiel vorliegenden Daten sind diese in dieser Aufgabe
durch Aufgabe 16.9 • Stellen Sie das lineare Ausgleichspro-
k 0 1 2 3 4 blem auf und lösen Sie es unter Verwendung der Normalglei-
chungen.
vk [m/s] 2.78 5.56 8.33 11.11 13.89
sk [m] 1.2 3.8 9.2 17 24.9 Aufgabe 16.10 • –
bezüglich der Geschwindigkeit in einer anderen Einheit ge-
geben. Berechnen Sie bezogen auf die obigen Daten ein Aufgabe 16.11 •• Orientieren Sie sich am Beispiel auf
Polynom  s(v) = β @v + @ γ v 2 , das die Summe der Fehler- Seite 601.
4
quadrate k=0 (sk − s(vk ))2 über alle reellen Koeffizienten
minimiert und bestimmen Sie damit Prognosewerte für die Aufgabe 16.12 •• –
Geschwindigkeiten 22.22 ms = 80 km m km
h , 27.78 s = 100 h und
Aufgabe 16.13 •• Nutzen Sie die zugehörigen Normal-
41.67 ms = 150 kmh und vergleichen Sie diese mit den Resulta- gleichungen und beachten Sie die Selbstfrage auf Seite 586.
ten laut Beispiel auf Seite 588. In welcher Beziehung stehen
die Koeffizienten der jeweiligen Lösungsdarstellung?
Aufgabe 16.14 • Nutzen Sie die Normalgleichungen.
Aufgabe 16.14 • Bestimmen Sie die Menge der Aus-
gleichslösungen und die Optimallösung zum linearen Aus-
!
gleichsproblem  Ax − b 2 = min mit
⎛ ⎞ ⎛ ⎞ Lösungen
1 0 1 1
⎜0 1 1⎟ ⎜1⎟
A=⎜ ⎟
⎝0 1 1⎠ und b = ⎝2⎠ .
⎜ ⎟
Verständnisfragen
1 0 1 4
Aufgabe 16.1 •• Die Eindeutigkeit der Lösung ist dabei
nur noch im Fall der Maximumnorm und nicht mehr bei der
Betrachtung der Betragssummennorm gegeben.
Hinweise
Aufgabe 16.2 • –

Verständnisfragen Aufgabe 16.3 • Die Aussage ist richtig.


Aufgabe 16.1 •• Bei  · 1 handelt es sich um die Be-
Aufgabe 16.4 • Nein.
tragssummennorm,
n die für jeden Vektor x ∈ Rn durch
x1 = i=1 |xi | definiert ist. Durch  · ∞ ist die Ma-
ximumnorm mit x∞ = maxi=1,...,n |xi | gegeben. Beweisaufgaben
Aufgabe 16.2 • Beachten Sie die notwendige Dimen- Aufgabe 16.5 • –
sion der Produktmatrizen.
Aufgabe 16.6 • –
Aufgabe 16.3 • Einfaches Nachrechnen der vier
Eigenschaften führt zum Ziel. Aufgabe 16.7 • –

Aufgabe 16.4 • Konstruieren Sie ein Gegenbeispiel. Rechenaufgaben


Aufgabe 16.8 ••
Beweisaufgaben 1
(t) = (104 0 (t) + 1 1 (t) + 209 2 (t))
Aufgabe 16.5 • Nutzen Sie aus, dass sich jede Permu- 340
tationsmatrix durch eine Permutation der Vektoren der Iden- 1  
= 210 t 2 + 102 t
titätsmatrix ergibt. 340
Lösungswege zu Kapitel 16 117

Das Polynom ist unabhängig von der Variation der Koeffizi- womit wir analog zum euklidischen Abstandsbegriff die Lö-
enten innerhalb des Lösungsraums der Normalgleichungen. sung gemäß
3
1 α = arg min f (α0 ) =
Aufgabe 16.9 • y(t) = − sin t − 3 cos t α 0 ∈R 2
erhalten.
Aufgabe 16.10 •
y-Werte: g(t) = 2.04 + 1.94 t, p(t) = 0.969 + 1.004 t 2
z-Werte: g(t) = −2.89 + 1.99 t, p(t) = −3.072 + 0.724 t 2

Aufgabe 16.11 ••
 
1 3 6
A/ = .
125 4 8

Aufgabe 16.12 ••
p(t) = 2 + 0.6 t

Aufgabe 16.13 ••
1 Abbildung 16.1 Geometrische Interpretation der Lösung für die Maximum-
s(v) = (5.71 v + 12.68 v 2 ) norm.
100

Aufgabe 16.14 •Die allgemeine Lösungsdarstellung


lautet ⎛ ⎞ ⎛ ⎞
5 −1
1⎝ ⎠
x= 3 + λ ⎝−1⎠ mit λ ∈ R.
2
0 1
Dabei stellt ⎛ ⎞
7
1⎝ ⎠
x= 1
6
8
die Optimallösung dar.

Abbildung 16.2 Geometrische Interpretation der Lösung für die Betragssum-


Lösungswege mennorm.

Verständnisfragen Aufgabe 16.2 • Um die Produkte BA und AB zu er-


möglichen, muss notwendigerweise B ∈ Rn×m gelten. Hier-
Aufgabe 16.1 •• Wegen
mit erhalten wir jedoch BA ∈ Rn×n und AB ∈ Rm×m ,
/   /
/ 1 1 / wodurch wegen m > n keine Übereinstimmung vorliegen
f (α0 ) := /
/ 1 α − / = |α0 − 1| + |α0 − 2|
0
2 /1 kann.

erhalten wir für α0 ∈


/ [1, 2] direkt Aufgabe 16.3 • Wegen AB = I ergeben sich die Be-
dingungen der Pseudoinversen unmittelbar aus
f (α0 ) = |2α0 − 3| > 1
BAB = BI = B und ABA = I A = A.
und für α0 ∈ [1, 2] stets
Die für die Moore-Penrose-Inversen geforderten Symmetrien
f (α0 ) = α0 − 1 + 2 − α0 = 1. erkennen wir leicht gemäß

Somit stellen alle α ∈ [1, 2] eine Lösung des Minimierungs- BA = I = AB.


problems dar.
Im Fall der Maximumnorm ergibt sich Aufgabe 16.4 • Schreiben wir A = (2, 1) und B =
/   / (0, 1) , so folgt
/ 1 1 /
f (α0 ) := /
/ 1 0α − / = max{|α0 − 1|, |α0 − 2|},
2 /∞ AB = 1 und damit (AB)/ = 1.
118 Lösungswege zu Kapitel 16

Für die jeweilige Pseudoinverse ergibt sich schreiben lässt. Die Moore-Penrose-Inversen stehen damit in
  der Beziehung
1 2  
A/ = und B / = (0, 1). 1 F−1
5 1 / S 0
B =V c U
0 0
Nachrechnen zeigt  
1 F −1 1
= V S 0 U  = A/ .
1
B / A/ =  = 1 = (AB)/ . c 0 0 c
5

Rechenaufgaben
Beweisaufgaben
Aufgabe 16.8 ••
Die Normalgleichungen lauten
Aufgabe 16.5 • Jede Permutationsmatrix P ∈ Rn×n ⎛ ⎞ ⎛ ⎞
lässt sich mittels einer Permutation π : {1, . . . , n} → 10 −20 0 3
⎝−20 74 34⎠ x = ⎝15⎠ ,
{1, . . . , n} darstellen. Die Form der Matrix lautet dabei unter
Verwendung der Koordinateneinheitsvektoren e1 , . . . , en 0 34 34 21

womit sich die Lösungsmenge in der Form


P = (eπ(1) , . . . , eπ(n) ).
⎛ 522 ⎞ ⎛ ⎞
340 −2
Mit ⎜ ⎟ ⎜ ⎟
x = ⎝ 210
340 ⎠
+λ ⎝−1⎠ mit λ ∈ R

1, falls i = j, 1
(P  P )ij = e
π(i) eπ(j ) =
0
     
0 sonst
= x = x 
ergibt sich die behauptete Orthogonalitätseigenschaft.
ergibt. Da der Kern der Matrix A A eindimensional ist,
schreibt sich die Optimallösung in der Form
Aufgabe 16.6 •
⎛ 522 ⎞ ⎛ ⎞ ⎛ 104 ⎞
„⇒“ Besitze A maximalen Rang.   340 −2 340
 x , x   ⎜ 210 ⎟ 209 ⎜ ⎟ ⎜ 1 ⎟
x=x + x = ⎝ ⎠ + ⎝ −1 ⎠ = ⎝ 340 ⎠ .
Aufgrund der Voraussetzung m ≥ n sind alle Spaltenvekto- x  22 340 340
209
ren der Matrix A linear unabhängig. Für x ∈ Kern(A A) 0 1 340
folgt aus
Aufgabe 16.9 • Wir erhalten das lineare Ausgleichs-
0 = A Ax, x = Ax, Ax = Ax22 !
problem Ax − b2 = min mit
direkt Ax = 0 und somit ergibt sich aufgrund der voraus- ⎛ ⎞ ⎛ ⎞
gesetzten linearen Unabhängigkeit der Spaltenvektoren der sin(−π ) cos(−π ) 0 −1
⎜sin(−π/2) cos(−π/2)⎟ ⎜−1 0 ⎟
Matrix x = 0. Folglich ist die Invertierbarkeit von A A ⎜ ⎟ ⎜ ⎟
nachgewiesen. A=⎜⎜ sin(0) cos(0) ⎟ ⎜ ⎟
⎟=⎜ 0 1 ⎟
⎝ sin(π/2) cos(π/2) ⎠ ⎝ 1 0 ⎠
„⇐“ Sei A A invertierbar. sin(π ) cos(π ) 0 −1
Wir führen einen Widerspruchsbeweis und nehmen an, dass und ⎛ ⎞
die Spalten von A nicht linear unabhängig sind. Dann exi- 2
stiert ein x ∈ Rn \ {0} mit Ax = 0, der offensichtlich auch ⎜4 ⎟
⎜ ⎟
A Ax = 0 erfüllt, womit ein Widerspruch zur Vorausset- b=⎜ ⎟
⎜ 1⎟ .
zung vorliegt. ⎝2⎠
0
Aufgabe 16.7 • Mit dem Satz zur Singulärwertzerle- Folglich ergeben sich die zugehörigen Normalgleichungen
gung erhalten wir die Darstellung der Matrix A in der Form gemäß     
  2 0 a1 −2
FS 0 = ,
A=U V , 0 3 a2 −1
0 0
womit die Lösung a1 = −1 und a2 = −1/3 lautet.
womit sich die Matrix B als
  Aufgabe 16.10 • Die Koeffizienten ergeben sich ein-
cF
S 0 fach durch Verwendung der Normalgleichungen. Die y-
B=U V
0 0 Daten kommen entsprechend ihrer Lage eher einer Parabel
Lösungswege zu Kapitel 16 119

nahe, während die z-Werte besser einer Geraden entgegen- und damit der Lösungsvektor gemäß
kommen. Demzufolge sind auch die Zusammenhänge  −1    
−2 −5 −7.00 2

4
4 x= = .
0 −2.23 −1.34 0.6
(yk − g(tk ))2 = 4.14 0 0.59 = (yk − p(tk ))2
k=0 k=0 Mittels der Normalgleichungen

4
4    
(zk − g(tk ))2 = 0.60 # 3.41 = (zk − p(tk ))2 4 10
x=
14
k=0 k=0 10 30 38
     
nicht weiter verwunderlich. = A A = A b

Aufgabe 16.11 •• Mit ergibt sich, wie auch über den QR-Ansatz berechnet, die Lö-
  sung zu    
1 3 4
W =  −1 14 2
5 4 −3 x = (A A) = .
38 0.6
erhalten wir Bezugnehmend auf die erste Annahme der Lösungsgestalt
     aufgrund der grafischen Betrachtung, erhalten wir hiermit
1 3 4 3 4 5 0
AW = = . durch p(t) = 2 + 0.6 t ein abweichendes Polynom. Verglei-
5 6 8 4 −3 10 0
chen wir die Fehlerterme für p und q, so zeigt sich in der Tat,
Die QR-Zerlegung dieser Matrix schreibt sich in der Form dass
  √ 
1 1 2 5 5 0
3
3
AW = √ ,
5 2 −1 0 0 (bk − p(tk ))2 = 3.2 < 4 = (bk − q(tk ))2
   k=0 k=0
=Q
gilt und q entgegen der ersten Vermutung nicht der Lösung
womit sich die Moore-Penrose-Inverse zu entspricht.
√   
5 1 3 6
/
A = W 25 0 Q =
0 0 125 4 8 Aufgabe 16.13 •• Das überbestimmte Gleichungssy-
stem lautet
ergibt. ⎛ ⎞ ⎛ ⎞
2.78 7.72 1.2
⎜ 5.56 30.86 ⎟   ⎜ 3.8 ⎟
Aufgabe 16.12 •• Trägt man die Punkte in ein Koor- ⎜ ⎟ @ ⎜ ⎟
⎜ 8.33 69.44 ⎟ β = ⎜ 9.2 ⎟ .
dinatensystem ein und legt gedanklich jeweils eine Gerade ⎜ ⎟ @
γ ⎜ ⎟
⎝11.11 123.46⎠ ⎝ 17 ⎠
durch die Punkte (1, 3) und (3, 5) sowie (2, 2) und
(4, 4) , so kommt eventuell die Vermutung auf, dass die 13.89 192.90 24.9
     
Lösung parallel und in der Mitte dieser beiden Geraden ver- =A =b
laufen könnte und folglich q(t) = 1 + t lautet. Gehen wir
von dieser visuellen Anschauung zurück zur mathematisch Mittels der zugehörigen Normalgleichungen
    
fundierten Vorgehensweise und betrachten das lineare Aus- 4.24 48.23 @
β 635.83
! 100 = ,
gleichsproblem Ax −b2 = min mit den eingehenden Grö- 48.23 582.87 @
γ 7667.43
     
ßen ⎛ ⎞ ⎛ ⎞
1 1 3 = A A = A b
⎜1 2⎟ ⎜2⎟
A=⎜ ⎟
⎝1 3⎠ und b = ⎝5⎠ .
⎜ ⎟ wodurch sich das Ergebnis
   
1 4 4 @
β 1 5.71
= (A A)−1 A b =
Mit der QR-Zerlegung @
γ 100 12.68
⎛ ⎞⎛ ⎞
−0.50 0.67 0.02 0.55 −2 −5 und hiermit die Lösungsdarstellung
⎜−0.50 0.22 −0.44 −0.71⎟ ⎜ 0 −2.23⎟
A=⎜ ⎟⎜
⎝−0.50 −0.22 0.81 −0.22⎠ ⎝ 0
⎟ 1
0 ⎠ s(v) = (5.71 v + 12.68 v 2 )
−0.50 −0.67 −0.39 0.38 0 0 100
     sowie die Prognosewerte
=Q =R
v [m/s] 22.22 27.78 41.67
ergibt sich ⎛ ⎞ s(v) [m] 63.90 99.44 222.56
−7.00
⎜−1.34⎟ ergeben. Die hier ermittelten Koeffizienten erfüllen in Bezug
b=Q b=⎜
F 
⎝ 1.66 ⎠

auf die im erwähnten Beispiel erhaltenen Werte den Zusam-
0.66 menhang β @ = 3.6 β und @ γ = 3.62 γ .
120 Lösungswege zu Kapitel 16

Aufgabe 16.14 • Die Normalgleichungen lauten Lösungsmenge hat demzufolge die Darstellung
⎛ ⎞ ⎛ ⎞ ⎛ ⎞ ⎛ ⎞
2 0 2 5 5 −1
1
⎝0 2 2⎠ x = ⎝3⎠ . x = ⎝3⎠ +λ ⎝−1⎠ mit λ ∈ R.
2
2 2 4 8 0 1
        
= x
= A A = A b
Mit ⎛ ⎞
−1
Offensichtlich ist die Matrix A A nicht invertierbar. Subtra- 1 ⎝ ⎠
ξ= √ −1
hieren der ersten und zweiten Zeile von der dritten ergibt das 3 1
äquivalente System
⎛ ⎞ ⎛ ⎞ liegt eine Orthonormalbasis des Kerns von A A vor. Laut
2 0 2 5 Seite 591 ergibt sich die Optimallösung somit gemäß
⎝0 2 2⎠ x = ⎝3⎠ .
x = x  − x  , ξ ξ
0 0 0 0 ⎛ ⎞ ⎛ ⎞ ⎛ ⎞
5 −1 7
1 ⎝ ⎠ −4 1 ⎝ ⎠ 1 ⎝ ⎠
Setzen wir x3 = t, so folgt aus der zweiten Zeile x2 = 3/2−t = 3 −√ √ −1 = 1 .
und entsprechend aus der ersten Zeile x1 = 5/2 − t. Die 2 3 3 1 6
0 8
Kapitel 17 Aufgabe 17.9 ••• Beweisen Sie mithilfe des Satzes von
Newton-Kantorowitsch auf Seite 639 die folgende Verallge-
meinerung:
Aufgaben Gibt es eine Matrix C ∈ Rm×m , sodass
1. Cf (x 0 ) ≤ η,
Verständnisfragen 2. Cf  (x 0 ) − I  ≤ τ ,
Aufgabe 17.1 • Warum ergibt jeder Schritt des Bisek- 3. Cf  (x) ≤ δ für x ∈ Ur (x 0 ),
tionsverfahrens eine weitere Ziffer in der Dualdarstellung und gelten
der Näherungslösung?
δη 1
h := ≤ , τ <1
Aufgabe 17.2 • Wie lautet der absolute Fehler im n- (1 − τ ) 2 2
ten Schritt des Bisektionsverfahrens?
und √
Aufgabe 17.3 • Geben Sie ein Beispiel einer Funktion 1−1 − 2h η
r ≥ r0 := ,
f auf [a, b], bei der f (a) · f (b) < 0 gilt, die aber keine h 1−τ
Nullstelle in [a, b] besitzt. dann hat die Gleichung f (x) = 0 eine Lösung ξ in der Kugel
x − x 0  ≤ r0 . Ist
Aufgabe 17.4 •• Die Funktion f (x) = x 42 besitzt im √
Punkt x = 0 eine 42-fache Nullstelle. Bleibt das Newton- 1 − 1 − 2h η
r1 := ,
Verfahren ohne Modifikationen anwendbar? Falls ja: Wie h 1−τ
groß ist die Konvergenzgeschwindigkeit?
dann ist für h < 1/2 diese Lösung eindeutig, wenn r < r1 .
Für h = 1/2 folgt die Eindeutigkeit im Fall r = r1 .
Beweisaufgaben
Aufgabe 17.5 • Zeigen Sie, dass das Iterationsverfah- Aufgabe 17.10 ••• Im Satz von Newton-Kantorowitsch
ren auf Seite 639 ist die Größe Gf  (x) abzuschätzen. Natürlich
(xn )p − q kann man immer
xn+1 = xn − , p ∈ N, q ∈ R ,
p(xn )p−1
Gf  (x) ≤ G f  (x)
die p-te Wurzel aus q liefert.
abschätzen, wobei die erste Norm auf der rechten Seite der
Aufgabe 17.6 •• Leonardo von Pisa (ca. 1180–1241), Ungleichung eine Matrixnorm und die zweite die Norm einer
genannt Fibonacci, berechnete auf heute unbekannte Weise Bilinearform ist, vergleiche (17.41) und (17.42). Zeigen Sie
die Nullstelle ξ = 1.368808107 der Gleichung stattdessen, wie das Produkt Gf  (x) tatsächlich aussieht,
f (x) = x 3 + 2x 2 + 10x − 20 . und beweisen Sie die Normabschätzung (17.42)
 
Betrachten Sie die Iteration xn+1 = (xn ) mit der Iterati- m m  m 
 ∂ 2f 
onsfunktion 
(Gf (x)) ∞ ≤ max  gkμ
μ
(x)

i=1 j =1 μ=1 
20 k=1,...,m ∂x ∂x
i j
(x) = 2 .
x + 2x + 10
a) Zeigen Sie die Konvergenz der Iteration auf [1, 2]. für diese Bilinearform.
b) Erklären Sie die Konvergenzgeschwindigkeit.

Aufgabe 17.7 • Wählt man in Aufgabe 17.6 die Itera- Rechenaufgaben


tionsfunktion Aufgabe 17.11 •• Die Iteration aus Aufgabe 17.6 lie-
20 − 2x 2 − x 3 fert erst im 24. Schritt die Ziffernfolge des Fibonacci,
(x) = , x24 = 1.368808107. Berechnen Sie mit der Aitken’schen
10
dann konvergiert die Iteration xn = (xn−1 ) auf [1, 2] nicht. 2 -Methode eine Verbesserung der Näherung aus den drei
Zeigen Sie, warum keine Konvergenz zu erwarten ist. Werten:

Aufgabe 17.8 • Zeigen√Sie, dass man das Heron-Ver- x10 = 1.368696397


fahren zur Berechnung von 2 allgemein zur Berechnung x11 = 1.368857688

von r für r > 0 verwenden kann und dass es in der Form x12 = 1.368786102
 
1 r
xn = xn−1 +
2 xn−1 Aufgabe 17.12 •• Wir gehen noch einmal zurück zur
geschrieben werden kann. Zeigen Sie weiter, dass es sich um Funktion
eine Variante des Newton-Verfahrens handelt. f (x) = x 3 + 2x 2 + 10x − 20
M. Brokate et al., Arbeitsbuch Grundwissen Mathematikstudium – Höhere Analysis, Numerik und
Stochastik, DOI 10.1007/978-3-642-54946-5_16, © Springer-Verlag Berlin Heidelberg 2016
122 Hinweise zu Kapitel 17

aus Aufgabe 17.6. Wenden Sie das Newton-Verfahren an mit beobachten. Erstellen Sie drei Dateien: In die k-te Datei
x0 = 1 und iterieren Sie so lange, bis sich Fibonaccis Ziffern- (k = 1, 2, 3) schreiben Sie alle diejenigen Punkte, bei denen
folge 1.368808107 ergibt. Rechnen Sie dazu mit 10 Nach- die Iteration gegen z(k) konvergiert. Plotten Sie die Inhalte
kommastellen. der drei Dateien mit jeweils anderer Farbe übereinander. Was
beobachten Sie?
Aufgabe 17.13 •• Zu berechnen sind die Schnittpunkte
des Kreises x 2 + y 2 = 2 mit der Hyperbel x 2 − y 2 = 1
a) direkt durch Einsetzen, Hinweise
b) mithilfe des Newton-Verfahrens für Systeme.
Wählen Sie x 0 = (1, 1) und rechnen Sie mit sechs Nach- Verständnisfragen
kommastellen. Brechen Sie nach der Berechnung von x 3 ab.
Aufgabe 17.1 • –
Aufgabe 17.14 ••• Stellen Sie das Newton-Verfahren
Aufgabe 17.2 • –
für das System
Aufgabe 17.3 • Eine solche Funktion kann nicht stetig
f1 (x, y) = 3x 2 y + y 2 − 1 = 0
sein.
f2 (x, y) = x 4 + xy 2 − 1 = 0
Aufgabe 17.4 •• –
auf. Dazu bestimmen Sie bitte graphisch eine Näherung für
die Nullstelle x 0 = (x0 , y0 ) im Rechteck (0, 2) × (0, 1)
und verwenden Sie diese Näherung als Startwert. Berech-
Beweisaufgaben
nen Sie die Matrix G = f  (x 0 ))−1 und finden Sie Ab- Aufgabe 17.5 • Führen Sie das Iterationsverfahren auf
schätzungen für Gf (x 0 ) und Gf  (x) im Rechteck das Newton-Verfahren zurück.
[0.93, 1] × [0.27, 0.34]. Verwenden Sie im R2 die Vektor-
norm x∞ =  max{|x|, |y|}, Aufgabe 17.6 •• a) Zeigen Sie:  ist eine Selbstabbil-
die verträgliche Matrixnorm
A∞ = max{ 2k=1 |a1k |, 2k=1 |a2,k |} und dementspre- dung von [1, 2] und sup1≤x≤2 | (x)| < 1.
chend Norm (17.42) für die symmetrische Bilinearform b) Drücken Sie den Fehler im n-ten Schritt aus durch einen
Gf  (x). Berechnen Sie in der Abschätzung für Gf  (x) Faktor mal den Fehler des (n − 1)-ten Schrittes und verwen-
nur den ersten Summanden. den Sie das ξ aus der Aufgabenstellung.

Aufgabe 17.15 ••• Wir wissen aus Aufgabe 17.5, dass Aufgabe 17.7 • –
das Iterationsverfahren
Aufgabe 17.8 • –
(xn )p − q
xn+1 = xn − , p ∈ N, q ∈ R,
p(xn )p−1 Aufgabe 17.9 ••• Weisen Sie die Voraussetzungen des
Satzes von Newton-Kantorowitsch auf Seite 639 nach. Um
die p-te Wurzel aus q liefert. Setzt man q = 1, p = 3, und
auf die Existenz von (f  (x 0 ))−1 schließen zu können, ver-
wendet das Verfahren auf komplexe Zahlen z = x + iy an,
wenden Sie das Störungslemma von Seite 283 mit den sich
dann lautet es
anschließenden Folgerungen in der folgenden Form:
 
z3 − 1 1 1 Ist U ∈ Rm×m mit
zn+1 = zn − n 2 = zn − zn − 2 .
3zn 3 zn
U  ≤ q < 1,
Leiten Sie aus dieser komplexen Form zwei Iterationsver-
fahren für Real- bzw. Imaginärteil von zn = xn + iyn her. dann hat die Matrix U − I eine Inverse mit
Schreiben Sie ein Computerprogramm in der Sprache Ihrer 1
(U − I )−1  ≤ .
Wahl, das das Quadrat [−2, 2] × [−2, 2] ∈ C in 8002 Punkte 1−q
zerlegt und jeden dieser Punkte als Startwert für die Itera-
tion verwendet. Iterieren Sie 2000 Mal. Sie werden je nach
Aufgabe 17.10 ••• Beachten Sie, dass G = f  (x 0 ) eine
Startwert Konvergenz gegen die drei möglichen dritten Ein-
Matrix mit konstanten Einträgen ist! Sie können daher die
heitswurzeln
zweite Fréchet-Ableitung des Produkts Gf  (x) berechnen.
z(1) = 1 + i0 = 1,
i√ Rechenaufgaben
z(2) = cos 60◦ − i sin 60◦ = 0.5 − 3,
2 Aufgabe 17.11 •• –
i√
z(3) = − sin 30◦ − i cos 30◦ = −0.5 − 3
2 Aufgabe 17.12 •• –
Lösungswege zu Kapitel 17 123

Aufgabe 17.13 •• – Lösungswege


Aufgabe 17.14 ••• Verwenden Sie die Abschätzung aus
Verständnisfragen
Aufgabe .
Aufgabe 17.1 • –
Aufgabe 17.15 ••• Um den Grenzwert numerisch zu er-
kennen, reicht es, nach der Iteration das Produkt von Real- Aufgabe 17.2 • –
und Imaginärteil auf = 0, < 0 und > 0 abzufragen.
Aufgabe 17.3 • Die Funktion
#
a+b
−1 : a ≤ x ≤ 4
f (x) = a+b
Lösungen 1 : 4 <x≤b

besitzt keine Nullstelle in [a, b], aber es gilt f (a) · f (b) =


Verständnisfragen −1 < 0.

Aufgabe 17.1 • Das Bisektionsverfahren liefert als be- Aufgabe 17.4 •• Für f (xn ) = (xn )42 ist f  (xn ) =
ste Schätzung der Nullstelle xn = (an +bn )/2, wobei [an , bn ] 42(xn )41 , also folgt für das Newton-Verfahren
das im n-ten Schritt durch Bisektion konstruierte Teilintervall
ist. Division durch 2 = (10)2 bedeutet im Dualsystem das f (xn ) (xn )42 1
xn+1 =xn − = xn − = xn − xn
Streichen der letzten Stelle des Dividenden („Rechtsshift“). f  (xn ) 42(xn )41 42
41
= xn .
Aufgabe 17.2 • Als absoluter Fehler im n-ten Schritt 42
ergibt sich Das Verfahren ist also uneingeschränkt anwendbar. Da wir
bn − an
|ξ − xn | ≤ . eine lineare Gleichung erhalten haben, kann die Konvergenz
2n nicht quadratisch sein. In der Tat gilt
41
Aufgabe 17.3 • – |xn−1 − 0| = |xn − 0|,
42
Aufgabe 17.4 •• – und damit ist die Konvergenz linear.

Beweisaufgaben
Beweisaufgaben

Aufgabe 17.5 • Die Zahl p q ist Nullstelle der Funk-
Aufgabe 17.5 • –
tion f (x) = x p − q. Dann ist f  (x) = px p−1 und das
Newton-Verfahren liefert
Aufgabe 17.6 •• –
f (xn ) (xn )p − q
xn+1 = xn − 
= xn − .
Aufgabe 17.7 • – f (xn ) p(xn )p−q

Aufgabe 17.8 • – Aufgabe 17.6 •• a) Der Nenner N(x) := x 2 + 2x + 10


der Iterationsfunktion  ist auf [1, 2] wegen N  (x) = 2x +
Aufgabe 17.9 ••• – 2 > 1 und N  (x) = 2 > 0 monoton wachsend und es
existiert in diesem Intervall keine Nullstelle. Es gilt daher
Aufgabe 17.10 ••• – min1≤x≤2 N(x) = N(1) = 13 und max1≤x≤2 N(x) =
N(2) = 18. Also ist  auf [1, 2] eine monoton fallende
Funktion mit
Rechenaufgaben
10
Aufgabe 17.11 •• – min (x) = (2) = = 1.1
1≤x≤2 9
und
Aufgabe 17.12 •• – 20
max (x) = (1) = ≈ 1.54.
1≤x≤2 13
Aufgabe 17.13 •• –
Somit ist  eine Selbstabbildung  : [1, 2] → [1, 2]. Wir
untersuchen für x ∈ [1, 2]
Aufgabe 17.14 ••• –
−40(x + 1) −40(x + 1)
 (x) = =
Aufgabe 17.15 ••• – + 2x + 10)
(x 2 2 N(x)2
124 Lösungswege zu Kapitel 17

und erhalten und dann folgen die an als arithmetische Mittel an = (an−1 +
bn−1 )/2 und die bn werden jeweils so angepasst, dass an ·
−40(x + 1) 40
 (x) ≤ 2
=− (x + 1), bn = 2 erfüllt ist, also bn = 2/an und damit bn−1 = 2/an−1 .
13 169 Setzen wir das in das arithmetische Mittel für an ein, so folgt
also | (x)| ≤ 120  
169 < 1. Nach (17.17) haben wir damit die an−1 + bn−1 1 2
Lipschitz-Konstante an = = an−1 + .
2 2 an−1
C := sup | (x)| < 1 Damit ist bereits die geforderte Form erreicht. Will man die
1≤x≤2
Wurzel aus einer anderen positiven Zahl als 2 ziehen, dann
gefunden. Die Iterationsfunktion  ist also auf [1, 2] folgt
eine Kontraktion und die Iteration ist konvergent für alle  
an−1 + bn−1 1 r
x0 ∈ [1, 2]. an = = an−1 + .
2 2 an−1
b) Der Fehler im n-ten Iterationsschritt ist en = ξ − xn und √
mit dem Mittelwertsatz der Differenzialrechnung folgt die Die Zahl r ist Nullstelle der Funktion f (x) = x 2 −r. Dann
Existenz einer Zahl η zwischen ξ und xn , sodass folgt f  (x) = 2x und das Newton-Verfahren liefert

en = ξ − xn = (ξ ) − (xn−1 ) =  (η)(ξ − xn−1 ) f (xn−1 ) (xn−1 )2 − r


xn =xn−1 − = xn−1 −
=  (η)en−1 f  (xn−1 ) 2xn−1
 
2(xn−1 ) − (xn−1 ) − r
2 2 1 r
gilt. Wenn n hinreichend groß ist, können wir η ≈ ξ anneh- = = xn−1 − .
2xn−1 2 xn−1
men. Setzen wir das aus der Aufgabe bekannte ξ ein, dann
folgt
Aufgabe 17.9 ••• Wir verwenden Voraussetzung 2.,
 (1.368808107) Cf  (x 0 ) − I  = I − Cf  (x 0 ) ≤ τ , und den Satz von
−40(1.368808107 + 1) Banach aus dem Hinweis und setzen
=
((1.368808107)2 + 2 · 1.368808107 + 10)2
U := Cf  (x 0 ) − I .
≈ −0.44.
Wegen Voraussetzung 2. gilt
Der Fehler im n-Schritt ist also nur das −0.44-Fache des
vorhergehenden Fehlers, d. h., der Fehler wird von Schritt zu U  = Cf  (x 0 ) − I  ≤ τ < 1,
Schritt nicht einmal halbiert. Für jede neue gültige Stelle des
Resultats werden 2 bis 3 Iterationen benötigt. also folgt mit dem zitierten Störungslemma aus dem Hinweis

Aufgabe 17.7 • 1
Für die Ableitung der Iterationsfunk- (U − I )−1  = Cf  (x 0 ) ≤ .
tion ergibt sich 1−τ

1
 (x) = − (4x + 3x 2 ). Aufgabe 17.10 ••• Schreiben wir
10
⎛ ⎞
Die Ableitung ist auf [1, 2] monoton fallend und es gilt g11 · · · g1m
⎜ ⎟
G = ⎝ ... . . . ... ⎠ ,
7
min | (x)| = | (1)| = , gm1 · · · gmm
1≤x≤2 10
max | (x)| = | (2)| = 2. dann ist
1≤x≤2
⎛ ⎞⎛ ⎞
Damit kann  auf [1, 2] keine Kontraktion sein. Die Ablei- g11 · · · g1m f1 (x)
⎜ . ⎟⎜ . ⎟
tung der Iterationsfunktion wird bei etwa x = 1.277 kleiner Gf (x) = ⎝ ... ..
. .. ⎠ ⎝ .. ⎠
als −1, aber in [1, 1.277] liegt keine Nullstelle von f . Schon gm1 · · · gmm fm (x)
die erste Iterierte x1 liegt außerhalb von [1, 1.277] und damit ⎛ m ⎞ ⎛ G ⎞
in dem Bereich, in dem  nicht kontrahierend ist. i=1 g1i fi (x) f1 (x)
⎜ . ⎟ ⎜ . ⎟
=⎝ . ⎠ =: ⎝ .. ⎠ .
m .
Aufgabe 17.8 •√ Das klassische Heron-Verfahren zur i=1 gmi fi (x) fmG (x)
Berechnung von 2 sucht die Berechnung der Kantenlänge
eines Quadrats der Fläche 2. Dazu setzte man Da die gij Konstante sind, folgt für die erste Fréchet-
Ableitung
a0 = 1, b0 = 2 (Gf (x)) = Gf  (x),
Lösungswege zu Kapitel 17 125


also also sind die Schnittpunkte bei x1,2 = ± 3/2 ≈
⎛ m m ⎞ ±1.224745
∂fi
···
i=1 g1,i ∂x1 (x)
∂fi
i=1 g1,i ∂xm (x)
√ und wir erhalten die zugehörigen y-Werte als
⎜ ⎟ y1,2 = ± 1/2 ≈ ±0.707107.
(Gf (x)) = ⎜

..
.
..
.
..
.
⎟.
⎠ b) Das zu lösende System lautet
m ∂fi m ∂fi    2 
i=1 gm,i ∂x1 (x) · · · i=1 gm,i ∂xm (x) f (x, y) x + y2 − 2
f (x) = 1 = = 0.
Die zweite Fréchet-Ableitung ist eine dreifach indizierte Ma- f2 (x, y) x2 − y2 − 1
trix Wir erhalten die Funktionalmatrix
(Gf (x)) = (aij ), i, j, k = 1, . . . , m
(k)
 ∂f ∂f   
1 1
 ∂x ∂y 2x 2y
mit f (x) = ∂f = ,
2 ∂f2 2x −2y
∂x ∂y
(k) ∂ 2 fkG
m
∂ 2 fμ
aij = = gkμ (x), i, j = 1, . . . , m. daraus die Inverse
∂xi ∂xj ∂xi ∂xj  
μ=1
1 1
 −1 1
Die Normabschätzung (17.42) lautet damit f (x) = x
1
x ,
4 − y1
  y
m m  m 
 2
∂ fμ  und damit das Newton-Verfahren
(Gf (x)) ∞ ≤ max  g (x) .
 kμ 
i=1 j =1 μ=1 
∂xi ∂xj
x n+1 =x n − (f (x n ))−1 f (x n )
k=1,...,m
1 1
 
n 1 xn xn xn2 + yn2 − 2
=x −
Rechenaufgaben 4 y1 − y1 xn2 − yn2 − 1
n n
   xn 3
 x 3

2 − 4xn 2 + 4xn
n
Aufgabe 17.11 •• Wir suchen F
x11 aus der Beziehung xn
= − yn 1 = yn 1 .
2 − 4yn 2 + 4y
yn
(x11 )2
F
x11 = x12 − ,
2 x10 Wählen wir x0 = y0 = 1, dann folgen die Vektoren
     
also 5/4 1.225 1.224745
x1 = , x2 = , x3 =
(x12 − x11 )2 3/4 0.708333 0.707108
x11 =x12 −
F = 1.368786102
x12 − 2x11 + x10
Aufgabe 17.14 ••• Löst man die beiden quadratischen
(−0.000071586)2 Gleichungen für y in den Zeilen des Systems, dann erhält
− = 1.368808107.
−0.000232877 man
D
Mit der 2 -Methode erhält man also bereits nach der 12. 3 9 4
Iteration die Ziffernfolge des Fibonacci. y = − x2 ± x + 1,
2 4
?
Aufgabe 17.12 •• Das Newton-Verfahren ist für n = 0, 1 − x4
y= .
1, . . . gegeben durch x

f (xn ) x 3 + 2x 2 + 10x − 20 Wählt man in der oberen Gleichung für y das positive Vorzei-
xn+1 = xn − 
= xn − . chen der Wurzel, dann ergibt sich im Rechteck (0, 2) ×(0, 1)
f (xn ) 3x 2 + 4x + 10
ein Schnittpunkt der beiden Graphen bei etwa
Startend mit x0 = 1 erhält man folgende Ergebnisse.
x 0 = (x0 , y0 ) = (0.98, 0.32).
n xn
1 1.4117647059 Für die Jacobi-Matrix
 
2 1.3693364706 6xy 3x 2 + 2y
f  (x) =
3 1.3688081886 4x 3 + y 2 2xy
4 1.3688081078
folgt an der Stelle x 0
5 1.3688081078  
1.8816 3.5212
Die Ziffernfolge des Fibonacci erhält man also bereits in der f  (x 0 ) = .
3.867168 0.6272
vierten Iteration.
Als Inverse ergibt sich
Aufgabe 17.13 ••  
−0.05043 0.283124
a) Aus der Gleichung der Hyperbel folgt y2 = x2 − 1 und G := (f  (x 0 ))−1 =
0.310942 −0.151291
Einsetzen in die Kreisgleichung liefert  
g g
=: 11 12
2x 2 − 1 = 2, g21 g22
126 Lösungswege zu Kapitel 17

und mit und

f (x 0 ) = (f1 (x0 , y0 ), f2 (x0 , y0 )) = (0.024384, 0.02272) ∂ 2 f1


min (x) = 6 · 0.27 = 1.62
x ∈R ∂x12
ergibt sich
  ∂ 2 f1
0.005203 min (x) = 6 · 0.93 = 5.58,
Gf (x 0 ) = . x ∈R ∂x1 ∂x2
0.004145
∂ 2 f1
min (x) = 2,
Damit können wir die erste geforderte Größe angeben, x ∈R ∂x 2
2
∂ 2 f2
Gf (x 0 )∞ = 0.005203 ≤ 0.0053. min (x) = 12 · (0.93)2 = 10.3788,
x ∈R ∂x12
Die zweiten partiellen Ableitungen von f sind ∂ 2 f1
min (x) = 2 · 0.27 = 0.54,
x ∈R ∂x1 ∂x2
∂ 2 f1 ∂ 2 f1 ∂ 2 f1
(x) = 6y, (x) = 6x, min (x) = 2 · 0.93 = 1.86.
∂x12 ∂x1 ∂x2 x ∈R ∂x 2
2
∂ 2 f1 ∂ 2 f2
(x) = 2, (x) = 12x 2 , Für k = 1 lautet der erste abzuschätzende Term
∂x22 ∂x12
 
∂ 2 f2 ∂ 2 f2  ∂ 2 f1 ∂ 2 f2 
(x) = 2y, (x) = 2x,  
∂x1 ∂x2 ∂x22 −0.05043 2 (x) + 0.283124 2 (x) ,
 ∂x ∂x 
1 1

und damit folgt aus der Abschätzung in Aufgabe 17.10


der maximal wird bei Verwendung des Minimums für
  ∂ 2 f1 ∂ 2 f2
2 2  2  (x) und des Maximums für (x),
 2
∂ fμ  ∂x12 ∂x12
(Gf (x)) ∞ ≤ max  g (x) 
 kμ   
i=1 j =1 μ=1 
k=1,2 ∂xi ∂xj
 ∂ 2 f1 ∂ 2 f2 
#   
 ∂ 2f  −0.05043 2 (x) + 0.283124 2 (x)
 1 ∂ 2 f2   ∂x ∂x 
= max gk1 2 (x) + gk2 2 (x) 1 1
k=1,2  ∂x1 ∂x1 
  ≤ |−0.05043 · 1.62 + 0.283124 · 12| = 3.3158.
 ∂ 2 f1 ∂ 2 f2 
 
+2 gk1 (x) + gk2 (x)
 ∂x1 ∂x2 ∂x1 ∂x2 
 :
 ∂ 2f ∂ 2 f2  Aufgabe 17.15 ••• Wegen zn = xn + iyn ist zn2 =
 1 
+ gk1 2 (x) + gk2 2 (x) , xn2 − yn2 + 2ixn yn . Dann folgt
 ∂x ∂x 
2 2
1 1 x 2 − yn2 − 2ixn yn
wobei für die gemischten Ableitungen der Satz von Schwarz = 2 = 2n .
zur Anwendung gekommen ist. Da die Abschätzung im
2
zn xn − yn + 2ixn yn
2 (xx − yn2 )2 + 4xn2 yn2
Rechteck R := [0.93, 1] × [0.27, 0.34] gesucht ist, berech-
nen wir die Minima und Maxima der zweiten Ableitungen Damit wird aus der Iteration
von f ,
xn+1 + iyn+1
 
∂ 2f
1 1 xn2 − yn2 − 2ixn yn
max (x) = 6 · 0.34 = 2.04 = xn + iyn − xn + iyn − 2
x ∈R ∂x 2 3 (xx − yn2 )2 + 4xn2 yn2
1
∂ 2 f1 2 xn2 − yn2
max (x) = 6 · 1 = 6, = xn +
x ∈R ∂x1 ∂x2 3 3(xn2 − yn2 )2 + 12xn2 yn2
∂ 2 f1  
max (x) = 2, 2 2xn yn
x ∈R ∂x 2
+i yn − ,
2 3 3(xn2 − yn2 )2 + 12xn2 yn2
∂ 2 f2
max (x) = 12 · (1)2 = 12, also die beiden reellen Iterationen
x ∈R ∂x12
∂ 2 f1 2 xn2 − yn2
max (x) = 2 · 0.34 = 0.68, xn+1 = xn + ,
x ∈R ∂x1 ∂x2 3 3(xn − yn2 )2 + 12xn2 yn2
2

∂ 2 f1 2 2xn yn
max (x) = 2 · 1 = 2, yn+1 = yn − .
x ∈R ∂x 2
2 3 3(xn − yn2 )2 + 12xn2 yn2
2
Lösungswege zu Kapitel 17 127

Die Inhalte der drei Dateien sind in Abbildung 17.1 in drei un-
terschiedlichen Färbungen zu sehen. Offenbar ist die Struktur
der Mengen der drei Attraktoren z(k) , k = 1, 2, 3, außer-
ordentlich kompliziert. Man kann zeigen, dass diese Men-
gen fraktale Dimension besitzen, d. h., es sind keine zwei-
dimensionalen Mengen, sondern ihre Dimension (die soge-
nannte Hausdorff-Dimension) liegt zwischen 1 und 2. Die
Mengen heißen Julia-Mengen nach Gaston Maurice Julia
(1893–1978). Das Studium solcher Mengen ist die Aufgabe
der Komplexen Dynamik, aber jeder Anwender von Ite-
rationsverfahren bei nichtlinearen Gleichungen oder Glei-
chungssystemen sollte wissen, dass häufig eine sehr kom-
plizierte Dynamik hinter dem Konvergenzverhalten des ge-
wählten Verfahrens steht.

Abbildung 17.1 Die unterschiedlich gefärbten Mengen von Attraktoren: z(1)


(rot), z(2) (grün) und z(3) (blau).

Ein Pseudocode lautet wie folgt.


d := 4/800;
for (i = 0; i < 801; i + +)
for (j = 0; j < 801; j + +)
u = −2 + i · d; v = −2 + j · d;
x := u; y := v;
for (k = 0; k ≤ 2000; k + +)
xneu := 23 x + 3(x 2 −yx2 )−y
2 2
2 +12x 2 y 2 ;

yneu := 23 y − 3(x 2 −y 22xy


)2 +12x 2 y 2
;
u := xneu ; v := yneu ;
endfor(k);
if (u · v = 0) schreibe x, y in Datei 1;
if (u · v < 0) schreibe x, y in Datei 2;
if (u · v > 0) schreibe x, y in Datei 3;
endfor(j );
endfor(i);
Kapitel 18 Aufgabe 18.7 ••• Zeigen Sie, dass für eine nicht-
negative und irreduzible Matrix A ∈ Rn×n die Eigenschaft

Aufgaben (I + A)n−1 > 0.

gilt, wobei I die Einheitsmatrix ist.


Verständnisfragen
Aufgabe 18.1 • Gegeben sei das Butcher-Array eines Aufgabe 18.8 •• Sei A ∈ Rn×n nicht-negativ und
Runge-Kutta-Verfahrens in der Form 0 ≤ x ∈ R  nnicht der Nullvektor. Zeigen Sie, rx :=
n

γ γ 0 j =1 akj xj
mink∈{1,...,n} xk ist nicht-negativ und das Supre-
1−γ 1 −γ mit xk >0
1/4 3/4 mum aller ξ ≥ 0 für die Ax ≥ ξ x gilt.

Begründen Sie, für welche Parameter γ ∈ R es sich um


Aufgabe 18.9 ••• Sei A ∈ Rn×n nicht-negativ und irre-
ein explizites, respektive implizites Verfahren handelt. Für
duzibel sowie r := supx ≥0 {rx } mit rx aus Aufgabe 18.8 und
welche Parameter γ ∈ R besitzt das Verfahren die Konsis- x  =0
2 3
tenzordnung p = 1 respektive p = 2? Qn := (I + A)n−1 x ∈ Rn | x ≥ 0 und x = 1 .
Zeigen Sie, dass
Aufgabe 18.2 •• Zeigen Sie, dass das Einschrittverfah-
ren
2 − t r = sup {ry }
yi+1 = yi y ∈Q n
2 + t
konsistent genau von der Ordnung p = 2 zur Differenzial- gilt.
gleichung y  (t) = −y(t) ist.

Aufgabe 18.3 • Zeigen Sie, dass außer der impliziten Aufgabe 18.10 ••• Sei A ∈ Rn×n nicht-negativ und ir-
Mittelpunktsregel kein einstufiges Runge-Kutta-Verfahren reduzibel sowie r (aus Aufgabe 18.8) und die Menge der
der Ordnung p = 2 existiert. Extremalvektoren der Matrix A durch {z ∈ Rn≥0 \ {0} | Az ≥
rz ∧ w ∈ Rn≥0 : Aw > rw} gegeben. Zeigen Sie: z ist ein
positiver Eigenvektor der Matrix A zum Eigenwert r > 0.
Beweisaufgaben D. h. Az = rz und z > 0.
Aufgabe 18.4 • Weisen Sie ohne Verwendung des
Satzes zur maximalen Ordnung expliziter Runge-Kutta- Aufgabe 18.11 ••• (Satz von Perron-Frobenius)
Verfahren folgende Aussage nach: Jedes explizite dreistufige Zeigen Sie, dass zu jeder nicht-negativen Matrix A ∈ Rn×n
Runge-Kutta-Verfahren dritter Ordnung besitzt eine Stabili- ein nicht-negativer Eigenwert λ = ρ(A) mit zugehörigem
tätsfunktion der Form nicht-negativen Eigenvektor x ≥ 0 existiert.
ξ2 ξ3
R(ξ ) = 1 + ξ + + .
2 6 Aufgabe 18.12 • Weisen Sie die Konservativität des
auf Seite 693 vorgestellten modifizierten Patankar-Runge-
Aufgabe 18.5 •• Weisen Sie nach, dass das implizite Kutta-Verfahrens nach.
Einschrittverfahren
 
1 1 Aufgabe 18.13 •• Füreine gegebene Matrix A ∈ Cn×n
yi+1 = yi + t f ti + t, (yi+1 + yi )
2 2 sei die Folge der Matrizen Ak k∈N beschränkt. Zeigen Sie,
mit dem Startwert y0 = Fy0 die exakte Lösung des Anfangs- dass dann der Spektralradius der Bedingung ρ(A) ≤ 1 ge-
wertproblems y  (t) = −2at, y(t0 ) = F
y0 für ti = t0 + it nügt und für jeden Eigenwert λ ∈ C von A mit |λ| = 1 alge-
liefert. braische und geometrische Vielfachheit übereinstimmen.

Aufgabe 18.6 •• Weisen Sie ohne Verwendung des Sat-


zes zur Konsistenz von Runge-Kutta-Verfahren nach, dass die Rechenaufgaben
explizite Mittelpunktsregel
t Aufgabe 18.14 •• Berechnen Sie das Stabilitätsgebiet
yi+1/2 = yi + f (ti , y(ti )) des expliziten Euler-Verfahrens und stellen Sie es graphisch
2
  dar.
t
yi+1 = yi + tf ti + , yi+1/2
2 Aufgabe 18.15 •• Bestimmen Sie die Konsistenzord-
die Konvergenzordnung p = 2 besitzt, falls f hinreichend nung des auf Seite 667 vorgestellten klassischen Runge-
glatt ist. Kutta-Verfahrens.
M. Brokate et al., Arbeitsbuch Grundwissen Mathematikstudium – Höhere Analysis, Numerik und
Stochastik, DOI 10.1007/978-3-642-54946-5_17, © Springer-Verlag Berlin Heidelberg 2016
Hinweise zu Kapitel 18 129

Aufgabe 18.16 •• Bestimmen Sie die Konsistenzord- Hinweise


nung des zweistufiges SDIRK-Verfahrens mit dem Butcher-
Array
Verständnisfragen
γ γ 0 √
3± 3 Aufgabe 18.1 • Betrachten Sie die Gestalt der Matrix
1 − γ 1 − 2γ γ für γ = .
6 im Butcher-Array und nutzen Sie den Satz zur Konsistenz
1 1
2 2 von Runge-Kutta-Verfahren.

Aufgabe 18.17 •• Bestimmen Sie den Stabilitätsbe- Aufgabe 18.2 •• Betrachten Sie die Verfahrensfunk-
reich des Verfahrens tion und führen Sie eine Taylor-Entwicklung durch.

yi+1 = yi + t (μf (ti , yi ) + (1 − μ)f (ti+1 , yi+1 )) Aufgabe 18.3 • –


für μ ∈ [0, 1]. Für welche Werte von μ ist das Verfahren
A-stabil? Beweisaufgaben

Aufgabe 18.18 • Wie müssen die freien Koeffizienten Aufgabe 18.4 • Nutzen Sie den Satz zur Konsistenz
des Runge-Kutta-Verfahrens von Runge-Kutta-Verfahren.

0 0 0 0 Aufgabe 18.5 •• Führen Sie eine vollständige Induk-


c2 c2 0 0 tion unter Berücksichtigung der exakten Lösung y(t) =
c3 0 c3 0 F
y0 − a(t 2 − t02 ) durch.
0 0 1
Aufgabe 18.6 •• Betrachten Sie die Vorgehensweise im
gewählt werden, damit das Verfahren die Ordnung p = 2 Beweis auf Seite 666 und nutzen Sie den Zusammenhang
besitzt. Kann das Verfahren die Konsistenzordnung p = 3 zwischen Konsistenz und Konvergenz bei Einschrittverfah-
erreichen? ren.
Aufgabe 18.19 • Bestimmen Sie die Konsistenzord- Aufgabe 18.7 ••• Es ist hinreichend (I + A)n−1 x > 0
nung des expliziten Runge-Kutta-Verfahrens für beliebige nicht-negative x ∈ Rn \ {0} zu zeigen. Betrach-
 
t t ten Sie dabei die durch die Iteration x k+1 := (I + A)x k mit
yi+1 = yi + tf ti + , yi + f (ti , yi ) nicht-negativem x 0 ∈ Rn \ {0} definierte Vektorfolge.
k k
in Abhängigkeit von k ∈ N. Aufgabe 18.8 •• –
Aufgabe 18.20 • Bestimmen Sie die Konsistenzord- Aufgabe 18.9 ••• rx = rα x für alle α > 0. Betrachten
nung des linearen Mehrschrittverfahrens Sie für „r ≤ supy ∈Qn {ry }“ die Aussage von Aufgabe 18.7
 
mit x = 1 und Aufgabe 18.8.
yi+2 + 4yi+1 − 5yi = t 4f (ti+1 , yi+1 ) + 2f (ti , yi ) .
Aufgabe 18.10 ••• –
Aufgabe 18.21 •• Zeigen Sie, dass die Vektoren
(1, ξk , ξk2 , . . . , ξkm− )T ∈ Cm , k = 1, . . . , m linear unab- Aufgabe 18.11 ••• Zeigen Sie die Aussage zunächst für
hängig sind, wenn die Größen ξ1 , . . . , ξm ∈ C paarweise irreduzible, nicht-negative Matrizen. Wegen Aufgabe 18.10
verschieden sind. reicht es, r ≥ λ für alle λ ∈ σ (A) zu zeigen. Nutzen Sie
dazu die Aussagen der vorherigen Aufgaben. Beweisen Sie
Aufgabe  18.22 •• Sei ξ eine Nullstelle des Polynoms anschließend die Aussage für reduzible Matrizen, indem Sie
p(ξ ) = m
j =0 αj ξ mit αm  = 0. Dann gilt für die Lö-
j die Nulleinträge von A durch ε > 0 ersetzen und ε → 0
sungsfolge (yn )n∈N0 der homogenen Differenzengleichung betrachten.
m
j =0 α j y i+m = 0 bei den Anfangswerten yi = ξ i , i =
0, . . . , m − 1 die Darstellung yn = ξ n für alle n ∈ N0 . Aufgabe 18.12 • –

Aufgabe 18.23 • Berechnen Sie das charakteristische Aufgabe 18.13 •• –


Polynom zur Matrix
⎛ ⎞ Rechenaufgaben
0 1
⎜ .. .. ⎟ ••
⎜ . . ⎟ Aufgabe 18.14 Betrachten Sie das Beispiel auf
⎜ ⎟
A=⎜ ⎜ . .
.. ..
⎟ ∈ Rm×m .

Seite 672.
⎜ ⎟
⎝ 0 1 ⎠ Aufgabe 18.15 •• Nutzen Sie die Box zu Butcher-
−a0 ... −am−1 Bäumen auf Seite 668.
130 Lösungswege zu Kapitel 18

Aufgabe 18.16 •• Verwenden Sie den auf Seite 666 Aufgabe 18.13 •• –
aufgeführten Satz zur Konsistenz von Runge-Kutta-
Verfahren.
Rechenaufgaben
Aufgabe 18.17 •• Man stelle die Stabilitätsfunktion R
auf und bestimme die Menge der Werte ξ ∈ C− , für die Aufgabe 18.14 •• S = {ξ ∈ C | |1 + ξ | < 1}
|R(ξ )| < 1 gilt.
Aufgabe 18.15 •• Die Konsistenzordnung ist p = 4.
Aufgabe 18.18 • Nutzen Sie den auf Seite 666 aufge-
führten Satz zur Konsistenz von Runge-Kutta-Verfahren. Aufgabe 18.16 •• Es handelt sich um ein Verfahren
dritter Ordnung.
Aufgabe 18.19 • –
Aufgabe 18.17 •• –
Aufgabe 18.20 • Nutzen Sie den Satz zur Konsistenz
linearer Mehrschrittverfahren auf Seite 675. Aufgabe 18.18 • c2 beliebig und c3 = 21 .
Aufgabe 18.21 •• Betrachten Sie das Polynom
Aufgabe 18.19 • –
p : C → C, p(ξ ) = m−1 j
j =0 αj ξ .

Aufgabe 18.22 •• – Aufgabe 18.20 • Das Verfahren besitzt genau die Kon-
sistenzordnung p = 3.
Aufgabe 18.23 • Führen Sie eine sukzessive Entwick-
lung nach der jeweils letzten Spalte durch. Aufgabe 18.21 •• –

Aufgabe 18.22 •• –

Lösungen Aufgabe 18.23 • –

Verständnisfragen
Aufgabe 18.1 • Das Verfahren ist genau dann explizit,
wenn γ = 0 gilt. Konsistenzordnung p = 1 liegt für alle γ Lösungswege
vor. Konsistenzordnung p = 2 gilt ausschließlich für γ = 0.5.
Verständnisfragen
Aufgabe 18.2 •• –
Aufgabe 18.1 • Es handelt sich genau dann um ein ex-
Aufgabe 18.3 • – plizites Verfahren, wenn innerhalb des Butcher-Arrays eine
strikte linke untere Dreiecksmatrix vorliegt. In der betrach-
teten Aufgabenstellung ist dies genau dann der Fall, wenn
Beweisaufgaben γ = 0 gilt. Die Bedingungen für die Konsistenzordnung
Aufgabe 18.4 • – p = 1 sind für alle γ ∈ R erfüllt und es liegt genau dann ein
Verfahren der Ordnung p = 2 vor, wenn
Aufgabe 18.5 •• –
1
2
1 3
= bj cj = γ + (1 − γ )
Aufgabe 18.6 •• – 2 4 4
j =1

Aufgabe 18.7 ••• – 1


gilt, wodurch γ = 2 folgt.
Aufgabe 18.8 •• –
Aufgabe 18.2 •• Das Verfahren schreibt sich in der
Aufgabe 18.9 ••• – Form
−2
yi+1 = yi + t yi .
Aufgabe 18.10 ••• – 2 + t
Mit der Taylor-Entwicklung
Aufgabe 18.11 ••• –
−2 1 1
= −1 + t − t 2 + O(t 3 )
Aufgabe 18.12 • – 2 + t 2 12
Lösungswege zu Kapitel 18 131

erhalten wir unter Berücksichtigung der speziellen Differen- y(ti ) = F


y0 − a(ti2 − t02 ) für ein beliebiges, aber festes i ∈ N0 ,
zialgleichung dann folgt der Induktionsschritt durch
   
1 1 1 1
yi+1 = yi + t −1 + t − t 2 yi + O(t 4 ) yi+1 = yi + t f ti + t, (yi+1 + yi )
2 12 2 2
1
t 2 t 3 =Fy0 − a(ti2 − t02 ) − 2t a(ti + t)
= y(ti ) − ty(ti ) + y(ti ) − y(ti ) + O(t 4 ) 2
2 12
=Fy0 − a(ti2 − t02 + 2t ti + t 2 )
t 2  t 3 
= y(ti )+ty  (ti )+ y (ti )+ y (ti )+ O(t 4 ) y0 − a((ti + t)2 − t02 )
=F
2 12
t 3  =F 2
y0 − a((ti+1 − t02 ) = y(ti+1 ).
= y(ti+1 )− y (ti )+ O(t 4 ) = y(ti+1 )+ O(t 3 ).
12

Da die Lösung der Differenzialgleichung als Exponential- Aufgabe 18.6 •• Nutzen wir eine Taylorentwicklung
funktion keine verschwindende dritte Ableitung aufweist, für die Lösung der zugrunde liegenden Differenzialgleichung
liegt eine Methode genau dritter Ordnung vor.
y(ti + t)

Aufgabe 18.3 • t 2 
Das Butcher-Array eines einstufigen = y(ti ) + ty  (ti ) + y (ti ) + O(t 3 )
Runge-Kutta-Verfahrens lautet 2
t 2 df
= y(ti ) + tf (ti , y(ti )) + (ti , y(ti )) + O(t 3 )
c1 a11 2 dt
b1 t 2  
= y(ti ) + tf (ti , y(ti )) + ft + fy · f (ti , y(ti ))
2
Um ein Verfahren mindestens erster Ordnung vorliegen zu + O(t 3 )
haben, müssen die Bedingungen c1 = a11 und b1 = 1 erfüllt
sein. Die Bedingung für die Konsistenzordnung p = 2 lautet und ebenso für die Näherungslösung
somit 21 = b1 c1 = c1 , sodass alle Koeffizienten eindeutig
yi+1
bestimmt sind und die implizite Mittelpunktsregel vorliegt.  
Mit 13 = b1 c12 ist diese Verfahren zudem genau von zweiter t t
= yi + tf ti + , yi + f (ti , yi )
Ordnung. 2 2
 t
= yi + t f (ti , yi ) + ft (ti , yi )
2

Beweisaufgaben t
+ fy (ti , yi ) f (ti , yi ) + O(t 2 )
2
Aufgabe 18.4 • Mit
t 2  
⎛ ⎞ ⎛ ⎞ = yi + tf (ti , yi ) + ft + fy · f (ti , yi ) + O(t 3 ),
0 0 0 0 0 0 2
A = ⎝a21 0 0⎠ und A2 = ⎝ 0 0 0⎠ so folgt für die Konsistenzbetrachtung mit der Voraussetzung
a31 a32 0 a21 a32 0 0 yi = y(ti ) durch einfache Subtraktion der beiden obigen
Darstellungen
folgt
y(ti + t) = O(t 3 ).
R(ξ ) = 1 + ξ bT (I + ξ A + ξ 2 A2 )e Hiermit liegt die Konsistenzordnung und mit dem Satz zur
2
= 1 + ξ(b1 + b2 + b3 ) + ξ (b2 a21 + b3 (a31 + a32 )) Konvergenz von Einschrittverfahren auch die Konvergenz-
ordnung p = 2 vor.
+ ξ 3 b3 a32 a21 .
Aufgabe 18.7 ••• Dazu definiere die Folge
Berücksichtigen wir die Ordnungsbedingung gemäß des
oben erwähnten Satzes, so ergibt sich b1 + b2 + b3 = 1, x k+1 := (I + A)x k
b2 a21 + b3 (a31 + a32 ) = b2 c2 + b3 c3 = 1/2 sowie
b3 a32 a21 = 1/6 und folglich mit x 0 := x. Im Folgenden wird gezeigt, dass, sofern x k noch
Einträge gleich null besitzt, die Anzahl der Nulleinträge von
ξ2 ξ3 x k+1 kleiner ist. Wegen
R(ξ ) = 1 + ξ + + .
2 6 x k+1 := x k + 
A xk
≥0

Aufgabe 18.5 •• Der Induktionsanfang ist durch die kann sich die Anzahl der positiven Einträge offensichtlich
Anfangsbedingung y0 = F
y0 = y(t0 ) gegeben. Gelte yi = nicht reduzieren.
132 Lösungswege zu Kapitel 18

Angenommen, x k und x k+1 hätten die gleiche Anzahl m Aufgabe 18.10 ••• Sei e = (1, . . . , 1)T , so ist, da A ≥ 0
an Nulleinträgen, mit 1 ≤ m < n. Sei P ∈ Rn×n eine und irreduzibel, Ae > 0. Folglich ist
Permutationsmatrix mit ⎧ ⎫
 +   + ⎪
⎨ n ⎪
⎬ n
x k+1 xk j =1 aij xj j =1 aij ej
P x k+1 = , P xk = r = sup min ≥ min
0 0 ⎪
x ≥0 ⎩ i∈{1,...,n} xi ⎪
⎭ i∈{1,...,n} ei
x  =0 mit xi >0

mit x+ +
k+1 , x k ∈ Rm
>0 := {x ∈ R |x ≥ 0}. Betrachte nun
m
n
= min aij > 0.
i∈{1,...,n}
T j =1
P x k+1 = P x k + P AP P x k
Da Qn kompakt und rz stetig auf Qn ist, existiert ein z ∈ Qn
respektive die äquivalente Formulierung
mit rz = r. Würde ein w ∈ Rn≥0 mit Aw > rw existie-
 +   +    + ren, so wäre nach Aufgabe 18.8 rw > r, da Aw ≥ rw w
x k+1 xk A11 A12 xk
= + mit Gleichheit in mindestens einer Komponente gälte, was
0 0 A21 A22 0
der Supremumseigenschaft von r widerspräche. Somit ist z
mit A11 ∈ R(n−m)×(n−m) , A22 ∈ Rm×m und A12 , AT21 ∈ positiver Extremalvektor.
R(n−m)×m . Wegen x + k > 0 folgt A21 = 0, was der Irre- Sei nun z ∈ Qn ein Extremalvektor mit Az − rz = η ≥ 0
duzibilität von A widerspricht. Folglich hat x k+1 weniger und η = 0. Multiplikation von links mit (I + A)n−1 > 0
Nulleinträge als x k . Da x maximal n − 1 Nulleinträge besit- (Aufgabe 18.7) liefert
zen kann und sich die Anzahl der Nulleinträge mindestens
um einen pro Produkt verringert, ist spätestens A (I + A)n−1 z −r (I + A)n−1 z = (I + A)n−1 η > 0
     
=w =w
x n−1 = (I + A)n−1 x
und somit
ein positiver Vektor.
Aw − rw > 0,
Aufgabe 18.8 •• Wegen A ≥ 0 und x ≥ 0 ist rx ≥ 0
also Aw > rw, was ein Widerspruch dazu darstellt, dass z
klar. Betrachte nun
⎛ ⎞ ⎛ ⎞ Extremalvektor ist. Folglich ist η = 0. Somit ist wegen
n n
aij xj
(Ax)i = ⎝ aij xj ⎠ = ⎝ xi ⎠ Az = rz
xi
j =1 i j =1 i
⎛ ⎧ ⎫ ⎞ z ein positiver Eigenvektor zum Eigenwert r.
⎨ a x ⎬
n
⎜ kj j ⎟
≥ ⎝ min xi ⎠ = (rx x)i = rx xi . Aufgabe 18.11 ••• Falls A irreduzibel ist, sei λ ein Ei-
k∈{1,...,n} ⎩ xk ⎭
mit xk >0 j =1 genwert von A mit zugehörigem Eigenvektor y. Dann gilt
i

A ||y| = A|y|,
|λ||y| = |λy| = |Ay| ≤ | 
≥0
Offensichtlich gilt (Ax)k = rx xk und wegen xk > 0 folglich
(Ax)k < ξ xk für alle ξ > rx . womit wegen der Aufgaben 18.8, 18.9 und 18.10

Aufgabe 18.9 ••• „≥“: Nach Definition gilt r = |λ| ≤ r|y | ≤ r


supx ∈Rn \{0} {rx }. Wegen Qn ⊂ Rn≥0 \ {0} folgt direkt, dass
≥0
folgt, also r = ρ(A). Zusammen mit der Aussage von Auf-
sup {ry } ≤ sup {rx } . gabe 18.4 folgt die Behauptung.
y∈Qn x ∈Rn≥0 \{0}
Falls A reduzibel ist, betrachte die Matrix
„≤“: Sei x ∈ Rn≥0 \ {0} o. B. d. A. x = 1. Betrachte ⎛ ⎞
R 11 R 12 · · · R 1m
⎜ 0 R 22 · · · R 2m ⎟
Ax ≥ rx x. ⎜ ⎟
P AP T = ⎜ . .. .. .. ⎟
⎝ .. . . . ⎠
Multiplikation von links mit (I + A)n−1 > 0 (Aufgabe 18.7) 0 · · · 0 R mm
liefert
wobei P ∈ Rn×n eine Permutationsmatrix ist und R ii qua-
n−1 n−1
A (I + A) x . ≥ rx (I + A) x. dratische irreduzible Matrizen, nicht notwendigerweise glei-
     
=:y =:y cher Größe, für i = 1, . . . , m sind. Da A ≥ 0 ist, gilt
offensichtlich auch R ii ≥ 0. Sofern nicht alle R ii 1 × 1
Nach Definition ist y ∈ Qn . Wegen Aufgabe 18.8 ist rx ≤ ry . Matrizen mit R ii = 0 sind (und somit ρ(A) = 0) sei
Lösungswege zu Kapitel 18 133

R jj = 0. Mit der Aussage zu irreduziblen, nicht-negativen und wir erkennen, dass die erste Komponente des Vektors
Matrizen folgt, dass R jj einen positiven Eigenwert gleich des J ki e2 betragsmäßig unbeschränkt mit k anwächst. Folglich
Spektralradius ρ(R jj ) besitzt. ρ(A) ≥ 0 folgt dann wegen gilt
σ (A) = ∪m i=1 σ (R ii ).
Zum Nachweis eines nicht-negativen Eigenvektors zu ρ(A) Ak ∞ = J k ∞ ≥ J ki ∞ → ∞, k → ∞
betrachte Aε := A + ε mit ε > 0. Somit ist Aε offen-
sichtlich irreduzibel und positiv. Mit der Aussage zu irre- und es liegt wiederum ein Widerspruch zur Potenz-
duziblen, nicht-negativen Matrizen existiert also ein Eigen- beschränktheit von A vor.
wert λε = ρ(Aε ) mit zugehörigem Eigenvektor x ε > 0 mit
||x ε || = 1. Da die Eigenwerte als Polynomnullstellen stetig
von ε abhängen und die Menge {x | ||x|| = 1} kompakt ist, Rechenaufgaben
findet man eine konvergente Teilfolge x εν → x mit x ≥ 0,
sodass Aufgabe 18.14 •• Graphisch erhalten wir die folgende
Abbildung:
0 = lim Aεν x εν − λεν x εν = Ax − λx
ν→∞

gilt. Folglich ist x ≥ 0 Eigenvektor zu ρ(A).


1.5

Aufgabe 18.12 • Die Eigenschaft folgt unmittelbar 0.5

aus

Im( )
0


N
(yi,n+1 − yi,n ) −0.5

i=1 −1

t 0
N
yj,n+1 −1.5
= (pij (y n ) + pij (y (1) )) (1) −2.5 −2 −1.5 −1
Re( )
−0.5 0 0.5

2 y
i,j =1 j
Abbildung 18.1 Stabilitätsgebiet des expliziten Euler-Verfahrens.
yi,n+1 1
− (dij (y n ) + dij (y (1) )) (1) = 0.
   y
i
pj i (y n )+pj i (y (1) ) Aufgabe 18.15 •• Da es sich beim klassischen Runge-
Kutta-Verfahren um eine explizite Methode handelt und
durch elementares Nachrechnen leicht gezeigt werden kann,
Aufgabe 18.13 •• Angenommen, es gilt ρ(A) > 1,
dass die auf Seite 668 aufgeführten Bedingungen erfüllt sind,
dann ergibt sich
handelt es sich um eine Methode 4. Ordnung.
Ak ∞ ≥ ρ(Ak ) ≥ ρ(A)k → ∞, k → ∞,
Aufgabe 18.16 •• Einfaches Nachrechnen der Bedin-
womit ein Widerspruch zur Potenzbeschränktheit der Matrix
gungen aus dem oben erwähnten Satz zeigt die minimale
A vorliegt.
Konsistenzordnung. Nutzen wir zudem die auf Seite 668 auf-
Sei ρ(A) = 1, wobei ein Eigenwert λi mit |λi | = 1 geführten Bedingungen, so wird deutlich, dass die Konsis-
existiert, bei dem algebraische und geometrische Vielfach- tenzordnung genau p = 3 ist.
heit nicht übereinstimmen. Dann ergibt sich bei der Jordan-
Normalform J = T −1 AT das zugehörige Jordan-Kästchen
Aufgabe 18.17 •• Ausgehend von der Testgleichung
die Form
y(t) = λy(t) ergibt sich für das Verfahren yi+1 =
⎛ ⎞ 1+tλμ
λi 1 1+tλ(μ−1) yi und folglich die Stabilitätsfunktion
⎜ .. .. ⎟
⎜ . . ⎟
Ji = ⎜ ⎜ ⎟ ∈ Rni ×ni mit ni ≥ 2 1 + ξμ
. ⎟
⎝ .. 1 ⎠ R(ξ ) = .
1 + ξ(μ − 1)
λi
Mit ξ = a + ib, a, b ∈ R ergibt sich
auf. Mit e2 = (0, 1, 0, . . . , 0)T gilt
⎛ k−1 ⎞ |R(ξ )| < 1 ↔ 2a < (1 − 2μ)(a 2 + b2 ). (18.10)
kλi
⎜ λk ⎟
⎜ i ⎟
⎜ ⎟ Wir betrachten eine Fallunterscheidung bezüglich μ. Für
J ki e2 = ⎜ 0 ⎟
⎜ . ⎟
⎝ .. ⎠ μ = 21 ergibt sich die Bedingung 2a < 0. Das Stabilitäts-
0 gebiete ist somit C− und die Methode ist A-stabil.
134 Lösungswege zu Kapitel 18

1
μ> 2 folgen aus (18.10) die äquivalenten Ungleichungen Aufgabe 18.20 • Die Koeffizienten bezogen auf die
allgemeine Darstellung eines linearen Mehrschrittverfahrens
2a lauten
a2 − + b2 < 0 und
1 − 2μ
 2 α0 = −5, α1 = 4, α2 = 1, β0 = 2, β1 = 4, β2 = 0.
1 1
a− + b2 > .
1 − 2μ (1 − 2μ)2 Einfaches Nachrechnen der Ordnungsbedingungen im Satz
zur Konsistenz linearer Mehrschrittverfahren liefert
Als Stabilitätsgebiet erhalten wir alle Punkte in C− , die au-
ßerhalb des abgeschlossenen Kreises um (1/(1−2μ), 0)T
2

mit Radius 1/(1 − 2μ) liegen. Das Verfahren ist somit ge- q=0: αj = 0,
nau dann A-stabil, wenn μ = 0 gilt und wir das implizite j =0
Euler-Verfahren vorliegen haben.
2
2
μ < 21 erhalten wir analog zur obigen Herleitung als Sta- q=1: αj j 1 = 6 = 1 · βj j 0 ,
bilitätsgebiet alle Punkte im offenen Kreis um (1/(1 − j =0 j =0
2μ), 0)T mit Radius 1/(1 − 2μ). Keine der Methoden ist
2
2
A-stabil. q=2: αj j 2 = 8 = 2 · βj j 1 ,
j =0 j =0

Aufgabe 18.18 • Zunächst  sind die Bedingungen


2
2
s s q=3: αj j 3 = 12 = 3 · βj j 2 ,
ν=1 a j ν = cj für j = 1, 2, 3 und j =1 bj = 1 offensicht-
j =0 j =0
lich erfüllt und es liegt folglich ein Verfahren mindestens der
Ordnung p = 1 vor. Aus der Bedingung sj =1 bj cj = 21
2
2
folgt aufgrund der vorliegenden Gewichte b1 , b2 , b3 direkt q=4: αj j 4 = 20 = 16 = 4 · βj j 3 ,
 j =0 j =0
c3 = 21 bei beliebigem c2 . Wegen sj =1 bj cj2 = 41  = 13
liegt damit ein Verfahren genau zweiter Ordnung vor und
eine Konsistenzordnung p = 3 kann nicht erreicht werden. womit genau die Konsistenzordnung p = 3 vorliegt.

Aufgabe 18.19 • Das zugehörige Butcher-Array hat Aufgabe 18.21 •• Sei (α0 , . . . , αm−1 )T Lösung des
die Form Gleichungssystem
0 0 0 ⎛ ⎞⎛ ⎞ ⎛ ⎞
1 ξ1 . . . ξ1m−1 α0 0
1 1
0 . ⎜. . . ⎟ ⎜ .. ⎟ ⎜ .. ⎟
k k ⎝ .. .. .. ⎠ ⎝ . ⎠ = ⎝ . ⎠ ,
0 1 1 ξm . . . ξmm−1 αm−1 0

Es gilt somit so folgt für das im Hinweis angegebene Polynom p direkt


p(ξk ) = 0 für k = 1, . . . , m. Wegen p ∈ m−1 ergibt sich

2
2 aufgrund der Verschiedenheit der Nullstellen p ≡ 0 und für
bj = 1 und aj ν = cj für j = 0, 1 , die Koeffizienten somit die Aussage α0 = . . . = αm−1 = 0.
j =1 ν=1 Somit ist die obige Matrix regulär und die Zeilenvektoren
damit linear unabhängig.
womit das Verfahren für alle k ∈ N mindestens von erster
Ordnung genau ist. Des Weiteren ergibt sich
Aufgabe 18.22 •• Formal ist eine vollständige Induk-
tion durchzuführen,
 die auf der folgenden Idee beruht. Aus

2
1 1 0 = ξi m j = m α ξ i+j folgt mit ξ i+m =
bj cj = 0 · 0 + 1 · = . j =0 αj ξ j =0 j

j =1
k k − α1m m−1
j =0 jα ξ i+j bei gegebenen m Daten

Folglich ist das Verfahren für k ∈ N\{2} genau erster Ord- yi = ξ i , yi+1 = ξ i+1 , . . . , yi+m−1 = ξ i+m−1
nung und für k = 2 mindestens zweiter Ordnung. Abschlie-
ßend erhalten wir für k = 2 aus die Schlussfolgerung

1
m−1

2
1 1 1
bj cj2 = 2 = = , yi+m = − αj yi+j
k 4 3 αm
j =1 j =0

1
m−1
die Aussage, dass das Verfahren für k = 2 genau zweiter =− αj ξ i+j = ξ i+m .
αm
Ordnung ist. j =0
Lösungswege zu Kapitel 18 135

Aufgabe 18.23 • Der Nachweis ergibt sich aus einer Sukzessive Anwendung liefert
einfachen Entwicklung  
⎛ ⎞ m m−1 2 λ −1
λ −1 p(λ) = −λ − λ am−1 − . . . − λ a2 − det
a0 a1
⎜ .. .. ⎟
⎜ . . ⎟ = −λm − λm−1 am−1 − . . . − λ2 a2 − λa1 − a0 .
⎜ ⎟
p(λ) = − det ⎜
⎜ . .. . ..


⎜ ⎟
⎝ λ −1 ⎠
a0 . . . am−2 λ + am−1
⎛ ⎞
λ −1
⎜ .. .. ⎟
⎜ . . ⎟

= − (λ + am−1 ) det ⎜ ⎟
. ⎟
⎝ . . −1⎠
λ
  
=λm−1
⎛ ⎞
λ −1
⎜ .. .. ⎟
⎜ . . ⎟
⎜ ⎟
− det ⎜
⎜ . .
.. ..


⎜ ⎟
⎝ λ −1 ⎠
a0 ... am−2
= − λm − λm−1 am−1 − λm−2 am−2
⎛ ⎞
λ −1
⎜ .. .. ⎟
⎜ . . ⎟
⎜ ⎟
− det ⎜
⎜ . .
.. ..
⎟.

⎜ ⎟
⎝ λ −1 ⎠
a0 ... am−3
Kapitel 19 a) 1 2 3 4

1
Aufgaben 2
2
Verständnisfragen b) c) 1 3
3
Aufgabe 19.1 • In einer Schachtel liegen fünf von 1
bis 5 nummerierte Kugeln. Geben Sie einen Grundraum für 4

die Ergebnisse eines stochastischen Vorgangs an, der darin 4


besteht, rein zufällig zwei Kugeln mit einem Griff zu ziehen.

Aufgabe 19.2 • Geben Sie jeweils einen geeigneten 1 3


Grundraum für folgende stochastischen Vorgänge an: d)

2 4
a) Drei nicht unterscheidbare 1-€-Münzen werden gleich-
zeitig geworfen. Schaltbilder zu Stromkreisen
b) Eine 1-€-Münze wird dreimal hintereinander geworfen.
c) Eine 1-Cent-Münze und eine 1-€-Münze werden gleich-
zeitig geworfen. a) In der zweiten Versuchsreihe treten mindestens zwei Tref-
fer auf,
Aufgabe 19.3 • Eine technische Anlage bestehe aus b) bei beiden Versuchsreihen treten unterschiedlich viele
einem Generator, drei Kesseln und zwei Turbinen. Jede dieser Treffer auf,
sechs Komponenten kann während eines gewissen, definier- c) die zweite Versuchsreihe liefert weniger Treffer als die
ten Zeitraums ausfallen oder intakt bleiben. Geben Sie einen erste,
Grundraum an, dessen Elemente einen Gesamtüberblick über d) in jeder Versuchsreihe gibt es mindestens einen Treffer.
den Zustand der Komponenten am Ende des Zeitraums lie-
fern. Aufgabe 19.8 •• Ein Würfel wird höchstens dreimal
geworfen. Erscheint eine Sechs zum ersten Mal im j -ten Wurf
Aufgabe 19.4 • Es seien A, B, C, D Ereignisse in (j = 1, 2, 3), so erhält eine Person aj €, und das Spiel ist be-
einem Grundraum . Sei E das Ereignis, dass von den Ereig- endet. Hierbei sei a1 = 100, a2 = 50 und a3 = 10. Erscheint
nissen A, B, C, D höchstens zwei eintreten. Drücken Sie das auch im dritten Wurf noch keine Sechs, so sind 30 € an die
verbal beschriebene Ereignis E durch A, B, C und D aus. Bank zu zahlen, und das Spiel ist ebenfalls beendet. Beschrei-
ben Sie den Spielgewinn mithilfe einer Zufallsvariablen auf
einem geeigneten Grundraum.
Aufgabe 19.5 •• In einem Stromkreis befinden sich
vier nummerierte Bauteile, die jedes für sich innerhalb eines
Aufgabe 19.9 • Das gleichzeitige Eintreten der Ereig-
gewissen Zeitraums intakt bleiben oder ausfallen können. Im
nisse A und B ziehe das Eintreten des Ereignisses C nach
letzteren Fall ist der Stromfluss durch das betreffende Bauteil
sich. Zeigen Sie, dass dann gilt:
unterbrochen. Es bezeichnen Aj das Ereignis, dass das j -te
Bauteil intakt bleibt (j = 1, 2, 3, 4) und A das Ereignis, dass
P(C) ≥ P(A) + P(B) − 1 .
der Stromfluss nicht unterbrochen ist. Drücken Sie für jedes
der vier Schaltbilder das Ereignis A durch A1 , A2 , A3 , A4
aus. Aufgabe 19.10 •• Es sei c ∈ (0, ∞) eine beliebige
(noch so große) Zahl. Gibt es Ereignisse A, B in einem ge-
Aufgabe 19.6 • In der Situation von Aufgabe 19.3 sei eigneten Wahrscheinlichkeitsraum, sodass
die Anlage arbeitsfähig (Ereignis A), wenn der Generator,
mindestens ein Kessel und mindestens eine Turbine intakt P(A ∩ B) ≥ c · P(A) · P(B)
sind. Die Arbeitsfähigkeit des Generators, des i-ten Kessels
und der j -ten Turbine seien durch die Ereignisse G, Ki und gilt?
Tj (i = 1, 2, 3; j = 1, 2) beschrieben. Drücken Sie A und
Ac durch G, K1 , K2 , K3 und T1 , T2 aus. Aufgabe 19.11 • Ist es möglich, dass von drei Ereig-
nissen, von denen jedes die Wahrscheinlichkeit 0.7 besitzt,
Aufgabe 19.7 • Ein Versuch mit den möglichen Er- nur genau eines eintritt?
gebnissen Treffer (1) und Niete (0) werde 2n-mal durchge-
führt. Die ersten (bzw. zweiten) n Versuche bilden die sog. Aufgabe 19.12 • Zeigen Sie, dass es unter acht paar-
erste (bzw. zweite) Versuchsreihe. Beschreiben Sie folgende weise disjunkten Ereignissen stets mindestens drei gibt, die
Ereignisse mithilfe geeigneter Zählvariablen: höchstens die Wahrscheinlichkeit 1/6 besitzen.
M. Brokate et al., Arbeitsbuch Grundwissen Mathematikstudium – Höhere Analysis, Numerik und
Stochastik, DOI 10.1007/978-3-642-54946-5_18, © Springer-Verlag Berlin Heidelberg 2016
Aufgaben zu Kapitel 19 137

Aufgabe 19.13 • Mit welcher Wahrscheinlichkeit ist 1 2 3 4 5 6 7


beim Lotto 6 aus 49 616 624 638 626 607 649 617
8 9 10 11 12 13 14
a) die zweite gezogene Zahl kleiner als die erste? 598 636 605 623 600 561 610
b) die dritte gezogene Zahl kleiner als die beiden ersten Zah- 15 16 17 18 19 20 21
len? 588 623 615 618 610 585 594
c) die letzte gezogene Zahl die größte aller 6 Gewinnzahlen? 22 23 24 25 26 27 28
627 611 619 652 659 648 577
29 30 31 32 33 34 35
Aufgabe 19.14 •• Auf einem m × n-Gitter mit den Ko- 593 602 649 629 643 615 615
ordinaten (i, j ), 0 ≤ i ≤ m, 0 ≤ j ≤ n (siehe nachstehende 36 37 38 39 40 41 42
Abbildung für den Fall m = 8, n = 6) startet ein Robo- 618 610 658 617 616 639 623
ter links unten im Punkt (0, 0). Er kann wie abgebildet pro 43 44 45 46 47 48 49
Schritt nur nach rechts oder nach oben gehen. 663 612 570 592 621 612 649

n Aufgabe 19.19 • Zeigen Sie, dass durch die Werte


pk := 1/(k(k + 1)), k ≥ 1, eine Wahrscheinlichkeitsver-
teilung auf der Menge N der natürlichen Zahlen definiert
. wird.
.
.
Aufgabe 19.20 • Bei einer Qualitätskontrolle können
1
Werkstücke zwei Arten von Fehlern aufweisen, den Fehler
0
0 1 ··· m A und den Fehler B. Aus Erfahrung sei bekannt, dass ein
zufällig herausgegriffenes Werkstück mit Wahrscheinlichkeit

a) Auf wie viele Weisen kann er den Punkt (m, n) rechts 0.04 den Fehler A hat,
oben erreichen? 0.005 beide Fehler aufweist,
b) Wie viele Wege von (0, 0) nach (m, n) gibt es, die durch 0.01 nur den Fehler B hat.
den Punkt (a, b) verlaufen? a) Mit welcher Wahrscheinlichkeit weist das Werkstück den
Fehler B auf?
b) Mit welcher Wahrscheinlichkeit ist das Werkstück fehler-
Aufgabe 19.15 • Wie viele Möglichkeiten gibt es, k
haft bzw. fehlerfrei?
verschiedene Teilchen so auf n Fächer zu verteilen, dass im j -
c) Mit welcher Wahrscheinlichkeit besitzt das Werkstück
ten Fach kj Teilchen liegen (j = 1, . . . , n, k1 , . . . , kn ∈ N0 ,
genau einen der beiden Fehler?
k1 + · · · + kn = k)?
Aufgabe 19.21 • Es seien A, B Ereignisse in einem
Aufgabe 19.16 • Es sei f eine auf einer offenen Teil- Wahrscheinlichkeitsraum (, A, P). Zeigen Sie:
menge des Rn definierte stetig differenzierbare reellwertige
a) P(Ac ∩ B c ) + P(A) + P(Ac ∩ B) = 1,
Funktion. Wie viele verschiedene partielle Ableitungen k-ter
Ordnung besitzt f ? b) P(A ∩ B) − P(A)P(B) = P(Ac ∩ B c ) − P(Ac )P(B c ).

Aufgabe 19.22 • Beim Zahlenlotto6 aus 49 beobach-


Aufgabe 19.17 • Aus sieben Männern und sieben tet man häufig, dass sich unter den sechs Gewinnzahlen min-
Frauen werden sieben Personen rein zufällig ausgewählt. Mit destens ein Zwilling, d. h. mindestens ein Paar (i, i + 1) be-
welcher Wahrscheinlichkeit enthält die Stichprobe höchstens nachbarter Zahlen befindet. Wie wahrscheinlich ist dies?
drei Frauen? Ist das Ergebnis ohne Rechnung einzusehen?
Aufgabe 19.23 • Sollte man beim Spiel mit einem fai-
ren Würfel eher auf das Eintreten mindestens einer Sechs in
Rechenaufgaben vier Würfen oder beim Spiel mit zwei echten Würfeln auf das
Eintreten mindestens einer Doppelsechs (Sechser-Pasch) in
Aufgabe 19.18 • Im Lotto 6 aus 49 ergab sich nach 24 Würfen setzen? (Frage des Antoine Gombault Chevalier
5047 Ausspielungen die nachstehende Tabelle der Gewinn- de Méré (1607–1684))
häufigkeiten der einzelnen Zahlen.
Aufgabe 19.24 • Bei der ersten Ziehung der Glücks-
a) Wie groß sind die relativen Gewinnhäufigkeiten der Zah- spirale 1971 wurden für die Ermittlung einer 7-stelligen Ge-
len 13, 19 und 43? winnzahl aus einer Trommel, die Kugeln mit den Ziffern
b) Wie groß wäre die relative Gewinnhäufigkeit, wenn jede 0, 1, . . . , 9 je 7mal enthält, nacheinander rein zufällig 7 Ku-
Zahl gleich oft gezogen worden wäre? geln ohne Zurücklegen gezogen.
138 Aufgaben zu Kapitel 19

a) Welche 7-stelligen Gewinnzahlen hatten hierbei die Man mache sich klar, dass A nur dann gleich der vollen Po-
größte und die kleinste Ziehungswahrscheinlichkeit, und tenzmenge von  ist, wenn jedes Aj einelementig (und somit
wie groß sind diese Wahrscheinlichkeiten?  insbesondere abzählbar) ist.
b) Bestimmen Sie die Gewinnwahrscheinlichkeit für die
Zahl 3 143 643. Aufgabe 19.30 • Es seien A und B Ereignisse in einem
c) Wie würden Sie den Ziehungsmodus abändern, um al- Grundraum . Zeigen Sie:
len Gewinnzahlen die gleiche Ziehungswahrscheinlich- a) 1A∩B = 1A · 1B ,
keit zu sichern?
b) 1A∪B = 1A + 1B − 1A∩B ,
c) 1A+B = 1A + 1B ,
Aufgabe 19.25 •• Bei der Auslosung der 32 Spiele der
ersten Hauptrunde des DFB-Pokals 1986 gab es einen Eklat, d) 1{Ac } = 1 − 1A ,
als der Loszettel der Stuttgarter Kickers unbemerkt buchstäb- e) A ⊆ B ⇐⇒ 1A ≤ 1B .
lich unter den Tisch gefallen und schließlich unter Auslosung
des Heimrechts der zuletzt im Lostopf verbliebenen Mann- Aufgabe 19.31 • Es seien (, A, P) ein Wahrschein-
schaft Tennis Borussia Berlin zugeordnet worden war. Auf lichkeitsraum und (An ) eine Folge in A mit An ↓ A. Zeigen
einen Einspruch der Stuttgarter Kickers hin wurde die ge- Sie:
samte Auslosung der ersten Hauptrunde neu angesetzt. Ku- P(A) = lim P(An ).
n→∞
rioserweise ergab sich dabei wiederum die Begegnung Tennis
Borussia Berlin – Stuttgarter Kickers. Aufgabe 19.32 • Es seien (, A) ein Messraum und
P : A → [0, 1] eine Funktion mit
a) Zeigen Sie, dass aus stochastischen Gründen kein Ein-
wand gegen die erste Auslosung besteht. P(A + B) = P(A) + P(B), falls A, B ∈ A mit A ∩ B = ∅,
b) Wie groß ist die Wahrscheinlichkeit, dass sich in der zwei- P(B) = limn→∞ P(Bn ) für jede Folge (Bn ) aus A mit
ten Auslosung erneut die Begegnung Tennis Borussia Bn ↑ B.
Berlin – Stuttgarter Kickers ergibt?
Zeigen Sie, dass P σ -additiv ist.

Aufgabe 19.26 •• Die Zufallsvariable Xk bezeichne Aufgabe 19.33 ••• Beweisen Sie die Formel des Ein-
die k-kleinste der 6 Gewinnzahlen beim Lotto 6 aus 49. Wel- und Ausschließens auf Seite 717 durch Induktion über n.
che Verteilung besitzt Xk unter einem Laplace-Modell?
Aufgabe 19.34 •• In einer geordneten Reihe zweier
Aufgabe 19.27 •• Drei Spieler A, B, C spielen Skat. verschiedener Symbole a und b heißt jede aus gleichen Sym-
Berechnen Sie unter einem Laplace-Modell die Wahrschein- bolen bestehende Teilfolge maximaler Länge ein Run. Als
lichkeiten Beispiel betrachten wir die Anordnung b b a a a b a, die mit
einem b-Run der Länge 2 beginnt. Danach folgen ein a-Run
a) Spieler A erhält alle vier Buben, der Länge 3 und jeweils ein b- und ein a-Run der Länge 1.
b) irgendein Spieler erhält alle Buben, Es mögen nun allgemein
 m Symbole a und n Symbole b vor-
c) Spieler A erhält mindestens ein Ass, liegen, wobei alle m+n
m Anordnungen im Sinne von Aus-
wahlen von m der m + n Komponenten in einem Tupel für
d) es liegen ein Bube und ein Ass im Skat. die a’s (die übrigen Komponenten sind dann die b’s) gleich
wahrscheinlich seien. Die Zufallsvariable X bezeichne die
Aufgabe 19.28 • Eine Warenlieferung enthalte 20 in- Gesamtanzahl der Runs. Zeigen Sie:
takte und 5 defekte Stücke. Wie groß ist die Wahrscheinlich-  n−1
keit, dass eine Stichprobe vom Umfang 5 2 m−1
s−1 s−1
P(X = 2s) = m+n , 1 ≤ s ≤ min(m, n) ,
a) genau zwei defekte Stücke enthält? m
n−1m−1 n−1m−1
b) mindestens zwei defekte Stücke enthält? s s−1 +
P(X = 2s + 1) = m+ns−1 s
,
m

Beweisaufgaben 1 ≤ s < min(m, n).


∞
Aufgabe 19.29 •• Es sei  = n=1 An eine Zerle- Aufgabe 19.35 •• Es seien M1 eine k-elementige und
gung des Grundraums  in paarweise disjunkte Mengen M2 eine n-elementige Menge, wobei n ≤ k gelte. Wie viele
A1 , A2 , . . .. Zeigen Sie, dass das System surjektive Abbildungen f : M1 → M2 gibt es?
# :
Aufgabe 19.36 •• Es seien A1 , . . . , An die in (19.34)
A= B ⊆  | ∃T ⊆ N mit B = An definierten Ereignisse. Zeigen Sie:
n∈T
r · (r − 1)
P (Ai ∩ Aj ) = (1 ≤ i = j ≤ n).
eine σ -Algebra über  ist. (r + s) · (r + s − 1)
Hinweise zu Kapitel 19 139

Aufgabe 19.37 ••• Es fallen rein zufällig der Reihe nach Rechenaufgaben
Teilchen in eines von n Fächern. Die Zufallsvariable Xn be-
zeichne die Anzahl der Teilchen, die nötig sind, damit zum Aufgabe 19.18 • –
ersten Mal ein Teilchen in ein Fach fällt, das bereits belegt
ist. Zeigen Sie: Aufgabe 19.19 • –
 
a) 1 − exp − k(k−1)
2n ≤ P(Xn ≤ k), Aufgabe 19.20 • –
 
k(k−1)
b) P(Xn ≤ k) ≤ 1 − exp − 2(n−k+1) ,
Aufgabe 19.21 • –
c) für jedes t > 0 gilt
   
Xn t2 Aufgabe 19.22 • Man betrachte das komplementäre
lim P √ ≤ t = 1 − exp − . Ereignis.
n→∞ n 2

Aufgabe 19.23 • –

Hinweise Aufgabe 19.24 • Unterscheiden Sie gedanklich die 7


gleichen Exemplare jeder Ziffer.
Verständnisfragen
Aufgabe 19.1 • – Aufgabe 19.25 •• Nummeriert man alle Mannschaf-
ten gedanklich von 1 bis 64 durch, so ist das Ergebnis
Aufgabe 19.2 • – einer regulären Auslosung ein 64-Tupel (a1 , . . . , a64 ), wo-
bei Mannschaft a2i−1 gegen Mannschaft a2i Heimrecht hat
Aufgabe 19.3 • – (i = 1, . . . , 32).

Aufgabe 19.4 • – Aufgabe 19.26 •• –

Aufgabe 19.5 •• – Aufgabe 19.27 •• –

Aufgabe 19.6 • –
Aufgabe 19.28 • –
Aufgabe 19.7 • –

Aufgabe 19.8 •• – Beweisaufgaben


Aufgabe 19.29 •• –
Aufgabe 19.9 • –

Aufgabe 19.10 •• Wählen Sie  := {1, . . . , n} und ein Aufgabe 19.30 • –


Laplace-Modell.
Aufgabe 19.31 • –
Aufgabe 19.11 • Betrachten Sie einen Laplace-Raum
der Ordnung 10.
Aufgabe 19.32 • –
Aufgabe 19.12 • –
Aufgabe 19.33 ••• –
Aufgabe 19.13 • Stellen Sie Symmetriebetrachtungen
an. Aufgabe 19.34 •• Um die Längen der a-Runs festzule-
gen, muss man bei den in einer Reihe angeordneten m a’s
Aufgabe 19.14 •• – Trennstriche anbringen.

Aufgabe 19.15 • – Aufgabe 19.35 •• Formel des Ein- und Ausschließens!

Aufgabe 19.16 • Es kommt nur darauf an, wie oft nach


jeder einzelnen Variablen differenziert wird. Aufgabe 19.36 •• –

Aufgabe 19.17 • – Aufgabe 19.37 ••• Starten Sie mit (19.40).


140 Lösungswege zu Kapitel 19

Lösungen Aufgabe 19.25 •• –

Aufgabe 19.26 •• –
Verständnisfragen
Aufgabe 19.1 • – Aufgabe 19.27 •• –

Aufgabe 19.2 • – Aufgabe 19.28 • –

Aufgabe 19.3 • –
Beweisaufgaben
Aufgabe 19.4 • –
Aufgabe 19.29 •• –
Aufgabe 19.5 •• a) A = A1 ∩ A2 ∩ A3 ∩ A4
Aufgabe 19.30 • –
b) A = A1 ∪ A2 ∪ A3 ∪ A4 c) A = A1 ∩ (A2 ∪ A3 ∪ A4 )
d) A = (A1 ∪ A2 ) ∩ (A3 ∪ A4 ).
Aufgabe 19.31 • –
Aufgabe 19.6 • A = G ∩ (K1 ∪ K2 ∪ K3 ) ∩ (T1 ∪ T2 ),
Ac = Gc ∪ (K1c ∩ K2c ∩ K3c ) ∪ (T1c ∩ T2c ). Aufgabe 19.32 • –

Aufgabe 19.7 • – Aufgabe 19.33 ••• –

Aufgabe 19.8 •• – Aufgabe 19.34 •• –


n−1  
r n (n − r)k
Aufgabe 19.9 • – Aufgabe 19.35 •• r=0 (−1) r

Aufgabe 19.10 •• – Aufgabe 19.36 •• –

Aufgabe 19.11 • – Aufgabe 19.37 ••• –

Aufgabe 19.12 • –

Aufgabe 19.13 • – Lösungswege


Aufgabe 19.14 •• –
Verständnisfragen
Aufgabe 19.15 • – Aufgabe 19.1 • Da nur festgestellt werden kann, wel-
n+k−1 che Zweier-Teilmenge der 5 Kugeln gezogen wurde, ist
Aufgabe 19.16 • k .
 := {{1, 2}, {1, 3}, {1, 4}, {1, 5}, {2, 3}, {2, 4},
Aufgabe 19.17 • 1/2. {2, 5}, {3, 4}, {3, 5}, {4, 5}}
= {M ⊆ {1, 2, 3, 4, 5} | |M| = 2}
Rechenaufgaben
ein angemessener Grundraum.
Aufgabe 19.18 • –
Aufgabe 19.2 • a) Da nur festgestellt werden kann,
Aufgabe 19.19 • – wie oft Wappen oder Zahl fällt, ist  := {0, 1, 2, 3} ein mög-
licher Ergebnisraum. Dabei stehe j ∈  für das Ergebnis,
Aufgabe 19.20 • – dass j mal Wappen und 3 − j mal Zahl auftritt.

Aufgabe 19.21 • – b) In diesem Fall besitzt man die vollständige Information


über die Ergebnisse Wappen (W ) oder Zahl (Z) in jedem der
Aufgabe 19.22 • – drei Würfe. Ein angemessener Grundraum ist somit

 := {(W, W.W ), (W, W, Z), (W, Z, W ), (W, Z, Z),


Aufgabe 19.23 • –
(Z, W, W ), (Z, W, Z), (Z, Z, W ), (Z, Z, Z)}
Aufgabe 19.24 • – = {Z, W }3 .
Lösungswege zu Kapitel 19 141

Natürlich kann man die Ergebnisse Z und W auch als 1 bzw. Aufgabe 19.8 •• Wie bei Modellierungsproblemen üb-
0 notieren. lich gibt es auch hier mehrere Möglichkeiten. Eine besteht
darin, den Grundraum  := {1, 2, 3, 4, 5, 6}3 zu wählen,
c) Ein möglicher Grundraum ist also gedanklich auch für den Fall, dass im ersten oder zwei-
 := {(E, W ), (E, Z), (Z, W ), (Z, Z)}. ten Wurf eine Sechs fällt, weiterzuwürfeln. Hierbei steht im
Tripel ω = (a1 , a2 , a3 ) die Komponente aj für das Ergeb-
Dabei stehe E für Eichenblatt, und die erste Komponente nis des j -ten Wurfs. Der Spielgewinn wird dann auf diesem
bezeichne das Ergebnis der 1-Cent-Münze. Grundraum durch die Zufallsvariable X :  → R mit


⎪ 100, falls a1 = 6
Aufgabe 19.3 • Nummerieren wir zwecks Identifizie- ⎪

⎨50, falls a ≤ 5 und a = 6,
rung die Kessel von 1 bis 3 und die Turbinen von 1 bis 2, so X(ω) :=
1 2
ist die Menge ⎪10, falls max(a1 , a2 ) ≤ 5 und a3 = 6,



⎩−30, sonst,
 := {(a1 , . . . , a6 ) | aj ∈ {0, 1} für j = 1, . . . , 6}
(ω = (a1 , a2 , a3 ) ∈ ) beschrieben. Dabei bedeutet der
ein möglicher Grundraum. Dabei stehe eine 1 (0) für intaktes
negative Wert −30 einen Verlust. Eine andere Möglichkeit
Verhalten (Ausfall); a1 beschreibe den Zustand des Gene-
besteht darin, den auf den ersten Blick attraktiven, weil einfa-
rators, aj +1 den des j -ten Kessels (j = 1, 2, 3) und aj +4 ˜ := {0, 1, 2, 3} zu wählen. Hier modelliert
chen Grundraum 
denjenigen der j -ten Turbine (j = 1, 2).
j ∈ {1, 2, 3} das Ergebnis die erste Sechs tritt im j -ten Wurf
Aufgabe 19.4 • „Am elegantesten“ ist es, die Indika- auf, und 0 bedeutet, dass keiner der drei Würfe eine Sechs
torsumme X := 1A + 1B + 1C + 1D zu definieren. Das ergibt. In diesem Fall beschreibt die durch X̃(0) := −30,
gesuchte Ereignis ist dann E = {X ≤ 2}. Eine andere Mög- X̃(1) = 100, X̃(2) = 50 und X̃(3) = 10 definierte Zufalls-
variable X̃ : ˜ → R den Spielgewinn. Der Vorteil des ersten
lichkeit besteht darin, das Ereignis E in die Ereignisse Ej
aufzuspalten, wobei genau j der vier Ereignisse eintreten Raumes offenbart sich, wenn wir etwa die Wahrscheinlich-
(j = 0, 1, 2). Dann wäre E = E0 + E1 + E2 zu setzen. keit dafür angeben wollen, dass das Spiel mit einer Zahlung
Hierbei gilt an die Bank endet oder die Frage stellen, ob das Spiel für
den Spieler vorteilhaft ist. Die Ergebnisse ω ∈  würde man
E0 = Ac B c C c D c , unter der Annahme, dass der Würfel exakt gefertigt ist, als
E1 = AB c C c D c + Ac BC c D c + Ac B c CD c + Ac B c C c D, gleich wahrscheinlich ansehen. Für die Ergebnisse im Grund-
raum  ˜ trifft dies nicht zu.
E2 = ABC c D c + AB c CD c + Ac BCD c + AB c C c D
+Ac BC c D + Ac B c CD. Aufgabe 19.9 • Die verbal beschriebene Vorausset-
zung besagt A ∩ B ⊆ C, und somit folgt P(A ∩ B) ≤ P(C).
Man beachte, dass Ej = {X = j } gilt (j = 0, 1, 2). Weiter gilt
Aufgabe 19.5 •• a) Es muss jedes der Ereignisse ein- P(A ∩ B) = 1 − P((A ∩ B)c ) = 1 − P(Ac ∪ B c )
treten. b) Es muss mindestens eines der Ereignisse eintreten. ≥ 1 − P(Ac ) − P(B c )
c) Es muss A1 und dazu noch mindestens eines der anderen
= 1 − (1 − P(A)) − (1 − P(B))
Ereignisse eintreten. d) Es müssen mindestens eines der Er-
eignisse A1 , A2 und mindestens eines der Ereignisse A3 , A4 = P(A) + P(B) − 1.
eintreten.
Zusammen folgt die Behauptung.
Aufgabe 19.6 • Die Darstellung für A ergibt sich aus Aufgabe 19.10 •• Mit der obigen Wahl von  und P
der Bedingung, dass der Generator und mindestens einer der als Gleichverteilung auf  sei A := {1, 2, . . . , k} und B :=
Kessel und mindestens eine der Turbinen arbeiten muss so- {2, 3, . . . , k + 1} gesetzt, wobei k ≤ n − 1. Dann gilt
wie der Tatsache, dass das logische und dem Durchschnitt-
szeichen und das nicht ausschließende logische oder dem k k−1
P(A) = P(B) = , P(A ∩ B) =
Vereinigungszeichen entspricht. Die Darstellung für Ac folgt n n
aus der de Morgan’schen Regel. und somit
P(A ∩ B) (k − 1)n
= .
Aufgabe 19.7 • Es sei  := {0, 1}2n .Für j = 1, P(A) · P(B) k2
. . . , n bezeichne Aj := {(a1 , . . . , a2n ) ∈  | aj = 1} das Wählt man jetzt zu gegebenem c die Zahl m als kleinste
Ereignis, dass der j -te Versuch einen Treffer n ergibt. Dann natürliche Zahl, die größer oder gleich c+1 ist sowie n := m2 ,
beschreiben die Zufallsvariablen X := j =1 1{Aj } und k := m, so geht obige Gleichung in
2n
Y := j =n+1 1{Aj } die Trefferanzahlen in der ersten bzw.
P(A ∩ B) (m − 1)m2
zweiten Versuchsreihe. Hiermit nehmen die verbal beschrie- = =m−1≥c
benen Ereignisse formal folgende Gestalt an: a) {Y ≥ 2} P(A) · P(B) m2
b) {X = Y } c) {Y < X} d) {X ≥ 1} ∩ {Y ≥ 1}. über, was zu zeigen war.
142 Lösungswege zu Kapitel 19

Aufgabe 19.11 • Ja. Ein Beispiel ist der Grundraum 561/5047 ≈ 0.111,
 := {0, 1, 2, 3, 4, 5, 6, 7, 8, 9} mit der Gleichverteilung 610/5047 ≈ 0.121,
P. Für die Ereignisse A := {1, 2, 3, 4, 5, 6, 7}, B := 663/5047 ≈ 0.131.
{3, 4, 5, 6, 7, 8, 9}, C := {0, 2, 3, 4, 5, 6, 7} gilt P(A) =
Dabei wurde auf 3 Nachkommastellen gerundet.
P(B) = P(C) = 0.7 sowie P(A ∩ B c ∩ C c ) = P({1}) = 0.1,
P(Ac ∩ B ∩ C c ) = P({8, 9}) = 0.2, P(Ac ∩ B c ∩ C) = b) Wenn jede Zahl nach 5047 Ausspielungen k mal gezogen
P({0}) = 0.1. wurde, gilt k · 49 = 6 · 5047 und somit k = 618. Die relative
Häufigkeit ist dann 618/5047 = 6/49 ≈ 0.122.
Aufgabe 19.12 • Würde etwa nur P(A1 ) ≤ 1/6 und
P(A2 ) ≤ 1/6 gelten, so wäre P(Aj ) > 1/6 für j = 3, . . . , 8. Aufgabe 19.19 • Wegen pk ≥ 0 ist nur die Normie-
Da sich die Ereignisse paarweise ausschließen, wäre dann rungsbedingung ∞ k=1 pk = 1 zu zeigen. Wegen
P(A3 ∪ . . . ∪ A8 ) = P(A3 ) + . . . + P(A8 ) > 6 · 1/6 = 1,
was ein Widerspruch zu P(A) ≤ 1, A ∈ A, wäre. 1 1 1
= −
k(k + 1) k k+1
Aufgabe 19.13 • a) Für die beiden ersten Zahlen gibt 
es genauso viele Paare (i, j ) mit i < j wie es Paare mit i > j gilt unter Ausnutzung eines Teleskop-Effektes nk=1 pk =
gibt. Die Wahrscheinlichkeit ist somit 1/2. 1 − 1/n, woraus die Behauptung folgt. Diese Verteilung ent-
steht im Zusammenhang mit einem Urnenmodell, wenn auf
b) Eine der drei ersten Zahlen ist die kleinste. Aus Symme- das erstmalige Auftreten einer roten Kugel gewartet wird.
triegründen ist die Antwort 1/3.
c) Eine der 6 gezogenen Zahlen ist die größte aller 6 Zahlen. Aufgabe 19.20 • Es sei A bzw. B das Ereignis, dass
Die Antwort ist ebenfalls aus Symmetriegründen 1/6. das Werkstück den Fehler A bzw. den Fehler B aufweist. Aus
der Aufgabenstellung ist bekannt: P(A) = 0.04, P(A ∩ B) =
Aufgabe 19.14 •• a) Da von insgesamt
  m+n Schritten 0.005, P(B ∩ Ac ) = 0.01.
m für ’rechts’ zu wählen sind, gibt es m+n
m Möglichkeiten.
a): Mit Eigenschaft b) auf Seite 714 folgt P(B) = P(A ∩
b) Mit
a+b  a) und der Multiplikationsregel ist die Anzahl gleich
m−a B) + P(Ac ∩ B) = 0.005 + 0.01 = 0.015.
a · n−b .
b): Aus Eigenschaft f) auf Seite 714 ergibt sich P(A ∪ B) =
Aufgabe 19.15 • Es gibt
  P(A)+ P(B)− P(A∩B) = 0.04+0.015−0.005 = 0.05. Ein
k k! Werkstück ist also mit Wahrscheinlichkeit 0.05 fehlerhaft und
=
k1 , . . . , kn k1 ! · . . . · kn ! somit (komplementäre Wahrscheinlichkeit) mit Wahrschein-
k lichkeit 0.95 fehlerfrei.
Möglichkeiten, nämlich k1 Möglichkeiten, k1 Teilchen in
 1
Fach 1 zu legen, dann k−k k2 Möglichkeiten, von den rest- c): Es ist P(A ∩ B c ) + P(B ∩ Ac ) = P(A ∪ B) − P(A ∩ B) =
lichen Teilchen k2 für Fach 2 auszuwählen usw. (vgl. die 0.05 − 0.005 = 0.045.
Herleitung des Multinomialkoeffizienten auf Seite 724).
Aufgabe 19.21 • a): Die Behauptung folgt aus der
Aufgabe 19.16 • Es handelt sich um k-Kombinationen Darstellung  = A + Ac ∩ B c + Ac ∩ B und der endli-
aus n Objekten (den Variablen) mit Wiederholung. chen Additivität von P sowie P() = 1.

Aufgabe 19.17 • Die zufällige Anzahl X der Frauen b): Es gilt P(Ac ∩ B c ) = 1 − P(A ∪ B) = 1 − P(A) − P(B) +
in der Stichprobe besitzt die hypergeometrische Verteilung P(A ∩ B) sowie P(Ac ) = 1 − P(A), P(B c ) = 1 − P(B).
Hyp(n, r, s) mit n = r = s = 7. Es folgt Hieraus folgt die Behauptung.
3 3 7 7 

P(X ≤ 3) = P(X = k) =
k 7−k
14 Aufgabe 19.22 • Jede 6-Kombination
k=0 k=0 7
1 ≤ b1 < . . . < b6 ≤ 49
12 + 72 + 21 + 352
2 1
= =.
3432 2 ohne Zwilling lässt sich gemäß
Dieses Ergebnis ist auch aus Symmetriegründen klar, da es
genauso viele Frauen wie Männer gibt und das zu {X ≤ 3} aj := bj − j + 1, j = 1, . . . , 6,
komplementäre Ereignis {X ≥ 4} gleichbedeutend damit ist,
dass die Stichprobe höchstens drei Männer enthält. zu einer 6-Kombination 1 ≤ a1 . . . < a6 ≤ 44 aus
K644 (oW ) „zusammenziehen“ und umgekehrt. Die gesuchte
  49
Wahrscheinlichkeit ist somit 1 − 44
6 / 6 = 0.495 . . .
Rechenaufgaben
Aufgabe 19.18 • a) Die relativen Häufigkeiten der Aufgabe 19.23 • Die Wahrscheinlichkeit, mindestens
Zahlen 13, 19 und 43 sind eine Sechs in vier Würfen zu werfen, ist 1 − (5/6)4 ≈ 0.518
Lösungswege zu Kapitel 19 143

(vgl. die Ausführungen auf Seite 715). Die Wahrscheinlich- der Multiplikationsregel und der Anzahlformel für Kombi-
keit, mindestens eine Doppelsechs in 24 Doppelwürfen zu nationen ohne Wiederholung folgt
werfen, berechnet sich analog zu 1 − (35/36)24 ≈ 0.491 . j −1 49−j 
Man sollte also eher auf das Eintreten von mindestens einer k−1 · 6−k
P (Xk = j ) = 49 , j = k, k + 1, . . . , k + 43.
Sechs in vier Würfen wetten.
6

Aufgabe 19.24 •
a) 7-stellige Gewinnzahlen mit lauter verschiedenen (glei- Aufgabe 19.27 •• Wir verwenden den Grundraum
chen) Ziffern hatten die größte (kleinste) Wahrschein-
 := {(A, B, C) | A + B + C ⊆ K, |A + B + C| = 30}
lichkeit, gezogen zu werden. Als Grundraum  kann
die Menge der 7-Permutationen ohne Wiederholung aus auf Seite 724 mit der Gleichverteilung P auf . Die Teilmen-
{01 , 02 , . . . , 07 , 11 , 12 , . . . , 17 , . . . , 91 , 92 , . . . , 97 } ge- gen S, Bu und As bezeichnen den Skat, die vier Buben bzw.
wählt werden, wenn man jede Ziffer gedanklich von 1 die vier Asse.
bis 7 nummeriert. Bei Annahme eines Laplace-Modells
besitzt jede Zahl mit lauter verschiedenen (bzw. glei- a): Das beschriebene Ereignis ist D1 := {(A, B, C) ⊆  |
chen) Ziffern die gleiche Wahrscheinlichkeit 77 /(70)7 Bu ⊆ A}. Wegen Bu ⊆ A stehen zur Bildung der Teilmenge
(bzw. 7!/(70)7 ). Der Quotient von größter zu kleinster der Karten für A nur 28 Karten zur Verfügung. Nach der
Ziehungswahrscheinlichkeit ist 77 /7! ≈ 163.4. Multiplikationsregel folgt
b) 7 · 7 · 7 · 6 · 7 · 6 · 5/(70)7 ≈ 7.153 · 10−8 282212
|D1 | 6 10 10 21
c) Jede Ziffer der Gewinnzahl wird aus einer separaten P(D1 ) = = = ≈ 0.00584.
Trommel, welche die Ziffern 0, 1, . . . , 9 je einmal ent- || 32! 3596
10!3 ·2!
hält, gezogen. Gleichwertig hiermit ist das 7-fache Ziehen
mit Zurücklegen aus einer Trommel, die jede der Ziffern
b): Das beschriebene Ereignis D2 ist die Vereinigung der
0, 1, . . . , 9 einmal enthält.
drei paarweise disjunkten Ereignisse, dass entweder A oder
B oder C alle vier Buben erhalten. Mit a) folgt aus Symme-
Aufgabe 19.25 •• a) Seien  = P64 64 (oW ) die Menge
triegründen
aller regulären Auslosungen mit der am Ende des Hinweises
gemachten Interpretation sowie P die Gleichverteilung auf . 63
Ohne Beschränkung der Allgemeinheit sei 1 die Nummer der P(D2 ) = 3 · P(D1 ) = ≈ 0.0175.
3596
Stuttgarter Kickers und 2 die von Tennis Borussia Berlin. Das
Ereignis „Mannschaft j hat gegen Mannschaft k Heimrecht“
c): Gefragt ist nach P(D3 ), wobei D3 := {(A, B, C) ∈  |
ist formal durch Aj k := {(a1 , . . . , a64 ) ∈  | a2i−1 = j und
A ∩ As = ∅}. Das komplementäre Ereignis D3c bedeutet,
a2i = k für ein i ∈ {1, . . . , 32}} gegeben. Wegen || = 64!
dass A kein Ass erhält, also seine 10 Karten aus den 28
und |Aj k | = 32 · 1 · 62! gilt nach der Multiplikationsregel
Nicht-Assen ausgewählt werden. Nach der Multiplikations-
P(Aj k ) = |Aj k |/|| = 1/126, 1 ≤ j  = k ≤ 64, also ins-
regel folgt
besondere P(A21 ) = 1/126. Dieses Ergebnis kann auch so
eingesehen werden: Für Mannschaft 1 gibt es 63 gleichwahr- 282212
10 10 10 1463
scheinliche Gegner, wobei nach Auswahl des Gegners noch P(D3 ) = 1−P(D3c ) = 1− 32!
= 1− ≈ 0.797.
2 Möglichkeiten für das Heimrecht vorhanden sind. 7192
10!3 ·2!
Die Menge der möglichen Paarungen der „nicht regulären“
ersten Auslosung ist 0 := {(a1 , . . . , a64 ) ∈  | 1 ∈ d): Das beschriebene Ereignis ist D4 := {(A, B, C) ⊆  |
{a63 , a64 }}. Dabei sei im Folgenden P0 die Gleichvertei- |Bu ∩ S| = 1 = |As ∩ S|}. Die günstigen Fälle hierfür sind,
lung auf 0 . Setzen wir für j  = 1, k  = 1, j  = k dass zunächst sowohl von den vier Buben als auch von den
A0j k := {(a1 , . . . , a64 ) ∈ 0 | a2i−1 = j und a2i = k für ein vier Assen je eine Karte für den Skat ausgewählt werden,
i = 1, . . . , 31}, so liefert die Multiplikationsregel P0 (A0j k ) = wofür es 16 Möglichkeiten gibt. Danach kann man die restli-
|A0j k |/|0 | = 31 · 1 · 2 · 61!/(2 · 63!) = 1/126 = P(Aj k ). chen 30 Karten beliebig auf die drei Spieler verteilen. Nach
der Multiplikationsformel ergibt sich
Mit A01k := {(a1 , . . . , a64 ) ∈ 0 | a63 = 1, a64 = k} und
302010
A0k1 := {(a1 , . . . , a64 ) ∈ 0 | a63 = k, a64 = 1} (k  = 1) |D4 | 16 · 1
10 10 10
ergibt sich ebenso P0 (A01k ) = 62!/(2 · 63!) = 1/126 = P(D4 ) = = = ≈ 0.0323.
|| 32! 31
10!3 ·2!
P0 (A0k1 ).
b) 1/126.
Aufgabe 19.28 • Wir interpretieren die defekten Ex-
Aufgabe 19.26 •• Das Ereignis {Xk = j } tritt genau emplare als rote und die intakten Exemplare als schwarze Ku-
dann ein, wenn k − 1 der Lottozahlen kleiner als j , die k- geln und unterstellen Ziehen ohne Zurücklegen. Dann besitzt
kleinste gleich j und 6 − k Lottozahlen größer als j sind. Mit die Anzahl X der defekten Exemplare in einer rein zufälligen
144 Lösungswege zu Kapitel 19

Stichprobe die hypergeometrische Verteilung Hyp(n, r, s) Aufgabe 19.32 • Es seien A1 , A2 , . . . paarweise dis-
mit n = 5, r = 5, s = 20. Es folgt junkte Mengen aus A sowie Bn := A1 + . . . + An , n ≥ 1.
5 20  Dann gilt wegen der endlichen Additivität
P(X = k) =
k 5−k
25 , k = 0, 1, . . . , 5,
n

5
P(Bn ) = P(Aj ),
j =1
und somit auf drei Nachkommastellen genau
und die Stetigkeit von unten liefert
⎛ ⎞
P(X = 0) = 0.292, P(X = 1) = 0.328, P(X = 2) = 0.215. ∞

lim P(Bn ) = P ⎝ Aj ⎠ .
Es folgt a) P(X = 2) = 0.215 und b) P(X ≥ 2) = 1−P(X ≤ 1) n→∞
j =1
= 1 − 0.292 − 0.328 = 0.380.
Es gilt also ⎛ ⎞



Beweisaufgaben P⎝ Aj ⎠ = P(Aj ),
j =1 j =1
Aufgabe 19.29
 •• Setzt man T = N, so folgt  ∈ A.
Ist B = was zu zeigen war.
n∈T n ∈ A, so stellt sich mit 
A S := N \ T
das Komplement von B in der Form B c = n∈S An dar.
Aufgabe 19.33 ••• Der Induktionsanfang n = 2 ist mit
Also gilt B c ∈ A. Sind B1 , B2 , . . . ∈ A paarweise disjunkt,
(19.17) erbracht, und die Beweisidee für den Induktions-
so gibt ⊆ N mit
es paarweise disjunkte Mengen T1 , T2 , . . . 
schluss von n auf n + 1 ist die Gleiche wie bei der Herleitung
Bj = n∈Tj An ∈ A für jedes j ≥ 1. Mit T := ∞ j =1 Tj von (19.18) aus (19.17): Sind A1 , . . . , An+1 Ereignisse, so
gilt dann
setzen wir kurz Bn := A1 ∪. . .∪An und erhalten mit (19.17)


⎛ ⎞
Bj = An = An 4
n+1
 
P⎝ Aj ⎠ = P Bn ∪ An+1
j =1 j =1 n∈Tj n∈T
j =1
∞  
und somit j =1 Bj ∈ A. = P(Bn ) + P(An+1 ) − P Bn ∩ An+1 .
Man mache sich klar, dass A nur dann gleich der vollen Po- Hier stehen mit Bn = A1 ∪ . . . ∪ An und Bn ∩ An+1 =
; n
tenzmenge von  ist, wenn jedes Aj einelementig (und somit j =1 (Aj ∩An+1 ) Vereinigungen von jeweils n Ereignissen,
 insbesondere abzählbar) ist. sodass wir zweimal die Induktionsvoraussetzung anwenden
können. Mit Sr wie in (19.19) und
 
Aufgabe 19.30 • a) ergibt sich, indem man für ω ∈  S̃r := P Ai1 ∩ . . . ∩ Air ∩ An+1
die Fälle ω ∈ A ∩ B und ω ∈ / A ∩ B unterscheidet. Im 1≤i1 <...<ir ≤n
ersten Fall sind beide Seiten der zu beweisenden Identität ergibt sich
gleich Eins, im zweiten gleich Null. Zum Nachweis von b) ⎛ ⎞
betrachtet man wegen  = AB c +AB+BAc +Ac B c die vier 4
n+1
n
P⎝ Aj ⎠ = (−1)r−1 · Sr + P(An+1 )
Fälle ω ∈ AB c , ω ∈ AB, ω ∈ BAc und ω ∈ Ac B c . In jedem
j =1 r=1
der ersten drei Fälle nehmen beide Seiten der zu zeigenden
Gleichheit den Wert Eins an, im letzten Fall den Wert Null.
n
− (−1)r−1 · S̃r
Gleichung c) folgt wegen A ∩ B = ∅ und 1∅ ≡ 0 aus b).
r=1
Gleichung d) folgt aus c) mit B = Ac , da 1 ≡ 1. Gilt A ⊆
B, so folgt für jedes ω ∈  durch Unterscheiden der beiden = S1 + P(An+1 ) + (−1)n · S̃n
Fälle ω ∈ A und ω ∈ Ac die Ungleichung 1A (ω) ≤ 1B (ω). n

Gilt umgekehrt die letzte Ungleichung für jedes ω ∈ , so + (−1)r−1 (Sr + S̃r−1 ).
gilt sie insbesondere für jedes ω ∈ A, was A ⊆ B und somit r=2

e) beweist. Es
n+1bleibt zu zeigen, dass die rechte Seite die Gestalt
r−1 S
r=1 (−1) n+1,r mit
Aufgabe 19.31 • Wir verwenden, dass P nach Fol-
gerung h) auf Seite 714 stetig von unten ist und setzen Sn+1,r = P(Ai1 ∩ . . . ∩ Air )
1≤i1 <...<ir ≤n+1
Bn := Acn , n ≥ 1, sowie B := Ac . Aus An ↓ A folgt
dann Bn ↑ B und somit nach Teil c) und Teil h) der oben annimmt. Zerlegt man diese Summe danach, ob ir ≤ n oder
zitierten Folgerung ir = n + 1 gilt, so folgt
Sn+1,1 = S1 + P(An+1 ),
1 − P(A) = P(B) = lim P(Bn ) = lim (1 − P(An ))
n→∞ n→∞ Sn+1,r = Sr + S̃r−1 , 2 ≤ r ≤ n,
= 1 − lim P(An ).
n→∞ Sn+1,n+1 = S̃n (= P(A1 A2 . . . An+1 )),
Hieraus folgt die Behauptung. was zu zeigen war.
Lösungswege zu Kapitel 19 145

Aufgabe 19.34 •• Im Fall einer geraden Anzahl 2s von Vorgehensweise liefert


Runs gibt es jeweils s a-Runs und s b-Runs, die sich abwech-
|Ai ∩ Aj | = r · (r − 1) · (r + s − 2)n−2
seln.
m−1
Um die Länge der s a-Runs festzulegen, gibt es genau
s−1 Möglichkeiten, denn man muss hierzu nur s − 1 der
m−1 Lücken zwischen den in eine Reihe gelegt gedachten m und somit
a’s auswählen
 und dort Trennstriche anbringen. In gleicher |Ai ∩ Aj | r(r − 1)
Weise ist n−1 P(Ai ∩ Aj ) = = .
s−1 die Anzahl der Möglichkeiten, die Längen (r + s)n (r + s)(r + s − 1)
der s b-Runs festzulegen. Der Faktor 2 im Zähler rührt da-
her, dass der erste Run ein a-Run oder auch ein b-Run sein
kann. Der Fall einer ungeraden Anzahl von Runs folgt ohne Aufgabe 19.37 ••• a): Mit der Ungleichung log t ≤ t−1,
neue Überlegung. Hier hat man entweder s + 1 b-Runs und t > 0, folgt
s a-Runs oder s b-Runs und s + 1 a-Runs.

k−1  
k−1
j j (k − 1)k
log 1 − ≤ − =− ,
Aufgabe 19.35 •• Die Formel des Ein- und Ausschlie- n n 2n
j =1 j =1
ßens gilt offenbar auch für die Funktion M → |M|, die end-
lichen Mengen deren Elementanzahl zuordnet, denn diese sodass Darstellung (19.40) die Behauptung liefert.
Funktion tritt ja in Laplace-Modellen bei der Definition der
Wahrscheinlichkeit gemäß P(A) = |A|/|| auf. Im Folgen- b): Wir verwenden jetzt die nach Ersetzen von t durch 1/t
den nehmen wir o.B.d.A. den Fall M1 = {1, 2, . . . , k} und in obiger Logarithmus-Ungleichung folgende Abschätzung
M2 = {1, 2, . . . , n} an. Bezeichnet Aj die Menge derjenigen log t ≥ 1 − 1/t, t > 0. Hiermit ergibt sich
Funktionen f : M1 → M2 , die das Element j nicht anneh-  

k−1 k−1 

men, für die also f (i)  = j , i ∈ M1 , gilt, so ist die Menge j
log 1 − ≥ − 1 − (1 − j/n)−1
der nicht surjektiven Abbildungen gleich ∪nj=1 Aj . Mit n
j =1 j =1

1
k−1
Sr := |Ai1 ∩ . . . ∩ Air | j
= −
1≤i1 <...<ir ≤n n 1 − j/n
j =1

1
gilt nach der Formel des Ein- und Ausschließens k−1
j
≥ −
4n
 n n 1 − (k − 1)/n
j =1
 Aj  = (−1)r−1 Sr .
(k − 1)k
j =1 r=1 = −
2n(n − k + 1)
Offenbar gilt A1 ∩. . .∩An = ∅, denn irgendeinen Wert muss
und daraus b).
ja eine Funktion von M1 nach M2 annehmen. Für r < n ist
Ai1∩. . .∩Air die Menge derjenigen Funktionen f : M1 → M2 , c): Zu vorgegebenem t > 0 existiert für jede genügend große
die jeden der r Werte i1 , . . . , ir auslässt. Für jedes i ∈ M1 √
Zahl n eine natürliche Zahl kn mit kn ≤ nt ≤ kn + 1, und
gibt es also nur n − r mögliche Funktionswerte. Nach der es folgt
Anzahlformel für Permutationen mit Wiederholung gilt dann

|Ai1 ∩ . . . ∩ Air | = (n − r)k , und es folgt P(Xn ≤ kn ) ≤ P(Xn ≤ nt) ≤ P(Xn ≤ kn + 1).

 
4n
 n−1 r−1 n
Wegen
 
Aj = (−1) (n − r)k .
j =1
r (kn − 1)kn (kn − 1)kn
r=1 lim = lim = t2
n→∞ 2n n→∞ 2(n − kn + 1)
Die Anzahl der surjektiven Abbildungen ist somit
liefern die in a) und b) erhaltenen Schranken beim Grenz-

n−1  
n−1   übergang n → ∞
r−1 n r n
nk − (−1) k
(n − r) = (−1) (n − r)k .  
r r √ t2
r=1 r=0
lim sup P(Xn ≤ nt) ≤ exp − ,
n→∞ 2
 
Aufgabe 19.36 •• Analog zur Herleitung von |Aj | in √ t2
lim inf P(Xn ≤ nt) ≥ exp − ,
(19.42) besetzen wir zur Bestimmung von |Ai ∩Aj | zuerst die n→∞ 2
i-te, danach die j -te Stelle und danach die restlichen Stellen
des Tupels (a1 , . . . , an ) (z.B. von links nach rechts). Diese was zu zeigen war.
Kapitel 20 Kirschen werden an zwei Maschinen entkernt. Maschine A
liefert 70% dieser Kirschen, wobei 8% der von A gelieferten
Kirschen den Kern noch enthalten. Maschine B produziert
Aufgaben 30% der benötigten Kirschen, wobei 5% der von B geliefer-
ten Kirschen den Kern noch enthalten. Bei einer abschließen-
den Gewichtskontrolle werden 95% der Pralinen, in denen
Verständnisfragen ein Kirschkern enthalten ist, aussortiert, aber auch 2% der
Aufgabe 20.1 •• (3-Kasten-Problem von Joseph Ber- Pralinen ohne Kern.
trand (1822–1900)) a) Modellieren Sie diesen mehrstufigen Vorgang geeignet.
Drei Kästen haben je zwei Schubladen. In jeder Schublade Wie groß ist die Wahrscheinlichkeit, dass eine Praline mit
liegt eine Münze, und zwar in Kasten 1 je eine Gold- und Kirschkern in den Verkauf gelangt?
in Kasten 2 je eine Silbermünze. In Kasten 3 befindet sich b) Ein Kunde kauft eine Packung mit 100 Pralinen. Wie groß
in einer Schublade eine Gold- und in der anderen eine Sil- ist die Wahrscheinlichkeit, dass nur gute Pralinen, also
bermünze. Es wird rein zufällig ein Kasten und danach aufs Pralinen ohne Kirschkern, in der Packung sind?
Geratewohl eine Schublade gewählt, in der sich eine Gold-
münze befinde. Mit welcher bedingten Wahrscheinlichkeit ist Aufgabe 20.7 •• Ein homogenes Glücksrad mit den
dann auch in der anderen Schublade des gewählten Kastens Ziffern 1, 2, 3 wird gedreht. Tritt das Ergebnis 1 auf, so wird
eine Goldmünze? das Rad noch zweimal gedreht, andernfalls noch einmal.

Aufgabe 20.2 •• Es seien A, B und C Ereignisse in a) Modellieren Sie diesen zweistufigen Vorgang.
einem Wahrscheinlichkeitsraum (, A, P). b) Das Ergebnis im zweiten Teilexperiment sei die Ziffer
bzw. die Summe der Ziffern. Mit welcher Wahrschein-
a) A und B sowie A und C seien stochastisch unabhängig.
lichkeit tritt das Ergebnis j auf, j = 1, . . . , 6?
Zeigen Sie an einem Beispiel, dass nicht unbedingt auch
A und B ∩ C unabhängig sein müssen. c) Mit welcher Wahrscheinlichkeit ergab die erste Drehung
b) A und B sowie B und C seien stochastisch unabhän- eine 1, wenn beim zweiten Teilexperiment das Ergebnis
gig. Zeigen Sie anhand eines Beispiels, dass A und C 3 auftritt?
nicht notwendig unabhängig sein müssen. Der Unabhän-
gigkeitsbegriff ist also nicht transitiv ! Aufgabe 20.8 •• Beim Skatspiel werden 32 Karten rein
zufällig an drei Spieler 1, 2 und 3 verteilt, wobei jeder 10
Aufgabe 20.3 • Es bezeichne Xn , n ≥ 1, die Anzahl Karten erhält; zwei Karten werden verdeckt als Skat auf den
roter Kugeln nach dem n-ten Zug im Pólya’schen Urnenmo- Tisch gelegt. Spieler 1 gewinnt das Reizen, nimmt den Skat
dell von Seite 738 mit c > 0. Zeigen Sie: Mit der Festsetzung auf und will mit Karo-Buben und Herz-Buben einen Grand
X0 := r ist (Xn )n≥0 eine nicht homogene Markov-Kette. spielen. Mit welcher Wahrscheinlichkeit besitzt
a) jeder der Gegenspieler einen Buben?
Aufgabe 20.4 • Es sei (Xn )n≥0 eine Markov-Kette mit
Zustandsraum S. Ein Zustand i ∈ S heißt wesentlich, falls b) jeder der Gegenspieler einen Buben, wenn Spieler 1 bei
gilt: Spieler 2 den Kreuz-Buben (aber sonst keine weitere
∀j ∈ S : i → j ⇒ j → i. Karte) sieht?
Andernfalls heißt i unwesentlich. Ein wesentlicher Zustand c) jeder der Gegenspieler einen Buben, wenn Spieler 1 bei
führt also nur zu Zuständen, die mit ihm kommunizieren. Spieler 2 einen (schwarzen) Buben erspäht (er ist sich
Zeigen Sie: Jede Kommunikationsklasse hat entweder nur jedoch völlig unschlüssig, ob es sich um den Pik-Buben
wesentliche oder nur unwesentliche Zustände. oder den Kreuz-Buben handelt)?

Aufgabe 20.9 • Zeigen Sie, dass im Beispiel von La-


Rechenaufgaben place auf Seite 742 die A-posteriori-Wahrscheinlichkeiten
Aufgabe 20.5 • Zeigen Sie, dass für eine Zufallsva- P(Ak |B) für jede Wahl von A-priori-Wahrscheinlichkeiten
riable X mit der in (20.13) definierten Pólya-Verteilung P(Aj ) für n → ∞ gegen die gleichen Werte null (für k ≤ 2)
Pol(n, r, s, c) gilt: und eins (für k = 3) konvergieren.

s r
lim Pc (X = 0) = , lim Pc (X = n) = . Aufgabe 20.10 •• Drei-Türen-Problem, Ziegenpro-
c→∞ r +s c→∞ r +s blem
Dabei haben wir die betrachtete Abhängigkeit der Verteilung In der Spielshow Let’s make a deal! befindet sich hinter einer
von c durch einen Index hervorgehoben. von drei rein zufällig ausgewählten Türen ein Auto, hinter
den beiden anderen jeweils eine Ziege. Ein Kandidat wählt
Aufgabe 20.6 •• Eine Schokoladenfabrik stellt Prali- eine der Türen aufs Geratewohl aus; diese bleibt aber vorerst
nen her, die jeweils eine Kirsche enthalten. Die benötigten verschlossen. Der Spielleiter öffnet daraufhin eine der beiden
M. Brokate et al., Arbeitsbuch Grundwissen Mathematikstudium – Höhere Analysis, Numerik und
Stochastik, DOI 10.1007/978-3-642-54946-5_19, © Springer-Verlag Berlin Heidelberg 2016
Aufgaben zu Kapitel 20 147

anderen Türen, und es zeigt sich eine Ziege. Der Kandidat Aufgabe 20.15 • Eine Urne enthalte eine rote und eine
kann nun bei seiner ursprünglichen Wahl bleiben oder die schwarze Kugel. Es wird rein zufällig eine Kugel gezogen.
andere verschlossene Tür wählen. Er erhält dann den Preis Ist diese rot, ist das Experiment beendet. Andernfalls werden
hinter der von ihm zuletzt gewählten Tür. Mit welcher Wahr- die schwarze Kugel sowie eine weitere schwarze Kugel in
scheinlichkeit gewinnt der Kandidat bei einem Wechsel zur die Urne gelegt und der Urneninhalt gut gemischt. Dieser
verbleibenden verschlossenen Tür das Auto, wenn wir unter- Vorgang wird so lange wiederholt, bis die (eine) rote Kugel
stellen, dass gezogen wird. Die Zufallsvariable X bezeichne die Anzahl
der dazu benötigten Züge. Zeigen Sie:
a) der Spielleiter weiß, hinter welcher Tür das Auto steht,
diese Tür nicht öffnen darf und für den Fall, dass er 1
P(X = k) = , k ≥ 1.
eine Wahlmöglichkeit hat, mit gleicher Wahrscheinlich- k(k + 1)
keit eine der beiden verbleibenden Türen wählt?
b) der Spielleiter aufs Geratewohl eine der beiden verblei- Aufgabe 20.16 •• In der auf Seite 742 ff. geschilderten
benden Türen öffnet, und zwar auch auf die Gefahr hin, Situation habe sich eine Person r-mal einem ELISA-Test un-
dass das Auto offenbart wird? terzogen. Wir nehmen an, dass die einzelnen Testergebnisse
– unabhängig davon, ob eine Infektion vorliegt oder nicht –
als stochastisch unabhängige Ereignisse angesehen werden
Aufgabe 20.11 •• Eine Mutter zweier Kinder sagt:
können. Zeigen Sie: Die bedingte Wahrscheinlichkeit, dass
a) „Mindestens eines meiner beiden Kinder ist ein Junge.“ die Person infiziert ist, wenn alle r Tests positiv ausfallen, ist
b) „Das älteste meiner beiden Kinder ist ein Junge.“ in Verallgemeinerung von (20.26) durch
q · pse
r
Wie schätzen Sie jeweils die Chance ein, dass auch das andere
Kind ein Junge ist? q · pse
r + (1 − q) · (1 − p )r
sp
gegeben. Was ergibt sich speziell für q = 0.0001, pse =
Aufgabe 20.12 • 95% der in einer Radarstation ein- 0.999, psp = 0.998 und r = 1, 2, 3?
treffenden Signale sind mit einer Störung überlagerte Nutzsi-
gnale, und 5% sind reine Störungen. Wird ein gestörtes Nutz- Aufgabe 20.17 • Von einem regulären Tetraeder seien
signal empfangen, so zeigt die Anlage mit Wahrscheinlich- drei der vier Flächen mit jeweils einer der Farben 1, 2 und
keit 0.98 die Ankunft eines Nutzsignals an. Beim Empfang 3 gefärbt; auf der vierten Fläche sei jede dieser drei Farben
einer reinen Störung wird mit Wahrscheinlichkeit 0.1 fälsch- sichtbar. Es sei Aj das Ereignis, dass nach einem Wurf des
licherweise ein Nutzsignals angezeigt. Mit welcher Wahr- Tetraeders die unten liegende Seite die Farbe j enthält (j =
scheinlichkeit ist ein als Nutzsignal angezeigtes Signal wirk- 1, 2, 3). Zeigen Sie:
lich ein (störungsüberlagertes) Nutzsignal? a) Je zwei der Ereignisse A1 , A2 und A3 sind unabhängig.
b) A1 , A2 , A3 sind nicht unabhängig.
Aufgabe 20.13 •• Es bezeichne ak ∈ {m, j } das Ge-
schlecht des k-jüngsten Kindes in einer Familie mit n ≥ 2 Aufgabe 20.18 •• Es sei (, P (), P) ein La-
Kindern (j = Junge, m = Mädchen, k = 1, . . . , n). P sei die place’scher Wahrscheinlichkeitsraum mit
Gleichverteilung auf der Menge  = {m, j }n aller Tupel
a) || = 6 (echter Würfel),
(a1 , . . . , an ). Weiter sei
b) || = 7.
A = {(a1 , . . . , an ) ∈  | |{a1 , . . . , an } ∩ {j, m}| = 2} Wie viele Paare (A, B) unabhängiger Ereignisse mit 0 <
= {„die Familie hat Kinder beiderlei Geschlechts“} , P(A) ≤ P(B) < 1 gibt es jeweils?
B = {(a1 , . . . , an ) ∈  | |{j : 1 ≤ j ≤ n, aj = m}| ≤ 1}
Aufgabe 20.19 • Ein kompliziertes technisches Gerät
= {„die Familie hat höchstens ein Mädchen“} . bestehe aus n Einzelteilen, die innerhalb eines festen Zeit-
raumes unabhängig voneinander mit derselben Wahrschein-
Beweisen oder widerlegen Sie: A und B sind stochastisch lichkeit p ausfallen. Das Gerät ist nur funktionstüchtig, wenn
unabhängig ⇐⇒ n = 3. jedes Einzelteil funktionstüchtig ist.

Aufgabe 20.14 •• Zwei Spieler A und B drehen in un- a) Welche Ausfallwahrscheinlichkeit besitzt das Gerät?
abhängiger Folge abwechselnd ein Glücksrad mit den Sek- b) Durch Parallelschaltung identischer Bauelemente zu je-
toren A und B. Das Glücksrad bleibt mit Wahrscheinlichkeit dem der n Einzelteile soll die Ausfallsicherheit erhöht
p im Sektor A stehen. Gewonnen hat derjenige Spieler, wel- werden. Bei Ausfall eines Bauelements übernimmt dann
cher als Erster erreicht, dass das Glücksrad in seinem Sektor eines der noch funktionierenden Parallel-Elemente des-
stehen bleibt. Spieler A beginnt. Zeigen Sie: sen Aufgabe. Zeigen Sie: Ist jedes Einzelteil k-fach par-
√ allel geschaltet, und sind alle Ausfälle voneinander unab-
Gilt p = (3 − 5)/2 ≈ 0.382, so ist das Spiel fair, d. h., hängig, so ist die Ausfallwahrscheinlichkeit des Gerätes
beide Spieler haben die gleiche Gewinnchance. gleich 1 − (1 − pk )n .
148 Aufgaben zu Kapitel 20

c) Welche Ausfallwahrscheinlichkeiten ergeben sich für das Ereignis, im j -ten Zug eine rote Kugel zu erhalten (j =
n = 200, p = 0.0015 und die Fälle k = 1, k = 2 1, . . . , n). Zeigen Sie: Für jedes k = 1, . . . , n und jede Wahl
und k = 3? von i1 , . . . , ik mit 1 ≤ i1 < . . . < ik ≤ n gilt

Aufgabe 20.20 • Zeigen Sie durch Nachweis der 


k−1
r + jc
Markov-Eigenschaft, dass Partialsummen unabhängiger Z- P(Ai1 ∩ . . . ∩ Aik ) = P(A1 ∩ . . . ∩ Ak ) = .
r + s + jc
j =0
wertiger Zufallsvariablen (siehe Seite 755) eine Markov-
Kette bilden.
Aufgabe 20.27 • Es seien (, A, P) ein Wahrschein-
Aufgabe 20.21 • Es seien Y0 , Y1 , . . . unabhängige und lichkeitsraum und A, B ∈ A. Beweisen oder widerlegen Sie:
je Bin(1, p) verteilte Zufallsvariablen, wobei 0 < p < 1. Die
Folge (Xn )n≥0 sei rekursiv durch Xn := 2Yn + Yn+1 , n ≥ 0, a) A und ∅ sowie A und  sind unabhängig.
definiert. Zeigen Sie, dass (Xn ) eine Markov-Kette bildet, b) A und A sind genau dann stochastisch unabhängig, wenn
und bestimmen Sie die Übergangsmatrix. gilt: P(A) ∈ {0, 1}.
c) Gilt A ⊆ B, so sind A und B genau dann unabhängig,
Aufgabe 20.22 •• Es sei X0 , X1 , . . . eine Markov-
wenn P(B) = 1 gilt.
Kette mit Zustandsraum S. Zeigen Sie, dass für alle k, m, n
mit 0 ≤ k < m < n und alle h, j ∈ S die d) A ∩ B = ∅ ⇒ A und B sind stochastisch unabhängig.
sogenannteChapman-Kolmogorov-Gleichung e) Es gelte 0 < P(B) < 1 und A ∩ B = ∅. Dann folgt:
P(Xn = j |Xk = h) P(AC |B) = P(A|B C ) ⇐⇒ P(A) + P(B) = 1.

= P(Xm = i|Xk = h) · P(Xn = j |Xm = i) Aufgabe 20.28 •• Es sei  := Pnn = {(a1 , . . . , an ) |
i∈S 1 ≤ aj ≤ n, j = 1, . . . , n; ai = aj für i = j } die Menge
gilt. der Permutationen der Zahlen 1, . . . , n. Für k = 1, . . . , n
bezeichne
Aufgabe 20.23 • Leiten Sie im Fall des Bediensy-
stems mit drei Zuständen auf Seite 756 die invariante Vertei- Ak := {(a1 , . . . , an ) ∈  | ak = max(a1 , . . . , ak )}
lung α = (α0 , α1 , α2 ) her. Warum sind die Voraussetzungen
des Ergodensatzes erfüllt? das Ereignis, dass an der Stelle k ein „Rekord“ auftritt. Zeigen
Sie: Unter einem Laplace-Modell gilt:
Aufgabe 20.24 •• Beim diskreten Diffusionsmodell
a) P(Aj ) = 1/j, j = 1, . . . , n.
von Bernoulli-Laplace für den Fluss zweier inkompressi-
bler Flüssigkeiten befinden sich in zwei Behältern A und b) A1 , . . . , An sind stochastisch unabhängig.
B jeweils m Kugeln. Von den insgesamt 2m Kugeln seien
m weiß und m schwarz. Das System sei im Zustand j , Aufgabe 20.29 ••• Es sei  := {ω = (a1 , . . . , an ) |
j ∈ S := {0, 1, . . . , m}, wenn sich im Behälter A genau aj ∈ {0, 1} für 1 ≤ j ≤ n} = {0, 1}n , n ≥ 3, und p :  →
j weiße Kugeln befinden. Aus jedem Behälter wird unab- [0, 1] durch

⎨2−n+1 , falls  aj ungerade
hängig voneinander je eine Kugel rein zufällig entnommen ⎪ n
und in den jeweils anderen Behälter gelegt. Dieser Vorgang
p(ω) := j =1
wird in unabhängiger Folge wiederholt. Die Zufallsvariable ⎪
⎩ 0 sonst
Xn beschreibe den Zustand des Systems nach n solchen Zie-
hungsvorgängen, n ≥ 0. Leiten Sie die Übergangsmatrix der definiert. Ferner sei
Markov-Kette (Xn )n≥0 her und zeigen Sie, dass die invari-
ante Verteilung eine hypergeometrische Verteilung ist. Aj := {(a1 , . . . , an ) ∈  | aj = 1} , 1 ≤ j ≤ n.

Zeigen Sie:
Beweisaufgaben 
a) Durch P(A) := ω∈A p(ω), A ⊆ , wird ein Wahr-
Aufgabe 20.25 •• Es seien (, A, P) ein Wahrschein- scheinlichkeitsmaß auf P () definiert.
lichkeitsraum und C1 , C2 , . . . endlich oder abzählbar-
b) Je n − 1 der Ereignisse A1 , . . . , An sind unabhängig.
unendlich viele paarweise disjunkte
 Ereignisse mit positiver
Wahrscheinlichkeit sowie C := j ≥1 Cj . Besitzt A ∈ A c) A1 , . . . , An sind nicht unabhängig.
die Eigenschaft, dass P(A|Cj ) nicht von j abhängt, so gilt
Aufgabe 20.30 •• Es seien A1 , . . . , An Ereignisse in
P(A|C) = P(A|C1 ) . einem Wahrscheinlichkeitsraum (, A, P). Zeigen Sie, dass
A1 , . . . , An genau dann unabhängig sind, wenn die Indika-
Aufgabe 20.26 •• Im Pólya’schen Urnenmodell von torfunktionen 1{A1 }, . . . , 1{An } unabhängig sind.
Seite 738 sei
Aufgabe 20.31 •• Beweisen Sie die Identitäten in
Aj := {(a1 , . . . , an ) ∈  | aj = 1} (20.38).
Hinweise zu Kapitel 20 149

Aufgabe 20.32 ••• Es sei (, A, P) ein diskreter Wahr- Rechenaufgaben


scheinlichkeitsraum. Weiter sei A1 , A2 , . . . ∈ A eine Folge
unabhängiger Ereignisse mit pn := P(An ), n ≥ 1. Zeigen Aufgabe 20.5 • –
Sie:
∞ Aufgabe 20.6 •• Sehen Sie die obigen Prozentzahlen
min(pn , 1 − pn ) < ∞ . als Wahrscheinlichkeiten an.
n=1
Aufgabe 20.7 •• –
Aufgabe 20.33 •• Es seien An , n ≥ 1, Ereignisse in
einem Wahrscheinlichkeitsraum (, A, P). Zeigen Sie: Aufgabe 20.8 •• –
a) lim sup Acn = (lim inf An )c ,
n→∞ n→∞ Aufgabe 20.9 • –
b) lim inf Acn = (lim sup An )c ,
n→∞ n→∞ Aufgabe 20.10 •• Aus Symmetriegründen kann ange-
c) lim sup An \ lim inf An = lim sup(An ∩ Acn+1 ). nommen werden, dass der Kandidat Tür Nr. 1 wählt.
n→∞ n→∞ n→∞

Aufgabe 20.34 •• Es seien An , Bn , n ≥ 1, Ereignisse Aufgabe 20.11 •• Nehmen Sie an, dass die Geschlech-
in einem Wahrscheinlichkeitsraum (, A, P). Zeigen Sie: ter der Kinder stochastisch unabhängig voneinander und
Mädchen- sowie Jungengeburten gleichwahrscheinlich sind.
a) lim sup An ∩ lim sup Bn ⊇ lim sup(An ∩ Bn ),
n→∞ n→∞ n→∞
b) lim sup An ∪ lim sup Bn = lim sup(An ∪ Bn ), Aufgabe 20.12 • Interpretieren Sie die Prozentzahlen
n→∞ n→∞ n→∞ als Wahrscheinlichkeiten.
c) lim inf An ∩ lim inf Bn = lim inf (An ∩ Bn ),
n→∞ n→∞ n→∞
d) lim inf An ∪ lim inf Bn ⊆ lim inf (An ∪ Bn ). Aufgabe 20.13 •• –
n→∞ n→∞ n→∞
Geben Sie Beispiele für strikte Inklusion in a) und d) an. Aufgabe 20.14 •• –

Aufgabe 20.35 •• Es seien X1 , X2 , . . . stochastisch un- Aufgabe 20.15 • –


abhängige Zufallsvariablen auf einem Wahrscheinlichkeits-
raum (, A, P) mit P(Xj = 1) = p und P(Xj = 0) = 1−p, Aufgabe 20.16 •• –
j ≥ 1, wobei 0 < p < 1. Zu vorgegebenem r ∈ N und
(a1 , . . . , ar ) ∈ {0, 1}r sei Ak das Ereignis Aufgabe 20.17 • –
<
r
Ak := {Xk+l−1 = al } , k ≥ 1. Aufgabe 20.18 •• –
l=1
Aufgabe 20.19 • –
Zeigen Sie: P(lim supk→∞ Ak ) = 1.
Aufgabe 20.20 • –
Aufgabe 20.36 •• Es seien A ⊆ N und 1 der größte
gemeinsame Teiler von A. Für m, n ∈ A gelte m + n ∈ A. Aufgabe 20.21 • Yn und Yn+1 sind durch Xn be-
Zeigen Sie: Es gibt ein n0 ∈ N, sodass n ∈ A für jedes stimmt.
n ≥ n0 .
Aufgabe 20.22 •• Beachten Sie die verallgemeinerte
Markov-Eigenschaft.
Hinweise Aufgabe 20.23 • –
2m m m2
Verständnisfragen Aufgabe 20.24 •• Es ist m = k=0 k .
Aufgabe 20.1 •• 2/3.
Beweisaufgaben
Aufgabe 20.2 •• Für Teil a) kann man Aufgabe 20.17
verwenden. Aufgabe 20.25 •• –

Aufgabe 20.3 • – Aufgabe 20.26 •• –

Aufgabe 20.4 • – Aufgabe 20.27 • –


150 Lösungen zu Kapitel 20

Aufgabe 20.28 •• – Aufgabe 20.12 • –

Aufgabe 20.29 ••• – Aufgabe 20.13 •• –

Aufgabe 20.30 •• Wie sieht σ (1{Aj }) aus? Aufgabe 20.14 •• –

Aufgabe 20.31 •• Für A1 ∈ A1 , . . . , Al ∈ Al gilt Aufgabe 20.15 • –

Z1−1 (A1 × . . . × Al ) = ∩lj =1 Xj−1 (Aj ). (20.11) Aufgabe 20.16 •• –

Aufgabe 20.17 • –
Aufgabe 20.32 ••• –
Aufgabe 20.18 •• –
Aufgabe 20.33 •• –
Aufgabe 20.19 • –
Aufgabe 20.34 •• –
Aufgabe 20.20 • –
Aufgabe 20.35 •• Es reicht, die Aussage für eine Teil-
folge von (Ak ) zu zeigen. Aufgabe 20.21 • –

Aufgabe 20.36 •• Da 1 größter gemeinsamer Teiler von Aufgabe 20.22 •• –


A ist, gibt es ein k ∈ N und a1 , . . . , ak ∈ A sowie
n1 , . . . , nk ∈ Z mit 1 = kj =1 nj aj . Fasst man die positiven Aufgabe 20.23 •
und negativen Summanden zusammen, so gilt 1 = P − N 1 u v
mit P , N ∈ A, und n0 := (N + 1)(N − 1) leistet das Ver- α0 = , α1 = , α2 = ,
1+u+v 1+u+v 1+u+v
langte. Stellen Sie n ≥ n0 in der Form n = qN + r mit
0 ≤ r ≤ N − 1 dar. Es gilt dann q ≥ N − 1. wobei
p p 2 (1 − q)
u= , v= .
q(1 − p) q 2 (1 − p)

Lösungen Aufgabe 20.24 •• Die invariante Verteilung ist die hy-


pergeometrische Verteilung Hyp(m, m, m).
Verständnisfragen
Beweisaufgaben
Aufgabe 20.1 •• –
Aufgabe 20.25 •• –
Aufgabe 20.2 •• –
Aufgabe 20.26 •• –
Aufgabe 20.3 • –
Aufgabe 20.27 • –
Aufgabe 20.4 • –
Aufgabe 20.28 •• –

Rechenaufgaben Aufgabe 20.29 ••• –


Aufgabe 20.5 • – Aufgabe 20.30 •• –
Aufgabe 20.6 •• – Aufgabe 20.31 •• –

Aufgabe 20.7 •• – Aufgabe 20.32 ••• –

Aufgabe 20.8 •• a) 10/19, b) 10/19, c) 20/29. Aufgabe 20.33 •• –

Aufgabe 20.9 • – Aufgabe 20.34 •• –

Aufgabe 20.10 •• a) 2/3. b) 1/2. Aufgabe 20.35 •• –

Aufgabe 20.11 •• – Aufgabe 20.36 •• –


Lösungswege zu Kapitel 20 151

Lösungswege Rechenaufgaben
Aufgabe 20.5 • Nach (20.13) gilt
Verständnisfragen
Aufgabe 20.1 •• In einem zweistufigen Verfahren wird s  s + jc
n−1
Pc (X = 0) = ·
jede der sechs Schubladen mit gleicher Wahrscheinlichkeit r +s r + s + jc
j =1
1/6 ausgewählt. Wird eine Goldmünze gefunden, so handelt
es sich entweder um die Goldmünze in der einen Schublade
von Kasten 1 oder um die Goldmünze in der anderen Schub- sowie
lade von Kasten 1 oder um die (eine) Goldmünze in Kasten r  r + jc
n−1
3. In zwei der drei Fälle ist in der anderen Schublade eine Pc (X = n) = ·
r +s r + s + jc
Goldmünze. Die Wahrscheinlichkeit ist also 2/3. j =1

Mit (s +j c)/(r +s +j c) → 1 und (s +j c)/(r +s +j c) → 1


Aufgabe 20.2 ••
bei c → ∞ für jedes j ≥ 1 folgt die Behauptung.
a) Es kann A = A1 , B = A2 und C = A3 mit den Ereignis-
sen A1 , A2 und A3 aus Aufgabe 20.17 gesetzt werden.
Aufgabe 20.6 ••
b) Ein möglicher Wahrscheinlichkeitsraum ist  :=
{1, 2, 3, 4} mit der Gleichverteilung P auf . Mit A := a) Wir modellieren die Situation durch ein dreistufiges Ex-
{1, 2}, B := {2, 3} und C := {3, 4} folgt periment mit dem Grundraum  := 1 × 2 × 3 sowie
der Festsetzung
P(A) = P(B) = P(C) = 1/2
P({(ω1 , ω2 , ω3 )}) := p1 (ω1 )·p2 (ω1 |ω2 )·p3 (ω1 , ω2 |ω3 )
und
P(A ∩ B) = P(B ∩ C) = 1/4, für (ω1 , ω2 , ω3 ) ∈ . Dabei werden die drei Stufen wie
sodass A und B sowie B und C unabhängig sind. We- folgt beschrieben:
gen P(A ∩ C) = P(∅) = 0 sind jedoch A und C nicht 1 := {A, B} (Kirsche von Maschine A bzw. B) und
unabhängig. p1 (A) := 0.7, p1 (B) := 0.3.
2 := {mK, oK} (Kirsche mit Kern, Kirsche ohne
Aufgabe 20.3 • Aufgrund der Ziehungsvorschrift ist
Kern) und
die Zusammensetzung der Urne nach dem n + 1-ten Zug
nur durch das Mischungsverhältnis nach dem n-ten Zug und
die Farbe der im n + 1-ten Zug entnommenen Kugel be- p2 (A|mK) := 0.08, p2 (A|oK) := 0.92,
stimmt. Die Folge X0 , X1 , . . . ist also nach Konstruktion eine p2 (B|mK) := 0.05, p2 (B|oK) := 0.95.
Markov-Kette. Unter der Bedingung Xn = k enthält die Urne
nach dem n-ten Zug r + s + nc Kugeln, von denen k rot und 3 := {V , nV } (Praline im Verkauf, Praline nicht im
r + s + nc − k schwarz sind. Es gilt somit Verkauf) und
k
P(Xn+1 = k + 1|Xn = k) = , p3 (ω1 , mK|V ) := 0.05, p3 (ω1 , mK|nV ) := 0.95,
r + s + nc
r + s + nc − k p3 (ω1 , oK|V ) := 0.98, p3 (ω1 , oK|nV ) := 0.02.
P(Xn+1 = k|Xn = k) =
r + s + nc

und P(Xn+1 = j |Xn = k) = 0, falls j ∈ / {k, k + 1}. Die Man beachte, dass p3 nicht von ω1 abhängt.
Übergangswahrscheinlichkeiten hängen also vom Zeitpunkt Sei C := {(A, mK, V ), (B, mK, V )} das Ereignis, dass
n ab, sodass keine Homogenität vorliegt. eine Praline mit Kirschkern in den Verkauf gelangt. Dann
ist
Aufgabe 20.4 • Es seien i, j ∈ S mit i → j . Der
Zustand i sei wesentlich. Wir behaupten, dass auch j we- P(C) = 0.7 · 0.08 · 0.05 + 0.3 · 0.05 · 0.05 = 0.00355.
sentlich ist. Zum Beweis betrachten wir ein beliebiges k ∈ S
mit j → k. Wegen i → j gilt dann mit der Transitivität der
Mit 0.355-prozentiger Wahrscheinlichkeit gelangt also
Relation → auch i → k. Da i wesentlich ist, folgt k → i und
eine Praline mit Kirschkern in den Verkauf.
somit wegen i → j auch k → j , was zeigt, dass j wesentlich
ist. Mit mindestens einem wesentlichen Zustand sind somit b) Wir müssen zunächst berechnen, mit welcher Wahr-
alle Zustände einer Kommunikationsklasse wesentlich, was scheinlichkeit eine Praline ohne Kirschkern in den Ver-
die Behauptung liefert. kauf gelangt, d. h. die Wahrscheinlichkeit des Ereig-
152 Lösungswege zu Kapitel 20

nisses Wegen P(Bj |Ai ) = p2 (i|j ) folgt mit a)


D := {(A, oK, V ), (B, oK, V )}.

3
6
P(B1 ) = p1 (i)p2 (i|1) = ,
Diese ist gegeben durch 27
i=1

3
7
P(D) = 0.7 · 0.92 · 0.98 + 0.03 · 0.95 · 0.98 = 0.91042. P(B2 ) = p1 (i)p2 (i|2) = ,
27
i=1

Folglich ist die Wahrscheinlichkeit, dass eine in den Ver-


3
8
kauf gelangte Praline keinen Kirschkern enthält, gleich P(B3 ) = p1 (i)p2 (i|3) = ,
27
i=1

P(oK ∩ V ) 1
P(oK|V ) = P(B4 ) = p1 (1)p2 (1|4) = ,
P(V ) 9
2
P(oK ∩ V ) P(B5 ) = p1 (1)p2 (1|5) = ,
= 27
P(mK ∩ V ) + P(oK ∩ V )
1
P(D) P(B6 ) = p1 (1)p2 (1|6) = .
= 27
P(C) + P(D)
91042 c) Es ist
= ≈ 0.9961
91397
P(A1 ∩ B3 ) P(A1 )P(B3 |A1 )
P(A1 |B3 ) = =
Damit ergibt sich die Wahrscheinlichkeit p, dass in einer P(B3 ) P(B3 )
Packung mit 100 Pralinen nur gute sind, zu 1/3 · 2/9 1
= = .
8/27 4
 100
91042
p = ≈ 0.6776. Aufgabe 20.8 ••
91397
a) Da Spieler
 1 seine Karten und den Skat kennt, sind für
ihn alle 20
10 Kartenverteilungen (Möglichkeiten, aus 20
Aufgabe 20.7 •• Karten 10 für Spieler 2 auszuwählen) gleichwahrschein-
  20
a) Als Grundraum kann  := {(i, j ) | 1 ≤ i ≤ 3, 1 ≤ lich. Die gesuchte Wahrscheinlichkeit ist 2 · 18
9 / 10 =
10
j ≤ 6} gewählt werden. Dabei bezeichne i das Ergeb- 19 ≈ 0.526. Der Faktor 2 rührt daher, dass wir festlegen
nis der ersten Drehung und j das Ergebnis des zweiten müssen, wer den Kreuz-Buben erhält.
Teilexperiments. Da das Glücksrad homogen ist, wäh- b) Unter der gegebenen Information sind für Spieler 1 alle
len wir als Startwahrscheinlichkeiten p1 (i) := 1/3 für 19
9 Möglichkeiten, Spieler 2 noch 9 Karten zu geben,
i = 1, 2, 3. Im Fall i = 1 wird das Rad noch zweimal gleichwahrscheinlich. Die Wahrscheinlichkeit,
  19 dass Spie-
gedreht. Damit ergeben sich die Übergangswahrschein- ler 2 den Pik-Buben nicht erhält, ist 189 / 9 = 10
19 . Der
lichkeiten p2 (1|1) = 0, p2 (1|2) = 1/9, p2 (1|3) = 2/9, Vergleich mit a) zeigt, dass die gegebene Information die
p2 (1|4) = 3/9, p2 (1|5) = 2/9 und p2 (1|6) = 1/9. Da Aussicht auf verteilt sitzende Buben nicht verändert hat.
für i = 2 und i = 3 nur eine weitere Drehung erfolgt, c) Wir müssen genau modellieren, auf welche Weise die er-
gilt p2 (i|j ) = 1/3 für i ∈ {1, 2} und j ∈ {1, 2, 3} so- haltene Information zu uns gelangt. Kann Spieler 1 über-
wie p2 (i|j ) = 0 für i ∈ {1, 2} und j ∈ {4, 5, 6}. Das haupt nur die ganz links in der Hand von Spieler 2 befind-
WahrscheinlichkeitsmaßP auf (der Potenzmenge von) liche Karte sehen, so ergibt sich die gleiche Antwort wie in
 ist durch P(A) := (i,j )∈A p(i, j ) mit p(i, j ) := a) und b), wenn wir unterstellen, dass Spieler 2 seine Kar-
p1 (i)p2 (i|j ) definiert. ten in der aufgenommenen rein zufälligen Reihenfolge in
b) Es sei Bj := {(1, j ), (2, j ), (3, j )} das Ereignis, dass das der Hand hält (bitte nachrechnen nen Buben grundsätz-
zweite Teilexperiment den Wert j ergibt (j = 1, 2, 3). lich auf der linken Seite seiner Hand einsortiert, so ist
Weiter sei Ai := {(i, j ) | 1 ≤ j ≤ 6} das Ereignis, dass die gegebene Information gleichwertig damit, dass Spie-
die erste Drehung den Wert i liefert. Nach der Formel von ler 2 mindestens einen der beiden schwarzen Buben erhält
der totalen Wahrscheinlichkeit gilt (Ereignis
20 B). Bezeichnet P die Gleichverteilung auf allen
10 möglichen Kartenverteilungen der Gegenspieler, so
18 20

3 gilt P(B) = 1 − P(B c ) = 1 − 10 / 10 = 29/38 und
P(Bj ) = P(Ai ) · P(Bj |Ai ) somit P (Spieler 2 erhält genau einen schwarzen Buben
  20
i=1 |B) = (2 · 189 / 10 )/(29/38) = 20/29 = 0.689 . . . Die
1 gegenüber b) „schwächere Information“ hat somit unter
3
= · P(Bj |Ai ). der gemachten Annahme die Aussicht auf verteilte Buben
3
i=1 erheblich vergrößert.
Lösungswege zu Kapitel 20 153

Aufgabe 20.9 • Die Behauptung ergibt sich unmittel- Im Gegensatz zu den in a) gemachten Annahmen ist es also
bar aus der für k ∈ {0, 1, 2, 3} geltenden Darstellung jetzt für den Kandidaten egal, ob er bei seiner ursprüngli-
 n chen Wahl bleibt oder zur verbleibenden verschlossenen Tür
P(Ak ) k3 wechselt.
P(Ak |B) =  n  n .
P(A1 ) 13 + P(A2 ) 23 + P(A3 ) · 1
Aufgabe 20.11 •• Als Grundraum kann die Menge
Aufgabe 20.10 ••  := {(m, m), (w, m), (m, w), (m, m)} mit der Gleichver-
teilung P auf  gewählt werden. Dabei gibt die erste bzw.
a) Es bezeichne Aj das Ereignis, dass sich das Auto hinter
zweite Komponente das Geschlecht des älteren bzw. jünge-
Tür Nr. j befindet und Bj das Ereignis, dass der Spiel-
ren Kindes an.
leiter Tür Nr. j öffnet (j = 1, 2, 3). Da das Auto rein
zufällig platziert wird, gilt P(Aj ) = 1/3 (j = 1, 2, 3). a) Es können sich verschiedene Situationen abgespielt ha-
Aufgrund unserer Annahmen, dass der Kandidat Tür Nr. ben, wie es zu der gemachten Aussage kam. Wenn die
1 wählt, ergeben sich aus der Aufgabenstellung die be- Frau gebeten wurde, den Satz „Mindestens eines meiner
dingten Wahrscheinlichkeiten Kinder ist ein Junge“ zu sagen, falls er zutrifft, so ist die
P(B1 |A1 ) = 0, P(B2 |A1 ) = P(B3 |A1 ) = 1/2, gesuchte bedingte Wahrscheinlichkeit gleich
P(B1 |A2 ) = 0, P(B2 |A2 ) = 0, P(B3 |A2 ) = 1,
P({(m, m)}) 1
P(B1 |A3 ) = 0, P(B2 |A3 ) = 1, P(B3 |A3 ) = 0. = .
P({(m, m), (m, w), (w, m)}) 3
Nehmen wir an, dass der Spielleiter Tür Nr. 2 öffnet, so
folgt nach der Bayes-Formel Wenn die Frau jedoch gebeten wurde, den Satz „Minde-
P(A3 ∩ B2 ) P(A3 )P(B2 |A3 ) stens eines meiner beiden Kinder ist ...“ mit „ein Junge“
P(A3 |B2 ) = = 3
P(B2 ) j =1 P(Aj )P(B2 |Aj )
oder „ein Mädchen“ zu ergänzen und bei unterschied-
lichen Geschlechtern der Kinder aufs Geratewohl eine
1 2
= = . dieser beiden Ergänzungen gibt, liegt das untenstehende
1/2 + 0 + 1 3 Baumdiagramm für ein zweistufiges Experiment vor, bei
Der Kandidat gewinnt also das Auto mit der Wahrschein- dem in der ersten Stufe die Gleichverteilung auf allen vier
lichkeit 2/3, wenn er zur verbleibenden verschlossenen Geschlechterkombinationen erzeugt wird. Liegen zwei
Tür wechselt. Das gleiche Ergebnis erhält man, wenn der Jungen bzw. zwei Mädchen vor, so gibt die Mutter jeweils
Moderator Tür 3 öffnet, denn es gilt auch P(A2 |B3 ) = mit Wahrscheinlichkeit eins die einzig mögliche Ergän-
2/3 zung „ein Junge“ bzw. „ein Mädchen“. Im Fall verschie-
b) Die Ereignisse Ai , Bj seien wie in a) definiert. Wie oben dener Geschlechter wählt sie rein zufällig eine der beiden
gilt P(Aj ) = 1/3 (j = 1, 2, 3). Im Gegensatz zu a) öff- möglichen Ergänzungen aus.
net jetzt der Moderator aufs Geratewohl eine der beiden
Türen 2 oder 3. Wir haben es also daraufhin mit den be-
dingten Wahrscheinlichkeiten
P(B1 |A1 ) = 0, P(B2 |A1 ) = P(B3 |A1 ) = 1/2,
P(B1 |A2 ) = 0, P(B2 |A2 ) = P(B3 |A2 ) = 1/2,
P(B1 |A3 ) = 0, P(B2 |A3 ) = P(B3 |A3 ) = 1/2.
zu tun (beachten Sie, dass die vom Kandidaten gewählte Tür
Nr. 1 nicht geöffnet werden darf). Nehmen wir an, der Spiel-
leiter öffnet Tür 3, und es zeigt sich eine Ziege. Die Bedin-
gung ist also jetzt B3 ∩ Ac3 . Wegen A2 ⊆ Ac3 folgt
P(A2 ∩ B3 ∩ Ac3 )
P(A2 |B3 ∩ Ac3 ) =
P(B3 ∩ Ac3 )
Abbildung 20.1 Baumdiagramm zum Zwei-Jungen-Problem.
P(A2 ∩ B3 )
=
P(B3 ) − P(B3 ∩ A3 )
Die von der Mutter gegebene Ergänzung findet sich im
P(A2 )P(B3 |A2 )
= 3 Baumdiagramm am Ende der Pfeile der zweiten Stufe.
j =1 P(Aj)P(B3 |Aj )− P(A3)P(B3 |A3 ) Bezeichnet A das Ereignis, dass die Mutter die Ergänzung
1/2 „ein Junge“ gibt, so folgt wegen
=
1/2 + 1/2
1 1 1 1 1 1 1
= . P(A) = ·1+ · + · =
2 4 4 2 4 2 2
154 Lösungswege zu Kapitel 20

P({m, m} ∩ A) ben und danach die rote Kugel gezogen wird. Aufgrund des
P({(m, m)}|A) =
P(A) Ziehungsmodus ist die Wahrscheinlichkeit hierfür nach der
P({(m, m)})P(A|{(m, m)}) ersten Pfadregel
=
1/2
1 2 k−1 1 1
1/4 · 1 1 P(X = k) = · · ... · · = .
= = , 2 3 k k+1 k(k + 1)
1/2 2
also eine andere Antwort.
Aufgabe 20.16 •• Es bezeichne N das Ereignis, dass
b) Die Antwort ist P ({mm}|{wm, mm}) = 1/2. Man be- mindestens einer der r Tests negativ ausfällt. Weiter sei K
achte, dass die Geschlechter der beiden Kinder stocha- das Ereignis, dass die Person krank ist und q := P(K) die
stisch unabhängig voneinander sind. A-priori-Wahrscheinlichkeit für K. Die vorausgesetzte Un-
abhängigkeit bedeutet P(N c |K) = pse r und P(N|K c ) =
Aufgabe 20.12 • Es sei A das Ereignis, dass ein ein-
1 − (1 − psp ) . Die Bayes-Formel liefert dann
r
treffendes Signal ein störungsüberlagertes Nutzsignal dar-
stellt, und B bezeichne das Ereignis, dass ein eintreffendes q · pse
r
Signal eine Störung ist. Weiter stehe C für das Ereignis, P(K|N c ) = .
q · pse
r + (1 − q) · (1 − p )r
sp
dass ein Nutzsignal angezeigt wird. Interpretieren wir die in
der Aufgabenstellung angegebenen Prozentzahlen als Wahr- Im Fall q = 0.0001, pse = 0.999, psp = 0.998 nimmt
scheinlichkeiten, so gilt nach Voraussetzung P(A) = 0.95, diese Wahrscheinlichkeit für r = 1, 2, 3 die Werte 0.0476 (!),
P(B) = 0.05, P(C|A) = 0.98, P(C|B) = 0.1. Gesucht ist 0.9615 (!) und 0.99992 (!) an.
P(A|C). Es gilt
Aufgabe 20.17 •
P(A ∩ C) P(C|A)P(A)
P(A|C) = = a) Da jede Farbe auf genau zwei der vier Flächen vertreten
P(C) P(C|A)P(A) + P(C|B)P(B)
= 0.9946 . . . ist, gilt P(A1 ) = P(A2 ) = P(A3 ) = 1/2. Weiter gilt
P(A1 A2 ) = P(A1 A3 ) = P(A2 A3 ) = 1/4, denn nur eine
Fläche enthält zwei der drei Farben. Folglich sind die drei
Aufgabe 20.13 •• Das komplementäre Ereignis Ac be- Ereignisse paarweise unabhängig.
deutet, dass die Familie nur Jungen oder nur Mädchen hat.
b) Wegen 1/4 = P(A1 A2 A3 ) = P(A1 ) · P(A2 ) · P(A3 ) sind
Wegen der Gleichverteilungsannahme folgt P(Ac ) = 2 ·
A1 , A2 , A3 nicht unabhängig.
(1/2)n und somit P(A) = 1 − 2 · (1/2)n . Das Ereignis B
setzt sich zusammen aus den beiden Fällen, dass kein Mäd- Aufgabe 20.18 •• In einem Laplace-Raum der Ord-
chen oder genau ein Mädchen vorhanden ist. Es gilt somit nung n ist die Unabhängigkeit von A und B zu
P(B) = (1/2)n + n · (1/2)n . Das Ereignis A ∩ B ist gleich-
bedeutend damit, dass die Familie genau ein Mädchen hat. n · |A ∩ B| = |A| · |B|
Es gilt also P(A ∩ B) = n · (1/2)n und somit
 n  2n  2n äquivalent, und nach Voraussetzung gilt 1 ≤ |A| ≤ |B| ≤
1 1 1 n − 1.
P(A ∩ B) = P(A)P(B) ⇔ −2 − 2n =0
2 2 2
a) Es gibt 360 solcher Paare (A, B) , und zwar je 180 mit
⇔ 2n − 2 − 2n = 0 |A| = 2, |B| = 3, |A ∩ B| = 1 und 180 mit |A| = 3,
⇔ 2n−1 = n + 1 |B| = 4, |A ∩ B| = 2.
⇔ n = 3. b) Es kann kein solches Paar geben, denn die Gleichung
n|A ∩ B| = |A| · |B| mit 1 ≤ |A| ≤ |B| ≤ n − 1 ist nicht
erfüllbar, wenn n eine Primzahl ist.
Aufgabe 20.14 •• Spieler A gewinnt genau dann, wenn
entweder beim ersten Versuch A auftritt oder für ein k ∈ N
Aufgabe 20.19 •
k-mal die Sequenz BA und danach A. Mit q := 1 − p folgt
somit wegen der stochastischen Unabhängigkeit der Ergeb- a) Die Wahrscheinlichkeit, dass alle Bauteile funktionieren,
nisse der einzelnen Drehungen ist (1 − p)n . Die Ausfallwahrscheinlichkeit ist somit 1 −
∞ (1 − p)n .
p
P(A gewinnt) = p (qp)k = . b) Geht man zum komplementären Ereignis über und nutzt
1 − qp
k=0 die Unabhängigkeit aus, so ergibt sich die Wahrschein-
Dieser Ausdruck ist genau dann 1/2, wenn p Lösung der lichkeit, dass ein Bauteil intakt ist, zu 1 − p k . Die Wahr-
quadratischen Gleichung p 2 − 3p + 1 = 0 ist. Die √ einzige scheinlichkeit, dass alle Bauteile intakt sind, ist somit we-
Lösung dieser Gleichung im Intervall [0, 1] ist (3 − 5)/2. gen der vorausgesetzten Unabhängigkeit gleich (1−pk )n .
Komplementbildung liefert jetzt die Behauptung.
Aufgabe 20.15 • Es gilt X = k genau dann, wenn die c) Es ergeben sich die Werte 0.2593 . . . für k = 1,
ersten k − 1 Ziehungen jeweils eine schwarze Kugel erge- 0.0004498 . . . für k = 2 und 0.0000006749 . . . für k = 3.
Lösungswege zu Kapitel 20 155

Aufgabe 20.20 • Wegen Xn = Y0 + . . . + Yn gilt Aufgabe 20.22 •• Die σ -Additivität von P und die De-
Y0 = X0 und Yk = Xk − Xk−1 für k ≥ 1. Es folgt finition der bedingten Wahrscheinlichkeit liefern

P(Xn+1 = in+1 |Xn = in , . . . , X0 = i0 ) P(Xk = h|Xn = j )



P(Xn+1 = in+1 , Xn = in , . . . , X0 = i0 ) = P(Xk = h, Xm = i, Xn = j )
=
P(Xn = in , . . . , X0 = i0 ) i∈S

P(Yn+1 = in+1 − in , Yn = in − in−1 , . . . , Y0 = i0 ) = P(Xk = h, Xm = i)P(Xn = j |Xk = h, Xm = j ).
=
P(Yn = in − in−1 , . . . , Y0 = i0 ) i∈S

= P(Yn+1 = in+1 − in ). Nach der verallgemeinerten Markov-Eigenschaft kann im


letzten Faktor die Bedingung Xk = h weggelassen werden.
Dabei gilt das letzte Gleichheitszeichen aufgrund der Unab- Dividiert man dann durch P(Xk = h), so folgt die Behaup-
hängigkeit der Yj . Wegen tung.

P(Xn+1 = in+1 |Xn = in ) Aufgabe 20.23 • Die Voraussetzungen des Ergoden-


P(Xn+1 = in+1 , Xn = in ) satzes sind erfüllt, da alle 2-Schritt-Übergangswahrschein-
= lichkeiten positiv sind. Mit der Übergangsmatrix
P(Xn = in )
P(Xn+1 − Xn = in+1 − in , Xn = in ) ⎛ ⎞
= 1−p p 0
P(Xn = in ) P = ⎝q(1 − p) 1 − q(1 − p) − p(1 − q) p(1 − q)⎠
= P(Yn+1 = in+1 − in |Xn = in ) 0 q 1−q
= P(Yn+1 = in+1 − in )
nehmen die Gleichungen (20.49) die Gestalt
folgt die Behauptung. Hier ergibt sich das letzte Gleichheits- α0 = (1 − p)α0 + q(1 − p)α1
zeichen aus der Unabhängigkeit von Yn+1 und Xn , denn letz-
α1 = pα0 + (1 − q(1 − p) − p(1 − q))α1 + qα2
tere Zufallsvariable ist eine Funktion von Y0 , . . . , Yn .
α2 = p(1 − q)α1 + (1 − q)α2
Aufgabe 20.21 • Der Zustandsraum für Xn ist S = an. Aus der ersten und dritten Gleichung folgt
{0, 1, 2, 3}. Es gilt
pα0 p(1 − q)α1
α1 = = u α0 , α2 = = v α0
Xn = 3 ⇐⇒ Yn = 1, Yn+1 = 1, q(1 − p) q
Xn = 2 ⇐⇒ Yn = 1, Yn+1 = 0, und somit wegen α0 + α1 + α2 = 1
Xn = 1 ⇐⇒ Yn = 0, Yn+1 = 1,
α0 · (1 + u + v) = 1.
Xn = 0 ⇐⇒ Yn = 0, Yn+1 = 0
Hieraus ergibt sich unmittelbar die Behauptung.
und somit
Aufgabe 20.24 •• Sind 0 bzw. m weiße Kugeln in Be-
Yn+1 = 1{Xn ist ungerade}, hälter A, so befinden sich nach dem nächsten Ziehungsvor-
Yn = 1{Xn ≥ 2}. gang eine bzw. m − 1 weiße Kugeln in diesem Behälter. Es
gilt also
Folglich erhält man p0,1 = pm,m−1 = 1.
Es sei im Folgenden 1 ≤ j ≤ m − 1. Sind j weiße Kugeln
Xn+1 = 2 · Yn+1 + Yn+2 in Behälter A und damit m − j weiße Kugeln in Behälter B,
= 2 · 1{Xn ist ungerade} + Yn+2 . so befinden sich nach dem nächsten Ziehungsvorgang j − 1
weiße Kugeln in A, wenn aus A eine weiße und aus B eine
Da (X1 , . . . , Xn−1 ) von Yn+2 stochastisch unabhängig ist, schwarze Kugeln gezogen werden. Die Wahrscheinlichkeit
ist die Markov-Eigenschaft erfüllt. Für die Übergangswahr- hierfür ist (j/m)2 . In gleicher Weise ist ((m − j )/m)2 die
scheinlichkeiten pij ergibt sich Wahrscheinlichkeit, dass das System vom Zustand j in den
Zustand j + 1 übergeht. Das System bleibt im Zustand j ,
p01 = p21 = p, wenn aus den Behältern gleichfarbige Kugeln gezogen wer-
p00 = p20 = 1 − p, den, was mit Wahrscheinlichkeit 2j (m − j )/m2 geschieht.
Wir erhalten also für j ∈ {1, . . . , m−1} die Übergangswahr-
p13 = p33 = p,
scheinlichkeiten
p12 = p32 = 1 − p.  2    
j m−j 2 2j (m − j ) 2
pj,j −1 = , pj,j +1 = , pjj = .
Alle anderen Übergangswahrscheinlichkeiten sind gleich 0. m m m2
156 Lösungswege zu Kapitel 20

Aufgrund der Ziehungs- und Umverteilungsvorschrift gilt so wird durch die Festsetzung
ferner pij = 0, falls |i − j | > 1. Somit ist die Übergangs-
f ((a1 , . . . , an )) := (1, . . . , 1, b1 , . . . , bn−k )
matrix eine Tridiagonalmatrix. Wegen
eine bijektive Abbildung f : C → D definiert. Diese trans-

k−1
pj,j +1   m − j 2
k−1  2
m portiert die an den Stellen i1 , . . . , ik stehenden Einsen im
= =
pj +1,j j +1 k Tupel (a1 , . . . , an ) auf die ersten k Plätze und sortiert die üb-
j =0 j =0
rigen Komponenten des Tupels entsprechend ihrer ursprüng-
und lichen Reihenfolge ab der k + 1-ten Stelle des Tupels ein.
k−1
m−1  m  2
 2 Da die Abbildung f die Anzahl der Einsen des Tupels nicht
pj,j +1 m 2m ändert, folgt
1+ = =
k=0 j =0
pj +1,j k m
P({ω }) =
k=0
P(D) = P({f (ω)})
folgt aus (20.50) – mit m anstelle von s und der Änderung, ω ∈D ω∈C

dass k und j ab null und nicht ab 1 laufen – = P({ω}) = P(C).
m2 ω∈C
k
αk = 2m , k = 0, 1, . . . , m.
m Aufgabe 20.27 •
Die invariante Verteilung ist also die hypergeometrische Ver- a) Wegen A∩∅ = ∅ und P(∅) = 0 sind A und ∅ unabhängig.
teilung Hyp(n, r, s) mit n = r = s = m. Diese entsteht, In gleicher Weise sind wegen A ∩  = A und P() = 1
wenn man aus 2m Kugeln, von denen m weiß und m schwarz die Ereignisse A und  unabhängig.
sind, rein zufällig ohne Zurücklegen m Kugeln zieht und in b) A und A sind unabhängig genau dann, wenn P(A ∩ A) =
Behälter A legt, wobei die anderen m Kugeln in Behälter B P(A)2 und somit P(A) = P(A)2 gilt. Diese Gleichung
gelangen. Füllt man die Behälter zu Beginn auf diese Weise, besitzt nur die Lösungen P(A) = 0 und P(A) = 1.
so startet die Markov-Kette mit dieser in der Physik auch als c) Im Fall A ⊆ B gilt A ∩ B = A. Die Unabhängigkeit von
Gleichgewichtsverteilung bezeichneten Verteilung. A und B ist dann also zu P(A) = P(A) · P(B) äquivalent,
sodass aus P(B) = 1 die Unabhängigkeit von A und B
Beweisaufgaben folgt. Die Umkehrung muss nicht notwendig gelten, da
P(A) = 0 möglich ist.
Aufgabe 20.25 •• Es sei PC die bedingte Verteilung
b) A und B sind nur dann unabhängig, wenn P(A) = 0 oder
von P unter der Bedingung C, also PC (B) := P(B|C), B ∈ A.
P(B) = 0 gilt.
Wegen PC (C) = 1 liefert die Formel von der totalen Wahr-
scheinlichkeit e) Aus der Voraussetzung folgt
P(Ac ∩ B) P(B)− P(A ∩ B)
PC (A) = PC (Cj ) · PC (A|Cj ). P(Ac |B) = = = 1,
P(B) P(B)
j ≥1
P(A ∩ B c ) P(A)− P(A ∩ B) P(A)
P(A|B c ) = = = .
Mit P(B c ) 1 − P(B) 1− P(B)
P(A ∩ C ∩ Cj ) P(A ∩ Cj ) Beide Ausdrücke sind genau dann gleich, wenn P(A) +
PC (A|Cj ) = =
P(C ∩ Cj ) P(Cj ) P(B) = 1 gilt.
= P(A|Cj ) = P(A|C1 ),
 Aufgabe 20.28 ••
j ≥ 1, sowie j ≥1 PC (Cj ) = 1 folgt die Behauptung.
a) Die Lösung sollte zunächst intuitiv klar sein. Die Wahr-
scheinlichkeit, dass die an der k-ten Stelle stehende Zahl
Aufgabe 20.26 •• Nach Gleichung (20.12) hängt die
die größte der ersten k Zahlen ist, ist aus Symmetrie-
Wahrscheinlichkeit P({ω}) eines Tupels ω = (a1 , . . . , an )
gründen gleich 1/k, denn eine dieser Zahlen ist ja die
nur von der Anzahl a1 + . . . + an der Einsen im Tupel ab.
größte. Für einen formalen Beweis müssen wir die An-
Sind i1 , . . . , ik wie in der Aufgabenstellung, so sei
zahl aller (a1 , . . . , an ) ∈  bestimmen, für die ak =
C := {(a1 , . . . , an ) ∈  | ai1 = 1, . . . , aik = 1} max(a1 , . . . , ak ) ist. Hierzu wählen wir zunächst k der n
Zahlen
n für die ersten k Stellen des Tupels aus, was auf
und Weisen möglich ist. Dann platzieren wir die größte
k
D := {(a1 , . . . , an ) ∈  | a1 = 1, . . . , ak = 1}. dieser Zahlen an die k-te Stelle. Jetzt kann man die übri-
gen k − 1 der ausgewählten Zahlen beliebig auf die ersten
Zu zeigen ist P(C) = P(D), denn es gilt C = Ai1 ∩ . . . ∩ Aik k −1 Plätze des Tupels verteilen und die nicht ausgewähl-
und D = A1 ∩ . . . ∩ Ak . Sind b1 , . . . , bn−k mit 1 ≤ b1 < ten Zahlen auf die Plätze k + 1, . . . , n. Es folgt
. . . < bn−k ≤ n und  n
|Ak | (k − 1)! · (n − k)! 1
P(Ak ) = = k = .
{b1 , . . . , bn−k } = {1, . . . , n} \ {i1 , . . . , ik }, || n! k
Lösungswege zu Kapitel 20 157

b) Wir zeigen zunächst, dass 2 ≤ k ≤ n − 1 und i1 , . . . , ik mit 1 ≤ i1 < . . . < ik ≤ n


beliebig. Zu zeigen ist
1  k
P(Ai ∩ Aj ) = P(Ai ) · P(Aj ) = (20.12)
i·j P(Ai1 ∩ . . . ∩ Aik ) =
1
.
2
für jede Wahl von i, j mit 1 ≤ i < j ≤ n gilt, also
paarweise Unabhängigkeit besteht. Der allgemeine Fall Da die Wahrscheinlichkeit eines Tupels (a1 , . . . , an ) nur
erfordert dann nur einen größeren Schreibaufwand. Auch von der Anzahl seiner Einsen, nicht aber von der Stel-
hier sollte zunächst intuitiv klar sein, warum Ai und Aj lung der Einsen innerhalb des Tupels abhängt, können
unabhängig sind. Erhält man die Information, dass an der wir ohne Beschränkung der Allgemeinheit den Fall i1 =
j -ten Stelle des Tupels die größte der ersten j Zahlen 1, . . . , ik = k annehmen. Ein Tupel (a1 , . . . , an ) der Ge-
steht, so hat diese Information keinen Einfluss darauf, ob stalt (1, . . . , 1, ak+1 , . . . , an ) kann auf 2n−k−1 Weisen
die an der i-ten Stelle stehende Zahl die größte unter den durch Wahl von ak+1 , . . . , an zu einem Tupel mit einer
ersten i Zahlen ist oder nicht. ungeraden Anzahl von Einsen ergänzt werden. Da jedes
solche Tupel die Wahrscheinlichkeit 1/2n−1 besitzt, folgt
Für einen formalen Beweis wählen wir zunächst j der n
Zahlen für die ersten j Plätze und setzen die größte dieser 2n−k−1 1
Zahlen auf Platz j . Von den übrigen j − 1 Zahlen wählen P(A1 ∩ . . . ∩ Ak ) = n−1
= k,
2 2
wir i für die Plätze 1, . . . , i aus und setzen die größte
was zu zeigen war.
dieser Zahlen auf Platz i. Die übrigen Zahlen verteilen wir
beliebig auf die Plätze 1, . . . , i−1 und danach die j −i−1 c) Für ungerades n gilt P(A1 ∩ . . . ∩ An ) =
ursprünglich ausgewählten, aber noch nicht platzierten P({(1, . . . , 1)}) = 2−n+1 , und für gerades n gilt
Zahlen auf die Plätze i + 1, . . . , j . Die restlichen n − j P(A1 ∩ . . . ∩ An ) = 0. Wegen P(Aj ) = 1/2 folgt
9 n −n
Zahlen verteilen wir auf die Plätze j + 1, . . . , n. Es ergibt j =1 P(Aj ) = 2 , sodass A1 , . . . , An nicht unabhän-
sich gig sind.
  
n j −1 Aufgabe 20.30 •• Sind A1 , . . . , An unab-
|Ai Aj | = (i − 1)!(j − i − 1)!(n − j )!
j i hängig, so sind die Mengensysteme M1 :=
n! {∅, A1 , Ac1 , }, . . . , Mn := {∅, An , Acn , } unabhän-
= gig. Wegen σ (1{Aj }) = Mj , j = 1, . . . , n, sind dann
i·j
nach Definition 1{A1 }, . . . , 1{An } unabhängig. Umgekehrt
und somit (20.12). Sind allgemein 1 ≤ i1 < . . . < ik ≤ n folgt aus der Unabhängigkeit der Indikatorfunktionen,
mit k ≥ 2, so gilt dass M1 , . . . , Mn und damit auch die Teilsysteme
{A1 }, . . . , {An } unabhängig sind. Letzteres ist die Unabhän-
|Ai1 ∩ . . . ∩ Aik | gigkeit von A1 , . . . , An .
  k   
n ir − 1 Aufgabe 20.31 ••
= (ir −ir−1 − 1)! (i1 −1)!(n−ik )! Es ist nur
ik
r=2
ir−1 ⎛ ⎞
4l

=
n! σ (Z1 ) = σ ⎝ σ (Xj )⎠
i 1 · i 2 · . . . · ik j =1

und somit zu zeigen. Der Nachweis der zweiten Gleichung erfordert


keine neue Überlegung. Da jede der Mengen Xj−1 (Aj ),

k
1 
k
j = 1, . . . , l auf der rechten Seite von (20.11) (und da-
P(Ai1 ∩ . . . ∩ Aik ) = = P(Air ),
ir mit auch deren Durchschnitt) zu σ (∪lj =1 σ (Xj )) gehört, gilt
r=1 r=1
Z1−1 (H) ⊆ σ (∪lj =1 σ (Xj )), wobei H := {A1 × . . . × Al |
was zu zeigen war. A1 ∈ A1 , . . . , An ∈ An }. Wegen σ (H) = B1 = ⊗lj =1 Aj
gilt dann
Aufgabe 20.29 ••• ⎛ ⎞
4
l
a) Von den insgesamt 2n n-Tupeln ω = (a1 , . . . , an ) ist −1 −1
σ (Z1 ) = Z (B1 ) = σ (Z (H) ⊆ σ ⎝ σ (Xj )⎠ .
1 1
aus Symmetriegründen (betrachten Sie z. B. die „Flip- j =1
Abbildung“ T (ω) := (1 − a1 , . . . , 1 − an )) die Hälfte
„gerade“ (d. h., nj=1
aj ist gerade) und die Hälfte „unge- Für die umgekehrte Inklusion ⊇ reicht es aus, Xj−1 (Aj ) ∈
rade“. Folglich gilt ω∈ p(ω) = 1, sodass P ein Wahr- σ (Z1 ) für jedes j = 1, . . . , l und jedes Aj ∈ Aj zu zeigen.
scheinlichkeitsmaß ist. Dies folgt aber unmittelbar aus der Gleichung
b) Da für jedes i = 1, . . . , n aus Symmetriegründen genau Xj−1 (Aj ) = Z1−1 (1 ×. . .×j −1 ×Aj ×j +1 ×. . .×l ),
die Hälfte der „ungeraden“ n-Tupel an der i-ten Stelle eine
1 besitzt, gilt P(Ai ) = 1/2, 1 ≤ i ≤ n. Es seien k mit denn die rechte Seite gehört zu σ (Z1 ).
158 Lösungswege zu Kapitel 20

Aufgabe 20.32 ••• Es sei ω0 ∈  mit 0 < P({ω0 }) be- Diese Indizes müssen sich also unendlich oft abwechseln,
liebig. Für jedes n ≥ 1 gilt ω0 ∈ Bn , wobei Bn ∈ {An , Acn }. was zur Folge hat, dass es unendlich viele k ∈ N mit
Die Ereignisse B1 , B2 , . . . sind stochastisch unabhängig, und ω ∈ Ak und ω ∈ / Ak+1 geben muss.
folglich gilt für jedes k ≥ 1
Aufgabe 20.34 ••
0 < P({ω0 }) ≤ P(B1 ∩ . . . ∩ Bk )
a) Es ist

k
= P(Bn ) ∞ 4
< ∞
n=1 lim sup(An ∩ Bn ) = (Ak ∩ Bk )
  n→∞

k n=1 k=n
∞ 
= exp log(1 − (1 − P(Bn )) <∞ 4 ∞
4
n=1 ⊆ Ak ∩ Bk
 

k n=1 k=n k=n
≤ exp − (1 − P(Bn )) . <∞ 4∞ ∞ 4
< ∞

n=1 = Ak ∩ Bk
n=1 k=n n=1 k=n
Dabeihaben wir die Ungleichung log t ≤ t − 1 benutzt. Es = lim sup An ∩ lim sup Bn .
folgt ∞n=1 (1 − P(Bn )) <∞. Wegen min(pn , 1 − pn ) ≤ n→∞ n→∞
1 − P(Bn ) muss die Reihe ∞n=1 min(pn , 1 − pn ) nach dem
Majorantenkriterium konvergieren. b) Mit dem Distributivgesetz folgt
∞ 4
< ∞
Aufgabe 20.33 •• lim sup(An ∪ Bn ) = (Ak ∪ Bk )
n→∞
n=1 k=n
a) Nach der de Morgan’schen Regel gilt

∞ ∞

< 4 4
∞ 4
< ∞ = Ak ∪ Bk
lim sup Acn = Ack n=1 k=n k=n
n→∞ ∞ 4∞ ∞ ∞
n=1 k=n
∞ ∞ c < <4
4< = Ak ∪ Bk
= Ak n=1 k=n n=1 k=n
n=1 k=n = lim sup An ∪ lim sup Bn .
n→∞ n→∞
= (lim inf An )c .
n→∞
c) folgt, indem man in b) Acn und Bnc anstelle von An bzw.
b) folgt aus a), indem man An durch Acn ersetzt. Bn einsetzt und Aufgabe a) und b) verwendet.
c) Wegen A \ B = A ∩ B c ergibt sich zunächst mit der de d) ist die komplementäre Aussage zu a). Sie folgt analog zu
Morgan’schen Regel c), wenn man in a) Acn und Bnc anstelle von An bzw. Bn
einsetzt und Aufgabe a) und b) verwendet.
lim sup An \ lim inf An
n→∞ n→∞
∞  ∞ ∞ c Die folgende allgemeine Konstruktion liefert ein Beispiel

< 4 4< für strikte Inklusion in a) und d). Sind C und D dis-
= Ak ∩ Ak junkte Ereignisse mit C = ∅ und D  = ∅, und setzt man
n=1 k=n n=1 k=n
∞  ∞  A2k := C, A2k−1 := D für k ≥ 1 sowie B2k := D und
<∞ 4 ∞
< 4 B2k−1 := C für k ≥ 1, so gilt An ∩ Bn = ∅, n ≥ 1
= Ak ∩ Ack
und somit lim supn→∞ (An ∩ Bn ) = ∅. Andererseits gilt
n=1 k=n n=1 k=n

! ∞  ∞ " lim supn→∞ An = C + D und lim supn→∞ Bn = C + D,
< 4 4
= Ak ∩ Acl was strikte Inklusion in a) bedeutet. Wegen
n=1 k=n
∞ 
l=n
lim inf An = ∅ = lim inf Bn
<∞ 4 n→∞ n→∞
⊇ Ak ∩ Ack und lim inf n→∞ (An ∪ Bn ) = C + D ergibt sich auch ein
n=1 k=n
Beispiel für strikte Inklusion in d).
= lim sup(An ∩ Acn+1 ).
n→∞
Aufgabe 20.35 •• Es sei Bs := A1+(s−1)r , s ≥ 1. Da
Anstelle des Inklusionszeichens gilt aber auch „⊆“, denn Bs nur von X(s−1)r+1 , X(s−1)r+2 , . . . , Xsr abhängt und die

! ∞  ∞ " Indexmengen Is := {(s − 1)r + 1, (s − 1)r + 2, . . . , sr}
< 4 4
ω∈ Ak ∩ Alc für verschiedene s paarweise disjunkt sind, sind die Ereig-
n=1 k=n l=n nisse B1 , B2 , . . . (im Gegensatz zu A1 , A2 , . . .) stochastisch
unabhängig. Wegen der Unabhängigkeit der Xj gilt
bedeutet, dass es sowohl unendlich viele Indizes k mit
ω ∈ Ak als auch unendlich viele Indizes l mit ω ∈ Acl gibt. P(Bs ) ≥ δ := min(p, 1 − p)r > 0, s ≥ 1.
Lösungswege zu Kapitel 20 159


Da die Reihe ∞s=1 P(Bs ) divergiert, liefert Teil b) des Lem- Aufgabe 20.36 •• Unter Verwendung des Hinweises
mas von Borel-Cantelli sei n = qN + r mit 0 ≤ r ≤ N − 1. Nehmen wir an,
es gälte q < N − 1. Dann wäre
P(lim sup Bs ) = 1
s→∞
n = qN + r < N(N − 1) + N − 1 = (N + 1)(N − 1),
und damit wegen lim sups→∞ Bs ⊆ lim supk→∞ Ak auch
P(lim supk→∞ Ak ) = 1. was im Widerspruch zur Voraussetzung n ≥ n0 stünde. Somit
gilt q ≥ N − 1 und folglich q − r ≥ 0. Wegen 1 = P − N
Anmerkung: Steht Aj für das Ereignis, dass mein persönli- ergibt sich
cher Tipp beim Lotto 6 aus 49 in der j -ten Ausspielung einen
Sechser erzielt (die Wahrscheinlichkeit hierfür ist positiv), so n = qN + r(P − N) = (q − r)N + rP ,
besagt obiges Resultat, dass meine Zahlenreihe in einer Folge
von unendlich vielen Ausspielungen mit Wahrscheinlichkeit und aus q − r ≥ 0 und der Abgeschlossenheit von A gegen-
eins 176-mal direkt hintereinander jeweils einen Sechser lan- über der Addition folgt (q − r)N + rP ∈ A, was zu zeigen
det. war.
Kapitel 21 gelten muss, und bestätigen Sie diese Einsicht durch formale
Rechnung. Die bedingte Verteilung von X unter der Bedin-
gung X+Y = k ist also eine Gleichverteilung auf den Werten
Aufgaben 0, 1, . . . , k.

Aufgabe 21.6 •• Stellen Sie sich eine patriarchisch ori-


Verständnisfragen entierte Gesellschaft vor, in der Eltern so lange Kinder be-
kommen, bis der erste Sohn geboren wird. Wir machen zu-
Aufgabe 21.1 •• In der gynäkologischen Abteilung
dem die Annahmen, dass es keine Mehrlingsgeburten gibt,
eines Krankenhauses entbinden in einer bestimmten Wo-
dass Jungen- und Mädchengeburten gleich wahrscheinlich
che n Frauen. Es mögen keine Mehrlingsgeburten auftreten,
sind und dass die Geschlechter der Neugeborenen stocha-
und Jungen- bzw. Mädchengeburten seien gleich wahrschein-
stisch unabhängig voneinander sind.
lich. Außerdem werde angenommen, dass das Geschlecht der
Neugeborenen für alle Geburten stochastisch unabhängig sei. a) Welche Verteilung (Erwartungswert, Varianz) besitzt die
Sei an die Wahrscheinlichkeit, dass mindestens 60% der Neu- Anzahl der Mädchen in einer Familie?
geborenen Mädchen sind. b) Welche Verteilung (Erwartungswert, Varianz) besitzt die
a) Bestimmen Sie a10 . Anzahl der Jungen in einer Familie?
a) Es bezeichne Sn die Gesamtanzahl der Mädchen in einer
b) Beweisen oder widerlegen Sie: a100 < a10 .
aus n Familien bestehenden Gesellschaft. Benennen Sie
c) Zeigen Sie: limn→∞ an = 0. die Verteilung von Sn und zeigen Sie:
Aufgabe 21.2 •• Es werden unabhängig voneinander √ 1
P(|Sn − n| ≥ K 2n) ≤ , K > 0.
Kugeln auf n Fächer verteilt, wobei jede Kugel in jedes Fach K2
mit Wahrscheinlichkeit 1/n gelangt. Es sei Wn die (zufäl- Was bedeutet diese Ungleichung für K = 10 und eine aus
lige) Anzahl der Kugeln, die benötigt wird, bis jedes Fach 500 000 Familien bestehenden Gesellschaft?
mindestens eine Kugel enthält. Zeigen Sie:

n Aufgabe 21.7 • In einer Urne befinden sich 10 rote,
1
a) E(Wn ) = n · . 20 blaue, 30 weiße und 40 schwarze Kugeln. Es werden rein
j zufällig 25 Kugeln mit Zurücklegen gezogen. Es sei R (bzw.
j =1
B, W, S) die Anzahl gezogener roter (bzw. blauer, weißer,

n−1
1 1
n−1
b) V(Wn ) = n2 · − n · . schwarzer) Kugeln. Welche Verteilungen besitzen
j2 j
j =1 j =1 a) (R, B, W, S)?
b) (R + B, W, S)?
Aufgabe 21.3 •• Ein echter Würfel wird solange in un-
abhängiger Folge geworfen, bis die erste Sechs auftritt. Wel- c) R + B + W ?
che Verteilung besitzt die Anzahl der davor geworfenen Ein-
sen? Aufgabe 21.8 •• In einer Urne befinden sich r1 + · · · +
rs gleichartige Kugeln, von denen rj die Farbe j tragen. Es
Aufgabe 21.4 ••• Es werden n echte Würfel gleichzeitig werden rein zufällig n Kugeln nacheinander ohne Zurückle-
geworfen. Diejenigen, die eine Sechs zeigen, werden beisei- gen gezogen. Die Zufallsvariable Xj bezeichne die Anzahl
tegelegt, und die (falls noch vorhanden) übrigen Würfel wer- der gezogenen Kugeln der Farbe j , 1 ≤ j ≤ s. Die Verteilung
den wiederum gleichzeitig geworfen und die erzielten Sech- des Zufallsvektors (X1 , . . . , Xs ) heißt mehrdimensionale hy-
sen beiseitegelegt. Der Vorgang wird solange wiederholt, bis pergeometrische Verteilung. Zeigen Sie:
   
auch der letzte Würfel eine Sechs zeigt. Die Zufallsvariable r1 rs
· ... ·
Mn bezeichne die Anzahl der dafür nötigen Würfe. Zeigen k ks
a) P(X1 = k1 , . . . , Xs = ks ) =  1 ,
Sie: r1 + . . . + rs
  k n n
a) P(Mn > k) = 1 − 1 − 56 , k ∈ N0 . 0 ≤ kj ≤ rj , k1 + · · · + ks = n .
n b) Xj ∼ Hyp(n, rj , m − rj ), 1≤j ≤s.
n
k−1 k
b) E(Mn ) = (−1)  k .
k=1 1 − 56 Aufgabe 21.9 •• Die Zufallsvariable X besitze die
auf Seite 725 eingeführte hypergeometrische Verteilung
Aufgabe 21.5 •• Die Zufallsvariablen X und Y seien Hyp(n, r, s), d. h., es gelte
stochastisch unabhängig und je geometrisch verteilt mit Pa-    
r s
rameter p. Überlegen Sie sich ohne Rechnung, dass ·
k n−k
P(X = k) =   , 0 ≤ k ≤ n.
1 r +s
P(X = j |X + Y = k) = , j = 0, 1, . . . , k
k+1 n
M. Brokate et al., Arbeitsbuch Grundwissen Mathematikstudium – Höhere Analysis, Numerik und
Stochastik, DOI 10.1007/978-3-642-54946-5_20, © Springer-Verlag Berlin Heidelberg 2016
Aufgaben zu Kapitel 21 161

Leiten Sie analog zum Fall der Binomialverteilung den Er- Beweisaufgaben
wartungswert
r Aufgabe 21.17 ••• Beim Coupon-Collector-Problem
E(X) = n · oder Sammlerproblem wird einer Urne, die n gleichartige,
r +s
von 1 bis n nummerierte Kugeln enthält, eine rein zufäl-
von X auf zwei unterschiedliche Weisen her. lige Stichprobe von s Kugeln (Ziehen ohne Zurücklegen
bzw. „mit einem Griff“) entnommen. Nach Notierung
Aufgabe 21.10 • Zeigen Sie, dass die auf Seite 717 der gezogenen Kugeln werden diese wieder in die Urne
formulierte Formel des Ein- und Ausschließens aus der Jor- zurückgelegt und der Urneninhalt neu gemischt.
dan’schen Formel auf Seite 777 folgt.
Die Zufallsvariable X bezeichne die Anzahl der verschiede-
nen Kugeln, welche in den ersten k (in unabhängiger Folge
Aufgabe 21.11 •• In der Situation des Beispiels auf entnommenen) Stichproben aufgetreten sind. Zeigen Sie:
Seite 795 soll die mittlere quadratische Abweichung E(M −   
h(X1 ))2 durch geeignete Wahl einer Funktion h minimiert s k
a) EX = n · 1 − 1 −
werden. Dabei darf h nur die Werte 1, 2, . . . , 6 annehmen. n
     
r −j = n k
Zeigen Sie: Die unter diesen Bedingungen optimale Funktion r
n j r
h ist durch h(1) ∈ {3, 4}, h(2) = h(3) = 4, h(4) ∈ {4, 5}, b) P(X = r) = (−1)
r j s s
h(5) = 5 und h(6) = 6 gegeben. j =0
(0 ≤ r ≤ n).
Aufgabe 21.12 • Die Zufallsvariablen X und Y seien
stochastisch unabhängig, wobei X ∼ Bin(m, p) und Y ∼ Aufgabe 21.18 •• Es sei X eine N0 -wertige Zufallsva-
Bin(n, p), 0 < p < 1. Zeigen Sie: Für festes k ∈ riable mit EX < ∞ (für a)) und EX 2 < ∞ (für b)). Zeigen
{1, 2, . . . , m + n} ist die bedingte Verteilung von X unter Sie:
der Bedingung X + Y = k die hypergeometrische Verteilung 
Hyp(k, m, n). Ist dieses Ergebnis ohne Rechnung einzuse- a) EX = ∞ n=1 P(X ≥ n).

hen? b) EX 2 = ∞ n=1 (2n − 1)P(X ≥ n).

Aufgabe 21.13 •• Es seien X1 , X2 und X3 unabhängige Aufgabe 21.19 •• Es sei X eine Zufallsvariable mit der
Zufallsvariablen mit identischer Verteilung. Zeigen Sie: Eigenschaft b ≤ X ≤ c, wobei b < c. Zeigen Sie:
1
1 a) V(X) ≤ (c − b)2 .
E(X1 |X1 + X2 + X3 ) = · (X1 + X2 + X3 ) . 4
3 1 1
b) V(X) = (c − b)2 ⇐⇒ P(X = b) = P(X = c) = .
4 2
Aufgabe 21.14 •• Die Zufallsvariable X besitze die Bi- Aufgabe 21.20 •• Es sei X eine Zufallsvariable mit
nomialverteilung Bin(n, p). Zeigen Sie: EX = 0 und EX 2 < ∞. Zeigen Sie:

 0 J n K1 V(X)
1 + (1 − 2p)n P(X ≥ ε) ≤ ε > 0.
P X ∈ 0, 2, . . . , 2 · = . V(X) + ε 2
2 2

Aufgabe 21.21 ••
Aufgabe 21.15 •• Es sei (Mn )n≥0 ein Galton-Watson- a) X1 , . . . , Xn seien Zufallsvariablen mit EXj =: μ und
Prozess wie auf Seite 804 mit M0 = 1, EM1 = μ und V(Xj ) =: σ 2 für j = 1, . . . , n. Weiter existiere eine na-
V(M1 ) = σ 2 < ∞. Zeigen Sie mithilfe von Aufgabe 21.52: türliche Zahl k, sodass für |i −j | ≥ k die Zufallsvariablen
Xi und Xj unkorreliert sind. Zeigen Sie:
a) E(Mn ) = μn ,
⎛  ⎞
 
b) 1 n 
lim P ⎝ Xj − μ ≥ ε⎠ = 0 für jedes ε > 0 .
⎧ 2 n−1 n n→∞ n 
⎨ σ μ (μ − 1) j =1
, falls μ  = 1
V(Mn ) = μ−1

n · σ 2, falls μ = 1. b) Ein echter Würfel werde in unabhängiger Folge geworfen.
Die Zufallsvariable Yj bezeichne die beim j -ten Wurf
erzielte Augenzahl, und für j ≥ 1 sei Aj := {Yj < Yj +1 }.
Aufgabe 21.16 ••• Kann man zwei Würfel (möglicher-
Zeigen Sie mithilfe von Teil a):
weise unterschiedlich) so fälschen, d. h. die Wahrschein- ⎛  ⎞
lichkeiten der einzelnen Augenzahlen festlegen, dass beim  n 
1 5 
gleichzeitigen Werfen jede Augensumme 2, 3, . . . , 12 gleich lim P⎝ 1{Aj } −  ≥ ε⎠ = 0 für jedes ε > 0 .
wahrscheinlich ist?
n→∞  n j =1 12
162 Aufgaben zu Kapitel 21

Aufgabe 21.22 •• Es sei X eine N0 -wertige Zufallsva- sind. Die Zahl 0 besitzt die Farbe Grün. Man kann auf ge-
riable mit 0 < P(X = 0) < 1 und der Eigenschaft wisse Mengen von n Zahlen setzen und erhält dann im Ge-
winnfall in Abhängigkeit von n zusätzlich zum Einsatz das
P(X = m + k|X ≥ k) = P(X = m) (21.13) k(n)-fache des Einsatzes zurück. Die Setzmöglichkeiten mit
den Werten von n und k(n) zeigt die folgende Tabelle:
für jede Wahl von k, m ∈ N0 . Zeigen Sie: Es gibt ein p ∈
(0, 1) mit X ∼ G(p).
n Name k(n)

Aufgabe 21.23 •• Zeigen Sie: In der Situation und mit 1 Plein 35


den Bezeichnungen der Jordan’schen Formel auf Seite 777 2 Cheval 17
gilt 3 Transversale 11


n   4 Carré 8
j −1
P(X ≥ k) = (−1)j −k Sj , k = 0, 1, . . . , n . 6 Transversale simple 5
k−1
j =k 12 Douzaines, Colonnes 2
18 Rouge/Noir, Pair/Impair, Manque/Passe 1
Aufgabe 21.24 •• Wir betrachten wie auf Seite 718 die
Gleichverteilung P auf der Menge Es bezeichne X den Spielgewinn bei Einsatz einer Geldein-
heit. Zeigen Sie: Unabhängig von der gewählten Setzart gilt
 := {(a1 , . . . , an ) | {a1 , . . . , an } = {1, . . . , n}} , EX = −1/37. Man verliert also beim Roulette im Durch-
schnitt pro eingesetztem Euro ungefähr 2,7 Cent.
also eine rein zufällige Permutation der Zahlen 1, 2, . . . , n.
Mit Aj := {(a1 , a2 , . . . , an ) ∈  | aj= j } für j ∈ Aufgabe 21.28 ••• n Personen haben unabhängig von-
{1, . . . , n} gibt die Zufallsvariable Xn := nj=1 1{Aj } die einander und je mit gleicher Wahrscheinlichkeit p eine
Anzahl der Fixpunkte einer solchen Permutation an. Zeigen Krankheit, die durch Blutuntersuchung entdeckt werden
Sie: kann. Dabei sollen von den n Blutproben dieser Personen die
a) E(Xn ) = 1, Proben mit positivem Befund möglichst kostengünstig her-
ausgefunden werden. Statt alle Proben zu untersuchen bietet
1 (−1)j
n−k sich ein Gruppen-screening an, bei dem jeweils das Blut von
b) P(Xn = k) = , k = 0, 1, . . . , n, k Personen vermischt und untersucht wird. In diesem Fall
k! j!
j =0 muss nur bei einem positiven Befund jede Person der Gruppe
e−1 einzeln untersucht werden, sodass insgesamt k+1 Tests nötig
c) limn→∞ P(Xn = k) = , k ∈ N0 ,
k! sind. Andernfalls kommt man mit einem Test für k Personen
aus.
d) V(Xn ) = 1.
Es sei Yk die (zufällige) Anzahl nötiger Blutuntersuchungen
Aufgabe 21.25 ••• In der Situation des Beispiels von bei einer Gruppe von k Personen. Zeigen Sie:
Seite 796 (Warten auf den ersten Doppeltreffer) sei wn :=
a) EYk = k + 1 − k(1 − p)k .
P(X = n), n ≥ 2, gesetzt. Zeigen Sie: √
b) Für p < 1 − 1/ 3 3 = 0.3066 . . . gilt E(Yk ) < k.
a) wk+1 = q · wk + pq · wk−1 , k ≥ 3 , c) Welche Gruppengröße ist im Fall p = 0.01 in Bezug auf
∞
k=2 wk = 1 ,
b) die erwartete Ersparnis pro Person optimal?

∞ d) Begründen Sie die Näherungsformel k ≈ 1/ p für die
c) k=2 k · wk < ∞ (d. h., EX existiert). optimale Gruppengröße bei sehr kleinem p.

Rechenaufgaben Aufgabe 21.29 •• Beim Pokerspiel Texas Hold’em


wird ein 52-Blatt-Kartenspiel gut gemischt; jeder von insge-
Aufgabe 21.26 •• Die Verteilung des Zufallsvektors samt 10 Spielern erhält zu Beginn zwei Karten. Mit welcher
(X, Y ) sei gegeben durch Wahrscheinlichkeit bekommt mindestens ein Spieler zwei
Asse?
P(X = −1, Y = 1) = 1/8 P(X = 0, Y = 1) = 1/8
P(X = 1, Y = −1) = 1/8 P(X = 0, Y = −1) = 1/8 Aufgabe 21.30 •• Es sei X ∼ Bin(n, p) mit 0 < p <
P(X = 2, Y = 0) = 1/4 P(X = −1, Y = 0) = 1/4. 1. Zeigen Sie die Gültigkeit der Rekursionsformel
(n − k)p
Bestimmen Sie: P(X = k + 1) = · P(X = k),
a) EX b) EY c) V(X) d) V(Y ) e) E(XY ). (k + 1)(1 − p)
k = 0, . . . , n − 1,
Aufgabe 21.27 • Beim Roulette gibt es 37 gleich und überlegen Sie sich hiermit, für welchen Wert bzw. welche
wahrscheinliche Zahlen, von denen 18 rot und 18 schwarz Werte von k die Wahrscheinlichkeit P(X = k) maximal wird.
Aufgaben zu Kapitel 21 163

Aufgabe 21.31 •• In Kommunikationssystemen wer- Aufgabe 21.38 • Ein echter Würfel wird 8-mal in un-
den die von der Informationsquelle erzeugten Nachrichten in abhängiger Folge geworfen. Wie groß ist die Wahrschein-
eine Bitfolge umgewandelt, die an den Empfänger übertragen lichkeit, dass jede Augenzahl mindestens einmal auftritt?
werden soll. Um die durch Rauschen und Überlagerung ver-
ursachten Störungen zu unterdrücken und die Zuverlässigkeit Aufgabe 21.39 •• Beim Spiel Kniffel werden fünf Wür-
der Übertragung zu erhöhen, fügt man einer binären Quell- fel gleichzeitig geworfen. Mit welcher Wahrscheinlichkeit
folge kontrolliert Redundanz hinzu. Letztere hilft, Übertra- erhält man
gungsfehler zu erkennen und eventuell sogar zu korrigieren. a) einen Kniffel (5 gleiche Augenzahlen)?
Wir machen die Annahme, dass jedes zu übertragende Bit un-
b) einen Vierling (4 gleiche Augenzahlen)?
abhängig von anderen Bits mit derselben Wahrscheinlichkeit
p in dem Sinne gestört wird, dass 0 in 1 und 1 in 0 umgewan- c) ein Full House (Drilling und Zwilling, also z. B. 55522)?
delt wird. Die zu übertragenden Codewörter mögen jeweils d) einen Drilling ohne weiteren Zwilling (z. B. 33361)?
aus k Bits bestehen. e) zwei Zwillinge (z. B. 55226)?
a) Es werden n Wörter übertragen. Welche Verteilung besitzt f) einen Zwilling (z. B. 44153)?
die Anzahl X der nicht (d. h. in keinem Bit) gestörten g) fünf verschiedene Augenzahlen?
Wörter?
Aufgabe 21.40 •• Der Zufallsvektor (X1 , . . . , Xs ) be-
b) Zur Übertragung werden nur Codewörter verwendet, die
sitze die Multinomialverteilung Mult(n, p1 , . . . , ps ). Leiten
eine Korrektur von bis zu zwei Bitfehlern pro Wort gestat-
Sie aus (21.31) durch Zerlegung des Ereignisses {X1 = k1 }
ten. Wie groß ist die Wahrscheinlichkeit, dass ein über-
nach den Werten der übrigen Zufallsvariablen die Vertei-
tragenes Codewort korrekt auf Empfängerseite ankommt
lungsaussage X1 ∼ Bin(n, p1 ) her.
(evtl. nach Korrektur)? Welche Verteilung besitzt die An-
zahl der richtig erkannten unter n übertragenen Codewör-
Aufgabe 21.41 •• Leiten Sie die Varianz np(1 − p)
tern?
einer Bin(n, p)-verteilten Zufallsvariablen X über die Dar-
stellungsformel her.
Aufgabe 21.32 •• Peter wirft 10-mal in unabhängiger
Folge einen echten Würfel. Immer wenn eine Sechs auftritt, Aufgabe 21.42 •• Es seien X1 , . . . , Xn unabhängige
wirft Claudia eine echte Münze (Zahl/Wappen). Welche Ver- Zufallsvariablen mit gleicher Verteilung und der Eigenschaft
teilung besitzt die Anzahl der dabei erzielten Wappen? EX12 < ∞. Ferner seien μ := EX1 , σ 2 := V(X1 ) und

X̄n := nk=1 Xk /n. Zeigen Sie:
Aufgabe 21.33 •• Es sei X ∼ G(p). Zeigen Sie:
a) E(X̄n ) = μ.
1−p
a) E(X) = , b) V (X̄n ) = σ 2 /n.
p
1−p c) Cov(Xj , X̄n ) = σ 2 /n.
b) V(X) = . √
p2 d) ρ(X1 − 2X2 , X̄n ) = −1/ 5n.

Aufgabe 21.34 • Es sei X ∼ Po(λ). Zeigen Sie: Aufgabe 21.43 •• Der Zufallsvektor (X1 , . . . , Xs ) be-
sitze die Multinomialverteilung Mult(n, p1 , . . . , ps ), wobei
E(X) = V(X) = λ . p1 > 0, . . . , ps > 0 vorausgesetzt ist. Zeigen Sie:
a) Cov(Xi , Xj ) = −n · pi · pj (i = j ) ,
D
pi · pj
Aufgabe 21.35 •• Ein echter Würfel wird in unabhän- b) ρ(Xi , Xj ) = − (i = j ) .
giger Folge geworfen. Bestimmen Sie die Wahrscheinlich- (1 − pi ) · (1 − pj )
keiten folgender Ereignisse:
Aufgabe 21.44 •• In der Situation des zweifachen
a) mindestens eine Sechs in sechs Würfen, Wurfs mit einem echten Würfel seien Xj die Augenzahl des
b) mindestens zwei Sechsen in 12 Würfen, j -ten Wurfs sowie M := max(X1 , X2 ). Zeigen Sie:
c) mindestens drei Sechsen in 18 Würfen. M 2 + M(M − 1)/2
E(X1 |M) = .
2M − 1
Aufgabe 21.36 • Es sei (pn )n≥1 eine Folge aus (0, 1)
mit limn→∞ npn = λ, wobei 0 < λ < ∞. Zeigen Sie: Aufgabe 21.45 •• In einer Bernoulli-Kette mit Treffer-
  wahrscheinlichkeit p ∈ (0, 1) sei X die Anzahl der Versuche,
n k λk bis erstmalig
lim pn (1 − pn )n−k = e−λ · , k ∈ N0 .
n→∞ k k! a) die Sequenz 01 aufgetreten ist. Zeigen Sie: Es gilt
EX = 1/(p(1 − p)).
Aufgabe 21.37 • Es sei X ∼ Po(λ). Für welche Werte b) die Sequenz 111 aufgetreten ist. Zeigen Sie: Es gilt
von k wird P(X = k) maximal? EX = (1 + p + p 2 )/p 3 .
164 Hinweise zu Kapitel 21

Aufgabe 21.46 •• Wir würfeln in der Situation zwi- Hinweise


schen Angst und Gier auf Seite 798 k-mal und stoppen dann.
Falls bis dahin eine Sechs auftritt, ist das Spiel natürlich so-
Verständnisfragen
fort (mit dem Gewinn 0) beendet. Zeigen Sie, dass bei dieser
Strategie der Erwartungswert des Spielgewinns G durch Aufgabe 21.1 •• –
 k
5
EG = 3 · k · Aufgabe 21.2 •• Modellieren Sie Wn als Summe unab-
6 hängiger Zufallsvariablen.
gegeben ist. Welcher Wert für k liefert den größten Erwar-
tungswert? Aufgabe 21.3 •• Es kommt nicht auf die Zahlen 2 bis
5 an.
Aufgabe 21.47 •• In einer Bernoulli-Kette mit Treffer-
wahrscheinlichkeit p ∈ (0, 1) bezeichne Yj die Anzahl der Aufgabe 21.4 ••• Stellen Sie sich vor, jede von n Perso-
Nieten vor dem j -ten Treffer (j = 1, 2, 3). Nach Übungsauf- nen hat einen Würfel, und jede zählt, wie viele Versuche sie
gabe 21.5 besitzt Y1 unter der Bedingung Y2 = k eine Gleich- bis zu ersten Sechs benötigt.
verteilung auf den Werten 0, 1, . . . , k. Zeigen Sie: Unter der
Bedingung Y3 = k, k ∈ N0 , ist die bedingte Verteilung von Aufgabe 21.5 •• –
Y1 durch
Aufgabe 21.6 •• –
2(k + 1 − j )
P(Y1 = j |Y3 = k) = , j = 0, 1, . . . , k ,
(k + 1)(k + 2) Aufgabe 21.7 • –

gegeben. Aufgabe 21.8 •• –

Aufgabe 21.48 •• Es seien X1 , . . . , Xs unabhängige Aufgabe 21.9 •• –


Zufallsvariablen mit den Poisson-Verteilungen Xj ∼ Po(λj ),
j = 1, . . . , s. Zeigen Sie, dass der Zufallsvektor Aufgabe 21.10 • –
(X1 , . . . , Xs ) unter der Bedingung X1 + . . . + Xs = n,
n ∈ N, die Multinomialverteilung Mult(n, p1 , . . . , ps ) be- Aufgabe 21.11 •• –
sitzt. Dabei ist pj = λj /(λ1 + . . . + λs ), j ∈ {1, . . . , s}.
Aufgabe 21.12 • –
Aufgabe 21.49 • Es gelte X ∼ Nb(r, p). Zeigen Sie,
dass X die erzeugende Funktion Aufgabe 21.13 •• Verwenden Sie ein Symmetrieargu-
 r ment.
p
gX (t) = , |t| < 1 ,
1 − (1 − p)t
Aufgabe 21.14 •• Betrachten Sie die erzeugende Funk-
besitzt. tion von X an der Stelle −1.

Aufgabe 21.50 • Leiten Sie mithilfe der erzeugen- Aufgabe 21.15 •• –


den Funktion Erwartungswert und Varianz der Poisson-
Verteilung (vgl. Seite 785) und der negativen Binomialver- Aufgabe 21.16 ••• Sind X und Y die zufälligen Augen-
teilung (vgl. Seite 784) her. zahlen bei einem Wurf mit dem ersten bzw. zweiten Würfel
und g bzw. h die erzeugenden Funktionen von X bzw. Y , so
Aufgabe 21.51 •• Die Zufallsvariable X sei poisson- gilt g(t) = tP (t) und h(t) = tQ(t) mit Polynomen vom
verteilt mit Parameter λ. Zeigen Sie: Grad 5, die jeweils mindestens eine reelle Nullstelle besitzen
a) E[X(X − 1)(X − 2)] = λ3 . müssen.
b) EX 3 = λ3 + 3λ2 + λ.
c) E(X − λ)3 = λ. Beweisaufgaben
Aufgabe 21.17 ••• Stellen Sie X mithilfe einer geeigne-
Aufgabe 21.52 •• Zeigen Sie, dass in der auf Seite 803 ten Indikatorsumme dar.
beschriebenen Situation für die randomisierte Summe SN
gilt: k k
Aufgabe 21.18 •• Es ist n=1 1 = k und 2 n=1 n =
a) E(SN ) = EN · EX1 , k(k + 1).
b) V(SN ) = V(N) · (EX1 )2 + EN · V(X1 ).
Aufgabe 21.19 •• Setzen sie a := (b + c)/2 in Eigen-
Dabei sei EX12 < ∞ und EN 2 < ∞ vorausgesetzt. schaft a) der Varianz auf Seite 778.
Lösungen zu Kapitel 21 165

Aufgabe 21.20 •• Schätzen Sie den Indikator des Er- Aufgabe 21.42 •• –
eignisses {X ≥ ε} möglichst gut durch ein Polynom zweiten
Grades ab, das durch den Punkt (ε, 1) verläuft. Aufgabe 21.43 •• Es gilt Xi + Xj ∼ Bin(n, pi + pj ).

Aufgabe 21.21 •• – Aufgabe 21.44 •• –

Aufgabe 21.22 •• Leiten Sie mit k = 1 in (21.13) eine Aufgabe 21.45 •• Gehen Sie analog wie im Beispiel
Rekursionsformel für P(X = m) her. auf Seite 796 vor.
n
Aufgabe 21.23 •• Es gilt P(X ≥ k) = l=k P(X = l) Aufgabe 21.46 •• –
sowie (vollständige Induktion über m!)
    Aufgabe 21.47 •• (Y1 , Y3 ) hat die gleiche gemeinsame

m
j j −1 Verteilung wie (X1 , X1 + X2 + X3 ), wobei X1 , X2 , X3 un-
(−1)ν = (−1)m , m = 0, 1, . . . , j − 1.
ν m abhängig und je G(p)-verteilt sind.
ν=0

Aufgabe 21.48 •• –
Aufgabe 21.24 •• –
Aufgabe 21.49 • –
Aufgabe 21.25 ••• Verwenden Sie die im Beispiel auf
Seite 796 verwendeten Ereignisse A1 , A2 und A3 . Aufgabe 21.50 • –

Aufgabe 21.51 •• Verwenden Sie die erzeugende


Rechenaufgaben Funktion.
Aufgabe 21.26 •• –
Aufgabe 21.52 •• Verwenden Sie (21.59).
Aufgabe 21.27 • –

Aufgabe 21.28 ••• –


Lösungen
Aufgabe 21.29 •• Formel des Ein- und Ausschließens!
Verständnisfragen
Aufgabe 21.30 •• –
Aufgabe 21.1 •• –
Aufgabe 21.31 •• –
Aufgabe 21.2 •• –
Aufgabe 21.32 •• Sie brauchen nicht zu rechnen!
Aufgabe 21.3 •• G(1/2)
Aufgabe 21.33 •• Bestimmen Sie die Varianz, indem
Sie zunächst EX(X − 1) berechnen. Aufgabe 21.4 ••• –

Aufgabe 21.34 • Bestimmen Sie EX(X − 1). Aufgabe 21.5 •• –

Aufgabe 21.35 •• – Aufgabe 21.6 •• –

Aufgabe 21.36 • Es gilt 1−1/t ≤ log t ≤ t −1, t > 0. Aufgabe 21.7 • –

Aufgabe 21.37 • Betrachten Sie P(X = k + 1)/ Aufgabe 21.8 •• –


P(X = k).
Aufgabe 21.9 •• –
Aufgabe 21.38 • –
Aufgabe 21.10 • –
Aufgabe 21.39 •• Die Wahrscheinlichkeiten aus a) bis
g) addieren sich zu eins auf. Aufgabe 21.11 •• –

Aufgabe 21.40 •• Multinomialer Lehrsatz! Aufgabe 21.12 • –

Aufgabe 21.41 •• Bestimmen Sie zunächst EX(X−1). Aufgabe 21.13 •• –


166 Lösungswege zu Kapitel 21

Aufgabe 21.14 •• – Aufgabe 21.39 •• a) 6/65 , b) 150/65 , c) 300/65 , d)


1200/65 , e) 1800/65 , f) 3600/65 , g) 720/65 .
Aufgabe 21.15 •• –
Aufgabe 21.40 •• –
Aufgabe 21.16 ••• Nein.
Aufgabe 21.41 •• –

Beweisaufgaben Aufgabe 21.42 •• –


Aufgabe 21.17 ••• –
Aufgabe 21.43 •• –
Aufgabe 21.18 •• –
Aufgabe 21.44 •• –
Aufgabe 21.19 •• – Aufgabe 21.45 •• –

Aufgabe 21.20 •• – Aufgabe 21.46 •• –

Aufgabe 21.21 •• – Aufgabe 21.47 •• –

Aufgabe 21.22 •• – Aufgabe 21.48 •• –

Aufgabe 21.23 •• – Aufgabe 21.49 • –

Aufgabe 21.24 •• – Aufgabe 21.50 • –

Aufgabe 21.25 ••• – Aufgabe 21.51 •• –

Aufgabe 21.52 •• –
Rechenaufgaben
Aufgabe 21.26 •• EX = 1/4, EY = 0, EX 2 = 3/2,
EY 2 = 1/2, V(X) = 23/16, V(Y ) = 1/2, E(XY ) = −1/4.
Lösungswege
Aufgabe 21.27 • –
Verständnisfragen
Aufgabe 21.28 ••• –
Aufgabe 21.1 •• a) Bezeichnet Aj das Ereignis, dass
bei der j -ten Geburt ein Mädchen geboren wird, so sind auf-
Aufgabe 21.29 •• 0.04508 . . .
grund der getroffenen Annahmen A1 , . . . , An unabhängige
Ereignisse mit gleicher  Wahrscheinlichkeit 1/2. Somit be-
Aufgabe 21.30 •• – n
sitzt die Anzahl Xn = j =1 1{Aj } der Mädchen unter n
Geburten die Binomialverteilung Bin(n, 1/2). Damit wird
Aufgabe 21.31 •• –
10  
10 386
Aufgabe 21.32 •• – a10 = P(X10 ≥ 6) = 2−10 = = 0.376 . . .
j 1024
j =6
Aufgabe 21.33 •• –
b) Mit Xn wie in a) und der Tschebyschow-Ungleichung gilt
Aufgabe 21.34 • – V(X100 )
a100 = P(X100 ≥ 60) ≤ P(|X100 − 50| ≥ 10) ≤ .
100
Aufgabe 21.35 •• –
1 1
Wegen V(X100 ) = 100 · 2 · 2 = 25 folgt a100 < a10 .
Aufgabe 21.36 • –
c) Mit Rn := Xn /n ist
 
Aufgabe 21.37 • Der Maximalwert wird im Fall λ ∈
/N
 1 
für k = 2λ3 und für λ ∈ N für die beiden Werte k = λ und an = P(Xn ≥ n · 0.6) ≤ P Rn −  ≥ 0.1 .
2
k = λ − 1 angenommen.
Nach dem Schwachen Gesetz großer Zahlen konvergiert die
Aufgabe 21.38 • – letzte Wahrscheinlichkeit für n → ∞ gegen null.
Lösungswege zu Kapitel 21 167

Aufgabe 21.2 •• Die Zufallsvariable Xj bezeichne die Verteilung G(1/2) als Verteilung der Anzahl der geworfenen
Anzahl der Kugeln, die nötig sind, um das (j + 1)-te Fach Einsen vor der ersten Sechs heraus. Wer ein wenig rechnen
zu besetzen, wenn schon j Fächer besetzt sind. Dann gilt möchte, könnte so vorgehen: Es bezeichne X die Anzahl der
vor dem Auftreten der ersten Sechs geworfenen Einsen und
Wn = X0 + X1 + . . . + Xn−1 , Y die Anzahl der Würfe bis zum Auftreten der ersten Sechs.
Es gilt
und offenbar ist X0 = 1. Sind j < n Fächer besetzt, so be-  n−1
5 1
findet man sich unabhängig von den Nummern der bereits P(Y = n) = · , n ∈ N.
besetzten Fächer und unabhängig von der Dauer der bishe- 6 6
rigen Besetzungsvorgänge in der Situation, auf den ersten Unter der Bedingung Y = n mit n ≥ 2 tritt bei keinem der
Treffer in einer Bernoulli-Kette zu warten. Dabei bedeutet ersten n−1 Würfe eine Sechs auf, und bei jedem dieser Würfe
ein Treffer, eines der n − j noch nicht besetzten Fächer zu ist jede der Zahlen 1,2,3,4,5 gleich wahrscheinlich. Unter der
belegen. Die mit pj bezeichnete Trefferwahrscheinlichkeit Bedingung Y = n besitzt somit X die Binomialverteilung
ist also (n − j )/n. Die Zahl der Nieten vor dem ersten Tref- Bin(n − 1, 1/5), d. h., es gilt
fer besitzt die geometrische Verteilung G(pj ). Da der Treffer
mitgezählt wird, hat Xj − 1 die Verteilung G(pj ). Es gilt so-    k  
n−1 1 1 n−1−k
mit P(X = k|Y = n) = 1−
k 5 5
1 − pj n
EXj = 1 + = , für k = 0, . . . , n − 1 sowie P (X = k|Y = n) = 0 für k ≥ n.
pj n−j
Nach der Formel von der totalen Wahrscheinlichkeit ergibt
1 − pj nj sich für jedes feste k ∈ N0
V(Xj ) = = .
pj2 (n − j )2


Unterstellen wir für die Varianz die stochastische Unabhän- P(X = k)= P(Y = n) · P(X = k|Y = n)
gigkeit von X2 , . . . , Xn , so ergibt sich: n=k+1
∞  n−1     
5 1 n − 1 1 k 4 n−1−k
a) =
6 6 k 5 5
n=k+1

n
n−1
n  k ∞    
E(Wn ) = E(Xj ) = 1 1 n−1 2 n−1
n−j = .
j =1 j =0 6 4 k 3
  n=k+1
1 1 1
= n · 1 + + + ... + Die hier auftretende unendliche Reihe wird mit
2 3 n

b) d k d 1 k!
x = k = , |x| < 1,
dx k dx 1 − x (1 − x)k+1

n−1
n−1
nj k=0
V(Wn ) = V(Xj ) =
(n − j )2 zu
j =1 j =1
∞    j
∞  j −k

n−1
n(j − n) + n2 j 2 (2/3)k 2
= = (j )k
(n − j )2 j =k
k 3 k!
j =k
3
j =1


n−1
1
n−1
1 (2/3)k k!
= n2 · − n . = ·
k2 k k! (1/3)k+1
k=1 k=1
= 3 · 2k .
Im Fall n = 6 ist die Situation gedanklich gleichwertig damit,
einen echten Würfel solange wiederholt zu werfen, bis jede Insgesamt folgt
Augenzahl mindestens einmal aufgetreten ist. Man benötigt  k  k+1
hierfür nach a) im Mittel 6 · (1 + 1/2 + . . . + 1/6) = 14.7 1 1 k 1
P(X = k) = 3·2 = ,
Würfe. 6 4 2

Aufgabe 21.3 •• Die durch den Hinweis angedeutete was zu zeigen war.
begriffliche Lösung arbeitet mit einem einfachen Modell, das
die nicht interessierenden Zahlen 2 bis 5 einfach ausblendet. Aufgabe 21.4 ••• Wir verwenden den Hinweis und be-
Ihr Auftreten bedeutet nur Zeitverschwendung. Wir können zeichnen mit Xj die Anzahl der Würfe, die Person j bis
genauso gut eine echte Münze werfen und die eine Seite mit zum Auftreten der ersten Sechs benötigt. Dann besitzt die
6 und die andere mit 1 beschriften. Deuten wir die 6 als Tref- Zufallsvariable Mn die gleiche Verteilung wie das Maxi-
fer und die Eins als Niete, so schält sich die geometrische mum maxj =1,...,n Xj . Da X1 , . . . , Xn stochastisch unabhän-
168 Lösungswege zu Kapitel 21

gig sind und P(Xj > k) = (5/6)k gilt, ergibt sich Aufgabe 21.6 •• a) Die idealisierenden Annahmen sind
gleichwertig mit der Annahme einer Bernoulli-Kette, in der
P(Mn > k) = 1 − P(Mn ≤ k)
ein Sohn einen „Treffer“ und ein Mädchen eine „Niete“ be-
= 1 − P(X1 ≤ k, . . . , Xn ≤ k) deuten. Als Anzahl der Nieten vor dem ersten Treffer ist die
= 1 − P(X1 ≤ k)n mit M bezeichnete zufällige Anzahl der Mädchen in einer
= 1 − (1 − P(X1 > k))n Familie geometrisch verteilt mit Parameter 1/2. Nach dem
   k n Satz auf Seite 783 gilt somit insbesondere EM = 1 und
5 V(M) = 2.
= 1− 1−
6
b) Die Anzahl J der Jungen in einer Familie ist eine Zufalls-
und damit a). Mit Aufgabe a) und variable, die nur den Wert 1 annimmt. Somit gilt EJ = 1 und
n    k(n−j ) V(J ) = 0.
n n−j 5
P(Mn > k) = 1 − (−1)
j 6 c) Als Summe von n unabhängigen geometrisch G(1/2) ver-
j =0
   (n−j )k teilten Zufallsvariablen (den Anzahlen der Mädchen in den

n−1
5
n−j −1 n einzelnen Familien) besitzt Sn die negative Binomialvertei-
= (−1)
j 6 lung Nb(n, 1/2). Wegen ESn = n und V(Sn ) = 2n folgt mit
j =0
der Tschebyschow-Ungleichung
folgt
√ V(Sn ) 1

P(|Sn − n| ≥ K 2n) ≤ = 2.
K 2 · 2n K
E(Mn ) = P(Mn > k)
k=0 Geht man zum komplementären Ereignis über, so ergibt sich

n−1  
∞  (n−j )k für K = 10
n−j −1 n 5
= (−1)
j =0
j 6
k=0 P(490 001 ≤ S500 000 ≤ 509 999) ≥ 0.99.


n−1  
n−j −1 n 1 Wir werden in Aufgabe 23.40 sehen, dass für jede Wahl
= (−1)  n−j . √
j von a, b mit a < b die Wahrscheinlichkeit P(n + a n ≤
j =0 1− 5 √
6 Sn ≤ n + b n) für n → ∞ konvergiert. Insbesondere gilt
Führen wir jetzt den Summationsindex k := n − j ein, so limn→∞ P(Sn ≥ n) = 1/2.
ergibt sich b). Der Erwartungswert von Mn wächst recht lang-
sam mit n. So ist EM1 = 6, EM5 ≈ 13.02, EM10 ≈ 16.56, Aufgabe 21.7 •
EM20 ≈ 20.23 und EM50 ≈ 25.18. a) Es handelt sich um ein wiederholt in unabhängiger
Folge durchgeführtes Experiment mit den 4 Ausgän-
Aufgabe 21.5 •• Wir können X als Anzahl der Nie- gen rot, blau, weiß und schwarz, die in dieser Reihen-
ten vor dem ersten und Y als Anzahl der Nieten zwischen folge die Wahrscheinlichkeiten p1 := 10/100, p2 :=
dem ersten und dem zweiten Treffer in einer Bernoulli-Kette 20/100, p3 := 30/100 und p4 := 40/100 besitzen.
mit Trefferwahrscheinlichkeit p ansehen. Die Bedingung Nach der Erzeugungsweise der Multinomialverteilung
X + Y = k besagt, dass insgesamt k Nieten vor dem zwei- gilt (R, B, W, S) ∼ Mult(25; p1 , p2 , p3 , p4 ).
ten Treffer aufgetreten sind. Wegen der Unabhängigkeit und
b) In diesem Fall werden die Trefferarten rot und blau zu
Gleichartigkeit aller Versuche sollte der Zeitpunkt (= Num-
einer Trefferart zusammengefasst, die die Wahrschein-
mer des Versuchs) des ersten Treffers eine Gleichverteilung
lichkeit p1 + p2 besitzt. Wiederum nach der Erzeugungs-
auf den Werten 1, 2, . . . , k+1 und damit die um eins kleinere
weise der Multinomialverteilung gilt (R + B, W, S) ∼
Zahl der Nieten vor dem ersten Treffer eine Gleichverteilung
Mult(25; p1 + p2 , p3 , p4 ).
auf den Werten 0, 1, . . . , k besitzen. Diese Einsicht wird wie
folgt bestätigt: Für j ∈ {0, . . . , k} ist c) In diesem Fall werden rot, blau und weiß zu einer Treffer-
art vereinigt. Es gibt daneben nur noch das als Niete inter-
P(X = j, X + Y = k) pretierbare schwarz. Nach der Erzeugungsweise der Bi-
P(X = j |X + Y = k) =
P(X + Y = k) nomialverteilung gilt R+B+W ∼ Bin(25; p1 +p2 +p3 ).
P(X = j, Y = k − j )
=
P(X + Y = k) Aufgabe 21.8 •• Es sei kurz m := r1 + . . . + rs gesetzt.
P(X = j ) · P(Y = k − j ) Die einfachste Möglichkeit besteht darin, mit dem Grund-
= raum  := Knm (oW ) zu arbeiten, also alle Kugeln zu unter-
P(X + Y = k)
(1 − p)j p · (1 − p)k−j p scheiden, aber nicht darauf zu achten, in welcher Reihenfolge
= k+1 die Kugeln gezogen werden. Gedanklich gleichwertig hier-
k p (1 − p)
2 k
mit ist, die n Kugeln „blind mit einem Griff zu ziehen“. Die
1 Wahrscheinlichkeitsverteilung P sei die Gleichverteilung auf
= .
k+1 . Günstig für das Ereignis {X1 = k1 , . . . , Xs = ks } sind
Lösungswege zu Kapitel 21 169

n
diejenigen unter den n-Auswahlen aller m Kugeln, die für je- Aufgabe 21.10 • Mit X = j =1 1{Aj } und der Jor-
des j ∈ {1, . . . , s} genau kj Kugeln der Farbe j aufweisen.  
dan’schen Formel gilt wegen S0 = 1 und j0 = 1
Nach der Multiplikationsregel der Kombinatorik ist die Zahl
der günstigen Fälle gleich P(A1 ∪ . . . ∪ An ) = P(X ≥ 1)
      = 1 − P(X = 0)
r1 r2 rs
· · ... · , n
k1 k2 ks = 1− (−1)j Sj
j =0
denn es müssen unabhängig voneinander für jedes j aus den

n
rj Kugeln der Farbe j genau kj Kugeln ausgewählt wer- = (−1)j −1 Sj ,
den. Wegen || = m n folgt die Behauptung. Eine andere j =1
Möglichkeit besteht darin, die Reihenfolge zu beachten, in
der die Kugeln gezogen werden. Dann arbeitet man mit dem was zu zeigen war.
Grundraum Pnm (oW ) und geht wie im Beispiel auf Seite 725
vor. Aufgabe 21.11 •• Es gilt

1 1
6 6
Aufgabe 21.9 •• Die hypergeometrische Verteilung
E(M −h(X1 ))2 = (max(i, j )−h(i))2 = ai ,
Hyp(n, r, s) entsteht im Zusammenhang mit dem n-maligen 36 36
i,j =1 i=1
rein zufälligen Ziehen ohne Zurücklegen von Kugeln aus
einer Urne, die r rote und s schwarze Kugeln enthält. Be- 
wobei ai := i(i − h(i))2 + 6j =i+1 (j − h(i))2 von h(i)
zeichnet Aj das Ereignis, dass die j -tegezogene Kugel rot abhängt. Es ist
ist, so hat die Indikatorsumme X := nj=1 1{Aj } die Ver-
a6 = 6(6 − h(6))2 ,
teilung Hyp(n, r, s). Ein konkreter Grundraum, auf dem X
definiert ist, ist in dem auf Seite 725 beginnenden Beispiel woraus h(6) = 6 folgt. Weiter gilt
angegeben. In der Selbstfrage auf Seite 725 haben Sie sich
überlegt, dass P(Aj ) = r/(r + s), j = 1, . . . , n, gilt. Somit a5 = 5(5 − h(5))2 + (6 − h(5))2 .
ergibt sich
Dieser Ausdruck wird minimal, wenn h(5) := 5 gesetzt wird.

n
n
r In gleicher Weise fährt man mit den Werten 4, 3, 2 und 1 für
EX = E1{Aj } = P(Aj ) = n · . i fort.
r +s
j =1 j =1
Aufgabe 21.12 • Es ist
Eine Herleitung des Erwartungswertes mithilfe von (21.9) ist
wie folgt: Durch elementare Umformung von Binomialkoef- P(X = j |X + Y = k)
fizienten erhält man P(X = j, X + Y = k)
=
P(X + Y = k)

n
EX = k · P(X = k) P(X = j ) · P(Y = k − j )
k=0 =
    P(X + Y = k)
r s m  n  k−j

n ·
k n−k pj (1 − p)m−j k−j
j p (1 − p)n−(k−j )
= k·   = m+n
r +s
k p (1 − p)
k m+n−k
k=1
n m n 
   
r −1 s j k−j
n · = m+n ,
r k−1 (n − 1) − (k − 1)
= n· ·   k
r +s r −1+s
k=1
n−1 X+Y =k = Hyp(k, m, n).
und damit gilt wie behauptet PX
   
r −1 s

n−1 · Dieses Resultat erschließt sich wie folgt auch intuitiv: Den-
r j (n − 1) − j
= n· ·   . ken wir uns X und Y als Trefferzahlen in den ersten m
r +s r −1+s bzw. letzten n Versuchen einer Bernoulli-Kette vom Umfang
j =0
n−1 m+n mit Trefferwahrscheinlichkeit p, so besagt das Ereignis
n−1 X + Y = k, dass insgesamt k Treffer
 aufgetreten sind. Aus
Die letzte Summe ist gleich j =0 P(Y = j ), wobei Y Symmetriegründen sind alle m+n k Auswahlen derjenigen k
eine Zufallsvariable mit der hypergeometrischen Verteilung aller m + n Versuche mit dem Ausgang Treffer gleichwahr-
Hyp(n − 1, r − 1, s) ist, also gleich 1, und somit folgt die scheinlich. Interpretiert man die ersten m Versuche als rote
schon auf eleganterem Wege erhaltene Darstellung für E(X). und die übrigen als schwarze Kugeln und die Zuordnung
170 Lösungswege zu Kapitel 21

„Treffer“ zu einem Versuch als Ziehen einer von m + n Ku- auf den Werten 2, . . . , 12 besitzen soll, gilt nach dem Multi-
geln, so ist die Situation von Seite 725 mit r = m, s = n und plikationssatz für erzeugende Funktionen die Darstellung
n = k gegeben, und X = j bedeutet gerade, j rote Kugeln
zu ziehen. 1 2  t 2 t 11−1
g(t)h(t) = t + t 3 + . . . + t 12 = · , t = 1.
11 11 t − 1
Aufgabe 21.13 •• Wir setzen kurz Z := X1 + X2 + X3
Die Augensummen 2 und 12 können nur auftreten, wenn X
und betrachten ein beliebiges z ∈ R mit P(Z = z) > 0. Nach
und Y die Werte 1 und 6 mit positiven Wahrscheinlichkeiten
der Substitutionsregel gilt
annehmen. Aus diesem Grund können wir in den erzeugen-
den Funktionen g und h den Faktor t abspalten und erhalten
E(X1 + X2 + X3 |Z = z) = E(z|Z = z) = z.
die Darstellungen
Andererseits ist wegen der Additivität des bedingten Erwar-
g(t) = t · P (t), h(t) = t · Q(t), t ∈ R,
tungswertes
mit Polynomen P und Q vom jeweiligen Grad 5, wobei

3
E(X1 + X2 + X3 |Z = z) = E(Xj |Z = z).
P (0) = 0, P (1) = 0, Q(0) = 0, Q(1) = 0
j =1

Wegen der gemachten Annahmen sind die bedingten Erwar- gilt. Damit folgt
tungswerte E(Xj |Z = z) für jedes j = 1, 2, 3 gleich, und
1 t 11 − 1
wir erhalten P (t) · Q(t) = · , t ∈ R \ {0, 1},
z 11 t − 1
E(X1 |Z = z) = .
3
was bedeuten würde, dass weder P noch Q eine reelle Null-
Nach Definition der bedingten Erwartung folgt die Behaup- stelle besäßen. Da jedoch jedes Polynom fünften Grades min-
tung. destens eine reelle Nullstelle hat, muss die eingangs gestellte
Frage negativ beantwortet werden.
Aufgabe 21.14 •• Es ist


n
Beweisaufgaben
g(t) = P (X = k)t k = (1 − p + pt)n .
k=0 Aufgabe 21.17 ••• Es bezeichne Aj das Ereignis, dass
in keiner der k Stichproben die Kugeln mit der Nummer j
Aus auftritt, j ∈ {1, 2, . . . , n}. Dann ist

n

n
g(−1) = (−1)k P(X = k) = (1 − 2p)n X =n− 1{Aj }
k=0
j =1
und

n die Anzahl der verschiedenen Kugelnummern aus den k
1= P(X = k) Stichproben. Dass bei einer Ziehung  die Kugel Nr. j nicht
n
k=0 auftritt, hat die Wahrscheinlichkeit n−1
s / s . Wegen der sto-
chastischen Unabhängigkeit der Stichproben gilt
folgt durch Addition
 n−1 k
2n/23  s k
P(Aj ) = ns  = 1−
n
1 + (1 − 2p) = 2 P(X = 2k) n
s
k=0
und folglich
und hieraus die Behauptung.
  
s k
EX = n − n · P(A1 ) = n · 1 − 1 − .
Aufgabe 21.15 •• Aus der Reproduktionsgleichung n
(21.60) und Aufgabe 21.52 ergibt sich für jedes n ≥ 0
Für j ∈ {1, . . . , n − s} und i1 , . . . , ij mit 1 ≤ i1 < . . . <
E(Mn+1 ) = E(Mn ) · μ, ij ≤ n gilt
V(Mn+1 ) = V(Mn ) · μ2 + E(Mn ) · σ 2 ,  n−j  k
P(Ai1 ∩ . . . ∩ Aij ) = ns  ,
sodass die Behauptung durch Induktion über n folgt.
s

Aufgabe 21.16 ••• Wir verwenden die Bezeichnungen denn bei jeder der k unabhängigen Stichproben dürfen die s
des Hinweises. Da die Summe X + Y eine Gleichverteilung Kugeln nur aus der (n − j )-elementigen Menge aller Kugeln
Lösungswege zu Kapitel 21 171

mit Nummern aus {1, . . . , n} \ {i1 , . . . , ij } gewählt werden. b) Aus Gleichung (21.14) ist ersichtlich, dass das Ungleich-
Mit heitszeichen in a) genau dann zu einem Gleichheitszeichen

   n−j  k wird, wenn gilt:


n
Sj = P(Ai1 ∩ . . . ∩ Aij ) = ns  b+c
1≤i1 <...<ij ≤n
j s EX = , P(X = b) + P(X = c) = 1.
2
liefert die Jordan’sche Formel Diese beiden Gleichungen sind äquivalent zur Bedingung
 n  P(X = b) = P(X = c) = 1/2.

P(X = r) = P 1{Ai } = n − r
i=1
Aufgabe 21.20 •• Wir betrachten eine Parabel mit

n   einem auf der x-Achse aufliegenden Scheitelpunkt mit Ab-
j
= (−1)j −n+r Sj szissenwert a < ε, die durch den Punkt (ε, 1) geht, also die
n−r Funktion
j =n−r
(x − a)2
 
n   n−j k g(x) := , x ∈ R.
n j −n+r r
ns  (ε − a)2
= (−1)
r n−j Es gilt 1[ε,∞) (x) ≤ g(x), x ∈ R, und somit
j =n−r s
 
r    r−i  k
n i r 1{X(ω) ≥ ε} ≤ g(X(ω)), ω ∈ .
= (−1) ns  .
r i s
i=0 Da die Erwartungswertbildung monoton ist, folgt

E(X − a)2
Aufgabe 21.18 •• a) Mit dem ersten Hinweis folgt E1{X ≥ ε} = P(X ≥ ε) ≤ .
(ε − a)2
∞ ∞
 
k
EX = k P(X = k) = 1 P(X = k) Wegen EX = 0 ist der Zähler gleich EX 2 +a 2 = V(X)+a 2 .
k=1 k=1 n=1 Minimiert man die Funktion a → (V(X) + a 2 )/(ε − a)2 ),

∞  ∞ a < ε, bezüglich a, so ergibt sich als Lösung a = −V(X)/ε.

= P(X = k) = P(X ≥ n). Setzt man diesen Wert für a ein, so folgt die Behauptung nach
n=1 k=n n=1 direkter Rechnung.

b) Der zweite Hinweis liefert Aufgabe 21.21 •• a): Aus der Regel V(aX + b) =
a 2 V(X) und der Eigenschaft f) der Kovarianz auf Seite 789


folgt
EX 2 = k 2 P(X = k) = [k(k + 1) − k] · P(X = k) ⎛ ⎞
k=1 k=1
1 n
 k

 ∞ V(X̄n ) = 2 · V⎝ Xj⎠
n
= 2 n P(X = k) − k P(X = k) j =1
⎛ ⎞
k=1 n=1 k=1

∞  ∞ 1⎝ 2
= 2 nσ + 2 Cov(Xi , Xj )⎠.
= 2 n P(X = k) − P(X ≥ n) n
1≤i<j ≤n
n=1 k=n n=1
∞ ∞
Da die Unkorreliertheit von Xi und Xj für |i − j | ≥ k
= 2 nP(X ≥ n) − P(X ≥ n) und
 die Cauchy-Schwarz-Ungleichung die2 Abschätzung
1≤i<j ≤n | Cov(Xi , Xj )| ≤ n(k − 1)σ liefern, folgt
n=1 n=1

limn→∞ V(X̄n ) = 0 und somit die Behauptung wegen
= (2n − 1) P(X ≥ n).
E(X̄n ) = μ aus der Tschebyschow-Ungleichung.
n=1
b): Die Behauptung ergibt sich aus Teil a) mit Xj := 1{Aj }
und k = 2, denn es ist P(Y1 < Y2 ) = 5/12.
Aufgabe 21.19 •• a) Mit dem Hinweis gilt
 2  2 Aufgabe 21.22 •• Wir schreiben pj := P(X = j ) und
b+c b+c
E X− = V(X) + EX − . (21.14) setzen in k = 1 in (21.13). Es ergibt sich
2 2
P(X = m + 1, X ≥ 1)
Wegen b ≤ X ≤ c folgt |X − (b + c)/2| ≤ (c − b)/2, und = P(X = m), m ∈ N0 ,
P(X ≥ 1)
somit ergibt sich
 2 und somit wegen P(X ≥ 1) = 1 − p0 die Beziehung
b+c (c − b)2
V(X) ≤ E X − ≤ .
2 4 pm+1 = pm · (1 − p0 ), m ∈ N0 .
172 Lösungswege zu Kapitel 21

∞
Da 0 < p0 < 1 vorausgesetzt wurde, folgt hieraus pm > 0 Wegen ex = k=0 x
k /k!, x ∈ R, folgt c) aus b). Mit (21.32)
für jedes m ≥ 0, also
folgt
pm+1
= 1 − p0 , m ∈ N0 .    
pm 1 1 (n − 2)! 1
V(Xn ) = n · 1− + n(n − 1) − 2
Wir erhalten n n n! n
 k−1  = 1,
 pm+1
pk = · p0 = (1 − p0 )k · p0 ,
pm was d) zeigt.
m=0

k ∈ N0 . Folglich gilt X ∼ G(p0 ).


Aufgabe 21.25 ••• a) Mit A1 , A2 , A3 wie im Hinweis
Aufgabe 21.23 •• Die obige Identität zwischen Bino- gelten für k ≥ 3
mialkoeffizienten beweist man durch Induktion über m un-
ter Verwendung der Rekursionsformel (19.30). Mit der Jor- P(X = k + 1|A1 ) = P(X = k),
dan’schen Formel folgt dann P(X = k + 1|A2 ) = P(X = k − 1),

n P(X = k + 1|A3 ) = 0.
P(X ≥ k) = P(X = l)
l=k
  Die Behauptung folgt dann aus der Formel von der totalen
n n
j Wahrscheinlichkeit.
= (−1)j −l Sj
l
l=k j =l n
⎛ ⎞ b) Sei sn := k=1 wk . Mit a) folgt
n j  
⎝ (−1)j −l j ⎠ Sj
=
j =k l=k
l
n−1
⎛ ⎞ sn = w2 +w3 + (qwk + pqwk−1 )
n −k
j   k=3
= ⎝ (−1)ν j ⎠ Sj = w2 +w3 +q(sn − w2 − wn ) + pq(sn − wn − wn−1 )
ν
j =k ν=0

n  
j −1 und somit
= (−1)j −k Sj ,
k−1
j =k
sn = p−2 (pw2 + w3 − (q + pq)wn − pqwn−1 ),
was zu zeigen war.
also insbesondere die Beschränktheit der Folge (sn ).
Aufgabe 21.24 •• Auf Seite 718 wurde P(Aj ) = 1/n Da (sn ) als monoton wachsende Folge konvergiert, gilt
für 1 ≤ j ≤ n und allgemeiner limn→∞ wn = 0. Die letzte Darstellung für sn liefert dann
(n − j )! limn→∞ sn = p−2 (pw2 + w3 ) = 1.
P(Ai1 ∩ . . . ∩ Aij ) =
n! n
c) Es sei en := k=2 kwk . Aus Teil a) folgt
für 1 ≤ i1 < . . . < ij ≤ n gezeigt. Wegen

n
1
n−1
E(Xn ) = P(Aj ) = n · en = 2w2 + 3w3 + (k + 1)(qwk + pqwk−1 ),
n
j =1 k=3

(vgl. (21.11)) folgt a), und b) ergibt sich mit der Jordan’schen
also
Formel auf Seite 777, denn danach gilt
n  
j en ≤ 2w2 + 3w3 + qen + qsn + pqen + 2pqsn
P(X = k) = (−1)j −k Sj ,
k
j =k
mit sn wie in b). Somit ist die Folge (en ) beschränkt, was zu
wobei zeigen war.

Sj = P(Ai1 ∩ . . . ∩ Aij )
1≤i1 <...<ij ≤n
  Rechenaufgaben
n (n − j )!
= ·
j n! Aufgabe 21.26 •• Die gemeinsame Verteilung von X
1 und Y ist nachstehend in tabellarischer Form zusammen mit
= .
j! den Marginalverteilungen von X und Y aufgeführt.
Lösungswege zu Kapitel 21 173

j bezüglich k zu minimieren. Die folgende Tabelle zeigt die


−1 0 1 mithilfe eines Computers gewonnenen optimalen Gruppen-
−1 0 1/4 1/8 3/8 größen k0 für verschiedene Werte von p sowie die erwartete
prozentuale Ersparnis (1 − E(Yk0 )/k0 ) × 100% pro Person.
0 1/8 0 1/8 1/4
i P(X = i)
1 1/8 0 0 1/8
p 0.1 0.05 0.01 0.005 0.001 0.0001
2 0 1/4 0 1/4
k0 4 5 11 15 32 101
1/4 1/2 1/4 1
Ersparnis in % 41 57 80 86 94 98
P(Y = j )
Optimale Gruppengrößen und prozentuale Ersparnis pro Person beim
Gruppenscreening in Abhängigkeit von p
Es ergibt sich
3 1 2 1 d) Wir betrachten die Funktion x → 1/x − (1 − p)x , x ≥ 3,
EX = − + + = , und approximieren (1 − p)x für kleines p durch 1 − px.
8 8 4 4
1 1 Minimierung der Funktion x −→ 1/x − (1 − px) bezüg-
EY = − + = 0, √
lich x (1. Ableitung!) liefert x0 = 1/ p als Abszisse der
4 4 √
Minimalstelle. Für kleine Werte von p ist also k0 ≈ 1/ p
(−1)2 · 3 12 22 3
EX 2 = + + = , mit einer erwarteten prozentualen Ersparnis von ungefähr
8 8 4 2 √
(1 − 2 p) × 100% eine gute Näherung (vgl. die obige Ta-
(−1)2 12 1 belle).
EY 2 = + = ,
4 4 2
3 1 23 Aufgabe 21.29 •• Wir nummerieren die Spieler ge-
V(X) = − = ,
2 16 16 danklich von 1 bis 10 durch und bezeichnen mit Aj das Ereig-
1 nis, dass der j -te Spieler zwei Asse erhält (j = 1, . . . , 10).
V(Y ) = ,
2 Aus Sicht eines jeden Spielers wird zu Beginn zweimal ohne
1 1 1 Zurücklegen aus einer Urne gezogen, die vier rote Kugeln
E(XY ) = − − =− .
8 8 4 (Asse) und 48 schwarze Kugeln (restliche Karten) enthält.
Die Anzahl der Asse für Spieler j besitzt somit die Vertei-
Aufgabe 21.27 • Setzt man auf n Zahlen, so gilt lung Hyp(2, 4, 48), und es folgt
4 
37 − n n n(1 + k(n)) − 37 1
EX = − · 1 + k(n) · = . P(Aj ) = 52 2
 = = 0.00452 . . .
37 37 37 221
2
Da das Produkt n(1 + k(n)) für jede der Setzmöglichkeiten
36 beträgt, folgt die Behauptung. Da die Wahrscheinlichkeiten P(Ai ∩Aj ) für 1 ≤ i < j ≤ n aus
Symmetriegründen nicht von i und j abhängen und Schnitte
Aufgabe 21.28 ••• a) Die Zufallsvariable Yk nimmt die von mehr als zwei der Ai die leere Menge ergeben, liefert die
Werte 1 und k + 1 an. Im ersten Fall sind alle Personen Formel des Ein- und Ausschließens
⎛ ⎞
der Gruppe gesund. Im zweiten Fall liegt ein positiver Be- 410  
10
fund vor, und es müssen zusätzlich zur Gruppenuntersuchung P⎝ Aj ⎠ = 10 · P(A1 ) − P(A1 ∩ A2 ).
noch k Einzeluntersuchungen vorgenommen werden. Wegen 2
j =1
P(Yk = 1) = (1 − p)k und P(Yk = k + 1) = 1 − (1 − p)k    50
besitzt Yk den Erwartungswert Wegen P(A1 ∩ A2 ) = 24 /( 52 2 2 ) folgt durch direkte
Rechnung P(∪10
j =1 Aj ) = 0.04508 . . ..
E(Yk ) = (1 − p)k + (k + 1)(1 − (1 − p)k )
= k + 1 − k(1 − p)k . Aufgabe 21.30 •• Nach Definition des Binomialkoef-
fizienten gilt für k = 0, 1, . . . , n − 1
b) Damit sich im Mittel überhaupt eine Ersparnis durch Grup-  
√ n
penbildung ergibt, muss E(Yk ) < k und somit 1−p > 1/ k k P(X = k + 1) = p k+1 (1 − p)n−k−1
√ k+1
sein. Da die Funktion k → 1/ k k ihr Minimum für √ k =3  
− 3 n−k p n k
annimmt, folgt
√ notwendigerweise 1 p > 1/ 3 oder = · · p (1 − p)n−k .
p < 1 − 1/ 3 3 = 0.3066 . . .. Gruppenscreening lohnt sich k+1 1−p k
also nur für genügend kleines p, was auch zu erwarten war. Es folgt
c) Die optimale Gruppengröße k0 , die die erwartete Anzahl ⎧ ⎫ ⎧ ⎫

⎪ <⎪
⎪ ⎪
⎪ >⎪

E(Yk )/k von Tests pro Person minimiert, hängt natürlich von ⎪ ⎬
⎨ ⎪ ⎨ ⎪
⎪ ⎬
p ab und führt auf das Problem, die Funktion P(X = k + 1) = P(X = k) ⇐⇒ k = (n + 1)p − 1.

⎪ ⎪
⎪ ⎪
⎪ ⎪

⎩>⎪
⎪ ⎭ ⎩<⎪
⎪ ⎭
k → 1 + 1/k − (1 − p)k
174 Lösungswege zu Kapitel 21

Hieraus ergibt sich, dass das Maximum der Wahrscheinlich- und somit
keiten P(X = k) im Fall (n + 1)p ∈ / N für k = 2(n + 1)p3
und andernfalls für die beiden Werte k = (n + 1)p und V(X) = EX(X − 1) + EX − (EX)2
 2
k = (n + 1)p − 1 angenommen wird. 2(1 − p)2 1−p 1−p
= + −
p2 p p
Aufgabe 21.31 •• 1−p
= .
a) Die Wahrscheinlichkeit, dass kein Bit in einem aus k Bits p2
bestehenden Wort gestört wird und damit das Wort feh-
lerfrei übertragen wird, beträgt (1 − p)k . Bezeichnet Aj
das Ereignis, dass das j -te Codewort fehlerfrei übertragen Aufgabe 21.34 • Mit der Darstellungsformel gilt
wird, so hat X als Indikatorsumme der stochastisch unab- ∞

λk
hängigen Ereignisse A1 , . . . , An die Binomialverteilung EX = k P(X = k) = e−λ k
Bin(n, (1 − p)k ). k!
k=1 k=1


b) Für jedes Codewort besitzt die Anzahl der gestörten Bits λk−1
die Binomialverteilung Bin(k, p). Ein übertragenes Co- = e−λ λ = e−λ λ eλ
(k − 1)!
dewort kommt (eventuell nach Korrektur) fehlerfrei beim k=1
Empfänger an, wenn es an höchstens 2 Stellen gestört ist. = λ.
Die Wahrscheinlichkeit hierfür ist
  In gleicher Weise ist
k 2
r := (1 − p)k + kp(1 − p)k−1 + p (1 − p)k−2 . ∞

2
EX(X − 1) = k(k − 1) P(X = k)
Die Anzahl der richtig erkannten Wörter besitzt somit die k=2
Binomialverteilung Bin(n, r). ∞
λk
= e−λ k(k − 1)
k!
Aufgabe 21.32 •• Claudia kann 10-mal gleichzeitig k=1
mit Peter und unabhängig von ihm ihre Münze werfen. Ge- ∞
λk−2
zählt werden hierbei die Versuche, bei denen sowohl eine = e−λ λ2 = e−λ λ2 eλ
(k − 2)!
Sechs als auch Wappen auftritt. Die Wahrscheinlichkeit für k=2
einen solchen „Doppeltreffer“ ist 1/12. Somit besitzt die in = λ2
der Aufgabenstellung beschriebene „Anzahl der dabei erziel-
ten Wappen“ die Verteilung Bin(10, 1/12). und somit

Aufgabe 21.33 ••  Die beiden ersten Ableitungen der V(X) = EX(X − 1) + EX − (EX)2 = λ2 + λ − λ2 = λ.
geometrischen Reihe ∞
k=0 x = 1/(1 − x), |x| < 1, sind
k


1

2 Aufgabe 21.35 •• Indem man jeweils zum komple-
k x k−1 = , k(k−1) x k−2 = . mentären Ereignis übergeht, gilt mit p := 1/6 und q := 5/6
(1 − x)2 (1 − x)3
k=1 k=2
a) 1 − q 6 = 0.665 . . .
Hiermit und mit der Transformationsformel folgt   11
b) 1 − (q 12 + 121 pq ) = 0.618 . . .


  17 18 2 16
EX = k · P(X = k) = k p(1 − p)k c) 1 − (q 18 + 181 pq + 2 p q ) = 0.597 . . .
k=0 k=1
∞ Wir sehen auf Seite 889, dass die Wahrscheinlichkeit, in 6n

= p(1 − p) k(1 − p)k−1 Würfen mindestens n Sechsen zu erzielen, für n → ∞ gegen
k=1 1/2 konvergiert.
1 1−p
= p(1 − p) = . Aufgabe 21.36 • Wegen
(1 − (1 − p)) 2 p
 
In gleicher Weise ergibt sich n k 1 (n)k λk
pn = · · (npn )k →
∞ k k! n! k!

EX(X − 1) = k(k − 1) p(1 − p)k
und (1 − pn )−k → 1 ist nur
k=2
∞  npn n
1− → e−λ
= p(1 − p)2 k(k − 1) (1 − p)k−2 n
k=2
zu zeigen. Die im Hinweis gegebenen Ungleichungen liefern
2 2(1 − p)2
= p(1 − p)2 =
(1 − (1 − p))3 p2 log(1 − (npn )/n)n = n log(1 − (npn )/n) ≤ −npn → −λ
Lösungswege zu Kapitel 21 175

6 
und e) 2 · 4 · P(X1 = 2, X2 = 2, X3 = 1, X4 = 0, X5 = 0,
 npn n  npn  X6 = 0) = 1800/65 ≈ 0.23148,
log 1 − = n log 1 − 
n n f) 6 · 53 · P(X1 = 2, X2 = 1, X3 = 1, X4 = 1, X5 = 0,
  
npn −1 X6 = 0) = 3600/65 ≈ 0.46296,
≥ n 1− 1−
n g) 6 · P(X1 = 1, X2 = 1, X3 = 1, X4 = 1, X5 = 1,
npn X6 = 0) = 720/65 ≈ 0.09259.
= − → −λ,
1 − pn
Aufgabe 21.40 •• Das Ereignis {X1 = k1 } ist die
also zusammen
Vereinigung der für verschiedene Tupel (k2 , . . . , ks ) mit
 npn n
lim log 1 − = −λ. k2 + . . . + ks = n − k1 paarweise disjunkten Ereignisse
n→∞ n {X1 = k1 , . . . , Xs = ks }. Mit der Maßgabe, dass die
Hieraus folgt die Behauptung. Summen in der ersten und zweiten Zeile über alle Tupel
(k2 , . . . , ks ) ∈ Ns−1
0 mit k2 + · · · + ks = n − k1 laufen,
Aufgabe 21.37 • Wegen P(X = k) = e−λ λk /k! ist folgt

P(X = k + 1) λ P(X1 = k1 ) = P(X1 = k1 , . . . , Xs = ks )
= , k ∈ N0 ,
P(X = k) k+1 k
n! p11 (n − k1 )! k
= · · p 2 . . . psks
und damit k1 !(n − k1 )! k2 ! . . . ks ! 2
 
⎧ ⎫ ⎧ ⎫ n k

⎪ ⎪ ⎪ ⎪ = · p11 · (p2 + · · · + ps )n−k1
⎪>⎪
⎨ ⎪
⎬ ⎪<⎪

⎨ ⎪
⎬ k1
 
P(X = k + 1) = P(X = k) ⇐⇒ k + 1 = λ. n k
⎪ ⎪
⎪ ⎪ ⎪
⎪ ⎪
⎪ = · p11 · (1 − p1 )n−k1
⎩<⎪
⎪ ⎭ ⎩>⎪
⎪ ⎭ k1

(k1 = 0, 1, . . . , n).
Hieraus folgt die Behauptung.
Aufgabe 21.41 •• Nach der allgemeinen Darstellungs-
Aufgabe 21.38 • Ist Xj die Anzahl der Würfe, bei formel gilt
denen die Augenzahl j auftritt (j = 1, . . . , 6), so gilt

n  
(X1 , . . . , X6 ) ∼ Mult(8; 1/6, . . . , 1/6). Das beschriebene n k
EX(X − 1)= k(k − 1) p (1 − p)n−k
Ereignis tritt genau dann ein, wenn entweder eine Augenzahl k
k=2
3-mal und die übrigen fünf je einmal oder zwei Augenzah-
n  
len je 2-mal und die übrigen vier je einmal auftreten. Aus n−2 k−2
=n(n−1)p2 p (1−p)n−2−(k−2)
Symmetriegründen folgt k−2
k=2

P(X1 ≥ 1, . . . , X6 ≥ 1) = 6P(X1 = 3, X2 = · · · = X6 = 1) 
n−2 
n−2 j
=n(n − 1)p 2 p (1 − p)n−2−j
+15P(X1 = X2 = 2, X3 = ... = X6 = 1) j =0
j
   
8! 1 8 8! 1 8 2
= 6 + 15 =n(n − 1)p
3! 6 2!2! 6
= 0.114 . . . und somit

V(X) = E(X(X − 1)) + EX − (EX)2


Aufgabe 21.39 •• Das Spiel ist gleichwertig damit, = n(n − 1)p 2 + np − (np)2
einen Würfel fünfmal in unabhängiger Folge zu werfen.
= np(1 − p).
Sei Xj die Anzahl der Würfe mit j Augen. Es gilt
(X1 , . . . , X6 ) ∼ Mult(5; 1/6, . . . , 1/6). Hieraus folgt durch
Symmetriebetrachtungen Aufgabe 21.42 •• a) Da die Erwartungswertbildung
a) 6 · P(X1 = 5, X2 = 0, . . . , X6 = 0) = 6/65 ≈ 0.00077, ein lineares Funktional ist und alle Zufallsvariablen die glei-
che Verteilung und damit auch den gleichen Erwartungswert
besitzen, ergibt sich
b) 6 · 5 · P(X1 = 4, X2 = 1, X3 = 0, . . . , X6 = 0) =
150/65 ≈ 0.01929, 1
n
1
c) 6 · 5 · P(X1 = 3, X2 = 2, X3 = 0, . . . , X6 = 0) = EX̄n = EXj = n · μ = μ.
n n
300/65 ≈ 0.03858, j =1

d) 6 · 25 · P(X1 = 3, X2 = 1, X3 = 1, X4 = 0, X5 = 0, b) Wegen der Unabhängigkeit der Xi ist auch die Varianz-
X6 = 0) = 1200/65 ≈ 0.15432, bildung additiv, und mit der Regel V(aX + b) = a 2 V(X)
176 Lösungswege zu Kapitel 21

folgt Aufgabe 21.44 •• Im kanonischen Grundraum  =


{1, . . . , 6}2 stellt sich das Ereignis {M = j } in der
1
n
1 σ2 Form {M = j } = {(j, 1), (j, 2), . . . , (j, j − 1),
V(X̄n ) = · V(Xj ) = 2 · n · σ 2 = .
n2 n n (1, j ), (2, j ), . . . , (j − 1, j ), (j, j )} dar. Es folgt
j =1
1 1
c) Da die Kovarianzbildung ein bilineares Funktional ist, er- E(X1 |M = j ) = · · (j 2 +1+2 +. . .+j −1)
P(M = j ) 36
halten wir
1
= · (j 2 + j (j −1)/2)
1
n
2j − 1
Cov(Xj , X̄n ) = Cov(Xj , Xk ).
n
k=1 und somit
Im Fall k = j gilt Cov(Xj , Xk ) = 0, da aus der Un- M 2 + M(M − 1)/2
E(X1 |M) = .
abhängigkeit die Unkorreliertheit folgt. Im Fall k = j 2M − 1
ist Cov(Xj , Xj ) = V(Xj ) = σ 2 . Insgesamt ergibt sich
Cov(Xj , X̄n ) = σ 2 /n.
Aufgabe 21.45 •• a) Es sei  analog wie im Beispiel
d) Es ist auf Seite 796 die Menge aller endlichen Sequenzen aus Nul-
len und Einsen, die auf 01 enden und vorher diese Sequenz
Cov(X1 − 2X2 , X̄n ) nicht enthalten. Seien A0 bzw. A1 das Ereignis, dass der
ρ(X1 − 2X2 , X̄n ) =  .
V(X1 − 2X2 ) · V(X̄n ) erste Versuch eine Niete bzw. einen Treffer ergibt. Es gilt
A0 + A1 =  sowie P(A1 ) = p = 1 − P(A0 ). Weiter gilt
Zieht man die Bilinearität der Kovarianzbildung heran, so
folgt mit c) E(X|A1 ) = 1 + EX.

1
n Tritt A0 ein, so befindet man sich in der Situation, nach einem
Cov(X1 − 2X2 , X̄n ) = Cov(X1 , Xj ) gezählten Versuch auf den ersten Treffer in einer Bernoulli-
n
j =1 Kette zu warten. Die Zahl der Nieten vor dem ersten Tref-
2
n fer besitzt (geometrische Verteilung!) den Erwartungswert
− Cov(X2 , Xj ) (1 − p)/p. Da der Treffer mitgezählt wird und ein erster Ver-
n
j =1 such stattfand, gilt E(X|A0 ) = 1 + 1/p. Nach der Formel
σ2 vom totalen Erwartungswert folgt
= − .
n
EX = p(1 + EX) + q(1 + 1/p).
Weiter ist wegen der Unabhängigkeit von X1 und −2X2
Hieraus ergibt sich die Behauptung.
V(X1 −2X2 ) = V(X1 )+V(−2X2 ) = σ 2 +(−2)2 σ 2 = 5σ 2 .
b) Es sei analog zu a)  die Menge aller endlichen Sequenzen
Zusammen mit b) erhält man aus Nullen und Einsen, die auf 111 enden und vorher diese
Sequenz nicht enthalten. Seien A0 (bzw. A10 bzw. A110 bzw.
−σ 2 /n 1 A111 ) die Menge aller Sequenzen aus , die mit 0 (bzw. 10
ρ(X1 − 2X2 , X̄n ) =  = −√ .
5σ · σ /n
2 2 5n bzw. 110 bzw. 111) beginnen. Es gilt A0 + A10 + A110 +
A111 = , P(A0 ) = q, P(A10 ) = pq, P(A110 ) = p2 q,
P(A111 ) = p 3 sowie
Aufgabe 21.43 •• a) Wegen Xi +Xj ∼ Bin(n, pi +pj )
folgt E(X|A0 ) = 1 + EX,
E(X|A10 ) = 2 + EX,
V(Xi + Xj ) = n(pi + pj )(1 − pi − pj ).
E(X|A110 ) = 3 + EX,
Andererseits gilt E(X|A111 ) = 3.

V(Xi + Xj ) = V(Xi ) + V(Xj ) + 2 Cov(Xi , Xj ). Die Formel vom totalen Erwartungswert ergibt

Aus Xi ∼ Bin(n, pi ) folgt V(Xi ) = npi (1 − pi ) (analog für EX = q(1+ EX) + pq(2+ EX) + p 2 q(3+ EX) + 3p 3 .
Xj ), und man erhält
Hieraus folgt die Behauptung.
n(pi + pj )(1 − pi − pj )
= npi (1 − pi ) + npj (1 − pj ) + 2 Cov(Xi , Xj ). Aufgabe 21.46 •• Es seien Xj das Ergebnis des j -ten
Wurfs sowie A das Ereignis, dass in den (ersten) k Wür-
Hieraus ergeben sich die Behauptungen durch direkte Rech- fen keine Sechs auftritt. Unter der Bedingung A ist in je-
nung. dem der ersten k Würfe jede der Augenzahlen 1, 2, 3, 4, 5
Lösungswege zu Kapitel 21 177

gleich wahrscheinlich, es gilt also P(Xj = l|A) = 1/5 wegen der Unabhängigkeit von X1 , . . . , Xs
(l = 1, . . . , 5) und somit E(Xj |A) = 3. Aufgrund der Addi-
tivität des bedingten Erwartungswertes folgt dann P(X1 = k1 , . . . , Xs = ks |T = n)
P(X1 = k1 , . . . , Xs = ks )
=
E(G|A) = E(X1 + . . . + Xk |A) = 3k. P(T = n)
9s
j =1 P (X j = kj )
Wegen P(A) = (5/6)k und E(G|Ac ) = 0 liefert Formel =
P(T = n)
(21.45) 9s  k

j =1 e−λj λjj /kj !
=
E(G) = E(G|A)P(A) + E(G|Ac )P(Ac ) = 3k(5/6)k . e−λ λn /n!
 k1  ks
n! λ1 λs
= ... .
Der Maximalwert 6.02816 . . . wird für k = 5 und k = 6 k1 ! . . . ks ! λ λ
angenommen. Die auf Seite 798 vorgestellte Strategie, ab
einer gewürfelten Augensumme von 15 zu stoppen, ist also
etwas besser. Aufgabe 21.49 • Wir verwenden die Darstellung
 
Aufgabe 21.47 •• Wir verwenden den Hinweis, der −r r
P(X = k) = p (−(1 − p))k , k ∈ N0 ,
aus dem Additionsgesetz für die negative Binomialverteilung k
folgt. Wegen X1 + X2 ∼ Nb(2, p) und X1 + X2 + X3 ∼
Nb(3, p) folgt mit der Unabhängigkeit von X1 und X2 + X3 von Seite 783 und erhalten mithilfe der Binomialreihe (21.25)
für jedes t mit |t| < 1
P(Y1 = j |Y3 = k) = P(X1 = j |X1 + X2 + X3 = k) ∞  
−r
P(X1 = j, X2 + X3 = k − j ) gX (t) = p r (−(1 − p)t)k = pr (1 − (1 − p)t )−r .
= k
P(X1 + X2 + X3 = k) k=0
 +1 2
p(1 − p)j · k−jk−j p (1 − p)
k−j
= k+2
k p (1 − p)
3 k
Aufgabe 21.50 • a) Eine Zufallsvariable X mit der
2(k + 1 − j )
= . Poisson-Verteilung Po(λ) hat die erzeugende Funktion
(k + 1)(k + 2) g(t) = eλ(t−1) . Es gilt

Dieses Ergebnis wird auch begrifflich klar, wenn man es in


g  (t) = λeλ(t−1) , g  (t) = λ2 eλ(t−1)
der Form

k+1−j und somit g  (1) = λ = EX, g  (1) = λ2 = E(X(X − 1)).


P(Y1 = j |Y3 = k) = k+2 Mit (21.58) folgt V(X) = λ2 + λ − λ2 = λ.
2
b) Im Fall X ∼ Nb(r, p) gilt g(t) = pr (1 − (1 − p)t )−r und
schreibt. Die Information Y3 = k besagt, dass der (k + 3)- somit
te Versuch einen Treffer ergibt, also unter den ersten k + 2
Versuchen genau 2 Treffer und k Nieten sind. Wegen der Ge- g  (t) = pr r (1 − p)(1 − (1 − p)t )−(r+1) ,
dächtnislosigkeit der geometrischen
  Verteilung sollte für die
beiden Treffer jede der k+2 Auswahlen von 2 Plätzen unter g  (t) = pr r(r + 1)(1 − p)2 (1 − (1 − p)t)−(r+2)
2
den Versuchsnummern 1, 2, . . . , k + 2 gleichwahrscheinlich
sein. Das Ereignis {Y1 = j } bedeutet, dass der erste Treffer und folglich g  (1) = r(1 − p)/p = EX, g  (1) =
im (j +1)-ten Versuch auftritt. Damit bleiben genau k +1−j r(r + 1)(1 − p)2 /p2 = E(X(X − 1)). Mit (21.58) ergibt
günstige Fälle, den zweiten Treffer auf eine der Versuchs- sich
nummern j + 2, j + 3, . . . , k + 2 zu verteilen. Man beachte,  2
dass das Ergebnis nicht von der Trefferwahrscheinlichkeit p r(r + 1)(1 − p)2 r(1 − p) r(1 − p)
V(X) = + −
abhängt. p2 p p
r(1 − p)
= .
p2
Aufgabe 21.48 •• Es seien k1 , . . . , ks ∈ N0 mit
k1 + . . . + ks = n. Weiter sei λ := λ1 + . . . + λs so-
wie T = X1 + . . . + Xs gesetzt. Nach dem Additions- ∞
gesetz für die Poisson-Verteilung gilt T ∼ Po(λ). Da aus Aufgabe 21.51 •• a) Es sei g(t) = k=0 P(X =
{X1 = k1 , . . . , Xs = ks } das Ereignis {T = n} folgt, gilt k)t k = eλ(t−1) die erzeugende Funktion von X. Es ist
178 Lösungswege zu Kapitel 21

EX = λ und λ = V(X) = EX 2 − λ2 . Weiter ist Aufgabe 21.52 •• Sei h(t) := Et SN die erzeugende

Funktion von SN . Mit (21.59) gilt
g  (t) = k(k − 1)(k − 2)P(X = k)t k−3 = λ3 eλ(t−1) .
k=3 h (t) = ϕ  (g(t)) · g  (t),
 2
Also gilt g  (1) = E[X(X − 1)(X − 2)] = λ3 . h (t) = ϕ  (g(t)) · g  (t) + ϕ  (g(t)) · g  (t).

b) Es ist λ3 = EX(X − 1)(X − 2) = EX 3 − 3EX 2 + 2EX. Wegen h (1) = ESN und (21.58) folgt
Hieraus folgt
h (1) = ESN = ϕ  (1)g  (1) = EN · EX1 ,
3 3 2 3 2
EX = λ + 3(λ + λ) − 2λ = λ + 3λ + λ. h (1) = ESN (SN − 1)
 2
c) Mit der binomischen Formel ergibt sich = ϕ  (1) g  (1) + ϕ  (1)g  (1)
= EN(N − 1) · (EX1 )2 + EN · EX1 (X1 − 1).
E(X − λ)3 = EX 3 − 3λEX 2 + 3λ2 EX − λ3
= λ3 + 3λ2 + λ − 3λ3 − 3λ2 + 2λ3 Hieraus folgt der angegebene Ausdruck für die Varianz durch
= λ. direkte Rechnung.
Kapitel 22 a) Stimmen F und G auf einer in R dichten Menge (deren
Abschluss also ganz R ist) überein, so gilt F = G.
b) Die Menge
Aufgaben W (F ) := {x ∈ R | F (x + ε) − F (x − ε) > 0 ∀ ε > 0}

Verständnisfragen der Wachstumspunkte von F ist nichtleer und abgeschlos-


sen.
Aufgabe 22.1 •• Es sei F die Verteilungsfunktion einer c) Es gibt eine diskrete Verteilungsfunktion F mit der
Zufallsvariablen X. Zeigen Sie. Eigenschaft W (F ) = R.
a) P(a < X ≤ b) = F (b) − F (a), a, b ∈ R, a < b.
Aufgabe 22.9 •• Sei F die Verteilungsfunktion eines
b) P(X = x) = F (x) − F (x−), x ∈ R.
k-dimensionalen Zufallsvektors X = (X1 , . . . , Xk ). Zeigen
Sie: Für x = (x1 , . . . , xk ), y = (y1 , . . . , yk ) ∈ Rk mit x ≤ y
Aufgabe 22.2 •• Zeigen Sie, dass eine Verteilungs-
gilt
funktion höchstens abzählbar unendlich viele Unstetigkeits- y
x F = P(X ∈ (x, y]) ,
stellen besitzen kann.
wobei
Aufgabe 22.3 •• Die Zufallsvariable X besitze eine y
ρ 1−ρ1 ρ 1−ρk
x F := (−1)k−s(ρ) F (y1 1 x1 , . . . , yk k xk )
Gleichverteilung in (0, 2π). Welche Verteilung besitzt Y :=
sin X? ρ∈{0,1}k

und ρ = (ρ1 , . . . , ρk ), s(ρ) = ρ1 + . . . + ρk .


Aufgabe 22.4 • Die Zufallsvariablen X1 , . . . , Xn
seien stochastisch unabhängig. Die Verteilungsfunktion von Aufgabe 22.10 •• Für eine natürliche Zahl m sei Pm
Xj sei mit Fj bezeichnet, j = 1, . . . , n. Zeigen Sie: die Gleichverteilung auf der Menge m := {0, 1/m, . . . ,
   n (m − 1)/m}. Zeigen Sie: Ist [u, v], 0 ≤ u < v ≤ 1, ein
a) P max Xj ≤ t = Fj (t), t ∈ R , beliebiges Teilintervall von [0, 1], so gilt
j =1,...,n
j =1
1
  
n |Pm ({a ∈ m : u ≤ a ≤ v}) − (v − u)| ≤ . (22.15)
m
b) P min Xj ≤ t =1− (1 − Fj (t)), t ∈ R .
j =1,...,n
j =1
Aufgabe 22.11 •• Es seien r1 , . . . , rn ,s1 , . . . , sn ∈
Aufgabe 22.5 •• Es sei X eine Zufallsvariable mit [0, 1] mit |rj − sj | ≤ ε, j = 1, . . . , n, für ein ε > 0.
nichtausgearteter Verteilung. Zeigen Sie:
 
1 1 a) Zeigen Sie:
a) E > ,
X EX  
 

 n n

b) E(log X) < log(EX),  r − s  ≤ nε. (22.16)
 j j 
  j =1 j =1 
c) E eX > eEX .
b) Es seien Pnm die Gleichverteilung auf nm (vgl. Aufgabe
Dabei mögen alle auftretenden Erwartungswerte existieren, 22.10) sowie uj , vj ∈ [0, 1] mit uj < vj für j = 1, . . . , n.
und für a) und b) sei P(X > 0) = 1 vorausgesetzt. Weiter sei A := {(a1 , . . . , an ) ∈ nm : uj ≤ aj ≤ vj für
j = 1, . . . , n}. Zeigen Sie mithilfe von (22.16):
Aufgabe 22.6 • Der Zufallsvektor X = (X1 , . . . , Xs )  
 n 
sei multinomialverteilt mit Parametern n und p1 , . . . , ps .  n 
P (A) − (v − u )  ≤ n .
Zeigen Sie, dass die Kovarianzmatrix von X singulär ist.  m j j 
 j =1  m

Aufgabe 22.7 • Es sei X eine Zufallsvariable mit cha-


rakteristischer Funktion ϕX . Zeigen Sie: Aufgabe 22.12 •• Es sei zj +1 ≡ azj + b (mod m) das
iterative lineare Kongruenzschema des linearen Kongruenz-
X ∼ −X ⇐⇒ ϕX (t) ∈ R ∀t ∈ R. generators mit Startwert z0 , Modul m, Faktor a und Inkre-
ment b (siehe Seite 823). Weiter seien d ∈ N mit d ≥ 2
und

Beweisaufgaben Zi := (zi , zi+1 , . . . , zi+d−1 ) , 0 ≤ i < m.

Aufgabe 22.8 ••• Es seien F, G : R → [0, 1] Vertei- Dabei bezeichne u den zu einem Zeilenvektor u transpo-
lungsfunktionen. Zeigen Sie: nierten Spaltenvektor. Zeigen Sie:
M. Brokate et al., Arbeitsbuch Grundwissen Mathematikstudium – Höhere Analysis, Numerik und
Stochastik, DOI 10.1007/978-3-642-54946-5_21, © Springer-Verlag Berlin Heidelberg 2016
180 Aufgaben zu Kapitel 22

a) Zi − Z0 ≡ (zi − z0 )(1 a a 2 · · · a d−1 ) (mod m), i ≥ 0. Aufgabe 22.20 ••• Es sei (X1 , X2 ) ein zweidimensio-
b) Bezeichnet G die Menge der ganzzahligen Linearkombi- naler Zufallsvektor mit 0 < V(X1 ) < ∞, 0 < V(X2 ) < ∞.
nationen der d Vektoren Zeigen Sie: Mit ρ := ρ(X1 , X2 ) gilt für jedes ε > 0:
⎛ ⎞ ⎛ ⎞ ⎛ ⎞ ⎛ ⎞ 
1 0 0 42 0 * 1 1 + 1 − ρ2
⎜ ⎟ ⎜ ⎟ ⎜ ⎟ P⎝ |Xj − EXj | ≥ ε V(Xj ) ⎠ ≤
⎜ ⎟ ⎜ ⎟ ⎜ ⎟ .
⎜ a ⎟ ⎜m⎟ ⎜0⎟ ε2
⎜ ⎟, ⎜ ⎟ , ··· , ⎜ ⎟ , j =1
⎜ .. ⎟ ⎜ .. ⎟ ⎜ .. ⎟
⎜ . ⎟ ⎜.⎟ ⎜.⎟
⎝ ⎠ ⎝ ⎠ ⎝ ⎠
a d−1 0 m Aufgabe 22.21 ••• Es sei X eine Zufallsvariable mit
E|X| < ∞. Zeigen Sie: Ist a0 ∈ R mit
so gilt Zi − Z0 ∈ G für jedes i.
1 1
P(X ≥ a0 ) ≥ , P(X ≤ a0 ) ≥ ,
Aufgabe 22.13 •• Die Zufallsvariablen X1 , . . . , Xk , 2 2
k ≥ 2, seien stochastisch unabhängig mit gleicher,
überall positiver differenzierbarer Dichte f . Dabei hänge so folgt E|X − a0 | = mina∈R E|X − a|. Insbesondere gilt
9k also
j =1 (xj ) von (x1 , . . . , xk ) ∈ R nur über x1 + . . . + xk
f k 2 2

ab. Zeigen Sie: Es gibt ein σ > 0 mit E|X − Q1/2 | = min E|X − a|.
a∈R
 
1 x2
f (x) = √ exp − 2 , x ∈ R . Aufgabe 22.22 •• Die Zufallsvariable X sei symme-
σ 2π 2σ
trisch verteilt und besitze die stetige, auf {x | 0 < F (x) <
1} streng monotone Verteilungsfunktion F . Weiter gelte
Aufgabe 22.14 •• Leiten Sie die auf Seite 832 angege- EX 2 < ∞. Zeigen Sie:
bene Dichte gr,n der r-ten Ordnungsstatistik Xr:n über die

Beziehung Q3/4 − Q1/4 ≤ 8V(X) .
P(t ≤ Xr:n ≤ t + ε)
lim = gr,n (t)
ε→0 ε
Aufgabe 22.23 •• Es gelte X ∼ Nk (μ, ). Zeigen Sie,
für jede Stetigkeitsstelle t der Dichte f von X1 her. dass die quadratische Form (X − μ) −1 (X − μ) eine χk2 -
Verteilung besitzt.
Aufgabe 22.15 •• Leiten Sie die Darstellungsformel
für den Erwartungswert auf Seite 837 her. Aufgabe 22.24 • Zeigen Sie: Für die charakteristische
Funktion ϕX einer Zufallsvariablen X gelten:
Aufgabe 22.16 •• Es seien X eine Zufallsvariable und
p eine positive reelle Zahl. Man prüfe, ob die folgenden Aus- a) ϕX (−t) = ϕX (t), t ∈ R,
sagen äquivalent sind: b) ϕaX+b (t) = eitb ϕX (at), a, b, t ∈ R.
a) E|X|p < ∞,
∞   Aufgabe 22.25 •• Es sei X eine Zufallsvariable mit
b) P |X| > n1/p < ∞. charakteristischer Funktion ϕ und Dichte f . Weiter sei ϕ
n=1 reell und nichtnegativ, und es gelte c := ϕ(t) dt < ∞.
Zeigen Sie:
Aufgabe 22.17 ••
a) Es gilt c > 0, sodass durch g(x) := ϕ(x)/c, x ∈ R, eine
a) Es sei X eine Zufallsvariable mit E|X|p < ∞ für ein p > Dichte g definiert wird.
0. Zeigen Sie: Es gilt E|X|q < ∞ für jedes q ∈ (0, p). b) Ist Y eine Zufallsvariable mit Dichte g, so besitzt Y die
b) Geben Sie ein Beispiel für eine Zufallsvariable X mit charakteristische Funktion
E|X| = ∞ und E|X|p < ∞ für jedes p mit 0 < p < 1
an. 2π
ψ(t) = f (t) , t ∈ R.
c
Aufgabe 22.18 ••• Es sei X eine Zufallsvariable mit
EX 4 < ∞ und EX = 0, EX 2 = 1 = EX 3 . Zeigen Sie: Aufgabe 22.26 ••
EX 4 ≥ 2. Wann tritt hier Gleichheit ein?
a) Es seien X und Y unabhängige und je Exp(1)-verteilte
Zufallsvariablen. Bestimmen Sie Dichte und charakteri-
Aufgabe 22.19 •• Die Zufallsvariablen X1 , X2 , . . .
stische Funktion von Z := X − Y .
seien identisch verteilt, wobei E|X1 | < ∞. Zeigen Sie:
  b) Zeigen Sie: Eine Zufallsvariable mit der Cauchy-
1 Verteilung C(0, 1) besitzt die charakteristische Funktion
lim E max |Xj | = 0 .
n→∞ n j =1,...,n ψ(t) = exp(−|t|), t ∈ R.
Aufgaben zu Kapitel 22 181

c) Es seien X1 , . . . , Xn unabhängig und identisch verteilt Aufgabe 22.34 • Die Zufallsvariable X sei N(μ, σ 2 )-
mit Cauchy-Verteilung C(α, β). Dann gilt: verteilt. Wie groß ist die Wahrscheinlichkeit, dass X vom
Erwartungswert μ betragsmäßig um höchstens das k-Fache
1
n
Xj ∼ C(α, β) . der Standardabweichung σ abweicht, k ∈ {1, 2, 3}?
n
j =1
Aufgabe 22.35 • Zeigen Sie, dass die Verteilungs-
Aufgabe 22.27 ••• Es sei h eine positive reelle Zahl. Die funktion  der Standardnormalverteilung die Darstellung
Zufallsvariable X besitzt eine Gitterverteilung mit Spanne h, ∞
falls ein a ∈ R existiert, sodass PX ({a + hm | m ∈ Z}) = 1 1 1 (−1)k x 2k+1
(x) = +√ , x > 0,
gilt. (Beispiele für a = 0, h = 1: Binomialverteilung, Pois- 2 2π k=0 2k k!(2k + 1)
sonverteilung). Beweisen Sie die Äquivalenz der folgenden
Aussagen: besitzt.

a) X besitzt eine Gitterverteilung mit Spanne h. Aufgabe 22.36 •• Es sei F0 (x) := (1 + exp(−x))−1 ,
  
 2π  x ∈ R.
b) ϕX = 1.
h  a) Zeigen Sie: F0 ist eine Verteilungsfunktion, und es gilt

c) |ϕX (t)| ist periodisch mit Periode . F0 (−x) = 1 − F0 (x) für x ∈ R.
h
Aufgabe 22.28 •• Es sei X eine Zufallsvariable mit b) Skizzieren Sie die Dichte von F0 . Die von F0 erzeugte
charakteristischer Funktion ϕ. Zeigen Sie: Es gilt Lokations-Skalen-Familie heißt Familie der logistischen
Verteilungen. Eine Zufallsvariable X mit der Verteilungs-
1 T −ita funktion
lim e ϕ(t) dt = P(X = a), a ∈ R.
T →∞ 2T −T     
x − a −1 x−a
F (x) = 1 + exp − = F0
Rechenaufgaben σ σ
heißt logistisch verteilt mit Parametern a und σ , σ > 0,
Aufgabe 22.29 •
kurz: X ∼ L(a, σ ).
a) Zeigen Sie, dass die Festsetzung c) Zeigen Sie: Ist F wie oben und f = F  die Dichte von
1 F , so gilt
F (x) := 1 − , x ≥ 0,
1+x 1
f (x) = F (x)(1 − F (x)) .
und F (x) := 0 sonst, eine Verteilungsfunktion definiert. σ
b) Es sei X eine Zufallsvariable mit Verteilungsfunktion F . Die Verteilungsfunktion F genügt also einer logistischen
Bestimmen Sie P(X ≤ 10) und P(5 ≤ X ≤ 8). Differenzialgleichung.
c) Besitzt X eine Dichte?
Aufgabe 22.37 • Die Zufallsvariable X habe die
Aufgabe 22.30 •• Der Zufallsvektor (X, Y ) besitze Gleichverteilung U(0, 1). Welche Verteilung besitzt Y :=
eine Gleichverteilung im Einheitskreis B := {(x, y) : x 2 + 4X(1 − X)?
y 2 ≤ 1}. Welche marginalen Dichten haben X und Y ? Sind
X und Y stochastisch unabhängig? Aufgabe 22.38 •• Die Zufallsvariablen X1 , X2 besit-
zen die gemeinsame Dichte
Aufgabe 22.31 •• Die Zufallsvariable X habe die ste- √  
2 3 2 3 2
tige Verteilungsfunktion F . Welche Verteilungsfunktion be- f (x1 , x2 ) = exp − x1 − x1 x2 − x2 , (x1 , x2 ) ∈ R2 .
π 2 2
sitzen die Zufallsvariablen
a) Bestimmen Sie die Dichten der Marginalverteilungen von
a) X 4 , b) |X|, c) − X? X1 und X2 . Sind X1 , X2 stochastisch unabhängig?
b) Welche gemeinsame Dichte besitzen Y1 := X1 + X2 und
Y2 := X1 − X2 ? Sind Y1 und Y2 unabhängig?
Aufgabe 22.32 • Wie ist die Zahl a zu wählen, damit
die durch f (x) := a exp(−|x|), x ∈ R, definierte Funktion
Aufgabe 22.39 •• Die Zufallsvariablen X, Y seien un-
eine Dichte wird? Wie lautet die zugehörige Verteilungsfunk-
abhängig und je Exp(λ)-verteilt, wobei λ > 0. Zeigen Sie:
tion?
Der Quotient X/Y besitzt die Verteilungsfunktion
Aufgabe 22.33 • Der Messfehler einer Waage kann t
G(t) = , t > 0,
aufgrund von Erfahrungswerten als approximativ normalver- 1+t
teilt mit Parametern μ = 0 (entspricht optimaler Justierung) und G(t) = 0 sonst.
und σ 2 = 0.2025 mg2 angenommen werden. Wie groß ist
die Wahrscheinlichkeit, dass eine Messung um weniger als Aufgabe 22.40 •• In der kinetischen Gastheorie wer-
0.45 mg (weniger als 0.9 mg) vom wahren Wert abweicht? den die Komponenten Vj des Geschwindigkeitsvektors V =
182 Aufgaben zu Kapitel 22

(V1 , V2 , V3 ) eines einzelnen Moleküls mit Masse m als sto- Aufgabe 22.49 • Die Zufallsvariable X besitze die
chastisch unabhängige und je N(0, kT /m)-verteilte Zufalls- Weibull-Verteilung Wei(α, 1). Zeigen Sie: Es gilt
variablen betrachtet. Hierbei bezeichnen k die Boltzmann-  1/α
Konstante 1
* und T die absolute Temperatur. Zeigen Sie, dass X ∼ Wei(α, λ) .
λ
Y := V12 + V22 + V32 die Dichte
D   Aufgabe 22.50 •• Die Zufallsvariable X besitzt die
2  m 3/2 2 m y2
g(y) = y exp − 1(0,∞) (y) Weibull-Verteilung Wei(α, λ). Zeigen Sie:
 
π kT 2kT
" 1 + αk
a) EX k = , k ∈ N.
besitzt (sog. Maxwell’sche Geschwindigkeitsverteilung). λk/α
b) Q1/2 < E X.
Aufgabe 22.41 •• Die gemeinsame Dichte f der Zu-
fallsvariablen X und Y habe die Gestalt f (x, y) = ψ(x 2+y 2 )
Aufgabe 22.51 •• Zeigen Sie, dass eine χk2 -verteilte
mit einer Funktion ψ : R≥0 → R≥0 . Zeigen Sie: Der Quo-
Zufallsvariable X die Dichte
tient X/Y besitzt die Cauchy-Verteilung C(0, 1), also die
1 k x
Dichte
1 fk (x) := x 2 −1 e− 2 , x>0
g(t) = , t ∈ R. 2k/2 "(k/2)
π(1 + t 2 )
und fk (x) := 0 sonst besitzt.

Aufgabe 22.42 • Zeigen Sie unter Verwendung der


Aufgabe 22.52 •• Die Zufallsvariable X besitze die Lo-
Polarmethode (siehe Seite 827), dass der Quotient zweier
gnormalverteilung LN(μ, σ 2 ). Zeigen Sie:
unabhängiger standardnormalverteilter Zufallsvariablen die
Cauchy-Verteilung C(0, 1) besitzt. a) Mod(X) = exp(μ − σ 2 ),
b) Q1/2 = exp(μ),
Aufgabe 22.43 •• Es seien X1 und X2 unabhängige und c) E X = exp(μ + σ 2 /2),
je N(0, 1)-verteilte Zufallsvariablen: Zeigen Sie: d) V(X) = exp(2μ + σ 2 )(exp(σ 2 ) − 1).
 
X1 X2 1
* ∼ N 0, . Aufgabe 22.53 •• Die Zufallsvariable X hat eine Beta-
X12 + X22 4
verteilung mit Parametern α > 0 und β > 0, falls X die
Dichte
Aufgabe 22.44 •• Welche Verteilung besitzt der Quoti- 1
f (x) := x α−1 (1 − x)β−1 für 0 < x < 1
ent X/Y , wenn X und Y stochastisch unabhängig und je im B(α, β)
Intervall (0, a) gleichverteilt sind?
und f (x) := 0 sonst besitzt, und wir schreiben hierfür kurz
X ∼ BE(α, β). Dabei ist
Aufgabe 22.45 •• Der Zufallsvektor (X, Y ) besitze die
Dichte h := 2 1A , wobei A := {(x, y) ∈ R2 | 0 ≤ x ≤ y ≤ 1}, "(α)"(β)
B(α, β) :=
vgl. Seite 850. Zeigen Sie: "(α + β)
a) E X = 13 , E Y = 23 , die in (22.56) eingeführte Euler’sche Betafunktion. Zeigen
b) V(X) = V(Y ) = 1 Sie:
18 ,
1  α+j
k−1
c) Cov(X, Y ) = 36 , ρ(X, Y ) = 21 . a) EX k = , k ∈ N,
α+β +j
j =0
Aufgabe 22.46 • Der Zufallsvektor (X1 , . . . , Xk ) be- α αβ
b) EX = , V(X) = .
sitze eine nichtausgeartete Normalverteilung Nk (μ; ). Zei- α+β (α + β + 1)(α + β)2
gen Sie: Ist eine Diagonalmatrix, so sind X1 , . . . , Xk sto-
c) Sind V und W stochastisch unabhängige Zufallsvaria-
chastisch unabhängig.
blen, wobei V ∼ "(α, λ) und W ∼ "(β, λ), so gilt

Aufgabe 22.47 •• Zeigen Sie, dass in der Situation von V


∼ BE(α, β) .
Abb. 22.23 der zufällige Ankunftspunkt X auf der x-Achse V +W
die Cauchy-Verteilung C(α, β) besitzt.
Aufgabe 22.54 •• Die Zufallsvariable Z besitze eine
Aufgabe 22.48 • Es sei X ∼ C(α, β). Zeigen Sie: Gamma-Verteilung "(r, β), wobei r ∈ N. Die bedingte Ver-
teilung der Zufallsvariablen X unter der Bedingung Z = z,
a) Q1/2 = α, z > 0 sei die Poisson-Verteilung Po(z). Welche Verteilung
b) 2β = Q3/4 − Q1/4 . hat X?
Hinweise zu Kapitel 22 183

Hinweise Aufgabe 22.20 ••• Schätzen Sie die Indikatorfunktion


der Menge A := R2 \ (−ε, ε)2 durch eine geeignete qua-
Aufgabe 22.1 •• – dratische Form nach oben ab.

Aufgabe 22.2 •• – Aufgabe 22.21 ••• Es kann o.B.d.A. a0 = 0 gesetzt wer-


den. Betrachten Sie die Funktion x → |x − a| − |x| getrennt
für a > 0 und a < 0 und schätzen Sie nach unten ab.
Aufgabe 22.3 •• Machen Sie sich eine Skizze!
Aufgabe 22.22 •• Es kann o.B.d.A. EX = 0 angenom-
Aufgabe 22.4 • –
men werden. Dann gilt P(|X| ≥ Q3/4 ) = 0.5.
Aufgabe 22.5 •• –
Aufgabe 22.23 •• Es gilt X ∼ AY + μ mit = AA
und Y ∼ Nk (0, Ik ).
Aufgabe 22.6 • Sie müssen die Kovarianzmatrix nicht
kennen!
Aufgabe 22.24 • –
Aufgabe 22.7 • –
Aufgabe 22.25 •• Verwenden Sie Aufgabe 22.7.
Aufgabe 22.8 ••• –
Aufgabe 22.26 •• Verwenden Sie für b) Teil a) und
Aufgabe 22.25.
Aufgabe 22.9 •• Es ist P(X ∈ (x,y]) = F (y1 , . . . ,yk ) −
P(∪kj =1 Aj ), wobei Aj = {X1 ≤ y1 , . . . , Xj −1 ≤
Aufgabe 22.27 ••• Für die Richtung „b) ⇒ a)“ ist die
yj −1 , Xj ≤ xj , Xj +1 ≤ yj +1 , . . . , Xk ≤ yk }.
Implikation
 
Aufgabe 22.10 •• – 2π
ϕX = eiα ⇒ 0
h
Aufgabe 22.11 •• – ∞  

= 1 − cos x − α PX (dx)
−∞ h
Aufgabe 22.12 •• –
9 hilfreich.
Aufgabe 22.13 •• Der Ansatz kj =1 f (xj ) = g(x12 +
. . . + xk2 ) für eine Funktion g führt nach Logarithmieren Aufgabe 22.28 •• Gehen Sie wie beim Beweis des Sat-
und partiellem Differenzieren auf eine Differenzialgleichung zes über die Umkehrformeln vor.
für f .
Aufgabe 22.29 • –
n
Aufgabe 22.14 •• Bezeichnet NB := j =1 1{Xj ∈ B}
die Anzahl der Xj , die in die Menge B ⊆ R fallen, Aufgabe 22.30 •• –
so besitzt der Zufallsvektor (N(−∞,t) , N[t,t+ε] , N(t+ε,∞) )
die Multinomialverteilung Mult(n; F (t), F (t + ε) − F (t), Aufgabe 22.31 •• –
1 − F (t + ε)). Es gilt P(N[t,t+ε] ≥ 2) = O(ε2 ) für ε → 0.
Aufgabe 22.32 • –
Aufgabe 22.15 •• Integrieren Sie die Indikatorfunk-
tion der Menge B := {(x, y) ∈ R2 : x ≥ 0, 0 ≤ y < x} Aufgabe 22.33 • –
bezüglich des Produktmaßes PX⊗λ1 und beachten Sie dabei
den Satz von Tonelli. Aufgabe 22.34 • Verwenden Sie Tabelle 22.1.

Aufgabe 22.16 •• Setze Y := |X|p . Aufgabe 22.35 • Potenzreihenentwicklung von ϕ!

Aufgabe 22.17 •• – Aufgabe 22.36 •• –



Aufgabe 22.18 ••• Betrachten Sie für a := (1 + 5)/2 Aufgabe 22.37 • Versuchen Sie, direkt die Vertei-
das Polynom p(x) = (x − a)2 (x + 1/a)2 . lungsfunktion G von Y zu bestimmen.

Aufgabe 22.19 •• Verwenden Sie die Darstellungsfor- Aufgabe 22.38 •• –


mel auf Seite 778 und spalten Sie den Integrationsbereich
geeignet auf. Aufgabe 22.39 •• –
184 Lösungen zu Kapitel 22

Aufgabe 22.40 •• Sind Z1 , Z2 , Z3 unabhängig und je Aufgabe 22.9 •• –


N(0, 1)-normalverteilt, so besitzt Z := Z12 + Z22 + Z32 eine
χ32 -Verteilung. Aufgabe 22.10 •• –

Aufgabe 22.41 •• Verwenden Sie Gleichung (22.29) Aufgabe 22.11 •• –


sowie Polarkoordinaten.
Aufgabe 22.12 •• –
Aufgabe 22.42 • –
Aufgabe 22.13 •• –
Aufgabe 22.43 •• Polarmethode!
Aufgabe 22.14 •• –
Aufgabe 22.44 •• Die Verteilung hängt nicht von a ab.
Aufgabe 22.15 •• –
Aufgabe 22.45 •• –
Aufgabe 22.16 •• Die Aussagen sind äquivalent.
Aufgabe 22.46 • Welche Gestalt besitzt die gemein-
same Dichte von X1 , . . . , Xk ? Aufgabe 22.17 •• –
Aufgabe 22.47 •• –
Aufgabe 22.18 ••• Es gilt
Aufgabe 22.48 • –  
2 1
EX 4 = 2 ⇐⇒ P(X = a) = √ = 1−P X = − .
5+ 5 a
Aufgabe 22.49 • –
Aufgabe 22.19 •• –
Aufgabe 22.50 •• –
Aufgabe 22.20 ••• –
Aufgabe 22.51 •• Verwenden Sie die Faltungsformel.
Aufgabe 22.21 ••• –
Aufgabe 22.52 •• Für c) und d) ist bei Integralberech-
nungen die Substitution u = log x hilfreich.
Aufgabe 22.22 •• –
Aufgabe 22.53 •• a) Verwenden Sie (22.56) und die
Gleichung "(t + 1) = t"(t), t > 0. c) Bestimmen Sie zu- Aufgabe 22.23 •• –
nächst die Dichte von W/V .
Aufgabe 22.24 • –
Aufgabe 22.54 •• –
Aufgabe 22.25 •• –

Aufgabe 22.26 •• Es ist ϕZ (t) = 1/(1 + t 2 ), t ∈ R.


Lösungen
Aufgabe 22.27 ••• –
Aufgabe 22.1 •• –
Aufgabe 22.28 •• –
Aufgabe 22.2 •• –
Aufgabe 22.29 • b) P(X ≤ 10) = 10/11, P(5 ≤ X ≤
8) = 1/18.
Aufgabe 22.3 •• Die Verteilungsfunktion von Y ist
c) Ja.
G(y) = 21 + π1 arcsin y, −1 ≤ y ≤ 1.

Aufgabe 22.4 • – Aufgabe 22.30 •• f (x) = 2 1 − x 2 /π für |x| ≤ 1. X
und Y sind nicht unabhängig.
Aufgabe 22.5 •• –
Aufgabe 22.31 •• a) F (t 1/4 ) − F (−t 1/4 ) für t ≥ 0 b)
Aufgabe 22.6 • – F (t) − F (−t) für t ≥ 0 c) 1 − F (−t), t ∈ R.

Aufgabe 22.7 • – Aufgabe 22.32 • a = 1/2. Die Verteilungsfunktion ist


F (x) = 1 − exp(−x)/2 für x ≥ 0 und F (x) = 1 − F (−x)
Aufgabe 22.8 ••• – für x < 0.
Lösungswege zu Kapitel 22 185

Aufgabe 22.33 • 2(1) − 1 ≈ 0.6826 (2(2) − 1 ≈ Lösungswege


0.9544).

Aufgabe 22.34 • k = 1: 0.6826, k = 2: 0.9544, Verständnisfragen


k = 3: 0.9974 Aufgabe 22.1 •• a) Wegen {X ≤ b} = {X ≤ a}+{a <
X ≤ b} liefert die Additivität von P die Gleichung
Aufgabe 22.35 • –
F (b) = F (a) + P(a < X ≤ b),
Aufgabe 22.36 •• –
√ woraus a) folgt.
Aufgabe 22.37 • Es gilt G(y) = 1− 1 − y, 0 ≤ y ≤ 1. b) Gilt xn → x, wobei xn ≤ xn+1 , n ≥ 1, so bilden die
Mengen An := (−∞, xn ], n ≥ 1, eine aufsteigende Folge
Aufgabe 22.38 •• a) Die Dichte von X1 (und von X2 )
mit ∪∞
n=1 An = (−∞, x). Da P stetig von unten ist, ergibt
X
ist  
1 x12 sich
f1 (x1 ) = √ exp − 2 , x1 ∈ R.
σ 2π 2σ P(X < x) = PX ((−∞, x)) = lim PX (An ) = lim F (xn ).
n→∞ n→∞
X1 und X2 sind nicht stochastisch unabhängig.
und damit wegen P(X < x) = F (x−) die Behauptung.
b) Die gemeinsame Dichte von Y1 und Y2 ist
  Aufgabe 22.2 •• Es seien X eine Zufallsvariable mit
1 2 y22
g(y1 , y2 ) = √ exp −y1 − . Verteilungsfunktion F und D die Menge der Unstetigkeits-
π 2 2
stellen von F . Wegen der Monotonie von F ist jedes x ∈ D
Y1 und Y2 sind stochastisch unabhängig. eine Sprungstelle von F . Es gilt also D = {x ∈ R | P(X = x)
> 0}. Mit Dn := {x ∈ R | P(X = x) ≥ 1/n} stellt sich D in
Aufgabe 22.39 •• – der Form

4
D= Dn
Aufgabe 22.40 •• –
n=1

Aufgabe 22.41 •• – dar. Da die Menge Dn wegen der Normierungsbedingung für


ein Wahrscheinlichkeitsmaß höchstens n Elemente enthält,
Aufgabe 22.42 • – ist D als abzählbare Vereinigung endlicher Mengen abzähl-
bar. Ein alternativer Beweis verwendet, dass die Sprunginter-
Aufgabe 22.43 •• – valle (F (x−), F (x)] von F für verschiedene Unstetigkeits-
stellen x paarweise disjunkt sind, jedes Intervall mindestens
Aufgabe 22.44 •• Die Dichte von X/Y ist eine rationale Zahl enthält und die Menge der rationalen Zah-
1 len abzählbar ist.
g(t) = · (min(1, 1/t))2 für t > 0 und g(t) = 0 sonst.
2
Aufgabe 22.3 •• Die Zufallsvariable X nimmt Werte
Aufgabe 22.45 •• – im Intervall [−1, 1] an. Bezeichnet G die Verteilungsfunk-
tion von Y , so gilt zunächst
Aufgabe 22.46 • –
π 1
G(0) = P(Y ≤ 0) = P(π ≤ X ≤ 2π ) = = .
Aufgabe 22.47 •• – 2π 2
Für y mit 0 < y ≤ 1 ist das Ereignis {Y ≤ y} die Vereinigung
Aufgabe 22.48 • –
der drei sich paarweise ausschließenden Ereignisse {π ≤
X ≤ 2π}, {X ≤ arcsin y} und {π − arcsin y ≤ X < π}, vgl.
Aufgabe 22.49 • –
nachstehende Abbildung.
Aufgabe 22.50 •• – f (x ) = sinx
1
Aufgabe 22.51 •• – y

Aufgabe 22.52 •• –
x
− arcsin y
arcsin y

Aufgabe 22.53 •• –

Aufgabe 22.54 •• Die negative Binomialverteilung


−1
NB(r, p) mit p = β/(1 + β)).
186 Lösungswege zu Kapitel 22

Da die Wahrscheinlichkeit, dass X in ein Intervall fällt, gleich konvexe Funktion g(x) = − log x, und im Fall c) ist M = R
der durch 2π dividierten Intervalllänge ist, folgt sowie g(x) = exp(x). Da alle Funktionen strikt konvex sind
und die Verteilung von X nicht in einem Punkt degeneriert
π arcsin y π − (π − arcsin y) ist, folgt die Behauptung.
G(y) = + +
2π 2π 2π
1 1 Aufgabe 22.6 • Für einen Zufallsvektor X =
= + arcsin y.
2 π (X1 , . . . , Xs ) ∼ Mult(n; p1 , . . . , ps ) gilt
Aus Symmetriegründen gilt G(y) + G(−y) = 1, sodass man ⎛ ⎞
die Verteilungsfunktion auch für y ∈ [−1, 0) zu s
P⎝ Xj = n⎠ = 1
  j =1
1 1
G(y) = 1 − + arcsin(−y)
2 π (vgl. die Definition der Multinomialverteilung auf Seite 788).
1 1 Es besteht also eine lineare Beziehung zwischen den Kom-
= + arcsin y
2 π ponenten von X, was die Behauptung zeigt.
erhält. Es gilt somit
Aufgabe 22.7 • „⇒“: Besitzen X und −X dieselbe
1 1 Verteilung, so besitzen sie auch dieselbe charakteristische
G(y) = + arcsin y, −1 ≤ y ≤ 1. Funktion. Es gilt also ϕX (t) = ϕ−X (t), t ∈ R. Wegen
2 π
ϕ−X (t) = ϕX (t) ist ϕX (t) reellwertig.
Natürlich gilt G(y) = 0 für y < −1 und G(y) = 1 für
y > 1. Die Dichte von Y ist „⇐“: Gilt ϕX (t) ∈ R ∀ t ∈ R, so ist
1 ϕX (t) = ϕX (t) = ϕ−X (t), t ∈ R.
g(y) = , |y| < 1,
π(1 − y 2 )
Somit besitzen X und −X dieselbe charakteristische Funk-
und g(y) = 0 sonst. tion und damit nach dem Eindeutigkeitssatz auf Seite 857
f (x ) = sinx dieselbe Verteilung.
1
y
Beweisaufgaben
Aufgabe 22.8 ••• a) Es seien M := {x ∈ R | F (x) =
x G(x)} und x0 ∈ R beliebig. Da M in R dicht liegt, gibt es
− arcsin y
arcsin y

eine Folge (xn ) aus M, die von rechts gegen x0 konvergiert.


Wegen der rechtsseitigen Stetigkeit von F und G in x0 folgt
−1 F (x0 ) = limn→∞ F (xn ) = limn→∞ G(xn ) = G(x0 ).
Verteilungsfunktion von sin X b) Wir zeigen zunächst, dass das Komplement W (F )c offen
ist, womit W (F ) als abgeschlossen nachgewiesen wäre. Ist
Aufgabe 22.4 • Das Ereignis {maxj =1,...,n Xj ≤ t} x ∈ W (F )c , so gibt es ein ε > 0 mit F (x+ε)−F (x−ε) = 0.
tritt genau dann ein, wenn jedes der Ereignisse {Xj ≤ t}, Dann gehört aber auch jedes y mit |y − x| < ε/2 zu W (F )c ,
j = 1, . . . , n, eintritt; es gilt also denn es gilt wegen der Monotonie von F
 <
n F (y + ε/2) − F (y − ε/2) ≤ F (x + ε) − F (x − ε) = 0.
max Xj ≤ t = {Xj ≤ t}.
j =1,...,n
j =1 Um W (F ) = ∅ zu zeigen, können wir o.B.d.A. annehmen,
dass F stetig ist, denn andernfalls gäbe es mindestens eine
Behauptung a) folgt somit wegen der vorausgesetzten sto- Sprungstelle und damit einen Wachstumspunkt. Dann gibt
chastischen Unabhängigkeit. In gleicher Weise gilt es wegen F (x) → 0 bei x → −∞ und F (x) → 1 bei
 <
n x → ∞ ein x0 ∈ R mit F (x0 ) = 1/2. Wäre W (F ) = ∅, so
min Xj > t = {Xj > t} gäbe es zu jedem y ∈ R ein von y abhängendes ε > 0 mit
j =1,...,n F (y+ε)−F (y−ε) = 0, also insbesondere F (y+ε) = F (y).
j =1
9n Setzen wir A := {x ∈ R | F (x) = 1/2}, so ist A = ∅, denn
und somit P(minj =1,...,n Xj > t) = j =1 (1 − Fj (t)), was es gilt x0 ∈ A. Die Menge A ist nach oben unbeschränkt, denn
b) beweist. wäre u := sup A < ∞, so gälte zunächst wegen der Stetig-
keit von F die Gleichung F (u) = 1/2. Wegen der Annahme
Aufgabe 22.5 •• Die Voraussetzung der Jensen- W (F ) = ∅ gäbe es ein ε > 0 mit F (u + ε) = F (u) = 1/2,
Ungleichung auf Seite 834 ist erfüllt. In a) ist M = (0, ∞) was der Definition von u als Supremum der Menge A wi-
und g(x) = 1/x. Bei b) betrachte man die auf M = (0, ∞) derspräche. Folglich ist A nach oben unbeschränkt. Diese
Lösungswege zu Kapitel 22 187

Eigenschaft widerspricht aber der Konvergenz F (x) → 1 im Fall u = i/m (bzw. . . . = (j − i)/m im Fall u > i/m).
bei x → ∞. Wegen
c) Es seien Q =: {q1 , q2 , . . .} eine Aufzählung von Q, X eine j −i j +1−i
Zufallsvariable mit P(X = qj ) = 2−j für j ≥ 1 und F die ≤ v−u ≤
m m
Verteilungsfunktion von X. Wegen P(X ∈ Q) = 1 und der
Abzählbarkeit von Q ist F eine diskrete Verteilungsfunktion. im Fall u = i/m (bzw. (j −i−1)/m < v−u < (j +1−i)/m
Zu beliebigem x ∈ R und beliebigem ε > 0 gibt es minde- im Fall u > i/m) folgt die Behauptung.
stens eine rationale Zahl im Intervall (x − ε, x + ε]. Folglich
ist 0 < P(x − ε < X ≤ x + ε) = F (x + ε) − F (x − ε), was
Aufgabe 22.11 •• a) Wir zeigen die Behauptung durch
W (F ) = R zeigt.
Induktion über n, wobei der Induktionsanfang n = 1 unmit-
telbar klar ist. Der Induktionsschluss n → n + 1 folgt we-
Aufgabe 22.9 •• Die im Hinweis gegebene Darstellung
gen r1 . . . rn+1 − s1 . . . sn+1 = (r1 . . . rn − s1 . . . sn )rn+1 +
spiegelt die Tatsache wider, dass das Ereignis {X ≤ x} die
s1 . . . sn (rn+1 − sn+1 ) aus der Dreiecksungleichung und
Vereinigung der disjunkten Ereignisse {X ∈ (x, y]} und
0 ≤ rj , sj ≤ 1.
∪nj=1 Aj ist. Nach der Formel des Ein- und Ausschließens
auf Seite 717 gilt b) Wenden wir (22.16) auf rj := Pm ({aj ∈ m : uj ≤ aj ≤
⎛ ⎞ vj }) und sj := vj − uj an, wobei 0 ≤ uj < vj ≤ 1,
4
k k
P⎝ Aj ⎠ = (−1)r−1 Sr , j = 1, . . . , n, so ergibt sich unter Beachtung von (22.15) die
j =1 r=1
Behauptung.

wobei Aufgabe 22.12 •• a) Durch Induktion nach s ergibt sich


  aus dem iterativen Kongruenzschema unmittelbar
Sr = P Ai1 ∩ . . . ∩ Air .
1≤i1 <...<ir ≤n zi+s − zs ≡ a s (zi − z0 ) (mod m), i, s ≥ 0
Nun ist
ρ 1−ρ1 ρ 1−ρk
und somit die Behauptung. Dabei ist die Kongruenz modulo
{Ai1 ∩ . . . ∩ Air } = {X1 ≤ y1 1 x1 , . . . , Xk ≤ yk k xk }, m komponentenweise zu verstehen.
wobei ρ := (ρ1 , . . . , ρk ) ∈ {0, 1}k mit ρν = 0 für ν ∈ b) Nach Definition der Kongruenzrelation folgt aus a)
{i1 , . . . , ir } und ρν = 1 sonst. Nach Definition der Vertei- ⎛ ⎞ ⎛ ⎞
lungsfunktion ist damit 1 k1
⎜ ⎟ ⎜ ⎟
ρ 1−ρ1 ρ 1−ρk ⎜ ⎟ ⎜ ⎟
P(Ai1 ∩ . . . ∩ Air ) = F (y1 1 x1 , . . . , yk k xk ). ⎜ a ⎟ ⎜ k2 ⎟
Zi − Z0 = (zi − z0 ) ⎜
⎜ ..
⎟ + m⎜
⎟ ⎜ ..
⎟,
⎟ i ≥ 0,
Schreiben wir s(ρ) := ρ1 + . . . + ρk für die Anzahl der ⎜ . ⎟ ⎜ . ⎟
⎝ ⎠ ⎝ ⎠
Einsen im Tupel ρ, so folgt
a d−1 kd
ρ 1−ρ ρ 1−ρ
Sr = F (y1 1 x1 1 , . . . , yk k xk k ),
ρ∈{0,1}k : s(ρ)=k−r mit ganzen Zahlen k1 , . . . , kd . Die Differenz Zi −Z0 ist somit
eine ganzzahlige Linearkombination der d + 1 Vektoren
und somit
⎛ ⎞ ⎛ ⎞ ⎛ ⎞ ⎛ ⎞
P(X ∈ (x, y]) 1 m 0 0
⎜ ⎟ ⎜ ⎟ ⎜ ⎟ ⎜ ⎟
= F (y1 , . . . , yk ) − ⎜ ⎟ ⎜ ⎟ ⎜ ⎟ ⎜ ⎟
⎜ a ⎟ ⎜ 0 ⎟ ⎜ m ⎟ ⎜ 0 ⎟

k ⎜ ⎟, ⎜ ⎟, ⎜ ⎟, ..., ⎜ ⎟.
⎜ .. ⎟ ⎜ .. ⎟ ⎜ .. ⎟ ⎜ .. ⎟
− (−1)r−1
ρ
F (y1 1 x1
1−ρ1 ρ
, . . . , yk k xk
1−ρk
) ⎜ . ⎟ ⎜ . ⎟ ⎜ . ⎟ ⎜ . ⎟
⎝ ⎠ ⎝ ⎠ ⎝ ⎠ ⎝ ⎠
r=1 ρ∈{0,1}k : s(ρ)=k−r
a d−1 0 0 m
ρ 1−ρ1 ρ 1−ρk
= (−1)k−s(ρ) F (y1 1 x1 , . . . , yk k xk )
ρ∈{0,1}k
Da der Vektor (m, 0, . . . , 0) als ganzzahlige Linearkombi-
y
= x F, nation der übrigen Vektoren sogar redundant ist, liegen die
Differenzen Zi − Z0 für jedes i in der Menge G , die ein Gitter
was zu zeigen war. im Rd darstellt.
Aufgabe 22.10 •• Zu u, v ∈ [0, 1] mit 0 ≤ u < v ≤ 1 Die vom Pseudozufallszahlengenerator erzeugten Punkte
existieren i, j ∈ Z mit 0 ≤ i ≤ j ≤ m − 1 und i/m ≤ u < (xi , xi+1 , . . . , xi+d−1 ) , i ≥ 0, liegen somit aufgrund der
(i + 1)/m, j/m ≤ v < (j + 1)/m. Es gilt Normierungsvorschrift xi = zi /m auf einem Gitter, das sich
aus G durch Verschiebung um Z0 und Skalierung mit dem
Pm ({a ∈ m : u ≤ a ≤ v}) = (j − i + 1)/m Faktor 1/m ergibt.
188 Lösungswege zu Kapitel 22

Aufgabe 22.13 •• Aus Teilt man hier durch ε, so folgt wegen


F (t + ε) − F (t)

k   lim = f (t)
f (xj ) = g x12 + . . . + xk2 ε→0 ε
j =1 und F (t + ε) → F (t) bei ε → 0 die Behauptung.

für eine Funktion g : [0, ∞) → (0, ∞) folgt Aufgabe 22.15 •• Führt man die im Hinweis angege-
bene Integration aus, so ergibt sich

k   ∞ x
log f (xj ) = log g x12 + . . . + xk2 1B dPX⊗λ1 = 1B (x, y)λ1 (dy)PX (dx)
j =1
0 ∞ 0
und somit für festes m ∈ {1, . . . , k} = x PX (dx).
0

f  (xm ) g  (x12 + . . . + xk2 ) Integriert man in umgekehrter Reihenfolge, so folgt wegen


= 2xm , (x1 , . . . , xk ) ∈ Rk . 1 − F (y) = PX ((y, ∞))
f (xm ) g(x12 + . . . + xk2 ) ∞ ∞
1B dPX ⊗λ1 = 1B (x, y)PX (dx)λ1 (dy)
Es existiert somit ein c  = 0 mit 0 y

f  (t) = (1 − F (y)) dy.
= c, t ∈ R \ {0}. 0
2tf (t)
Nach dem Satz von Tonelli liefert iterierte Integration in un-
Diese Differenzialgleichung besitzt die allgemeine Lösung terschiedlicher Reihenfolge das gleiche Ergebnis, woraus
  ∞ ∞
f (t) = K exp ct 2 , t ∈ R, (1 − F (y)) dy = x PX (dx)
0 0

für ein K > 0. Da f eine Wahrscheinlichkeitsdichte ist, kann folgt. Integriert man die Indikatorfunktion der Menge
nur c < 0 gelten. Hieraus folgt die Behauptung. A := {(x, y) ∈ R2 | −∞ ≤ x ≤ y ≤ 0}
Aufgabe 22.14 •• Wir überlegen uns zunächst, dass bezüglich PX ⊗ λ1 , so erhält man in gleicher Weise
die zweite Aussage im Hinweis richtig ist. Wegen Nε := 0 0
N[t,t+ε] ∼ Bin(n, pε ), wobei pε = F (t +ε)−F (t) ∼ f (t)ε − F (y) dy = x PX (dx).
für ε → 0 gilt −∞ −∞

Aufgrund der Äquivalenz


P(Nε ≥ 2) = 1 − P(Nε = 0) − P(Nε = 1) ∞
= 1 − (1 − pε )n − npε (1 − pε )n−1 E|X| < ∞ ⇐⇒ |x| PX (dx) < ∞
  −∞
= 1 − 1 − npε + O(pε2 )
und
  ∞ 0
−npε 1 − (n − 1)pε + O(pε2 ) EX = x PX (dx) + x PX (dx)
0 −∞
= O(pε2 ) folgt die Behauptung.
= O(ε2 ).
Aufgabe 22.16 •• Sei Y := |X|p . Wir zeigen
Folglich ist ∞

E Y < ∞ ⇐⇒ P(Y > n) < ∞,
P(t ≤ Xr:n ≤ t + ε)=P(t ≤ Xr:n ≤ t + ε, Nε = 1) n=1
+P(t ≤ Xr:n ≤ t + ε, Nε ≥ 2) sodass a) und b) in der Tat äquivalent sind. Sei hierzu G(t) :=
=P(t ≤ Xr:n ≤ t + ε, Nε = 1) + O(ε ). 2 P(Y ≤ t), t ∈ R, die Verteilungsfunktion von Y . Wegen der
Monotonie von G gilt für jedes k ∈ N unter Verwendung der
Das Ereignis {t ≤ Xr:n ≤ t + ε, Nε = 1} tritt genau dann Darstellungsformel
ein, wenn r − 1 der Xj in das Intervall (−∞, t) fallen, ge-
k
k
nau eines der Xj im Intervall [t, t + ε] liegt und n − r der P(Y > n) = (1 − G(n))
Xj größer als t + ε sind. Gleichbedeutend hiermit ist, dass n=1 n=1
für den im Hinweis eingeführten Zufallsvektor das Ereignis k
{(N(−∞,t) = r −1, N[t,t+ε] = 1, N(t+ε,∞) = n−r)} eintritt, ≤ (1 − G(t)) dt
0
und die Wahrscheinlichkeit hierfür ist ∞
≤ (1 − G(t)) dt
n!
F (t)r−1 (F (t +ε)−F (t))(1−F (t +ε))n−r . 0
(r − 1)!(n − r)! = E Y,
Lösungswege zu Kapitel 22 189

was „⇒“ zeigt, wenn man k gegen unendlich streben lässt. Mit √
Die umgekehrte Implikation folgt aus der Darstellungsformel 1 1− 5
=−
und der für jedes k ∈ N geltenden Ungleichungskette a 2
gilt    
k+1 k+1
n 1 1
(1 − G(t)) dt = (1 − G(t)) dt 2 a− − a 2 −4+ 2 − 1 = 2,
0
a a
n=1 n−1

k+1 und somit folgt EX 4 ≥ 2. Weiter gilt
≤ (1 − G(n − 1))  
1
n=1 EX 4 = 2 ⇐⇒ P(X = a) + P X = − = 1.
a

k
= 1 − G(0) + P(Y > n) Setzen wir p := P(X = a), so liefert die Bedingung EX = 0
n=1 die Gleichung

 
≤ 1+ P(Y > n) 1 1 1
n=1
0 = p a − (1 − p) = p a + −
a a a
beim Grenzübergang k → ∞. und somit
1 2
p= = √ .
Aufgabe 22.17 •• 1 + a2 5+ 5
a) Es gilt (punktweise auf ) |X|q ≤ 1 + |X|p . Hieraus Direktes Nachrechnen ergibt, dass hiermit auch die Bedin-
folgt gungen EX 2 = EX 3 = 1 erfüllt sind.
E|X|q ≤ 1 + E|X|p < ∞.
Aufgabe 22.19 •• Zunächst existiert der rechts ste-
b) Es sei X ∼ C(0, 1) mit Dichte f (x) = 1/(π(1 + x 2 )), hende Erwartungswert, denn es ist
x ∈ R. Dann gilt

n
∞ max |Xj | ≤ |Xj |
|x| j =1,...,n
E|X| = dx = ∞. j =1
−∞ π(1 + x 2 )
und E|Xj | = E|X1 | < ∞. Nach der Darstellungsformel auf
Ist p ∈ (0, 1), so folgt Seite 837 gilt für jedes a mit 0 < a < ∞
∞   ∞  
2 xp
E|X|p = dx, E max |Xj | = P max |Xj | > t dt
π 0 1 + x2 j =1...,n 0 j =1...,n
∞  
und wegen ≤ a+ P max |Xj | > t dt.
j =1...,n
∞ ∞
a
xp 1
dx ≤ 1 + dx < ∞ Aufgrund der identischen Verteilung der Xj gilt
0 1 + x2 1 x 2−p
⎛ ⎞
  4n
ergibt sich E|X|p < ∞.
P max |Xj | > t = P ⎝ {|Xj | > t}⎠
j =1...,n
j =1
Aufgabe 22.18 ••• Multipliziert man das im Hinweis
angegebene Polynom aus, so ergibt sich
n
 
≤ P |Xj | > t
      j =1
1 3 1 1
p(x) = x 4 −2 a − x + a 2 −4+ 2 x 2 +2 a − x+1. = n P (|X1 | > t)
a a a

Wegen p(x) ≥ 0, x ∈ R, folgt aufgrund der Monotonie und und somit – wenn wir a := n setzen –
Linearität der Erwartungswertbildung   ∞

E max |Xj | ≤ n + n √ P (|X1 | > t) dt.
j =1...,n
0 ≤ Ep(X) n
 
4 1 Dividiert man beide Seiten durch n, so folgt die Behauptung,
= EX − 2 a − EX 3
a denn wegen
   
1 1 ∞
+ a 2 −4+ 2 EX 2 + 2 a − EX + 1
a a E|X1 | = P (|X1 | > t) dt < ∞
    0
1 1
= EX 4 − 2 a − + a 2 −4+ 2 + 1. ∞

a a gilt n P (|X1 | > t) dt → 0 für n → ∞.
190 Lösungswege zu Kapitel 22

Aufgabe 22.20 ••• Wir setzen Wegen P(X ≤ 0) ≥ 1/2 folgt


X j − E Xj E(|X − a| − |X|)
Yj :=  , j = 1, 2.
V(Xj ) = (|x − a| − |x|) PX (dx)

Dann gilt E Yj = 0, V(Yj ) = 1 (j = 1, 2) sowie ρ =
E(Y1 Y2 ). Zu zeigen ist
≥ a 1(−∞,0] (x)PX (dx) − a 1(0,∞) (x)PX (dx)
⎛ ⎞  = a (P(X ≤ 0) − P(X > 0))
4
2
2 3 1 + 1 − ρ2
P⎝ |Yj | > ε ⎠ ≤ . ≥ 0,
ε2
j =1 also E(|X − a|) ≥ E|X|. Für a < 0 gilt

Für a ∈ R mit |a| < 1 ist die quadratische Form ⎪
⎨a, falls x < a,
⎛ ⎞⎛ ⎞ |x − a| − |x| = 2x − a, falls a ≤ x < 0,
  ⎪

1 1 −a x −a, falls 0 ≤ x
g(x1 , x2 ) := x1 x2 ⎝ ⎠ ⎝ 1⎠
1 − a 2 −a 1 #
x2 a, falls x < 0,

x12 − 2ax1 x2 + x22 −a, falls x ≥ 0.
=
1 − a2 Hieraus folgt
positiv definit. Wegen E(|X − a| − |X|)

(−ax1 + x2 )2 (−ax2 + x1 )2 = (|x − a| − |x|) PX (dx)
g(x1 , x2 ) = x12 + = x22 +
1−a 2 1 − a2
≥ a 1(−∞,0) (x)PX (dx) − a 1[0,∞) (x)PX (dx)
gilt weiter
= a (P(X < 0) − P(X ≥ 0))
1
1{R2 \ (−ε, ε)2 } ≤ g(x1 , x2 ), (x1 , x2 ) ∈ R2 . ≥ 0,
ε2
da P(X < 0) ≤ 1/2 und P(X ≥ 0) ≥ 1/2. Es gilt also
Die Monotonie des Erwartungswertoperators liefert somit
⎛ ⎞ E|X| ≤ E|X − a|, a ∈ R,
42
2 3  
P⎝ |Yj | > ε ⎠ = E 1{R2 \ (−ε, ε)2 }(Y1 , Y2 ) was zu zeigen war.
j =1
Aufgabe 22.22 •• Da sich der Quartilsabstand und die
1 Varianz unter Translationen X → X + a nicht ändern,
≤ 2 E [g(Y1 , Y2 )]
ε kann o.B.d.A. angenommen werden, dass X symmetrisch
1 EY12 − 2a E(Y1 Y2 ) + EY22 um 0 verteilt ist. Wegen X ∼ −X folgt dann EX = 0 und
= 2
ε 1 − a2 Q1/4 = −Q3/4 . Aufgrund der Voraussetzung über F gilt
1 2 − 2aρ weiter Q3/4 > 0. Die Tschebyschow-Ungleichung liefert
= 2 .
ε 1 − a2 1 V(X)
= P(|X| ≥ Q3/4 ) ≤
Setzt man speziell 2 Q23/4

ρ und somit Q3/4 ≤ 2V(X). Wegen
a=  ,
1 + 1 − ρ2 Q3/4 − Q1/4 = 2 Q3/4
so folgt die Behauptung für den Fall ρ 2  = 1. Im Fall ρ 2 = folgt die Behauptung.
1 gilt P(|Y1 | = |Y2 |) = 1, sodass sich die Behauptung in
diesem Fall aus der Tschebyschow-Ungleichung ergibt. Aufgabe 22.23 •• Mit dem Hinweis gilt
(X − μ) −1
(X − μ) ∼ (AY) (AA )−1 AY
Aufgabe 22.21 ••• Es sei a0 := 0 gesetzt. Für a > 0 = Y A (A )−1 A−1 AY
gilt
= Y Y

⎪ falls x ≤ 0, k
⎨a, = Yj2 ,
|x − a| − |x| = −2x + a, falls 0 < x ≤ a,

⎩ j =1
−a, falls a < x
# wobei Y = (Y1 , . . . , Yk) .
Da Y1 , . . . , Yk stochastisch un-
a, falls x ≤ 0, abhängig und je N(0, 1)-verteilt sind, folgt die Behauptung

−a, falls x > 0. nach Definition der χk2 -Verteilung.
Lösungswege zu Kapitel 22 191

Aufgabe 22.24 • a) Es gilt b) Es ist



c := ϕZ (t) dz = π,
ϕX (−t) = e−itx PX (dx) −∞
und somit ist die durch
= eitx PX (dx) 1 1
g(t) := ϕZ (t) = , t ∈ R,
c π(1 + t 2 )
= ϕX (t).
definierte Funktion eine Dichte (die Dichte der Cauchy-
Dabei folgt das letzte Gleichheitszeichen aus der Definition Verteilung C(0, 1)). Nach Aufgabe 22.25 b) hat eine Zufalls-
des Erwartungswertes einer C-wertigen Zufallsvariablen. variable Y mit der Cauchy-Verteilung C(0, 1) die charakte-
b) Es gilt ristische Funktion
    2πfZ (t)
ϕaX+b (t) = E eit (aX+b) = eitb E eiatX ψ(t) = = e−|t| , t ∈ R.
c
= eitb ϕX (at). c) Es gilt Xj ∼ βYj + α, j = 1, . . . , n, wobei Yj ∼
C(0, 1). Nach Eigenschaft d) charakteristischer Funktionen
Aufgabe 22.25 •• a) Wegen ϕ(0) = 1 und der Stetig- auf Seite 855 hat Xj die (von j unabhängige) charakteristi-
keit von ϕ gilt c = ϕ(t) dt > 0. Da ϕ reellwertig und sche Funktion
nichtnegativ ist, ist g eine Dichte. b) Sei Y eine Zufallsva- ϕ(t) = eitα ϕYj (βt) = eitα e−β|t| , t ∈ R.
riable mit der Dichte g. Nach der Umkehrformel für Dichten d) Nach der Multiplikationsformel für charakteristische
auf Seite 856 gilt für die charakteristische Funktion von Y n
Funktionen hat X
j =1 j die charakteristische Funktion

2π 1 exp(inαt) exp(−nβ|t|). Wiederum nach Eigenschaft d) auf
ψ(t) = itx
e g(x) dx = e−i(−t)x ϕ(x) dx 
c 2π Seite 855 besitzt das arithmetische Mittel n−1 nj=1 Xj die
2π charakteristische Funktion ϕ. Das arithmetische Mittel hat
= f (−t). also die gleiche Verteilung wie jeder einzelne Summand.
c
Wegen der Reellwertigkeit von ϕ gilt nach Aufgabe 22.7 Aufgabe 22.27 ••• „a) ⇒ c)“: Aus der Voraussetzung
folgt mit pa,h,m := P(X = a + hm) die Darstellung
f (t) = f (−t), t ∈ R,


und damit ϕX (t) = pa,h,m eit (a+hm)

ψ(t) = f (t), t ∈ R. m=−∞
c
und somit
    ∞   
Aufgabe 22.26 •• a) Nach Teil a) des Satzes auf  2π   2π 
ϕX t +k = pa,h,m exp i(a + hm) t +k .
h 
Seite 831 besitzt Z die Dichte
∞ h m=−∞
fZ (t) = fX (t + s)fY (s) ds. Wegen exp(2π ikm) = 1 ergibt sich
−∞
  
Wegen fX (u) = fY (u) = exp(−u) für u ≥ 0 und fX (u) = 2π
exp i(a + hm) t +k = ei(a+hm)t e2πiak/ h
fY (u) = 0 sonst, folgt h
⎧ ∞
⎪ e−t und deshalb

⎨ e−(t+s) e−s ds = , t ≥ 0,   
2π    
k 
2  
fZ (t) = 0 ∞ t ϕX t +k  =  exp 2π i  |ϕX (t)|

⎪ e h h
⎩ e−(t+s) e−s ds = , t < 0,
−t 2 = |ϕX (t)|.
und somit
1 −|t| „c) ⇒ b)“: Diese Implikation folgt unmittelbar wegen
fZ (t) = e , t ∈ R. ϕX (0) = 1.
2
Die charakteristische Funktion von Z ist „b) ⇒ a)“: Sei ϕX (2π/ h) =: eiα mit 0 ≤ α < 2π . Es folgt

1  
ϕZ (t) = eitx e−|x| dx 2π
−∞ 2 0 = 1 − ϕX e−iα
 ∞  h
1 0  
= (it+1)x
dx + (it−1)x 2π
2
e e dx = 1 − exp i x e−iα PX (dx)
−∞ 0 h
     
1 1 1 2π
= + = 1 − exp i x−α PX (dx)
2 1 + it 1 − it h
  
1 2π
= . = 1 − cos x − α PX (dx).
1 + t2 h
192 Lösungswege zu Kapitel 22

Das letzte Gleichheitszeichen gilt wegen 0 ∈ R. Da der Aufgabe 22.30 •• X und Y haben die gemeinsame
Integrand nichtnegativ ist und die Gleichung 0 = 1 − Dichte
cos(2πx/ h − α) zu 1
h(x, y) := , falls x 2 + y 2 ≤ 1
 π

αh  und h(x, y) := 0 sonst. Wegen h(x, y) = 0, falls |x| > 1
x∈ + mhm ∈ Z
2π oder |y| > 1 ergibt sich die marginale Dichte von X aus der
gemeinsamen Dichte gemäß (22.14) zu
äquivalent ist, folgt die Behauptung mit a := αh/(2 π ),
denn ganz allgemein
gilt ja für Maßintegrale die Implikation √1−x 2
„f ≥ 0 und f dμ = 0 ⇒ f = 0 μ-f.ü.“ (siehe Folgerung 1 2
f (x) = √ 1 dy = 1 − x 2 , (22.17)
a) auf Seite 245). π − 1−x 2 π

Aufgabe 22.28 •• Es ist falls |x| ≤ 1 und f (x) = 0 für |x| > 1 (siehe nachstehende
Abbildung). Aus Symmetriegründen besitzt Y die gleiche
T T ∞
1 −ita 1 −ita marginale Dichte wie X.
e ϕ(t) dt= e eitx PX (dx) dt
2T −T 2T −T −∞
∞ T f (x )
1
= eit (x−a) dt PX (dx)
2T −∞ −T
1
∞ eit (x−a) T
=  PX (dx)
2T ∞ i(x − a) −T

1 2 sin(T (x −a)) X
= P (dx) 1 1 x
2T −∞ x−a

sin(T (x − a)) X Marginale Dichte der Gleichverteilung im Einheitskreis
= P (dx).
−∞ T (x − a)
X und Y sind nicht unabhängig, denn es gilt etwa P(X >
Dabei wurde beim zweiten Gleichheitszeichen der Satz von 0.8, Y > 0.8) = 0, aber P(X > 0.8) P(Y > 0.8) > 0.
Fubini verwendet. Der Integrand im letzten Integral konver-
giert für T → ∞ gegen 0, falls x  = a bzw. gegen 1, falls Aufgabe 22.31 •• a) Nach Definition ist die Vertei-
x = a. Da der Integrand betragsmäßig durch 1 nach oben lungsfunktion G einer Zufallsvariablen Y durch G(t) :=
beschränkt ist, ergibt sich mit dem Satz von der dominierten P(Y ≤ t), t ∈ R, gegeben. Hiermit folgt wegen {X 4 ≤
Konvergenz t} = ∅ für t < 0 und {X 4 ≤ t} = {−t 1/4 ≤ X ≤ t 1/4 }
T ∞ sowie P(−t 1/4 ≤ X ≤ t 1/4 ) = F (t 1/4 ) − F (−t 1/4 ) (hier
1
lim e−ita ϕ(t) dt = 1{a} (x) PX (dx) ging P(X = −t 1/4 ) = 0 und somit die Stetigkeit von F ein!)
T →∞ 2T −T −∞
#
= P(X = a). F (t 1/4 ) − F (−t 1/4 ), falls t ≥ 0,
4
G(t) := P(X ≤ t) =
0 sonst.
Rechenaufgaben
b) Analog zu oben ist
Aufgabe 22.29 • a) Die Funktion F ist monoton wach-
#
send und stetig, also insbesondere rechtsseitig stetig, und es F (t) − F (−t), falls t ≥ 0,
gilt F (x) → 0 für x → −∞ sowie F (x) → 1 für x → ∞. G(t) := P(|X| ≤ t) =
0 sonst.
Somit ist F eine Verteilungsfunktion.
die Verteilungsfunktion von |X|.
b) Es ist
c) Wegen der Stetigkeit von F gilt
P(X ≤ 10) = F (10) = 1 − 1/(1 + 10) = 10/11 ≈ 0.909
G(t) := P(−X ≤ t) = P(X ≥ −t) = 1 − P(X < −t)
sowie wegen der Stetigkeit von F
= 1 − P(X ≤ −t) = 1 − F (−t), t ∈ R.
P(5 ≤ X ≤ 8) = P(5 < X ≤ 8) = F (8) − F (5)
1 1 1
= − = ≈ 0.0556. Aufgabe 22.32 • Für jedes a > 0 liegt eine Borel-
6 9 18
messbare nichtnegative Funktion vor.
∞ Eine Dichte entsteht,
c) Die Funktion F ist mit Ausnahme des Punktes x = 0 stetig wenn die Normierungsbedingung −∞ f (x)dx = 1 erfüllt
differenzierbar, und die Ableitung ist F  (x) = 0 für x < 0 ist. Wegen f (x) = f (−x), x ∈ R, gilt
und F  (x) = 1/(1+x)2 für x > 0. Setzen wir f (x) := F  (x) ∞ ∞
x
für x = 0 und f (0) := 0, so gilt F (x) = −∞ f (t) dt für f (x) dx = 2a exp(−x) dx = 2a.
jedes x ∈ R. Also ist f eine Dichte von F . −∞ 0
Lösungswege zu Kapitel 22 193

Eine Dichte entsteht also für a = 1/2. Wegen der Symmetrie Da die Konvergenz dieser Reihe auf dem kompakten Intervall
der Dichte um 0 genügt die zugehörige Verteilungsfunktion [0, x] gleichmäßig ist, kann über diesem Intervall gliedweise
F der Gleichung F (x) + F (−x) = 1, x ∈ R. Insbesondere integriert werden. Wegen
gilt also F (0) = 1/2. Für x > 0 folgt x
1
(x) = + ϕ(t) dt
1 1 x 2 0
F (x) = + exp(−t) dt
2 2 0
x für x > 0 folgt die Behauptung.
1 1  1
= + (− exp(−t)) = 1 − exp(−x).
2 2 0 2 Aufgabe 22.36 •• a) Es gilt
1
Aufgabe 22.33 • Modellieren wir den Messfehler als lim F0 (x) = lim= 0,
x→−∞ 1 + e−x
x→−∞
Zufallsvariable X mit der Verteilung N(0, 0.2025), so gilt
1
wegen 0.2025 = 0.452 unter Verwendung von Tabelle 22.21 lim F0 (x) = lim = 1.
x→∞ x→∞ 1 + e−x
 
X F ist stetig auf R, und für x, y mit x ≤ y gilt
P(|X| ≤ 0.45) = P −1 ≤ ≤1
0.45
= (1) − (−1) = 2(1) − 1 1 1
F0 (x) = ≤ = F0 (y).
≈ 2 · 0.8413 − 1 = 0.6826. 1 + e−x 1 + e−y
Somit ist F0 eine Verteilungsfunktion. Außerdem gilt für je-
In gleicher Weise folgt des x ∈ R
 
X 1 1 + e−x − 1
P(|X| ≤ 0.9) = P −2 ≤ ≤2 1 − F0 (x) = 1 − =
0.45 1 + e−x 1 + e−x
= (2) − (−2) = 2(2) − 1 e −x 1
= = x
≈ 2 · 0.9772 − 1 = 0.9544. 1 + e−x e +1
= F0 (−x).

Aufgabe 22.34 • Gesucht ist


  b) Die Dichte f0 ergibt sich durch Differenziation von F0 zu

X − μ
P(|X − μ| ≤ k σ ) = P  ≤k e−x
σ f0 (x) = , x ∈ R.
(1 + e−x )2
@ := (X − μ)/σ standardnormalverteilt
für k = 1, 2, 3. Da X
ist, ergibt sich 1

@ ≤ k)
P(|X − μ| ≤ k σ ) = P(|X|
y
= (k) − (−k)
= 2(k) − 1.

Mit (1) = 0.8413, (2) = 0.9772 und (3) = 0.9987


folgt
x0 x1 x
0 1
P(|X − μ| ≤ σ ) = 0.6826,
Dichte der logistischen Verteilung L(0, 1)
P(|X − μ| ≤ 2σ ) = 0.9544,
P(|X − μ| ≤ 3σ ) = 0.9974 c) Aufgrund der Gleichung
 
(sog. k-Sigma-Grenzen). 1 x−a
f (x) = f0
σ σ
Aufgabe 22.35 • Die Dichte ϕ der Standard-Normal-
verteilung besitzt die Potenzreihenentwicklung ist
f0 (y) = F0 (y) (1 − F0 (y)) , y ∈ R,
  ∞
1 t2 1 (−t 2 /2)k zu zeigen. Wegen
ϕ(t) = √ exp − = √
2π 2 2π k!
k=0 e−y

1 − F0 (y) =
1 (−1)k t 2k 1 + e−y
= √ , t ∈ R.
2π 2k k!
k=0 folgt diese Gleichung sofort aus der Gestalt von f0 .
194 Lösungswege zu Kapitel 22

Aufgabe 22.37 • Wegen P(0 < X < 1) = 1 gilt X1 und X2 sind nicht stochastisch unabhängig, denn sonst
P(0 < Y < 1) = 1. Bezeichnet G die Verteilungsfunktion wäre
von Y , so gilt somit G(y) = 0, falls y ≤ 0 und G(y) = 1,  
falls y ≥ 1. Für y ∈ (0, 1) gilt (siehe nachstehende Abbil- 1 x12 + x22
f@(x1 , x2 ) := f1 (x1 )f2 (x2 ) = 2 exp −
dung des Graphen von x −→ 4 x (1 − x)) σ 2π 2σ 2

f 0 (x ) eine gemeinsame Dichte von X1 und X2 . Da f und f@ ver-


0.25 schieden sind (es reicht, eine Umgebung von (0, 0) zu be-
trachten), sind nach dem Satz über stochastische Unabhän-
gigkeit und Dichten auf Seite 821 X1 und X2 nicht stocha-
stisch unabhängig.

b) Es gilt
⎛ ⎞ ⎛ ⎞
Y1 X1
−1 0 1 x ⎝ ⎠ = A ⎝ ⎠,
Y2 X2

{x ∈ (0, 1) | 4x(1 − x) ≤ y} = (0, x0 ] + [x1 , 1), wobei ⎛ ⎞


1 1
wobei A=⎝ ⎠.
√ √ 1 −1
1− 1−y 1+ 1−y
x0 = , x1 = . Die Matrix A definiert eine bijektive Abbildung T (x) := Ax,
2 2
x = (x1 , x2 ) ∈ R2 , des R2 auf sich, wobei |det A| = 2.
Es folgt Setzen wir y = (y1 , y2 ) , so ist die Umkehrabbildung T −1
durch
G(y)=P(Y ≤ y) ⎛ ⎞⎛ ⎞
 √   √ 
1− 1−y 1+ 1−y 1 1 1 y
=P 0 < X ≤ +P ≤X<1 T −1 (y1 , y2 ) = ⎝ ⎠ ⎝ 1⎠ , (y1 , y2 ) ∈ R2 ,
2 2 2 1 −1 y
√ √ 2
1− 1−y 1+ 1−y
= +1−
2 2 gegeben. Mit dem Transformationssatz auf Seite 827 ergibt

=1 − 1 − y. sich die gemeinsame Dichte g von Y1 und Y2 zu
√ f ((y1 + y2 )/2, (y1 − y2 )/2)
Die Dichte von Y ist g(y) = (2 1 − y)−1/2 1(0,1) (y). g(y1 , y2 ) = .
|det A|
Aufgabe 22.38 •• a) Die Dichte f1 von X1 ergibt sich Setzt man (y1 + y2 )/2 für x1 und (y1 − y2 )/2 für x2 in die
durch Integration gemäß Definition der Dichte f (x1 , x2 ) ein, so wird der Exponenti-
√   ∞   alausdruck zu
2 3 3
f1 (x1 ) = exp − x12 exp − x22 − x1 x2 dx2 .    
π 2 −∞ 2 3 y1 + y2 2 (y1 + y2 )(y1 − y2 ) 3 y1 − y2 2
− − −
2 2 4 2 2
Quadratische Ergänzung liefert mit a := −x1 /3 und
1
τ 2 := 1/3 = −y12 − y22 .
2
   
2 4 2 ∞ 1 (x2 −a)2 Die gemeinsame Dichte von Y1 und Y2 ist somit
f1 (x1 ) = √ exp − x1 √ exp − 2
dx2 .
3π 3 −∞ σ 2π 2τ  
1 2 y22
g(y1 , y2 ) = √ exp −y1 −
Da das Integral aufgrund der Normierungsbedingung für die π 2 2
2
Dichte der N(a, τ )-Normalverteilung gleich 1 ist, folgt mit
1  y1 
σ 2 := 3/8 = ϕ ϕ(y2 ), (y1 , y2 ) ∈ R2 .
σ σ
 
1 x12 √
f1 (x1 ) = √ exp − 2 , x1 ∈ R, Dabei wurde kurz σ := 1/ 2 gesetzt, und ϕ bezeichnet wie
σ 2π 2σ früher die Dichte der Standardnormalverteilung N(0, 1). Die
Zufallsvariablen Y1 und Y2 besitzen also die Normalvertei-
und damit X1 ∼ N(0, σ 2 ). Weil die Dichte f symmetrisch lungen N(0, 1/2) bzw. N(0, 1), und sie sind nach dem Satz
in x1 und x2 ist, ist die Dichte f2 von X2 identisch mit f1 , über stochastische Unabhängigkeit und Dichten auf Seite 821
und somit folgt auch X2 ∼ N(0, σ 2 ). stochastisch unabhängig.
Lösungswege zu Kapitel 22 195

Aufgabe 22.39 •• Für t > 0 ist Mit (22.29) ergibt sich dann die Dichte von X/Y zu

G(t) = P(X < tY )
∞  ty  g(t) = f (ts, s) |s| ds
−λx
= λe dx λ e−λy dy
−∞

0 0
∞ = ψ((t 2 + 1)s 2 ) |s| ds
= (1 − exp(−λty)) λ e−λy dy −∞

0
∞ = 2 ψ((t 2 + 1)s 2 ) s ds
= 1−λ exp(−λ(t + 1)y) dy 0

0 2
1 = ψ(r 2 ) rdr
= 1− 1 + t2 0
t +1 1
t = .
= . π(1 + t 2 )
t +1

Wegen P(X/Y > 0) = 1 ist G(t) = 0 für t ≤ 0. Aufgabe 22.42 • Nach der auf Seite 827 formulierten
Aussage sind mit unabhängigen und je U(0, 1)-verteilten Zu-
Aufgabe 22.40 •• Es gilt fallsvariablen U1 , U2 die Zufallsvariablen
D 
kT Y1 := −2 log U1 sin (2π U2 ) ,
(V1 , V2 , V3 ) ∼ (Z1 , Z2 , Z3 ), 
m
Y2 := −2 log U1 cos (2π U2 )
wobei Z1 , Z2 , Z3 stochastisch unabhängig und je N(0, 1)-
normalverteilt sind. Somit gilt mit dem Hinweis unabhängig und je standardnormalverteilt. Aus diesem
D Grund besitzt der Quotient zweier unabhängiger und je stan-
*
kT √ dardnormalverteilter Zufallsvariablen die gleiche Verteilung
Y = V1 + V 2 + V 3 ∼
2 2 2 Z,
m wie Y1 /Y2 = tan(2π U2 ). Da die Tangensfunktion die Pe-
riode π besitzt, hat tan(2π U2 ) die gleiche Verteilung wie
wobei Z eine χ32 -Verteilung und somit die Dichte T := tan(π U2 ). Aus dem Verlauf des Graphen der Tangens-
 x funktion auf dem Intervall (0, π ) ergibt sich für t ≥ 0
1
f (x) = exp − x 3/2−1 , x > 0,  
23/2 "(3/2) 2 1 1
P(T ≤ t) = + P U2 ≤ arctan t
und f (x) := 0 sonst besitzt. Mit der Abkürzung a := 2 π
√ 1 1
(kT )/m hat Y nach dem Satz (Methode Verteilungsfunk- = + arctan t
tion) auf Seite 824 die Verteilungsfunktion 2 π
 
√ y2 und aus Symmetriegründen für t < 0 ebenfalls
G(y) := P(Y ≤ y) = P(a Z ≤ y) = P Z ≤ 2 , y > 0,
a
P(T ≤ t) = 1 − P(T ≤ −t)
 
sowie G(y) := 0 für y ≤ 0. Somit ist die Dichte von Y durch 1 1
= 1− + arctan(−t)
  2 π
y 2 2y 1 1
g(y) = f = + arctan t.
a2 a2 2 π
D   
2 m 3/2 2 m y2 Durch Ableiten erhält man die Dichte f (t) = 1/(π(1 + t 2 )).
= y exp −
π kT 2kT Man beachte, dass die in der Aufgabenstellung geschilderte
Situation ein Spezialfall von Aufgabe 22.41 ist.
für y > 0 und g(y) = 0 sonst gegeben.
Aufgabe 22.41 •• Durch Übergang zu Polarkoordina- Aufgabe 22.43 •• Es gilt (X1 , X2 ) ∼ (Y1 , Y2 ), wobei
ten x = r cos θ, y = r sin θ, dxdy = rdrdθ erhält man

∞ ∞ ∞ Y1 := −2 log U1 sin (2π U2 ) ,
2π 
1= ψ(x 2 +y 2 ) dxdy = ψ(r 2 )rdr 1dθ Y2 := −2 log U1 cos (2π U2 )
−∞ −∞ 0 0

und somit und die Zufallsvariablen U1 , U2 unabhängig und je



2 1 U(0, 1)-verteilt sind. Es folgt mit der Abkürzung R :=

ψ(r )rdr = .
0 2π −2 log U1 und der trigonometrischen Gleichung sin(2α) =
196 Lösungswege zu Kapitel 22

2 sin α cos α Nach der allgemeinen Transformationsformel (22.38) gilt


weiter
X1 X2 Y1 Y2
* ∼ *
X12 + X22 Y12 + Y22 E(XY ) = x y 2 1A (x, y) dxdy
 
R 2 sin (2πU2 ) cos (2πU2 ) 1 1
= = 2 x y dy dx
R 0 x
= R sin (2πU2 ) cos (2πU2 ) 1 1
=
1
R sin (4πU2 ) . = 2 x (1 − x 2 ) dx
2 0 2
1
Da sin (4πU2 ) die gleiche Verteilung wie sin (2πU2 ) besitzt = ,
4
und R sin (2πU2 ) standardnormalverteilt ist, folgt die Be-
und somit erhält man
hauptung.
1
Aufgabe 22.44 •• Da X und Y die gleichen Verteilun- Cov(X, Y ) = E(XY ) − EX EY = ,
36
gen wie aX0 und aY0 besitzen, wobei X0 und Y0 im In- Cov(X, Y ) 1
tervall (0, 1) gleichverteilt sind, kann o.B.d.A. a = 1 an- ρ(X, Y ) = √ = ,
V(X)V(Y ) 2
genommen werden. Die Dichten von X und Y sind dann
fX (s) = fY (s) = 1(0,1) (t), t ∈ R. Nach Teil c) des Satzes also c).
auf Seite 831 ist
Aufgabe 22.46 • Es sei =: diag(σ12 , . . . , σk2 ). Dann

gilt  
g(t) = 1(0,1) (ts)1(0,1) (s) |s| ds,
−∞
−1
= diag σ1−2 , . . . , σk−2

also unter Berücksichtigung der Positivitätsbereiche beider sowie det( ) = σ12 . . . σk2 , und die gemeinsame Dichte von
Dichten für t > 0 X1 , . . . , Xk hat somit die Gestalt
min(1,1/t) ⎛ ⎞
1 1 k
(xj − μj )2
g(t) = s ds = (min(1, 1/t))2 . f (x) = exp ⎝− ⎠
0 2 (2π )k/2 σ1 . . . σk 2σj2
j =1
Die zugehörige Verteilungsfunktion ist   
 k
1 (xj − μj )2
# = √ exp − ,
t/2, falls 0 ≤ t ≤ 1, j =1
σj 2π 2σj2
G(t) = P(X/Y ≤ t) =
1 − 2t1 , falls 1 < t < ∞,
x = (x1 , . . . , xk ) ∈ Rk . Die gemeinsame Dichte ist also
das Produkt der marginalen Dichten von X1 , . . . , Xk . Nach
sowie G(t) := 0 sonst.
dem Unabhängigkeitskriterium für Dichten auf Seite 821
sind X1 , . . . , Xk stochastisch unabhängig.
Aufgabe 22.45 •• Auf Seite 850 wurden die margina-
len Dichten f und g von X und Y zu Aufgabe 22.47 •• Wir beziehen uns im Folgenden auf
Abb. 22.23. Da zwischen  und X die Gleichung
f (x) = 2(1 − x), g(x) = 2x, 0 ≤ x ≤ 1,
 π X−α
tan  − =
und f (x) = g(x) := 0 sonst bestimmt. Hieraus ergibt sich 2 β
a) und b) wie folgt: besteht, folgt mit P( ≤ y) = y/π , 0 ≤ y ≤ π, für die
1 Verteilungsfunktion F von X
1  
EX = 2 x(1 − x) dx = , X−α x−α
0 3 F (x) = P(X ≤ x) = P ≤
1 β β
2   
EY = 2 y 2 dy = , π x−α
0 3 = P  ≤ + arctan
1 2 β
1  
EX 2 = 2 x 2 (1 − x) dx = , 1 1 x−α
0 6 = + arctan , x ∈ R.
1 2 π β
V(X) = EX 2 − (EX)2 = ,
18 Nach (22.44) gilt X ∼ C(α, β).
1 1
EY 2 = 2 y 3 dy = , Aufgabe 22.48 •Die Verteilungsfunktion von X ist
0 2
 
1 1 1 x−α
V(Y ) = EY 2 − (EY )2 = . F (x) = + arctan , x ∈ R.
18 2 π β
Lösungswege zu Kapitel 22 197

Es gilt der χk2 -Verteilung mit unabhängigen und je N(0, 1)-


normalverteilten Zufallsvariablen Y1 , . . . , Yk aus. Für den
1 Induktionsanfang k = 1 verwenden wir das Resultat (22.26)
F (α) =
,
2 über die Dichte des Quadrates einer Zufallsvariablen. Setzen
1 1 3 wir in (22.26) für f die Dichte ϕ der Standardnormalvertei-
F (α + β) = + arctan(1) = ,
2 π 4 lung ein, so ergibt sich unmittelbar der Induktionsanfang. Für
1 1 1 den Induktionsschluss von k auf k + 1 sei
F (α − β) = + arctan(−1) = .
2 π 4
X ∼ Y12 + Y22 + . . . + Yk2 + Yk+1
2
Da F stetig und streng monoton wachsend ist, folgt α = Q1/2
sowie α + β = Q3/4 , α − β = Q1/4 , was zu zeigen war. mit unabhängigen standardnormalverteilten Zufallsvariablen
Y1 , . . . , Yk+1 . Wir setzen
Aufgabe 22.49 • Wir schreiben wie üblich S ∼ T , U := Y12 + . . . + Yk2 , 2
V := Yk+1 .
wenn Zufallsvariablen S und T die gleiche Verteilung besit-
zen. Es sei X ∼ Wei(α, 1) und Y := (1/λ)1/α X. Weiter sei Nach dem Blockungslemma sind U und V stochastisch unab-
U ∼ Exp(1). Nach Erzeugungsweise der Weibull-Verteilung hängig, und X besitzt die gleiche Verteilung wie die Summe
(vgl. (22.49)) gilt X ∼ U 1/α und somit Y ∼ (U/λ)1/α . We- U + V . Die Dichte fk+1 von U + V erhalten wir über die
gen U/λ ∼ Exp(λ) folgt (wiederum aufgrund von (22.49)) Faltungsformel aus der Dichte fk von U (Induktionsvoraus-
die Behauptung. Alternativ kann man auch direkt die Vertei- setzung!) und die Dichte f1 von V (Induktionsanfang!). Es
lungsfunktion und dann die Dichte von Y ausrechnen. ergibt sich für t > 0

Aufgabe 22.50 •• a) Mit der in (22.48) gegebenen
fU +V (t)= fU (s) fV (t − s) ds
Dichte ergibt sich −∞
t e−s/2 s k/2−1
∞  e−(t−s)/2 (t − s)1/2−1
k k+α−1 α = dt
EX = α λ x exp −λx dx. 0 2k/2 "( 2k ) 21/2 "( 21 )
0
e−t/2 t
Substituiert man hier y = λ xα ,
so folgt = s k/2−1 (t − s)−1/2 dt
2(k+1)/2 "( 2k ) "( 21 ) 0

1 "(1 + k/α) e−t/2 t (k+1)/2−1 1
EX k = k/α y k/α e−y dy = . = uk/2−1 (1−u)1/2−1 du.
λ 0 λk/α 2(k+1)/2 "( 2k ) "( 21 ) 0

b) Nach a) gilt   Nach (22.55) gilt


" 1 + α1    
EX = . 1 " k
" 1
2 2
λ1/α uk/2−1 (1 − u)1/2−1 du =   ,
0 k+1
Da eine Wei(α, λ)-verteilte Zufallsvariable X die Vertei- " 2
lungsfunktion
und somit folgt
 α
F (x) = 1 − exp −λ x , x > 0, 1 t k+1
fU +V (t) =   e− 2 t 2 −1 = fk+1 (t),
k+1
besitzt, ist die Quantilfunktion durch 2(k+1)/2 " 2

 1/α was zu zeigen war.


−1 1
F (p) = − log(1 − p) , 0 < p < 1,
λ Aufgabe 22.52 •• a) Die Dichte der Lognormalvertei-
lung LN(μ, σ 2 ) ist
gegeben. Der Median Q1/2 = F −1 (1/2) ergibt sich somit  
zu 1 (log x − μ)2
(log 2)1/α f (x) = exp −
√ , x > 0,
Q1/2 = . σ x 2π 2σ 2
λ1/α
Wegen und f (x) = 0 sonst. Aufgrund der strengen Monotonie der
  Logarithmusfunktion können wir auch die Funktion
1/α 1
(log 2) < 1 < " 1+
α (log x − μ)2 √
x → log f (x) = − log x − − log(σ 2π )
folgt die Behauptung. 2σ 2
im Bereich x > 0 maximieren. Die Substitution y := log x
Aufgabe 22.51 •• Wir zeigen das Resultat durch Induk- zeigt, dass wir auch die Funktion
tion über k. Dabei gehen wir von der Erzeugungsweise
(y − μ)2
y  → −y −
X∼ Y12 + Y22 + . . . + Yk2 2σ 2
198 Lösungswege zu Kapitel 22

bezüglich y maximieren können. Der Wert y0 , an dem diese b) Die Behauptung folgt unmittelbar aus a) und V(X) =
Funktion ihr Maximum annimmt, ist y0 = μ−σ 2 . Die Rück- EX 2 − (EX)2 .
transformation x = exp(y) liefert dann a).
c) Wir bestimmen zunächst die Dichte von Y := W/V . Diese
b) Da die Lognormalverteilung LN(μ, σ 2 ) die Verteilungs- ergibt sich nach Teil c) des Satzes über die Dichte einer Dif-
funktion F (x) = ((log x − μ)/σ ), x > 0, besitzt, führt die ferenz, eines Produktes und eines Quotienten unabhängiger
Gleichung F (x) = 1/2 auf die Lösung x = exp(μ). Zufallsvariablen auf Seite 831 zu
c) Mit der Substitution u = log x und quadratischer Ergän- ∞
zung ergibt sich fY (t) = fW (ts) fV (s) s ds
0
∞  
λβ λα ∞
1 1 (log x − μ)2
EX = √ x · exp − dx = (ts)β−1 e−λts s α−1 e−λs s ds
σ 2π 0 x 2σ 2 "(β) "(α) 0
  ∞
∞ λα+β t β−1
1 u (u − μ)2 = s α+β−1 e−λ(t+1)s ds.
= √ e exp − du "(α)"(β) 0
σ 2π −∞ 2σ 2
   
μ2 (μ + σ 2 )2 Mit der Substitution u := λ(t + 1)s und
= exp − 2 exp ×
2σ 2σ 2
  ∞
∞ uα+β−1 e−u du = "(α + β)
1 (u − (μ + σ 2 ))2
× √ exp − du 0
σ 2π −∞ 2σ 2
 
folgt
= exp μ + σ 2 /2 .

λα+β t β−1 "(α + β)


fY (t) =
d) Wir substituieren wieder u = log x und erhalten "(α)"(β) (λ(t + 1))α+β
∞   t β−1
1 (log x − μ)2 =
2
EX = √ x exp − dx B(α, β)(t + 1)α+β
2π 0 2σ 2
∞  
1 (u − μ)2 für t > 0 und fY (t) = 0 sonst. Man beachte, dass der Skalen-
2u
=√ e exp − du parameter λ der Gammaverteilung nach (22.53) keine Rolle
2π 0 2σ 2
  spielt. Wir hätten ihn schon zu Beginn o.B.d.A. zu λ = 1

2(μ+σ 2 ) 1 (u − (μ + 2σ 2 ))2 setzen können.
=e √ exp − du
−∞ σ 2π 2σ 2
2 Die Dichte von
=e2(μ+σ ) .
V 1
Wegen V(X) = EX 2 − (EX)2 folgt jetzt d) zusammen mit X= =
V +W 1+Y
c) und direkter Rechnung.
ergibt sich jetzt nach dem Satz auf Seite 824 mit O = (0, ∞)
Aufgabe 22.53 •• Unter Verwendung von (22.56) und
und T (y) := 1/(1 + y), y ∈ O, sowie T  (y) = −1/(1 + y)2 ,
der Funktionalgleichung "(t + 1) = t"(t), t > 0, für die
T −1 (x) = 1/x − 1 zu
Gammafunktion ist
∞  
1
EX k = x f (x) dx fY x −1
−∞ fX (x) =
∞ 1
1
= x k+α−1 (1 − x)β−1 dx (1 + 1/x − 1)2
B(α, β) 0  
1−x
B(α + k, β) = x −2 fY
= x
B(α, β)
"(α + k) "(β) "(α + β) 1 (1 − x)β−1 α+β
= = x −2 x
"(α + k + β) "(α)"(β) B(α, β) x β−1
"(α + k) "(α + β) x α−1 (1 − x)β−1
= =
"(α) "(α + β + k) B(α, β)

k−1
α+j
= . für 0 < x < 1 und fX (x) = 0 sonst. Somit gilt X ∼
α+β +j BE(α, β).
j =0
Lösungswege zu Kapitel 22 199

Aufgabe 22.54 •• Die Dichte g der Verteilung "(r, β)


ist
β r r−1 −βz
g(z) = z e
"(r)
für z > 0 und g(z) = 0 sonst. Wie im Beispiel auf Seite 849
ist mit

zk
P(X = k|Z = z) = e−z , k ∈ N0 , z > 0,
k!
für jedes k ∈ N0

P(X = k) = P(X = k|Z = z) g(z) dz
0
∞ zk β r r−1 −βz
= e−z z e dz
0 k! "(r)

βr
= zk+r−1 exp(−(1 + β)z) dz
"(r) k! 0

βr
= uk+r−1 e−u du
(r − 1)! k! (1 + β)k+r 0
βr
= "(k + r)
(r − 1)! k! (1 + β)k+r
  r  k
k+r −1 β 1
= .
r −1 1+β 1+β

Nach Definition der negativen Binomialverteilung folgt


hieraus die Behauptung.
Kapitel 23 a) Zeigen Sie, dass die Folge (Xn ) nicht dem Kolmogorov-
Kriterium genügt.
b) Zeigen Sie mit Aufgabe 23.21, dass für (Xn ) ein starkes
Aufgaben Gesetz großer Zahlen gilt.

Aufgabe 23.9 •• Zeigen Sie, dass eine endliche Menge


Verständnisfragen Q von Wahrscheinlichkeitsmaßen auf B1 straff ist.
Aufgabe 23.1 • Zeigen Sie, dass die in (23.1) stehende Aufgabe 23.10 •• In einer Folge (Xn )n≥1 von Zufalls-
Menge zu A gehört. variablen habe Xn die charakteristische Funktion
Aufgabe 23.2 • Es sei (Xn )n≥1 eine Folge von Zu- sin(nt)
ϕn (t) := , t = 0,
fallsvariablen auf einem Wahrscheinlichkeitsraum (, A, P) nt
P
mit Xn ≤ Xn+1 , n ≥ 1, und Xn −→ X. Zeigen Sie: und ϕn (0) := 0. Zeigen Sie, dass Xn eine Gleichverteilung
f.s. in (−n, n) besitzt und folgern Sie hieraus, dass die Folge
Xn −→ X.
(Xn ) nicht nach Verteilung konvergiert, obwohl die Folge
Aufgabe 23.3 •• Zeigen Sie, dass in einem diskreten (ϕn ) punktweise konvergent ist. Welche Bedingung des Ste-
Wahrscheinlichkeitsraum die Begriffe fast sichere Konver- tigkeitssatzes von Lévy-Cramér ist verletzt?
genz und stochastische Konvergenz zusammenfallen.
Aufgabe 23.11 •• Es seien Y1 , Y2 , . . . Zufallsvariablen
Aufgabe 23.4 • Es seien X, X1 , X2 , . . . (als Spalten- und (an ), (σn ) reelle Zahlenfolgen mit σn > 0, n ≥ 1, und
vektoren aufgefasste) d-dimensionale Zufallsvektoren auf Yn − an D
P −→ Z
einem Wahrscheinlichkeitsraum (, A, P) mit Xn −→ X σn
und A, A1 , A2 , . . . reelle (k × d)-Matrizen mit An → A. für eine Zufallsvariable Z. Zeigen Sie: Sind (bn ) und (τn )
P
Zeigen Sie: An Xn −→ A X. reelle Folgen mit τn > 0, n ≥ 1, und (an − bn )/σn → 0
sowie σn /τn → 1, so folgt
Aufgabe 23.5 •• Es sei (Xn , Yn )n≥1 eine Folge un-
abhängiger, identisch verteilter zweidimensionaler Zufalls- Yn − bn D
−→ Z.
vektoren auf einem Wahrscheinlichkeitsraum (, A, P) mit τn
EX12 < ∞, EY12 < ∞, V(X1 ) > 0, V(Y1 ) > 0 und
Aufgabe 23.12 ••
1 
n
 
Xj − X n Yj − Y n a) Es seien Y, Y1 , Y2 , . . . Zufallsvariablen mit Verteilungs-
n D
j =1 funktionen F, F1 , F2 , . . ., sodass Yn −→ Y für n → ∞.
Rn := A
B
B1
n
 2 1
n
 2 Ferner sei t eine Stetigkeitsstelle von F und (tn ) eine
C Xj − X n Yj − Y n Folge mit tn → t für n → ∞. Zeigen Sie:
n n
j =1 j =1
lim Fn (tn ) = F (t).
der sogenannte empirische Korrelationskoeffizient von n→∞

(X1 , Y1 ), . . . , (Xn , Yn ), wobei X n := n−1 nj=1 Xj , Y n := b) Zeigen Sie, dass in den Zentralen Grenzwertsätzen von

n−1 nj=1 Yj . Zeigen Sie: Lindeberg-Feller und Lindeberg-Lévy jedes der „≤“-
Zeichen durch das „<“-Zeichen ersetzt werden kann.
f.s. Cov(X1 , Y1 ) c) Es sei Sn ∼ Bin(n, 1/2), n ∈ N. Bestimmen Sie den
Rn −→ √ = #(X1 , Y1 ).
V (X1 ) · V (Y1 ) Grenzwert
    
n √ 1
Aufgabe 23.6 • Zeigen Sie, dass für den Beweis des lim P Sn ≤ n sin +1 .
n→∞ 2 n
starken Gesetzes großer Zahlen o.B.d.A. die Nichtnegativität
der Zufallsvariablen Xn angenommen werden kann. Aufgabe 23.13 •• In der Situation und mit den Bezeich-
√ D
nungen der Box auf Seite 876 gilt n(In − I )/σf −→
Aufgabe 23.7 •• Formulieren und beweisen Sie ein N(0, 1). Es sei
starkes Gesetz großer Zahlen für Zufallsvektoren.  
1 2
n
Jn I2
Aufgabe 23.8 •• Für die Folge (Xn ) unabhängiger Zu- Jn := |B| · f (Uj ), σn2 := |B|2 − n2 .
n |B| |B|
fallsvariablen gelte j =1

1 Zeigen Sie:
P(Xn = 1) = P(Xn = −1) = (1 − 2−n ),
2 f.s.
a) σn2 −→ σf2 für n → ∞.
1
P(Xn = 2n ) = P(Xn = −2n ) = n−1 . √ D
2 b) n(In − I )/σn −→ N(0, 1) für n → ∞.
M. Brokate et al., Arbeitsbuch Grundwissen Mathematikstudium – Höhere Analysis, Numerik und
Stochastik, DOI 10.1007/978-3-642-54946-5_22, © Springer-Verlag Berlin Heidelberg 2016
Aufgaben zu Kapitel 23 201

Aufgabe 23.14 •• Zeigen Sie: b) Ein echter Würfel werde in unabhängiger Folge gewor-

n fen. Die Zufallsvariable Yj beschreibe die beim j -ten
nk 1
a) lim e−n = , Wurf erzielte Augenzahl, j ≥ 1. Zeigen Sie:
n→∞ k! 2
k=0
1
n

2n f.s. 5
−n n
k
1{Yj < Yj +1 } −→ .
b) lim e = 1. n 12
n→∞ k! j =1
k=0

Aufgabe 23.15 •• Die Zufallsvariable Sn besitze die Aufgabe 23.21 •• Es seien (Xn )n≥1 und (Yn )n≥1 Fol-
Binomialverteilung Bin(n, pn ), n ≥ 1, wobei 0 < pn < 1 gen von Zufallsvariablen auf einem Wahrscheinlichkeits-
und pn → p ∈ (0, 1) für n → ∞. Zeigen Sie: raum (, A, P) mit
Sn − npn ∞

D
√ −→ N(0, 1) für n → ∞. P(Xn  = Yn ) < ∞.
npn (1 − pn )
n=1

1 1
n n
f.s. f.s.
Beweisaufgaben Zeigen Sie: Yj −→ 0 ⇒ Xj −→ 0.
n n
j =1 j =1
Aufgabe 23.16 • Beweisen Sie den Satz zur Äquiva-
lenz der fast sicheren bzw. stochastischen Konvergenz von Aufgabe 23.22 •• Es sei (Xn ) eine Folge unabhän-
Zufallsvektoren zur jeweils komponentenweisen Konvergenz giger Zufallsvariablen auf einem Wahrscheinlichkeitsraum
auf Seite 871. (, A, P) mit Xn ∼ Bin(1, 1/n), n ≥ 1. Zeigen Sie:
Aufgabe 23.17 ••• Es sei (Xn )n≥1 eine Folge von 1
n
Zufallsvariablen auf einem Wahrscheinlichkeitsraum lim Xj = 1 P-fast sicher.
n→∞ log n
(, A, P). j =1

1
n
f.s. f.s.
a) Zeigen Sie: Xn −→ 0 ⇒ Xj −→ 0. Aufgabe 23.23 •• Es sei (Xn ) eine u.i.v.-Folge mit
n
j =1 X1 ∼ U(0, 1). Zeigen Sie:
b) Gilt diese Implikation auch, wenn fast sichere Konver-  
D
genz durch stochastische Konvergenz ersetzt wird? a) n 1 − max Xj −→ Exp(1) für n → ∞.
1≤j ≤n
Aufgabe 23.18 •• Es sei (Xn ) eine Folge unabhän- D
b) n min Xj −→ Exp(1) für n → ∞.
giger Zufallsvariablen auf einem Wahrscheinlichkeitsraum 1≤j ≤n
(, A, P) mit P(Xn = 1) = 1/n und P(Xn = 0) = 1 − 1/n,
n ≥ 1. Zeigen Sie, dass die Folge (Xn ) stochastisch, aber Aufgabe 23.24 •• Es seien X, X1 , X2 , . . . ; Y1 , Y2 , . . .
nicht fast sicher gegen null konvergiert. Zufallsvariablen auf einem Wahrscheinlichkeitsraum
D P
(, A, P) mit Xn −→ X und Yn −→ a für ein a ∈ R.
Aufgabe 23.19 •• Es sei V die Menge aller reellen Zu- Zeigen Sie:
fallsvariablen auf einem Wahrscheinlichkeitsraum (, A, P) D
Xn Yn −→ a X.
und d : V × V → [0, 1] durch
d(X, Y ) := inf{ε ≥ 0 | P(|X − Y | > ε) ≤ ε} Aufgabe 23.25 •• Es seien Xn , Yn , n ≥ 1, Zufallsva-
riablen auf einem Wahrscheinlichkeitsraum (, A, P) sowie
definiert. Zeigen Sie: Für X, Y, Z, X1 , X2 , . . . ∈ V gelten:
(an ), (bn ) beschränkte Zahlenfolgen mit limn→∞ an = 0.
a) d(X, Y ) = min{ε > 0 | P(|X − Y | > ε) ≤ ε} Weiter gelte Xn = OP (1) und Yn = OP (1). Zeigen Sie:
b) d(X, Y ) = 0 ⇐⇒ X = Y P-f.s., a) Xn + Yn = OP (1), Xn Yn = OP (1),
c) d(X, Z) ≤ d(X, Y ) + d(Y, Z), b) Xn + bn = OP (1), bn Xn = OP (1),
P c) an Xn = oP (1).
d) limn→∞ d(Xn , X) = 0 ⇐⇒ Xn −→ X.

Aufgabe 23.26 •• Es sei Xn ∼ N(μn , σn2 ), n ≥ 1. Zei-


Aufgabe 23.20 •••
gen Sie:
a) Es sei (Xn )n≥1 eine Folge identisch verteilter Zufalls-
variablen auf einem Wahrscheinlichkeitsraum (, A, P). Xn = OP (1) ⇐⇒ (μn ) und (σn2 ) sind beschränkte Folgen.
Es existiere ein k ≥ 1 so, dass Xm und Xn stochastisch
unabhängig sind für |m − n| ≥ k (m, n ≥ 1). Zeigen Sie: Aufgabe 23.27 ••• Es sei (, A, P) := ((0, 1), B1 ∩
(0, 1), λ1|(0,1) ) sowie N := {ω ∈  | ∃n ∈ N ∃ε1 , . . . , εn ∈
1
n
E|X1 | < ∞ ⇒
f.s.
Xj → EX1 . 
n {0, 1}, ε = 1, mit ω = nj=1 εj 2−j } die Menge aller Zah-
j =1 len in (0, 1) mit abbrechender dyadischer Entwicklung.
202 Aufgaben zu Kapitel 23

a) Zeigen Sie: P(N) = 0. a) Sn2 konvergiert P-fast sicher gegen σ 2 .


b) Jedes ω ∈  \ N besitzt eine  eindeutig bestimmte dya-
dische Entwicklung ω = ∞ j =1 X j (ω) 2 −j . Definieren b) Mit μ := EX1 und τ 2 := E(X1 − μ)4 − σ 4 > 0 gilt
wir zusätzlich Xj (ω) := 0 für ω ∈ N , j ≥ 1, so sind
√  2  D
X1 , X2 , . . . {0, 1}-wertige Zufallsvariablen auf . Zei- n Sn − σ 2 −→ N(0, τ 2 ).
gen Sie: X1 , X2 , . . . sind stochastisch unabhängig und je
Bin(1, 1/2)-verteilt.
c) Nach Konstruktion gilt Aufgabe 23.33 • Es seien z1 , . . . , zn , w1 , . . . , wn ∈
C mit |zj |, |wj | ≤ 1 für j = 1, . . . , n. Zeigen Sie:

n
lim Xj 2−j = id P-fast sicher, n 
n 
n
n→∞  
j =1  zj − wj  ≤ |zj − wj |
j =1 j =1 j =1
wobei id die Gleichverteilung U(0, 1) besitzt. Die
Gleichverteilung in (0, 1) besitzt die charakteristische
Funktion t −1 sin t. Zeigen Sie unter Verwendung des Ste- Aufgabe 23.34 •• Es seien W1 , W2 , . . . , eine u.i.v.-
tigkeitssatzes von Lévy-Cramér: Folge mit EW1 = 0 und 0 < σ 2 := V(W1 ) < ∞ sowie

sin t

  
t  reelle Zahlenfolge mit an = 0, n ≥ 1. Weiter sei
(an ) eine
= cos j , t ∈ R. Tn := nj=1 aj Wj . Zeigen Sie:
t 2
j =1
max1≤j ≤n |aj | Tn D
Aus lim * = 0 folgt √ −→ N(0, 1).
Aufgabe 23.28 •• Es seien μ ∈ R, (Zn ) eine Folge von
n→∞ n 2 V (T n )
j =1 aj
Zufallsvariablen und (an ) eine Folge positiver reeller Zahlen
mit
D P
an (Zn − μ) −→ N(0, 1) und Zn −→ μ Aufgabe 23.35 •• Es sei (Xn )n≥1 eine  Folge von un-
abhängigen Indikatorvariablen und Sn := nj=1 Xj . Zeigen
für n → ∞. Weiter sei g : R → R eine stetig differenzier- 
bare Funktion mit g  (μ)  = 0. Zeigen Sie: Sie: Aus ∞n=1 V(Xn ) = ∞ folgt die Gültigkeit des Zentra-
√ D
  len Grenzwertsatzes (Sn − ESn )/ V(Sn ) −→ N(0, 1).
D
an (g(Zn ) − g(μ)) −→ N 0, (g  (μ))2 für n → ∞

(sog. Fehlerfortpflanzungsgesetz). Rechenaufgaben


Aufgabe 23.36 •• Der Lufthansa Airbus A380 bietet
Aufgabe 23.29 •• Es seien X, X1 , X2 , . . . Zu-
insgesamt 526 Fluggästen Platz. Da Kunden manchmal ihren
fallsvariablen mit zugehörigen Verteilungsfunktionen
Flug nicht antreten, lassen Fluggesellschaften zwecks opti-
F, F1 , F2 , . . . Zeigen Sie: Ist F stetig, so gilt:
maler Auslastung Überbuchungen zu. Es sollen möglichst
D viele Tickets verkauft werden, wobei jedoch die Wahrschein-
Xn −→ X ⇐⇒ lim sup |Fn (x) − F (x)| = 0.
n→∞ x∈R lichkeit einer Überbuchung maximal 0.05 betragen soll. Wie
viele Tickets dürfen dazu maximal verkauft werden, wenn be-
kannt ist, dass ein Kunde mit Wahrscheinlichkeit 0.04 nicht
Aufgabe 23.30 •• Es seien X, X1 , X2 , . . . Zufallsva- zum Flug erscheint und vereinfachend angenommen wird,
riablen mit Verteilungsfunktionen F, F1 , F2 , . . . und zuge- dass das Nichterscheinen für verschiedene Kunden unabhän-
hörigen Quantilfunktionen F −1 , F1−1 , F2−1 , . . . Zeigen Sie: gig voneinander ist?
Aus Fn (x) → F (x) für jede Stetigkeitsstelle x von F folgt
Fn−1 (p) → F −1 (p) für jede Stetigkeitsstelle p von F −1 . Aufgabe 23.37 •• Da jeder Computer nur endlich viele
Zahlen darstellen kann, ist das Runden bei numerischen Aus-
Aufgabe 23.31 •• Zeigen Sie, dass aus dem Zen-
wertungen prinzipiell nicht zu vermeiden. Der Einfachheit
tralen Grenzwertsatz von Lindeberg-Feller derjenige von
halber werde jede reelle Zahl auf die nächstgelegene ganze
Lindeberg-Lévy folgt.
Zahl gerundet, wobei der begangene Fehler durch eine Zu-
fallsvariable R mit der Gleichverteilung U(−1/2, 1/2) be-
Aufgabe 23.32 •• Für eine u.i.v.-Folge (Xn ) mit 0 <
schrieben sei. Für verschiedene zu addierende Zahlen seien
σ 2 := V(X1 ) und EX14 < ∞ sei
diese Fehler stochastisch unabhängig. Addiert man 1200
Zahlen, so könnten sich die Rundungsfehler R1 , . . . , R1200
1
n
Sn2 := (Xj − X n )2 theoretisch zu ±600 aufsummieren. Zeigen Sie: Es gilt
n−1
j =1 ⎛ ⎞
n  1200


die sog. Stichprobenvarianz, wobei X n := n−1 j =1 Xj . P ⎝ Rj  ≤ 20⎠ ≈ 0.9554.
Zeigen Sie: j =1
Hinweise zu Kapitel 23 203

Aufgabe 23.38 •• Die Zufallsvariablen X1 , X2 , . . . Aufgabe 23.7 •• Der Durchschnitt endlich vieler Eins-
seien stochastisch unabhängig, wobei Xk ∼ N(0, k!), k ≥ 1. Mengen ist ebenfalls eine Eins-Menge.
Zeigen Sie:
Aufgabe 23.8 •• Wählen Sie in b) Yn := Xn 1
a) Es gilt der Zentrale Grenzwertsatz.
{Xn = ±1}.
b) Die Lindeberg-Bedingung ist nicht erfüllt.
Aufgabe 23.9 •• Die Vereinigung endlich vieler kom-
Aufgabe 23.39 •• In einer Bernoulli-Kette mit Treffer- pakter Mengen ist kompakt.
wahrscheinlichkeit p ∈ (0, 1) bezeichne Tn die Anzahl der
Versuche, bis der n-te Treffer aufgetreten ist. Aufgabe 23.10 •• Rechnen Sie die charakteristische
Funktion der Gleichverteilung U(0, 1) aus.
a) Zeigen Sie:
 √  Aufgabe 23.11 •• Beachten Sie das Lemma von Sluzki.
n + a n(1 − p)
lim P Tn > = 1−(a), a ∈ R.
n→∞ p
Aufgabe 23.12 •• –
b) Wie groß ist ungefähr die Wahrscheinlichkeit, dass bei
fortgesetztem Werfen eines echten Würfels die hunderts- Aufgabe 23.13 •• Verwenden Sie für b) das Lemma
te Sechs nach 650 Würfen noch nicht aufgetreten ist? von Sluzki.

Aufgabe 23.40 •• Wir hatten in Aufgabe 21.6 gesehen, Aufgabe 23.14 •• Deuten Sie die Summen wahrschein-
dass in einer patriarchisch orientierten Gesellschaft, in der lichkeitstheoretisch.
Eltern so lange Kinder bekommen, bis der erste Sohn ge-
boren wird, die Anzahl der Mädchen in einer aus n Fami- Aufgabe 23.15 •• Es liegt ein Dreiecksschema vor.
lien bestehenden Gesellschaft die negative Binomialvertei-
lung Nb(n, 1/2) besitzt. Zeigen Sie:
Beweisaufgaben
a) Für jede Wahl von a, b ∈ R mit a < b gilt
    Aufgabe 23.16 • –
 √ √  b a
lim P n + a n ≤ Sn ≤ b + n =  √ − √ .
n→∞ 2 2 Aufgabe 23.17 ••• Wählen Sie für b) unabhängige Zu-
1 fallsvariablen X1 , X2 , . . . mit P(Xn = 0) = 1 − n1 und
b) lim P(Sn ≥ n) = . P(Xn = 2n) = n1 , n ≥ 1, und schätzen Sie die Wahrschein-
n→∞ 2 
lichkeit P(n−1 nj=1 Xj > 1) nach unten ab. Verwenden Sie
dabei die Ungleichung log t ≤ t − 1 sowie die Beziehung

Hinweise
k
1
− log k − γ → 0 für k → ∞,
j
j =1
Verständnisfragen
Aufgabe 23.1 • Betrachten Sie die Ereignisse wobei γ die Euler-Mascheroni’sche Konstante bezeichnet.
{|Xn − X| ≤ 1/k}.
Aufgabe 23.18 •• Wenden Sie das Lemma von Borel-
Aufgabe 23.2 • Verwenden Sie die auf Seite 868 an- Cantelli einmal auf die Ereignisse An = {Xn = 1}, n ≥ 1,
gegebene Charakterisierung der fast sicheren Konvergenz. und zum anderen auf die Ereignisse Bn = {Xn = 0}, n ≥ 1,
an.
Aufgabe 23.3 •• In einem diskreten Wahrscheinlich-
keitsraum (, A, P) gibt es eine abzählbare Teilmenge Aufgabe 23.19 •• Überlegen Sie sich, dass das Infi-
0 ∈ A mit P(0 ) = 1. mum angenommen wird.

Aufgabe 23.4 • Verwenden Sie das Teilfolgenkrite- Aufgabe 23.20 ••• Betrachten Sie die Teilfolge
rium für stochastische Konvergenz. X1 , Xk+1 , X2k+1 , . . .

Aufgabe 23.5 •• Der Durchschnitt endlich vieler Eins- Aufgabe 23.21 •• Verwenden Sie das Lemma von
Mengen ist ebenfalls eine Eins-Menge. Borel-Cantelli.

Aufgabe 23.6 • Zerlegen Sie Xn in Positiv- und Ne- Aufgabe 23.22 •• Verwenden Sie das Kolmogorov-
gativteil. Kriterium und beachten Sie ∞
n=2 1/(n(log n) ) < ∞.
2
204 Lösungen zu Kapitel 23

Aufgabe 23.23 •• Nutzen Sie für b) die Verteilungs- Lösungen


gleichheit (X1 , . . . , Xn ) ∼ (1 − X1 , . . . , 1 − Xn ) aus.
Verständnisfragen
Aufgabe 23.24 •• Betrachten Sie die Fälle a = 0,
a > 0 und a < 0 getrennt. Aufgabe 23.1 • –

Aufgabe 23.2 • –
Aufgabe 23.25 •• –
Aufgabe 23.3 •• –
Aufgabe 23.26 •• Verwenden Sie für „⇐“ die Markov-
Ungleichung P(|Xn | > L) ≤ L−2 E Xn2 . Überlegen Sie sich Aufgabe 23.4 • –
für „⇒“ zunächst, dass die Folge (μn ) beschränkt ist.
Aufgabe 23.5 •• –
Aufgabe 23.27 ••• –
Aufgabe 23.6 • –

Aufgabe 23.28 •• Taylorentwicklung von g um μ! Aufgabe 23.7 •• Es sei (Xn )n≥1 eine Folge stocha-
stisch unabhängiger und identisch verteilter k-dimensionaler
Aufgabe 23.29 •• Schätzen Sie die Differenz Fn (x) − Zufallsvektoren auf einem Wahrscheinlichkeitsraum
F (x) mithilfe der Differenzen Fn (xj k ) − F (xj k ) ab, wobei (, A, P) mit EX∞ < ∞. Dann gilt
für k ≥ 2 xj k := F −1 (j/k), 1 ≤ j < k, sowie x0k := −∞,
1
n
xkk := ∞. f.s.
Xj −→ EX1 ,
n
j =1
Aufgabe 23.30 •• –
wobei EX1 der Vektor der Erwartungswerte der Komponen-
ten von X1 ist.
Aufgabe 23.31 •• Weisen Sie die Lindeberg-Bedin-
gung nach. Aufgabe 23.8 •• –

Aufgabe 23.32 •• Es ist Xj −X n = Xj −μ−(X n −μ). Aufgabe 23.9 •• –

Aufgabe 23.10 •• –
Aufgabe 23.33 • –
Aufgabe 23.11 •• –
Aufgabe 23.34 •• Prüfen Sie die Gültigkeit der
Lindeberg-Bedingung. Aufgabe 23.12 •• c) (1).

Aufgabe 23.13 •• –
Aufgabe 23.35 •• Mit aj = EXj gilt E(Xj − aj )4 ≤
aj (1 − aj ).
Aufgabe 23.14 •• –

Rechenaufgaben Aufgabe 23.15 •• –

Aufgabe 23.36 •• –
Beweisaufgaben
Aufgabe 23.37 •• Zentraler Grenzwertsatz! Aufgabe 23.16 • –

n Aufgabe 23.17 ••• b) Nein.


Aufgabe 23.38 •• Wie verhält sich n! zu k=1 k!?

Aufgabe 23.18 •• –
Aufgabe 23.39 •• Stellen Sie Tn als Summe von unab-
hängigen Zufallsvariablen dar. Aufgabe 23.19 •• –

Aufgabe 23.40 •• Verwenden Sie das Additionsgesetz Aufgabe 23.20 ••• –


für die negative Binomialverteilung und den Zentralen
Grenzwertsatz von Lindeberg-Lévy. Aufgabe 23.21 •• –
Lösungswege zu Kapitel 23 205

Aufgabe 23.22 •• – Aufgabe 23.2 • Aus der Voraussetzung folgt 0 ≤ X −


Xn+1 ≤ X − Xn für jedes n ≥ 1 und somit
Aufgabe 23.23 •• –
sup |Xk − X| = |Xn − X|.
k≥n
Aufgabe 23.24 •• –
Nach Voraussetzung gilt P(|Xn − X| > ε) → 0 für jedes
Aufgabe 23.25 •• –
ε > 0. Aus obiger Gleichheit und dem Kriterium für fast
sichere Konvergenz folgt die Behauptung.
Aufgabe 23.26 •• –

Aufgabe 23.3 •• In einem diskreten Wahrscheinlich-


Aufgabe 23.27 ••• –
keitsraum (, A, P) gibt es eine abzählbare Teilmenge
0 ∈ A mit P(0 ) = 1. Wir zeigen, dass aus der stocha-
Aufgabe 23.28 •• – P
stischen Konvergenz Xn −→ X die Konvergenz Xn (ω0 ) →
Aufgabe 23.29 •• – X(ω0 ) für jedes ω0 ∈ 0 mit P({ω0 }) > 0 folgt, womit
f.s.
Xn −→ X gezeigt wäre.
Aufgabe 23.30 •• –
Sei hierzu ω0 ∈ 0 mit P({ω0 }) > 0 beliebig, aber fest
Aufgabe 23.31 •• – gewählt. Würde Xn (ω0 ) nicht gegen X(ω0 ) konvergieren,
so gäbe es zu jedem ε > 0 eine Teilfolge (Xnj )j ≥1 mit
Aufgabe 23.32 •• – |Xnj (ω0 ) − X(ω0 )| > ε für jedes j ≥ 1. Es würde also

{ω0 } ⊆ {ω ∈  | |Xnj (ω) − X(ω)| > ε}


Aufgabe 23.33 • –
= {|Xnj − X| > ε}, j ≥ 1,
Aufgabe 23.34 •• –
und somit
Aufgabe 23.35 •• –
P({ω0 }) ≤ P(|Xnj − X| > ε), j ≥ 1,
Rechenaufgaben
gelten. Da mit (Xn ) auch die Teilfolge (Xnj ) stochastisch
Aufgabe 23.36 •• – gegen X konvergiert, gilt

Aufgabe 23.37 •• – lim P(|Xnj − X| > ε) = 0


j →∞

Aufgabe 23.38 •• – und somit P({ω0 }) = 0, im Widerspruch zur Voraussetzung.


f.s.
Folglich gilt Xn −→ X.
Aufgabe 23.39 •• –

Aufgabe 23.40 •• – Aufgabe 23.4 • Wir benutzen das Teilfolgenkriterium


auf Seite 870. Es sei (Xnk )k≥1 eine beliebige Teilfolge von
(Xn )n≥1 . Nach besagtem Kriterium existiert eine weitere
f.s.
Teilfolge (Xn )k≥1 mit Xn −→ X, also limk→∞ Xn (ω) =
k k k
Lösungswege X(ω) für jedes ω aus einer Eins-Menge 0 und somit wegen
An → A auch limk→∞ An Xn (ω) = AX(ω), ω ∈ 0 .
k k k
Verständnisfragen Die Behauptung folgt somit aus dem Teilfolgenkriterium.

Aufgabe 23.1 • Es gilt Aufgabe 23.5 •• Eine direkte Rechnung ergibt


0 1 < 4 < 1 n
lim Xn = X = |Xn − X| ≤ , 1
n − Xn Y n
j =1 Xj Yj
n→∞
k≥1 m≥1 n≥m
k Rn = D   
1 n 2 − X2 1 n 2 − Y2
n X
j =1 j n n Y
j =1 j n
denn ein ω ∈  liegt genau dann in der links stehenden
Menge, wenn es zu jedem k ∈ N ein m ∈ N gibt, sodass für
jedes n ≥ m die Ungleichung |Xn (ω) − X(ω)| ≤ 1/k gilt. Wegen E|X1 Y1 | ≤ (EX12 EY12 )1/2 < ∞ sowie E|X1 | < ∞,
Da jede der Mengen {|Xn − X| ≤ 1/k} zu A gehört und A E|Y1 | < ∞ können wir das Starke Gesetz großer Zahlen
gegenüber abzählbaren Durchschnitten und Vereinigungen jeweils auf die u.i.v.-Folgen (Xj Yj ), (Xj ), (Yj ), (Xj2 ) und
abgeschlossen ist, gilt {limn→∞ Xn = X} ∈ A. (Yj2 ) anwenden und erhalten auf Eins-Mengen 1 , . . . 5
206 Lösungswege zu Kapitel 23

1
n
Es folgt
Xj (ω)Yj (ω) → EX1 Y1 , ω ∈ 1 ,
n ∞ ∞ ∞ n
j =1 V(Xn ) 1 − 2−n + 2n 2
2
= 2

n n n2
1 1
n n
n=1 n=1 n=1
Xj (ω) → EX1 , ω ∈ 2 , Yj (ω) → EY1 , ω ∈ 3 , ∞
n n n
j =1 j =1 ≥ = ∞,
n2
1 2 1 2
n n n=1
Xj (ω) → EX12 , ω ∈ 4 , Yj (ω) → EY12, ω ∈ 5 .
n n sodass die Folge (Xn ) nicht dem Kolmogorov-Kriterium ge-
j =1 j =1
nügt.
Wegen 0 < V(X1 ) = EX12 − (EX1 )2 , 0 < V(Y1 ) =
b) Wir setzen Yn := Xn 1{Xn = ±1}, also
EY12 − (EY1 )2 gilt dann für jedes ω aus der Eins-Menge
1 ∩ . . . ∩ 5 die Konvergenz Rn (ω) → #(X1 , Y1 ). 1 1
P(Yn = 1) = P(Yn = −1) = (1−2−n ), P(Yn = 0) = n .
2 2
Aufgabe 23.6 • Es sei Xn = Xn+ − Xn− die Zerlegung
Es gilt
von Xn in Positivteil Xn+ = max(Xn , 0) und Negativteil
Xn− = − min(Xn , 0). Gilt ∞


1
P(Xn  = Yn ) = P(Yn = 0) = <∞
2n
1 + f.s. 1 − f.s.
n n
n=1 n=1 n=1
Xj −→ EX1+ , Xj −→ EX1−
n n
j =1 j =1 sowie V(Yn ) = EYn2 = 1 − 2−n . Nach dem Kolmogorov-
 f.s.
und somit punktweise Konvergenz der Folge der ersten bzw. Kriterium gilt n−1 nj=1 Yj −→ 0 und zusammen mit Auf-
 f.s.
zweiten Mittelwerte auf Eins-Mengen 1 bzw. 2 , so folgt gabe 23.21 dann auch n−1 nj=1 Xj −→ 0.
für jedes ω ∈ 1 ∩ 2 die Konvergenz
Aufgabe 23.9 •• Es seien Q = {Q1 , . . . , Qk } und
1 1 + 1 −
n n n
Xj (ω) = Xj (ω) − Xj (ω) ε > 0 beliebig. Wegen [−n, n] ↑ R und der Tatsache,
n n n dass ein Wahrscheinlichkeitsmaß stetig von unten ist, exi-
j =1 j =1 j =1
stiert zu jedem j ∈ {1, . . . , k} eine kompakte Menge Kj mit
→ EX1+ − EX1− = EX1 .
Qj (Kj ) ≥ 1 − ε. Die Menge K := K1 ∪ . . . ∪ Kk ist kom-
n f.s. pakt, und es gilt Qm (K) ≥ 1 − ε für jedes m = 1, . . . , k,
Wegen P(1 ∩ 2 ) = 1 gilt also n−1 j =1 Xj −→ EX1 . was zu zeigen war.
(1) (k)
Aufgabe 23.7 •• Es sei Xn = (Xn , . . . , Xn ), n ≥ 1, Aufgabe 23.10 •• Eine Zufallsvariable mit der Gleich-
(1) (k)
sowie EX1 = (EX1 , . . . , EX1 ). Nach Voraussetzung verteilung U(−n, n) besitzt die auf (−n, n) konstante Dichte
ist für jedes j ∈ {1, . . . , k} die j -te Komponentenfolge 1/(2n) und somit aus Symmetriegründen die charakteristi-
(j ) sche Funktion
(Xn )n≥1 eine Folge von unabhängigen identisch verteilten
(j ) n
Zufallsvariablen mit existierendem Erwartungswert EX1 . 1
ψ(t) = eitx dx
Nach dem starken Gesetz großer Zahlen gibt es eine Menge 2n −n
j ∈ A mit P(j ) = 1, sodass für jedes ω ∈ j die Kon- n
1
vergenz = cos(tx) dx
2n −n
1 (j ) n
n
(j )
lim Xk (ω) = EX1 =
1
cos(tx) dx
n→∞ n
k=1 n 0
besteht. Für die Menge 0 := ∩kj =1 j gilt P(0 ) = 1, und sin(nt)
= , t = 0,
für jedes ω ∈ 0 gilt die (vektorielle) Konvergenz nt
und ψ(0) = 0. Nach dem Eindeutigkeitssatz für charakteri-
1
n
lim Xl (ω) = EX1 , stische Funktionen gilt Xn ∼ U(−n, n). Ist k ∈ N beliebig,
n→∞ n
l=1 so gilt für n ≥ k

was zu zeigen war. 2k


P(|Xn | ≤ k) =
2n
Aufgabe 23.8 •• a) Es gilt EXn = 0 und und folglich limn→∞ P(|Xn | ≤ k) = 0. Also ist die Folge
(Xn ) nicht straff; sie kann somit auch nicht nach Verteilung
V(Xn ) = EXn2
konvergieren. Die Folge (ϕn ) konvergiert punktweise gegen
1 22n die durch ϕ(t) = 1 für t = 0 und ϕ(0) = 0 gegebene
= (1 − 2−n ) · 2 + n+1
2 2 Funktion. Da diese nicht stetig im Nullpunkt ist, liegt kein
= 1 − 2−n + 2n . Widerspruch zum Stetigkeitssatz von Lévy-Cramér vor.
Lösungswege zu Kapitel 23 207

Aufgabe 23.11 •• Es ist c) Bezeichnet


n
Sn −
Yn − bn Yn − an σ n an − bn σn Sn∗ = √
2
n
= · + · 2
τn σn τn σn τn
die standardisierte Zufallsvariable, so folgt mit tn :=
Yn − an
=: · un + vn . n sin( n1 )
σn     
n √ 1
Nach Voraussetzung gelten un → 1 und vn → 0, und so- P Sn ≤ n sin +1 = P(Sn∗ ≤ tn ).
2 n
mit folgt die Behauptung aus dem Lemma von Sluzki auf
Wegen tn → 1 folgt nach a)
Seite 881.     
n √ 1
lim P Sn ≤ n sin +1 = (1).
Setzt
n man speziell mit der n-ten harmonischen Zahl Hn = n→∞ 2 n
−1
j =1 j

Aufgabe 23.13 •• a) Da B f 2 (x) dx < ∞ vorausge-

n
1 setzt ist, gilt nach dem starken Gesetz großer Zahlen
an = Hn , σn2 = Hn − , bn = log n, τn2 = log n,
j2 f.s.
j =1
Jn −→ |B| · Ef 2 (U1 ) = f 2 (x) dx.
B
so liegen nach dem Beispiel auf Seite 892 die obigen Vor- Zusammen mit (23.13) folgt
aussetzungen für Rn = Yn (Anzahl der Rekorde) vor und es  
folgt die am Ende des Beispiels behauptete Asymptotik. 2 2 Jn In2
σn = |B| −
|B| |B|2
  2 
Aufgabe 23.12 •• a) Es sei ε > 0 beliebig. Wir wählen 1 1
f.s.
a > 0 so, dass t + a ∈ C (F ) und t − a ∈ C (F ) und −→ |B|2 f 2 (x) dx − f (x)dx
|B| B |B|2 B

F (t + a) ≤ F (t) + ε, F (t − a) ≥ F (t) − ε. = σf2 .

Für n ≥ n0 (ε) gilt t − a ≤ tn ≤ t + a. Wegen der Monotonie f.s. f.s.


D b) Nach a) gilt σn /σf −→ 1 und somit auch σf /σn −→ 1,
von Fn und Yn −→ Y ergibt sich für solche n P
also auch σf /σn −→ 1. Wegen
Fn (tn ) ≤ Fn (t + a) → F (t + a) ≤ F (t) + ε, √ √
n(In − I ) n(In − I ) σf
= ·
Fn (tn ) ≥ Fn (t − a) → F (t − a) ≤ F (t) − ε. σn σf σn
folgt die Behauptung aus Teil b) des Lemmas von Sluzki auf
Es folgt
Seite 881.
F (t) − ε ≤ lim inf Fn (tn ) ≤ lim sup Fn (tn ) ≤ F (t) + ε Aufgabe 23.14 •• a) Seien X1 , X2 , . . . unabhängige
n→∞ n→∞
und je Po(1)-verteilte Zufallsvariablen. Nach dem Additi-
und damit die Behauptung, da ε beliebig klein gewählt wer- onsgesetz für die Poisson-Verteilung gilt dann Sn := X1 +
den kann. . . . + Xn ∼ Po(n), und wegen EX1 = V(X1 ) = 1 liefert der
Zentrale Grenzwertsatz von Lindeberg-Lévy
b) Sei Sn∗ die standardisierte Partialsumme in den oben ge-
n
nk
nannten Sätzen. Nach Voraussetzung gilt e−n = P(Sn ≤ n)
k!
k=0  
lim P(Sn∗ ≤ t) = (t), t ∈ R. Sn − n
n→∞ √= P ≤0
n
Nun ist P(Sn∗ < t) ≤ P(Sn∗ ≤ t) und somit → (0)
1
= .
lim sup P(Sn∗ < t) ≤ (t). 2
n→∞
b) Mit den Bezeichnungen von a) gilt
Andererseits gibt es zu beliebig vorgegebenem ε > 0 ein
2n
nk
a > 0 mit (t − a) ≥ (t) − ε. Es folgt e−n = P(Sn ≤ 2n)
k!
k=0  
Sn − n 2n − n
(t) − ε ≤ (t − a) ⇐ P(Sn∗ ≤ t − a) ≤ P(Sn∗ < t) = P √ ≤ √
n n
 
und somit lim inf n→∞ P(Sn∗ < t) ≥ (t) − ε. Lässt man ε Sn − n √
= P √ ≤ n .
gegen 0 streben, so ergibt sich die Behauptung. n
208 Lösungswege zu Kapitel 23

Sei ε > 0 beliebig und a so, dass (a) ≥ 1−ε. Für genügend Aufgabe 23.17 ••• a) Für jedes ω ∈  folgt aus

großes n gilt n ≥ a und somit Xn (ω) → 0 nach
 dem Grenzwertsatz von Cauchy (vgl.
    Seite 875) n−1 nj=1 Xj (ω) → 0. Somit gilt unter der Vor-
Sn − n √ Sn − n
P √ ≤ n ≥ P √ ≤a aussetzung
n n
⎛ ⎞
→ (a)
n  
1
≥ 1 − ε. P ⎝ lim Xj = 0⎠ ≥ P lim Xn = 0 = 1.
n→∞ n n→∞
j =1
Insgesamt folgt

2n
nk b) Seien X1 , X2 , . . . wie im Hinweis. Wegen P(|Xn | > ε) =
lim inf e−n ≥1−ε P
n→∞ k! P(Xn = 2n) = 1/n → 0 gilt dann Xn −→ 0. Andererseits
k=0
gilt aufgrund der Unabhängigkeit von X1 , X2 , . . . und wegen
und damit die Behauptung, da ε > 0 beliebig war.
der Ungleichung log(1 + x) ≤ x für jedes n ≥ 3
Aufgabe 23.15 •• Es gilt Sn ∼ Xn,1 + . . . + Xn,n mit ⎛ ⎞ ⎛ ⎞
unabhängigen Zufallsvariablen Xn,j ∼ Bin(1, pn ), 1 ≤ j ≤ 1 n n
P⎝ Xj > 1⎠ = P ⎝ Xj > n⎠
n. Es liegt also ein Dreiecksschema {Xn,j : n ≥ 1, 1 ≤ j ≤ n
j =1 j =1
n} vor. Wir prüfen die Gültigkeit der Ljapunov-Bedingung ⎛ ⎞
(23.37) mit δ = 2 nach. Es gilt 4n
≥ P⎝ {Xj > n}⎠
σn2 = V(Sn ) = npn (1 − pn ) j =4n/25
⎛ ⎞
und an,j = E(Xn,j ) = pn . Wegen |Xn,j − an,j |4 ≤ 1 ergibt <
n
sich = 1 − P⎝ {Xj ≤ n}⎠
j =4n/25
1
n
n·1
E|Xn,j − an,j |4 ≤ 
n
σn4 n2 pn2 (1 − pn )2
j =1 = 1− P(Xj ≤ n)
→ 0 j =4n/25
n  
für n → ∞, was zu zeigen war. = 1− 1−
1
j
j =4n/25
Beweisaufgaben ⎛ ⎞

n  
1
f.s. = 1 − exp ⎝ log 1 − ⎠
Aufgabe 23.16 • a) : Es gelte Xn −→ X, also j
j =4n/25
Xn (ω) → X(ω), ω ∈ 0 , wobei 0 ∈ A und P(0 ) = 1. ⎛ ⎞
(j )
Dann folgt Xn (ω) → X (j ) (ω), ω ∈ 0 , für jedes j = n
1⎠
≥ 1 − exp ⎝− .
1, . . . , k, also die komponentenweise fast sichere Konver- j
(j ) f.s. j =4n/25
genz. Gilt umgekehrt Xn −→ X (j ) für jedes j = 1, . . . , k,
so existieren Mengen 1 , . . . , k ∈ A mit P(j ) = 1 und Mit dem Hinweis und einer mit o(1) bezeichneten Nullfolge
(j )
Xn (ω) → X (j ) (ω), ω ∈ j , für jedes j = 1, . . . , k. Für gilt
die Menge 0 := 1 ∩ . . . ∩ k gilt P(0 ) = 1, und für

n
1
jedes ω ∈ 0 konvergiert Xn (ω) gegen X(ω). Es gilt also = log n − log (4n/25 − 1)) + o(1)
f.s. j
Xn −→ X. j =4n/25
b) : Für jedes l = 1, . . . , k und jedes ε > 0 gilt ≥ log n − log (4n/25)) + o(1)
 
{|Xn(l) (l)
− X | > ε} ⊆ {Xn − X∞ > ε} 3
≥ log n − log n + o(1)
4
4
k
(j )  
= {|Xn − X (j ) | > ε}. 4
= log + o(1).
j =1 3
P (l) n
Aus Xn −→ X folgt also P(|Xn − X (l) | > ε) → 0, l = Somit kann n−1 j =1 Xj nicht stochastisch gegen null kon-
(j ) P
1, . . . , k, und umgekehrt zieht Xn −→ X (j ) , j = 1, . . . , k, vergieren.
die Abschätzung
Aufgabe 23.18 •• Es sei ε > 0. Wegen P(|Xn | ≥

k
P
P (Xn − X∞ > ε) ≤
(j )
P(|Xn −X (j )
| > ε) ε)
∞ = P(Xn = 1) = 1/n gilt Xn −→ 0. Andererseits gilt
j =1 n=1 P(Xn = 1) = ∞. Weil die Ereignisse An := {Xn =
1}, n ≥ 1, unabhängig sind, liefert Teil b) des Lemmas von
P
und somit Xn −→ X nach sich. Borel-Cantelli die Beziehung P(lim supn→∞ An ) = 1. Für
Lösungswege zu Kapitel 23 209

jedes ω ∈ lim sup An gilt Xn (ω) = 1 für unendlich viele n (ii) Gilt d(Xn , X) → 0, so existiert zu jedem δ > 0 ein
und somit n0 ∈ N mit d(Xn , X) < δ für jedes n ≥ n0 , d. h.P(|Xn −
lim sup Xn (ω) = 1. (23.18) X| > δ) ≤ δ. Setzt man δ := min(ε, η) zu beliebigen ε > 0,
n→∞
η > 0, so erhält man für jedes n ≥ n0
In gleicher Weise gilt aber auch P(lim supn→∞ Bn ) = 1,
wobei Bn := {Xn = 0} und somit P(|Xn − X| > ε) ≤ P(|Xn − X| > δ) ≤ δ ≤ η

lim inf Xn (ω) = 0 (23.19) P


n→∞ und damit Xn −→ X.

für jedes ω ∈ lim sup Bn . Für jedes ω aus der Eins-Menge Aufgabe 23.20 ••• Ohne Beschränkung der Allgemein-
lim sup An ∩ lim sup Bn gilt also sowohl (23.18) als auch heit seien alle Xn nichtnegativ (andernfalls betrachte man
(23.19), was zeigt, dass die Folge (Xn ) P-fast sicher nicht Positiv- und Negativteil getrennt). Nach Voraussetzung be-
konvergiert. stehen für j ∈ {1, . . . , k} die Folgen (X(l−1)k+j )l∈N aus un-
abhängigen und identisch verteilten Zufallsvariablen. Setzen
Aufgabe 23.19 •• a) Für εn > 0, n ≥ 1, mit εn ↓ d := wir JnK
d(X, Y ) gilt ln := + 1,
k
P(|X − Y | > εn ) ≤ εn ≤ εm so gilt
n
für m ≥ n. Wegen 1{|X − Y | > εn } ↑ 1{|X − Y | > d} folgt ln − 1 ≤ < ln ,
k
aus dem Satz von der monotonen Konvergenz
und es folgt
P(|X − Y | > d) = lim P(X − Y | > εn ) ≤ εm
1 1
n→∞ n ln k
Xj ≤ Xj
für jedes m ≥ 1. Also ist P(|X − Y | > d) ≤ d, d. h., das n n
j =1 j =1
Infimum wird angenommen.
1
k ln
b) Nach a) ist = X(l−1)k+j
n
j =1 l=1
d(X, Y ) = 0 ⇐⇒ P(|X−Y | > 0) = 0 ⇐⇒ P(X = Y ) = 1.
ln 1
k ln
= X(l−1)k+j .
n ln
c) Nach a) ist j =1 l=1

P(|X − Y | > d(X, Y )) ≤ d(X, Y ), Zu jedem j ∈ {1, . . . , k} existiert eine Menge j ∈ A mit
P(j ) = 1, sodass gilt:
P(|Y − Z| > d(Y, Z)) ≤ d(Y, Z).

1
ln
Aus der Teilmengenbeziehung
lim X(l−1)k+j (ω) = EX1 für jedes ω ∈ j .
n→∞ ln
{|X − Y | + |Y − Z| > d(X, Y ) + d(Y, Z)} l=1

⊂ {|X − Y | > d(X, Y )} ∪ {|Y − Z| > d(Y, Z)} Für 0 := ∩kj =1 j gilt P(0 ) = 1, und für jedes ω ∈ 0
erhält man
ergibt sich daher
ln 1
k ln
P(|X − Y | + |Y − Z| > d(X, Y ) + d(Y, Z)) 1
X(l−1)k+j (ω) → · k · EX1 = EX1 .
n ln k
≤ P(|X − Y | > d(X, Y )) + P(|Y − Z| > d(Y, Z)) j =1 l=1
≤ d(X, Y ) + d(Y, Z).
Daraus ergibt sich
Weiter folgt aus der Dreiecksungleichung |X − Z| ≤
1
n
|X − Y | + |Y − Z| lim sup Xj ≤ EX1 P-fast sicher.
n→∞ n
j =1
P(|X − Z| > d(X, Y ) + d(Y, Z)) ≤ d(X, Y ) + d(Y, Z),
In gleicher Weise zeigt man
also insgesamt d(X, Z) ≤ d(X, Y ) + d(Y, Z).
1
n
P
d) (i) Aus Xn −→ X folgt, dass zu jedem m ∈ N ein n(m) lim inf Xj ≥ EX1 P-fast sicher,
n→∞ n
existiert, sodass j =1
 
1 1 woraus die Behauptung folgt.
P |Xn − X| > ≤ für n ≥ n(m).
m m
b) Es sei Xj := 1{Yj < Yj +1 } für j ≥ 1. Dann ist (Xn )n≥1
Somit gilt d(Xn , X) → 0. eine Folge identisch verteilter Zufallsvariablen. Weiter sind
210 Lösungswege zu Kapitel 23

Xn und Xm stochastisch unabhängig, falls |n − m| ≥ 2. die die Stammfunktion G(x) = −1/ log x besitzt. Nach dem
Wegen Kolmogorov-Kriterium gilt
5
EX1 = P(Y1 < Y2 ) = n  
12 1 1
lim Xj − = 0 P-fast sicher.
n→∞ log n j
folgt die Behauptung aus Teil a). j =1

Wegen
Aufgabe 23.21 •• Aus der Voraussetzung folgt mit
1 1
n
dem Lemma von Borel-Cantelli lim = 1
n→∞ log n j
  j =1
P lim sup{Xn  = Yn } = 0. folgt die Behauptung. Für die Gültigkeit der letzten Limes-
n→∞
beziehung
 beachte man, dass die n-te harmonische Zahl
Damit gibt es ein 0 ∈ A mit P(0 ) = 1, und es gilt: Hn = nj=1 j −1 mittels Integralabschätzung (Vergleich mit
den Funktionen f (x) = 1/x und g(x) = 1/(1 + x)) die
∀ ω ∈ 0 ∃ n0 (ω) ∈ N : Xn (ω) = Yn (ω) ∀ n ≥ n0 . Ungleichungen
 f.s. log(n + 1) ≤ Hn ≤ 1 + log n
Wegen n−1 nj=1 Yj −→ 0 existiert ein 1 ∈ A mit
P(1 ) = 1, und es gilt: erfüllt.

1
n
∀ ω ∈ 1 : lim Yj (ω) = 0. Aufgabe 23.23 •• a) Für jedes t > 0 gilt
n→∞ n
j =1
   
P n 1 − max Xj ≤ t
1≤j ≤n
Setzen wir 2 := 0 ∩ 1 , so gilt P(2 ) = 1. Für jedes  
ω ∈ 2 und jedes n ≥ n0 (ω) folgt t
= P max Xj ≥ 1 −
1≤j ≤n n
1   
1
n n
  t
 Xj (ω) − Yj (ω) = 1 − P max Xj < 1 −
n n 1≤j ≤n n
j =1 j =1  n
1 0 −1
n t
1 = 1 − P X1 < 1 −
n

=  Xj (ω) + Xj (ω) n
n n  
j =1 j =n0 t n
= 1− 1− (falls n ≥ t)
n0 −1
1 1
n  n

− Yj (ω) + Yj (ω) → 1 − exp(−t).
n n
j =1 j =n0
Für t ≤ 0 gilt
0 −1
n
1      
≤ |Xj (ω)| + |Yj (ω)| .
n
j =1 P n 1 − max Xj ≤t = 0.
1≤j ≤n

Da diese obere Schranke für n → ∞ gegen null konvergiert, Da die durch F (t) = 1 − exp(−t) für t > 0 und F (t) =
folgt die Behauptung. 0 sonst, die Verteilungsfunktion der Exponentialverteilung
Exp(1) ist, folgt die Behauptung.
Aufgabe 23.22 •• Es sei an := log n für n ≥ 2. Wegen
Xn ∼ Bin(1, 1/n) gilt b) Wegen Xj ∼ 1 − Xj und der stochastischen Unabhängig-
keit der Xj gilt die Verteilungsgleichheit
 
1 1 1
EXn = , V(Xn ) = 1− . (X1 , . . . , Xn ) ∼ (1 − X1 , . . . , 1 − Xn ).
n n n
Es folgt
Es folgt
∞ ∞
max Xj ∼ max (1 − Xj )
V(Xn ) 1 1≤j ≤n 1≤j ≤n
≤ < ∞. = 1 − min Xj
an2 n(log n)2 1≤j ≤n
n=2 n=2

Dabei ergibt sich die Konvergenz der Reihe durch eine Inte- und damit
 
gralabschätzung mit der Funktion
n 1 − max Xj ∼ n min Xj .
1≤j ≤n 1≤j ≤n
1
g(x) := ,
x(log x)2 Hieraus folgt zusammen mit a) die Behauptung.
Lösungswege zu Kapitel 23 211

Aufgabe 23.24 •• Im Fall a = 0 gilt wegen Xn = b) folgt aus a) mit der Wahl P(Yn = bn ) = 1 für jedes n.
P
OP (1) nach Aufgabe c) Xn Yn −→ 0 und somit nach dem c) Es sei ε > 0 beliebig. Zu zeigen ist
D
Satz auf Seite 881 auch Xn Yn −→ 0. Wir können uns also
im Folgenden auf den Fall a  = 0 beschränken, wobei wir lim P(|an Xn | > ε) = 0.
n→∞
o.B.d.A. a > 0 annehmen. Sei Fn (t) := P(Xn Yn ≤ t),
Für jedes positive C folgt aus |an Xn | > ε, dass entwe-
F (t) := P(X ≤ t) und G(t) := P(aX ≤ t), t ∈ R. Sei t eine
der |an | > ε/C oder |Xn | > C (oder beides) gilt. Wäh-
beliebige Stetigkeitsstelle von G. Zu zeigen ist
len wir zu beliebig vorgegebenem η > 0 ein C so, dass
lim Fn (t) = G(t). (23.20) P(|Xn | > C) ≤ η, n ≥ 1, gilt (dies ist wegen der Straffheit
n→∞ von (Xn ) möglich), so müssen wir nur noch die Konvergenz
Zum Nachweis von (23.20) beschränken wir uns auf den Fall limn→∞ an = 0 ausnutzen, um zum gewünschten Ergebnis
t ≥ 0. Es sei ε mit 0 < ε < a beliebig. Es gilt zu gelangen. Wegen dieser Konvergenz gibt es ein von ε und
C abhängendes n0 , sodass für jedes n > n0 die Ungleichung
Fn (t) |an | ≤ ε/C gilt. Für solche n gilt also nach obiger Überle-
=P(Xn Yn ≤ t, |Yn − a| ≤ ε) + P(Xn Yn ≤ t, |Yn − a| > ε) gung die Inklusion
≤P(Xn (a − ε) ≤ t) + P(|Yn − a| > ε) {|an Xn | > ε} ⊂ {|Xn | > C}
 
t
=P Xn ≤ + P(|Yn − a| > ε). und folglich
a−ε
lim sup P(|an Xn | > ε) ≤ η.
Ist t/(a − ε) eine Stetigkeitsstelle von F , so folgt wegen n→∞
D
Xn −→ X Da η beliebig war, folgt die Behauptung.
 
t Aufgabe 23.26 •• „⇐“: Falls |μn | ≤ C und σn2 ≤ C,
lim sup Fn (t) ≤ F .
n→∞ a−ε n ≥ 1, für ein C < ∞, so liefert die Markov-Ungleichung
Lassen wir ε eine Nullfolge mit der Nebenbedingung EXn2 σn2 + μ2n C + C2
t/(a − ε) ∈ C (F ) durchlaufen, so folgt P(|Xn | > L) ≤ = ≤ .
L2 L 2 L2
 
t Wählt man zu vorgegebenem ε > 0
lim sup Fn (t) ≤ F = G(t). 
n→∞ a
C + C2
L := √ ,
Ganz analog zeigt man ε
 
t so folgt
lim inf Fn (t) ≥ F = G(t).
n→∞ a P(Xn ∈ [−L, L]) ≥ 1 − ε, n ≥ 1.
Dies zeigt, dass die Folge (Xn ) straff ist, also Xn = OP (1)
Aufgabe 23.25 •• a) Es sei ε > 0 gegeben. Wir wählen gilt.
C > 0 so, dass für jedes n ≥ 1
ε „⇒“: Wäre die Folge (μn ) unbeschränkt, so gäbe es eine
P(|Xn | ≤ C) ≥ 1 − , Teilfolge (μnk )k≥1 mit |μnk | → ∞ für k → ∞. Wegen
2
ε 1
P(|Yn | ≤ C) ≥ 1 − P(Xnk ≥ μnk ) = P(Xnk ≤ μnk ) =
2 2
gilt. Die Existenz eines solchen C ist wegen der Straffheit kann es dann zu vorgegebenem ε > 0 kein kompaktes Inter-
der Folgen (Xn ) und (Yn ) gesichert. Es folgt vall K mit P(Xn ∈ K) ≥ 1 − ε für jedes n ≥ 1 geben. Somit
muss die Folge (μn ) notwendigerweise beschränkt sein. Es
P(|Xn + Yn | ≤ 2C) ≥ P(|Xn | ≤ C, |Yn | ≤ C) gibt also ein C > 0 mit |μn | ≤ C für jedes n ≥ 1. Wäre die
ε
≥ 1−2· Folge (σn2 ) unbeschränkt, so gäbe es eine Teilfolge (σn2k )k≥1
2 mit σnk → ∞ für k → ∞. Für L > 0 gilt dann
= 1−ε  
Xnk − μnk L − μnk
für jedes n ≥ 1. Da das Intervall [−2C, 2C] kompakt ist, ist P(Xnk > L) = P >
σn σnk
die Folge (Xn + Yn ) straff. Wegen  k 
L − μnk
= 1−
P(|Xn Yn | ≤ C 2 ) ≥ P(|Xn | ≤ C, |Yn | ≤ C) σnk
ε  
≥ 1−2· L+C
2 ≥ 1−
σnk
= 1−ε
1
→ für k → ∞.
für jedes n ≥ 1 ist auch die Folge (Xn Yn ) straff. 2
212 Lösungswege zu Kapitel 23

Da L beliebig groß gewählt werden kann, gibt es auch in die- Aufgabe 23.28 •• Für jedes t ∈ R gilt
sem Fall zu vorgegebenem ε > 0 kein kompaktes Intervall K
mit P(Xn ∈ K) ≥ 1 − ε für jedes n ≥ 1. Konsequenterweise g(t) = g(μ) + g  (δ)(t − μ)
muss also auch die Folge (σn2 ) beschränkt sein. mit δ = δ(t, μ) und |δ − μ| ≤ |t − μ|. Somit ist (punktweise
auf dem zugrunde liegenden Wahrscheinlichkeitsraum)
Aufgabe 23.27 ••• a) Wegen N ⊂ Q ist N abzählbar.
Damit gilt P(N) = 0. g(Zn ) = g(μ) + g  (n )(Zn − μ) ,

b) Es gilt für jedes k ≥ 1 wobei


⎛ ⎞ |n − μ| ≤ |Zn − μ|. (23.21)
k−1 
2 
2j − 1 2j ⎠ Es folgt
P(Xk = 1) = P ⎝ ,
2k 2k
j =1 an (g(Zn ) − g(μ)) = g  (n ) · an (Zn − μ).
2k−1
1 2k−1 1 P P
= = = , Wegen Zn −→ μ und (23.21) gilt auch n −→ μ, und
2k 2k 2 mit der Rechenregel a) für stochastische Konvergenz auf
j =1
P
1 Seite 871 folgt g  (n ) −→ g  (μ). Mit Xn := an (Zn − μ)
P(Xk = 0) = 1 − P(Xk = 1) = . D
2 gilt nach Voraussetzung Xn −→ X ∼ N(0, 1). Wenden wir
Damit ist Xj Bin(1, 1/2)-verteilt für jedes j ≥ 1. Teil b) des Lemmas von Sluzki mit Xn und Yn := g  (n ) an,
so ergibt sich
Sind k ∈ N beliebig und aj ∈ {0, 1} für j = 1, . . . , k, so
D
folgt an (g(Zn ) − g(μ)) = Yn Xn −→ g  (μ) X.
⎛⎛ ⎞⎞  

k
aj aj
k
1 Wegen g  (μ) X ∼ N(0, g  (μ)2 folgt die Behauptung.
P(X1 = a1 , . . . , Xk = ak )=P ⎝⎝ , + k ⎠⎠
2j 2j 2 Aufgabe 23.29 •• Es ist nur die Richtung „⇒“ zu zei-
j =1 j =1
 k 
k gen. Sei k ∈ N mit k ≥ 2 beliebig. Wir setzen xj k :=
1
= = P(Xj = aj ) F −1 j
k für 1 ≤ j < k sowie x0k := −∞, xkk := ∞.
2
j =1 j
Da F stetig ist, gilt F (xj k ) = k für alle j ∈ {0, 1, . . . , k}
und somit die stochastische Unabhängigkeit von X1 , X2 , . . . sowie
1
c) Setzt man Yj := 2Xj − 1 für j ≥ 1, so sind Y1 , Y2 , . . . F (xj +1,k ) − F (xj k ) = , 0 ≤ j < k.
k
{−1, 1}-wertige Zufallsvariablen auf  mit P(Yj = −1) =
P(Yj = 1) = 1/2, und Yj besitzt die charakteristische Funk- Daraus folgt für jedes j ∈ {0, 1, . . . , k − 1} und jedes x mit
tion xj k < x < xj +1,k

1  −it  1
ϕj (t) = e + eit = cos t, j ≥ 1. Fn (xj k ) − F (xj k ) − = Fn (xj k ) − F (xj +1,k )
2 k
≤ Fn (x) − F (x)
Es gilt
≤ Fn (xj +1,k ) − F (xj k )

n 1
Zn := Yj 2−j = Fn (xj +1,k ) − F (xj +1,k ) + .
k
j =1
Also ergibt sich
n
n
= 2 Xj 2−j − 2−j 1
j =1 j =1 sup |Fn (x) − F (x)| ≤ max |Fn (xj k ) − F (xj k )| + .
x∈R 0≤j <k k
→ 2id − 1
D
=: Z P-fast sicher. Aufgrund der Verteilungskonvergenz Xn −→ X gilt für je-
des j ∈ {0, 1, . . . , k} die Konvergenz |Fn (xj k ) − F (xj k )|
Wegen id ∼ U(0, 1) gilt Z ∼ U(−1, 1), und damit besitzt → 0 für n → ∞ und folglich
Z die charakteristische Funktion ϕ(t) = t −1 sin t. Wegen
f.s. D 1
Zn −→ Z gilt auch Zn −→ Z, und nach dem Stetigkeitssatz lim sup sup |Fn (x) − F (x)| ≤ für jedes k ≥ 2.
n→∞ x∈R k
von Lévy-Cramér folgt
  Mit k → ∞ gilt dann

n
t sin t
ϕn (t) = cos j → ϕ(t) = für n → ∞. lim sup |Fn (x) − F (x)| = 0.
2 t n→∞ x∈R
j =1
Lösungswege zu Kapitel 23 213

Aufgabe 23.30 •• Es sei p ∈ (0, 1) beliebig. Wir zei- Aufgabe 23.32 •• a) Es gilt
gen in einem ersten Schritt, dass
1
n
 2
Sn2 = (Xj − μ) − (X n − μ)
lim inf Fn−1 (p) ≥ F −1 (p) (23.22) n−1
n→∞ j =1
⎛ ⎞
1 ⎝
gilt. Sei hierzu ε > 0 beliebig. Da die Stetigkeitsstellen von n

F in R dicht liegen, finden wir ein x ∈ C (F ) mit = (Xj − μ)2 − n(X n − μ)2 ⎠
n−1
j =1
F −1 (p) − ε < x < F −1 (p). n 1 n
n  2
= · (Xj − μ)2 − Xn − μ .
Hieraus folgt F (x) < p, und wegen der Konvergenz n−1 n n−1
j =1
Fn (x) → F (x) gilt dann auch für genügend großes n die
Ungleichung Fn (x) < p und somit x < Fn−1 (p). Somit Nach dem starken Gesetz großer Zahlen gelten

1
ergibt sich n
f.s.
(Xj − μ)2 −→ E(X1 − μ)2 = σ 2
lim inf Fn−1 (p) ≥ x > F −1 (p) − ε. n
j =1
n→∞
f.s. f.s.
Da ε > 0 beliebig war, folgt (23.22). und X n − μ −→ 0. Hieraus folgt Sn2 −→ σ 2 .

Wohingegen (23.22) ohne weitere Voraussetzungen an p gilt, b) Mit der in a) erhaltenen Darstellung von Sn2 ergibt sich
benötigen wir für die Ungleichung ⎛ ⎞
√ n 1 n

lim sup Fn−1 (p) ≤ F −1 (p), (23.23) n(Sn2 − σ 2 )= · √ ⎝ (Xj − μ)2 − nσ 2 ⎠


n−1 n
n→∞ j =1
√ √
dass p ein Stetigkeitspunkt von F −1 ist. Wir wählen ein be-
2
σ n n ( n(X n − μ))2
=+ − · √ .
liebiges q mit p < q < 1 und zu gegebenem ε > 0 ein n−1 n−1 n
x ∈ C (F ) mit Nach dem Zentralen Grenzwertsatz von Lindeberg-Lévy gilt
−1 −1
⎛ ⎞
F (q) < x < F (q) + ε. 1 n
D
Un := √ ⎝ (Xj − μ)2 − nσ 2 ⎠ −→ N(0, τ 2 ),
Da F monoton ist, folgt n
j =1

p < q ≤ F (F −1 (q)) ≤ F (x) und das Lemma von Sluzki liefert


n D
und somit – da Fn (x) → F (x) – auch Fn (x) > p für genü- · Un −→ N(0, τ 2 ).
n−1
gend großes n. Für solche n gilt dann
Nach dem Zentralen Grenzwertsatz von Lindeberg-Lévy

Fn−1 (p) ≤ x < F −1
(q) + ε konvergiert auch n(Xn − μ) in Verteilung. Deshalb gilt
nach Teil a) des Satzes über Straffheit und Verteilungskon-

und somit lim supn→∞ Fn−1 (p) ≤ F −1 (q) für jedes q mit vergenz auf Seite 886 n(X n − μ) = OP (1) und somit nach

p < q < 1. Ist p ∈ C (F −1 ), so lässt man q gegen p streben Teil a) von Aufgabe 23.25 auch ( n(X n − μ))2 = OP (1).
und erhält (23.23). Insgesamt folgt die Behauptung. Mit Teil c) von Aufgabe 23.25 ergibt sich dann

Aufgabe 23.31 •• In der Situation des Satzes von n ( n(X n − μ))2
· √ = oP (1)
Lindeberg-Lévy sind X1 , X2 , . . . unabhängige identisch ver- n−1 n
teilte Zufallsvariablen mit 0 < σ 2 := V(X1 ) < ∞. Setzen √
Wegen σ 2 n/(n − 1) → 0 folgt jetzt mit dem Lemma von
wir a := EX1 , Sn := X1 + . . . + Xn , σn2 := V(Sn ) = nσ 2 , Sluzki die Behauptung.
so gilt wegen der identischen Verteilung von X1 , . . . , Xn
Aufgabe 23.33 • Der Beweis erfolgt durch Induktion
1  √ 
n
Ln (ε) = E (Xk − a)2 1{|Xk − a| > εσ n} über n. Für n = 1 ist nichts zu zeigen. Schreiben wir kurz
nσ 2
k=1 
n 
n
1  √  an := zj , bn := wj ,
= 2 E (X1 − a)2 1{|X1 − a| > εσ n} .
σ j =1 j =1
√ so gilt
Die Funktionenfolge fn := (X1 − a)2 1{|X1 − a| > εσ n}
konvergiert punktweise auf  gegen 0, und sie erfüllt |fn | ≤ |an+1 − bn+1 | = |an zn+1 − bn wn+1 |
(X1 − a)2 . Aufgrund des Satzes von der dominierten Kon-
= |(an − bn )zn+1 + bn (zn+1 − wn+1 )|
vergenz folgt somit limn→∞ Ln (ε) = 0, ε > 0, was zeigt,
dass in der Situation des Satzes von Lindeberg-Lévy die ≤ |zn+1 ||an − bn | + |bn ||zn+1 − wn+1 |
Lindeberg-Bedingung erfüllt ist. ≤ |an − bn | + |zn+1 − wn+1 |.
214 Lösungswege zu Kapitel 23

Nehmen wir als Induktionsvoraussetzung |an − bn | ≤


 und es folgt
n
j =1 |zj −wj | an, so folgt der Induktionsschluss aus obiger
1
n
Ungleichungskette. 1
E(Xj − aj )4 ≤ 2 .
σn4 σn
j =1
Aufgabe 23.34 •• Mit Xnj := aj Wj , 1 ≤ j ≤ n, liegt
die Situation des Satzes von Lindeberg-Feller vor. Es gilt
Da nach Voraussetzung limn→∞ σn2 = ∞ gilt, ist die

n
n Ljapunov-Bedingung mit δ = 2 erfüllt, was die Behauptung
σn2 = V(Tn ) = V(aj Wj ) = aj2 V(Wj ) zeigt.
j =1 j =1

n
= σ2 aj2 . Rechenaufgaben
j =1
Aufgabe 23.36 •• Bezeichnet Sn die Anzahl erschei-
Weiter ist nender Passagiere bei n verkauften Tickets, so liefern die
gemachten Annahmen den Ansatz Sn ∼ Bin(n, p) mit
1  
n
Ln (ε) = E (aj Wj )2 1{|aj Wj | > σn ε} p = 0.96. Gesucht ist das größte
√ n, sodass P(Sn ≥ 527) ≤
σn2
j =1 0.05. Mit u := (526 − np)/ np(1 − p) gilt nach dem Zen-
n   tralen Grenzwertsatz von de Moivre-Laplace für großes n
1
= aj2 E Wj2 1{|Wj | > εσn /|aj |}
σn2 P(Sn ≥ 527) = 1 − P(Sn ≤ 526)
j =1  
n   Sn − np
1 = 1−P √ ≤u
≤ 2 aj2 E Wj2 1{|Wj | > εun } , np(1 − p)
σn
j =1 ≈ 1 − (u).
wobei Die Lösung n ergibt sich also approximativ aus der Gleichung
σn
un := . u = −1 (0.95) = 1.645. Quadriert man die u definierende
max1≤j ≤n |aj |
Gleichung und löst die nach Multiplikation mit np(1 − p)
Wegen der identischen Verteilung der Wj folgt
entstehende quadratische Gleichung nach n auf, so ergeben
⎛ ⎞ sich die Lösungen n1 ≈ 555.8 und n2 ≈ 540.1. Wegen u > 0
1 ⎝ 2 ⎠  2 
n
Ln (ε) ≤ 2 aj E W1 1{|W1 | > un ε} ergibt sich die (sogar exakte) Antwort „540 Tickets dürfen
σn verkauft werden“.
j =1
1  
= 2 E W12 1{|W1 | > un ε} . Aufgabe 23.37 •• Bei der Addition von n Zahlen ist
σ
Sn := R1 + . . . + Rn die Summe der Rundungsfehler. Da die
Nach Voraussetzung gilt un → ∞ für n → ∞, und somit Rj stochastisch unabhängig und identisch verteilt sind, liegt
liefert der Satz von der dominierten Konvergenz die Situation des Zentralen Grenzwertsatzes von Lindeberg-
  Lévy vor. Wegen ER1 = 0 und V(R1 ) = 12 1
gilt nach diesem
lim E W12 1{|W1 | > un ε} = 0. Satz
n→∞ √
12 Sn D
Also ist die Lindeberg-Bedingung erfüllt, und folglich gilt √ −→ N(0, 1) für n → ∞.
n
der Zentrale Grenzwertsatz.
Für n = 1200 folgt
Aufgabe 23.35 •• Wir überlegen uns zunächst die im  √ 
 12 S 
Hinweis formulierte Ungleichung. Da Xj Indikatorvariable  1200 
P  √  ≤ 2 = P (|S1200 | ≤ 20)
ist, gilt Xjk = Xj für k ∈ N. Hiermit folgt 1200
≈ (2) − (−2) = 2 (2) − 1
E(Xj − aj )4 = E(Xj − 4Xj aj + 6Xj aj2 − 4Xj aj3 + aj4 )
≈ 0.9554.
= aj − 4aj2 + 6aj3 − 3aj4
= aj (1 − aj )(1 − 3aj (1 − aj ))
Aufgabe 23.38 •• a) Mit Sn := X1 + . . . + Xn gilt
≤ aj (1 − aj ),
wegen des Additionsgesetzes für die Normalverteilung
da 0 ≤ aj (1 − aj ) ≤ 1/4.
Sn
Nun gilt Sn∗ = √ ∼ N(0, 1)
V(Sn )

n
D
σn2 := V(Sn ) = aj (1 − aj ), für jedes n und somit insbesondere Sn∗ −→ N(0, 1) für n →
j =1 ∞. Es gilt also der Zentrale Grenzwertsatz.
Lösungswege zu Kapitel 23 215

b) Mit σn2 := V(Sn ) = 1 + 2! + . . . + n! ergibt sich Mit dem Zentralen Grenzwertsatz von Lindeberg-Lévy folgt
⎛ ⎞
 √ 
n! ≤ σn2 ≤ n! + n(n − 1)! = 2n! Tn − pn n a n(1 − p)
P ⎝√ * ≤ a ⎠ = P Tn − ≤
n 1−p p p
und somit p2
1 n!  √ 
≤ 2 ≤ 1. n+a n(1 − p)
2 σn = P Tn ≤
p

Wegen Xk ∼ k! N mit N ∼ N(0, 1) folgt → (a) für n → ∞
und damit die Behauptung.
1  2 
n
Ln (ε) = 2
E Xk 1{|Xk | > εσn } b) Wir verwenden Teil a) mit p = 1/6 und n = 100 und
σn
k=1 setzen √
1  2  n + a n(1 − p)
≥ 2 E Xn 1{|Xn | > εσn } 650 = .
σn p
1  √  Hieraus folgt
= 2 n!E N 2 1{|N| > εσn / n!} 650
σn 6 − 100
a = * ≈ 0.913
1  √  100 · 56
≥ E N 2 1{|N| > 2ε} .
2
und somit P(Tn > 650) ≈ 1 − (0.913) ≈ 1 − 0.819 =
Somit ist die Lindeberg-Bedingung Ln (ε) → 0 ∀ ε > 0 nicht 0.181.
erfüllt.
Aufgabe 23.40 •• a) Nach dem Additionsgesetz für
Aufgabe 23.39 •• a) Die Zufallsvariable Tn − n zählt n negative Binomialverteilung auf Seite 784 gilt Sn ∼
die
die Zahl der Nieten vor dem n-ten Treffer und besitzt folglich j =1 Xj , wobei X1 , . . . , Xn unabhängig sind und die glei-
die negative Binomialverteilung NB(n, p), vgl. Seite 783. che geometrische Verteilung G(1/2) mit EXj = 1 und
Nach dem Additionsgesetz für die negative Binomialvertei- V(Xj ) = 2 besitzen. Nach dem Zentralen Grenzwertsatz
lung auf Seite 784 gilt Tn − n ∼ Y1 + . . . + Yn , wobei von Lindeberg-Lévy folgt somit
Y1 , . . . , Yn stochastisch unabhängig sind und die gleiche geo- Sn − n D
√ −→ N(0, 1)
metrische Verteilung G(p) besitzen. Somit ergibt sich 2n
und deshalb für a, b ∈ R mit a < b
Tn ∼ X1 + . . . + Xn ,  
 √ √  a Sn − n b
wobei Xj = Yj + 1, j = 1, . . . , n. Die Zufallsvariablen P n + a n ≤ Sn ≤ b + n = P √ ≤ √ ≤ √
2 2n 2
X1 , . . . , Xn sind unabhängig und identisch verteilt mit    
b a
→ √ −  √ .
1 2 2
EXj = EYj + 1 = ,
p b) Es ist
 
1−p Sn − n 1
V(Xj ) = V(Yj ) = . P(Sn ≥ n) = P √ ≥0 → 1 − (0) = .
p2 2n 2
Kapitel 24 a) Ist C : X → P () ein Konfidenzbereich für ϑ zur Konfi-
denzwahrscheinlichkeit 1−α, so ist für beliebiges ϑ0 ∈ 
die Menge Kϑ0 := {x ∈ X | C(x) 6 ϑ0 } ein kri-
Aufgaben tischer Bereich für einen Niveau-α-Test der Hypothese
H0 : ϑ = ϑ0 gegen die Alternative H1 : ϑ = ϑ0 .
b) Liegt für jedes ϑ0 ∈  ein nichtrandomisierter Niveau-
Verständnisfragen α-Test für H0 : ϑ = ϑ0 gegen H1 : ϑ = ϑ0 vor, so lässt
Aufgabe 24.1 •• Konstruieren Sie in der Situation von sich hieraus ein Konfidenzbereich zur Konfidenzwahr-
Aufgabe eine obere Konfidenzschranke für ϑ zur Konfiden- scheinlichkeit 1 − α gewinnen.
zwahrscheinlichkeit 1 − α.
Aufgabe 24.7 •• Es seien U und V unabhängige Zu-
Aufgabe 24.2 •• Die Zufallsvariablen X1 , . . . , Xn fallsvariablen, wobei U ∼ N(0, 1) und V ∼ χk2 , k ∈ N. Ist
seien stochastisch unabhängig mit gleicher Poisson- δ ∈ R, so heißt die Verteilung des Quotienten
Verteilung Po(λ), wobei λ ∈ (0, ∞) unbekannt sei.
Konstruieren Sie in Analogie zum Beispiel auf Seite 925 U +δ
Yk,δ := √
einen asymptotischen Konfidenzbereich zum Niveau 1 − α V /k
für λ. Welches konkrete 95%-Konfidenzintervall ergibt
nichtzentrale t-Verteilung mit k Freiheitsgraden und Nicht-
sich für die Daten des Rutherford-Geiger-Experiments auf
zentralitätsparameter δ. Zeigen Sie: Für die Gütefunktion
Seite 787?
(24.52) des einseitigen t-Tests gilt
Aufgabe 24.3 • In einem Buch konnte man lesen: „Die  
Wahrscheinlichkeit α für einen Fehler erster Art bei einem gn (ϑ) = P Yn−1,δ > tn−1;1−α ,
statistischen Test gibt an, wie oft aus der Beantwortung der √
wobei δ = n(μ − μ0 )/σ.
Testfrage falsch auf die Nullhypothese geschlossen wird.
Wird α = 0.05 gewählt und die Testfrage mit ja beantwortet,
Aufgabe 24.8 • a) Zeigen Sie die Beziehung Fr,s;p =
dann ist die Antwort ja in 5% der Fälle falsch und mithin in
1/Fs,r;1−p für die Quantile der F-Verteilung.
95% der Fälle richtig.“ Wie ist Ihre Meinung hierzu?
b) Weisen Sie nach, dass die Gütefunktion des einseitigen F -
Aufgabe 24.4 • Der Leiter der Abteilung für Material-
Tests für den Varianzquotienten eine streng monoton wach-
beschaffung hat eine Sendung von elektronischen Schaltern
senden Funktion von σ 2 /τ 2 ist.
mit einem Test zum Niveau 0.05 stichprobenartig auf Funk-
tionsfähigkeit überprüft. Bei der Stichprobe lag der Anteil
Aufgabe 24.9 •• Die Zufallsvariable X besitze eine Bi-
defekter Schalter signifikant über dem vom Hersteller be-
nomialverteilung Bin(3, ϑ), wobei ϑ ∈  := {1/4, 3/4}.
haupteten Ausschussanteil. Mit den Worten „Die Chance,
Bestimmen Sie die Risikomenge des Zwei-Alternativ-
dass eine genaue Überprüfung zeigt, dass die Sendung den
Problems H0 : ϑ = ϑ0 := 1/4 gegen H1 : ϑ = ϑ1 := 3/4.
Herstellerangaben entspricht, ist höchstens 5%“ empfiehlt er,
die Lieferung zu reklamieren und zurückgehen zu lassen. Ist
Aufgabe 24.10 •• Leiten Sie die Beziehung
seine Aussage richtig?
 
Aufgabe 24.5 • Der Statistiker einer Firma, die Werk- (n − 1) Q(X)−2/n − 1 = Tn2
stücke zur Weiterverarbeitung bezieht, lehnt eine Lieferung im Beispiel von Seite 948 her.
dieser Werkstücke mit folgender Begründung ab: „Ich habe
meinen Standard-Test zum Niveau 0.05 anhand einer zu-
Aufgabe 24.11 •• Es seien X1 , . . . , Xn unabhängige
fälligen Stichprobe durchgeführt. Diese Stichprobe enthielt
Zufallsvariablen mit gleicher stetiger Verteilungsfunktion F
einen extrem hohen Anteil defekter Exemplare. Wenn der
und empirischer Verteilungsfunktion Fn . Bestimmen Sie die
Ausschussanteil in der Sendung wie vom Hersteller behaup-
Verteilung von
tet höchstens 2% beträgt, ist die Wahrscheinlichkeit für das  
Auftreten des festgestellten oder eines noch größeren Anteils F = sup Fn (x) − F (x)
n
defekter Werkstücke in der Stichprobe höchstens 2.7%.“Der x∈R
Werkmeister entgegnet: „Bislang erwiesen sich 70% der von im Fall n = 1.
Ihnen beanstandeten Sendungen im Nachhinein als in Ord-
nung. Aller Wahrscheinlichkeit nach liegt auch in diesem Aufgabe 24.12 •• Die Zufallsvariablen X1 , . . . , X2n
Fall ein blinder Alarm vor.“ Muss mindestens eine der bei- seien stochastisch unabhängig mit gleicher symmetrischer
den Aussagen falsch sein? Verteilung. Es gebe also ein a ∈ R mit X1 − a ∼ a − X1 .
Zeigen Sie: Ist m := n/2, so gilt (im Fall E|X1 | < ∞)
Aufgabe 24.6 •• (Zusammenhang zwischen Konfidenz-
 
bereichen und Tests) Es sei (X , B, (Pϑ )ϑ∈ ) ein statistisches Xm:2n + Xm+1:2n
Modell. Zeigen Sie: E = a.
2
M. Brokate et al., Arbeitsbuch Grundwissen Mathematikstudium – Höhere Analysis, Numerik und
Stochastik, DOI 10.1007/978-3-642-54946-5_23, © Springer-Verlag Berlin Heidelberg 2016
Aufgaben zu Kapitel 24 217

Aufgabe 24.13 •• Es seien X1 , . . . , Xn unabhängige Aufgabe 24.19 • Zeigen Sie, dass die Gütefunktionen
Zufallsvariablen mit gleicher stetiger Verteilungsfunktion. des ein- bzw. zweiseitigen Gauß-Tests durch (24.47) bzw.
Zeigen Sie: In Verallgemeinerung von (24.88) gilt: durch (24.48) gegeben sind.
s−1  

  n j
P X(r) ≤ Qp < X(s) = p (1 − p)n−j Aufgabe 24.20 •• Weisen Sie für die Verteilungsfunk-
j tion  und die Dichte ϕ der Normalverteilung N(0, 1) die
j =r
Ungleichung
Aufgabe 24.14 • In welcher Form tritt die Verteilung
ϕ(x)
einer geeigneten Wilcoxon-Rangsummenstatistik bei der 1 − (x) ≤ , x > 0,
x
Ziehung der Lottozahlen auf?
nach. Zeigen Sie hiermit: Für die in (24.47) gegebene Gü-
Beweisaufgaben tefunktion gn (μ) des einseitigen Gauß-Tests gilt für jedes
μ > μ0 und jedes hinreichend große n
Aufgabe 24.15 •• Die Zufallsvariable X besitze eine
 
hypergeometrische Verteilung Hyp(n, r, s), wobei n, r ∈ N 1 n(μ − μ0 )2
bekannt sind und s ∈ N0 unbekannt ist. Der zu schätzende 1 − gn (μ) ≤ √ exp − .
2π e 2σ 2
unbekannte Parameter sei ϑ := r +s ∈  := {r, r +1, r +2,
. . .}. Zeigen Sie: Es existiert kein erwartungstreuer Schätzer
Die Wahrscheinlichkeit für einen Fehler zweiter Art konver-
T : X →  für ϑ. Dabei ist X := {0, 1, . . . , n} der Stich-
giert also exponentiell schnell gegen null.
probenraum für X.

Aufgabe 24.16 •• Zeigen Sie: Aufgabe 24.21 •• Die Zufallsvariable Q habe eine Fis-
her’sche Fr,s -Verteilung. Zeigen Sie:
a) Für ϑ ∈ [0, 1] und k ∈ {1, 2, . . . , n} gilt
n   ϑ a) Q besitzt die in (24.55) angegebene Dichte.
n j n!
ϑ (1−ϑ)n−j = t k−1 (1−t)n−k dt. s
j =k
j (k−1)!(n−k)! 0 b) E(Q) = , s > 2.
s−2
b) Die in (24.24), (24.25) eingeführten Funktionen 2s (r + s − 2)
2
c) V(Q) = , s > 4.
a(·), A(·) :  → X sind (schwach) monoton wach- r(s − 2)2 (s − 4)
send, a ist rechtsseitig und A linksseitig stetig, und es
gilt a ≤ A. Aufgabe 24.22 •• Die Zufallsvariablen X1 , X2 , . . . ,
c) Es gilt die Aussage (24.29). Xn , . . . seien stochastisch unabhängig und je Poisson-verteilt
Po(λ), wobei λ ∈ (0, ∞) unbekannt ist. Konstruieren Sie
Aufgabe 24.17 •• Zeigen Sie, dass für die in (24.27) analog zum „Binomialfall“ auf Seite 938 eine Testfolge
und (24.28) eingeführten Funktionen l(·) bzw. L(·) gilt: (ϕn ) zum asymptotischen Niveau α für das Testproblem
 α 1/n  α 1/n H0 : λ ≤ λ0 gegen H1 : λ > λ0 und weisen Sie deren Kon-
a) l(0) = 0, L(0) = 1− , l(n) = , L(n) = 1. sistenz nach. Dabei ist λ0 ∈ (0, ∞) ein vorgegebener Wert.
2 2
b) Für x = 1, 2, . . . , n − 1 ist
1) l(x) die Lösung ϑ der Gleichung Aufgabe 24.23 ••• √ Zeigen Sie, dass die Konstante Kλ in
n   (24.62) durch Kλ = 1/ 2π λ gegeben ist.
n j α
ϑ (1 − ϑ)n−j = ,
j 2
j =x
Aufgabe 24.24 •• Der Zufallsvektor X besitze
2) L(x) die Lösung ϑ der Gleichung eine nichtausgeartete k-dimensionale Normalverteilung
x   Nk (μ, ). Zeigen Sie, dass die quadratische Form
n j α
ϑ (1 − ϑ)n−j = . (X − μ) −1 (X − μ) eine χk2 -Verteilung besitzt.
j 2
j =0
Aufgabe 24.25 •• Beweisen Sie die Konsistenz des
Aufgabe 24.18 •• Es seien X1 , X2 , . . . unabhängige Chi-Quadrat-Tests.
und je Bin(1, ϑ)-verteilte Zufallsvariablen, wobei ϑ ∈  :=
(0, 1). Weiter sei hα :=  −1 (1 − α/2), wobei α ∈ (0, 1).
−1 n Aufgabe 24.26 •• Zeigen Sie, dass für die Risikomenge
Zeigen Sie: Mit Tn := n j =1 Xj und Wn := Tn (1 − Tn ) R aller Fehlerwahrscheinlichkeitspunkte (α(ϕ), β(ϕ)) von
gilt Tests ϕ : X → [0, 1] im Zwei-Alternativ-Problem gilt:
 
hα  hα 
lim Pϑ Tn − √ Wn ≤ ϑ ≤ Tn + √ Wn = 1 − α, a) R enthält die Punkte (1, 0) und (0, 1),
n→∞ n n b) R ist punktsymmetrisch zu (1/2, 1/2),
ϑ ∈ . c) R ist konvex.
218 Aufgaben zu Kapitel 24

Aufgabe 24.27 •• Es seien X1 , X2 , . . . , unabhän- Sie, dass ein Maximum-Likelihood-Schätzwert für ϑ zu x


gige Zufallsvariablen mit stetigen Verteilungsfunktionen durch
F1 , F2 , . . . Zeigen Sie: ⎧J K
⎨ k(N+1) , falls k(N+1) ∈
⎛ ⎞ F 0n n 1 / N,
ϑ (x) =
4 ⎩∈ k(N+1) , k(N+1) − 1 sonst,
P⎝ {Xi = Xj }⎠ = 0. n n
1≤i<j <∞ gegeben ist.

Aufgabe 24.28 •• Es seien X1 , X2 , . . . unabhängige Aufgabe 24.33 •• Es sei die Situation des Beispiels von
Zufallsvariablen mit gleicher stetiger Verteilungsfunktion Seite 911 (Taxi-Problem) zugrunde gelegt. Zeigen Sie:
F . Die Ordnungsstatistiken von X1 , . . . , Xn seien mit a) Die Folge (F
ϑn ) der ML-Schätzer ist asymptotisch erwar-
X1:n , . . . , Xn:n bezeichnet. Zeigen Sie: Ist für α ∈ (0, 1) tungstreu und konsistent für ϑ.
hα := −1 (1 − α/2) gesetzt, und sind zu p ∈ (0, 1) b) Der durch
rn , sn ∈ N durch
ϑn (x)n+1 − (F
F ϑn (x) − 1)n+1
 Tn (x) =
rn := 2np − hα np(1 − p)3, F
ϑn (x)n − (F
ϑn (x) − 1)n

sn := 2np + hα np(1 − p)3 definierte Schätzer Tn ist erwartungstreu für ϑ.

definiert, so gilt Aufgabe 24.34 •• Es seien X1 , . . . , Xn stochastisch


  unabhängige Zufallsvariablen mit gleicher Poisson-
lim P Xrn :n ≤ Qp ≤ Xsn :n = 1 − α. Verteilung Po(ϑ), ϑ ∈  := (0, ∞) sei unbekannt. Zeigen
n→∞
Sie:

a) Das arithmetische Mittel X n = n−1 nj=1 Xj ist der
Aufgabe 24.29 •• Die Zufallsvariable X −a besitze für ML-Schätzer für ϑ.
ein unbekanntes a ∈ R eine t-Verteilung mit s Freiheitsgra- b) Die Fisher-Information If (ϑ) ist
den, wobei s ≥ 3. Die Verteilungsfunktion von X sei mit Fs
bezeichnet. Zeigen Sie: n
If (ϑ) = , ϑ ∈ .
ϑ
a) Die auf Seite 41 eingeführte asymptotische relative Effi-
zienz von Qn,1/2 bezüglich X n als Schätzer für a ist c) Der Schätzer X n ist Cramér-Rao-effizient.
  Aufgabe 24.35 •• Ein Bernoulli-Experiment mit unbe-
s+1
4" 2
AREFs (Qn,1/2 , X n ) =
2
 . kannter Trefferwahrscheinlichkeit ϑ ∈ (0, 1) wird in unab-
(s − 2)π " 2 2s hängiger Folge durchgeführt. Beim (k + 1)-ten Mal (k ∈ N0 )
sei der erste Treffer aufgetreten.
b) Der Ausdruck in a) ist für s = 3 und s = 4 größer und für
s ≥ 5 kleiner als 1, und im Limes für s → ∞ ergibt sich der a) Bestimmen Sie den ML-Schätzwert F ϑ (k) für ϑ.
Wert 2/π. b) Ist der Schätzer F
ϑ erwartungstreu für ϑ?

Aufgabe 24.36 •• In der Situation des Beispiels auf


Aufgabe 24.30 •• Beweisen Sie die Aussagen a) und
Seite 911 (Taxi-Problem) sei
b) des Satzes über die H0 -Verteilung der Wilcoxon-Rang-
2
summenstatistik auf Seite 959. n
@
ϑn := Xj − 1.
n
j =1
Rechenaufgaben
Zeigen Sie, dass der Schätzer @
ϑn erwartungstreu für ϑ ist und
Aufgabe 24.31 • Es seien n ∈ N und k ∈ {0, . . . , n}. die Varianz
Zeigen Sie, dass die durch ϑ2 − 1
Vϑ (@
ϑn ) =
  3n
n k besitzt.
h(ϑ) = ϑ (1 − ϑ)n−k
k
Aufgabe 24.37 •• Es seien X1 , . . . , Xn unabhängige
definierte Funktion h : [0, 1] → [0, 1] für ϑ = k/n ihr Ma- Zufallsvariablen mit gleicher Exponentialverteilung Exp(ϑ),
ximum annimmt. ϑ ∈  := (0, ∞) sei unbekannt. Im Beispiel auf Seite 910
wurde der ML-Schätzer für ϑ zu
Aufgabe 24.32 •• In der Situation des Beispiels der n
Qualitätskontrolle auf Seite 905 mögen sich in einer rein zu- F
ϑn = n
j =1 Xj
fälligen Stichprobe x = (x1 , . . . , xn ) vom Umfang n genau
k = x1 + . . . + xn defekte Exemplare ergeben haben. Zeigen hergeleitet. Zeigen Sie:
Aufgaben zu Kapitel 24 219

a) Eϑ (F
ϑn ) = n
n−1 ϑ, n ≥ 2. Aufgabe 24.43 •• a) Leiten Sie die in (24.35) angege-
bene Dichte der tk -Verteilung her.
b) Vϑ (F
ϑn ) = n2 ϑ 2
(n−1)2 (n−2)
, n ≥ 3.
b) Zeigen Sie: Besitzt X eine tk -Verteilung, so existieren Er-
c) Die Schätzfolge (F
ϑn ) ist konsistent für ϑ. wartungswert und Varianz von X genau dann, wenn k ≥ 2
bzw. k ≥ 3 gelten. Im Fall der Existenz folgt
Aufgabe 24.38 •• Es seien X1 , . . . , Xn stochastisch
unabhängige identisch verteilte Zufallsvariablen mit k
E(X) = 0, V(X) = .
EX12 < ∞. Zeigen Sie: Mit σ 2 := V(X1 ) gilt k−2
⎛ ⎞
1
n Aufgabe 24.44 •• a) Zeigen Sie: In der Situation des
E⎝ (Xj − X n )2 ⎠ = σ 2 . Taxi-Problems auf Seite 911 ist die durch
n−1 0 1
j =1
C (x1 , . . . , xn ) := ϑ ∈  | ϑ ≤ α −1/n max xj
j =1,...,n

Aufgabe 24.39 •• Die Zufallsvariablen X1 , . . . , Xn definierte Abbildung C ein Konfidenzbereich für ϑ zum Ni-
seien stochastisch unabhängig und je N(μ, σ 2 )-verteilt, wo- veau 1 − α.
bei μ und σ 2 unbekannt seien. Als Schätzer für σ 2 betrachte
b) Wir groß muss n mindestens sein, damit die größte beob-
man
n achtete Nummer, versehen mit einem Sicherheitsaufschlag
Sn (c) := c (Xj − Xn )2 , c > 0. von 10% (d. h. 1.1 · maxj =1,...,n xj ) eine obere Konfidenz-
j =1 schranke für ϑ zum Niveau 0.99 darstellt, also
 
Für welche Wahl von c wird die mittlere quadratische Ab-
Pϑ ϑ ≤ 1.1 · max Xj ≥ 0.99 ∀ϑ ∈ 
weichung E(Sn (c) − σ 2 )2 minimal? j =1,...,n

gilt?
Aufgabe 24.40 •• Die Zufallsvariablen X1 , . . . , Xn
seien stochastisch unabhängig und je gleichverteilt U[0, ϑ], Aufgabe 24.45 •• Um die Übertragbarkeit der Krank-
wobei ϑ ∈  := (0, ∞) unbekannt sei. Zeigen Sie: heit BSE zu erforschen, wird 275 biologisch gleichartigen
a) Der ML-Schätzer für ϑ ist F
ϑn := maxj =1,...,n Xj . Mäusen über einen gewissen Zeitraum täglich eine bestimmte
b) Der Schätzer Menge Milch von BSE-kranken Kühen verabreicht. Inner-
n+1 halb dieses Zeitraums entwickelte keine dieser Mäuse irgend-
ϑn∗ := F
ϑn
n welche klinischen Symptome, die auf eine BSE-Erkrankung
hindeuten könnten. Es bezeichne ϑ die Wahrscheinlichkeit,
ist erwartungstreu für ϑ. Bestimmen Sie Vϑ (ϑn∗ ).
dass eine Maus der untersuchten Art unter den obigen Ver-
c) Der Momentenschätzer für ϑ ist
suchsbedingungen innerhalb des Untersuchungszeitraumes
1 BSE-spezifische Symptome zeigt.
n
@
ϑn := 2 · Xj .
n a) Wie lautet die obere Konfidenzschranke für ϑ zur Garan-
j =1
tiewahrscheinlichkeit 0.99?
d) Welcher der Schätzer ϑn∗ und @ ϑn ist vorzuziehen, wenn b) Wie viele Mäuse müssten anstelle der 275 untersucht wer-
als Gütekriterium die mittlere quadratische Abweichung den, damit die obere Konfidenzschranke für ϑ höchstens
zugrunde gelegt wird? 10−4 ist?
c) Nehmen Sie vorsichtigerweise an, die obere Konfidenz-
schranke aus Teil a) sei die „wahre Wahrscheinlichkeit“
Aufgabe 24.41 •• Die Zufallsvariablen X1 , . . . , Xn
ϑ. Wie viele Mäuse mit BSE-Symptomen würden Sie
seien unabhängig und je "(α, λ)-verteilt. Der Parameter
dann unter 10 000 000 Mäusen erwarten?
ϑ := (α, λ) ∈  := (0, ∞)2 sei unbekannt. Zeigen Sie:
Die Loglikelihood-Gleichungen führen auf
Aufgabe 24.46 •
F
αn 1
n
d a) In einer repräsentativen Umfrage haben sich 25% aller
Xn = , log Xj = αn ) − log F
log "(F λn .
F
λn n dα 1250 Befragten für die Partei A ausgesprochen. Wie ge-
j =1
nau ist dieser Schätzwert, wenn wir die Befragten als
rein zufällige Stichprobe aus einer Gesamtpopulation von
vielen Millionen Wahlberechtigten ansehen und eine Ver-
Aufgabe 24.42 •• Zeigen Sie, dass die folgenden Ver-
trauenswahrscheinlichkeit von 0.95 zugrunde legen?
teilungsklassen einparametrige Exponentialfamilien bilden:
b) Wie groß muss der Stichprobenumfang mindestens sein,
a) {Bin(n, ϑ), 0 < ϑ < 1}, damit der Prozentsatz der Wähler einer Volkspartei (zu
b) {Po(ϑ), 0 < ϑ < ∞}, erwartender Prozentsatz ca. 30%) bis auf ± 1% genau
c) {Exp(ϑ), 0 < ϑ < ∞}. geschätzt wird (Vertrauenswahrscheinlichkeit 0.95)?
220 Hinweise zu Kapitel 24

Aufgabe 24.47 •• Um zu testen, ob in einem Paket, das Hinweise


100 Glühbirnen enthält, höchstens 10 defekte Birnen ent-
halten sind, prüft ein Händler jedes Mal 10 der Birnen und
nimmt das Paket nur dann an, wenn alle 10 in Ordnung sind.
Verständnisfragen
Beschreiben Sie dieses Verhalten testtheoretisch und ermit- Aufgabe 24.1 •• Es ist Pϑ (max(X1 , . . . , Xn ) ≤ t) =
teln Sie das Niveau des Testverfahrens. (t/ϑ)n , 0 ≤ t ≤ ϑ.

Aufgabe 24.48 •• Es sei die Situation des Beispiels von Aufgabe 24.2 •• Verwenden Sie den Zentralen Grenz-
Seite 932 (Konsumenten- und Produzentenrisiko) zugrunde wertsatz von Lindeberg-Lévy auf Seite 887.
gelegt. Eine Verbraucherorganisation möchte dem Herstel-
ler nachweisen, dass die mittlere Füllmenge μ kleiner als Aufgabe 24.3 • –
μ0 := 1000 ml ist. Hierzu wird der Produktion eine Stich-
probe vom Umfang n entnommen. Die gemessenen Füllmen- Aufgabe 24.4 • –
gen werden als Realisierungen unabhängiger und je N(μ, 4)
normalverteilter Zufallsvariablen angenommen. Aufgabe 24.5 • –

a) Warum wird als Hypothese H0 : μ ≥ μ0 und als Alter- Aufgabe 24.6 •• –


native H1 : μ < μ0 festgelegt?
b) Zeigen Sie: Ein Gauß-Test zum Niveau 0.01 lehnt H0 Aufgabe 24.7 •• –
genau dann ab, wenn das Stichprobenmittel X n die Un-

gleichung Xn ≤ μ0 − 4.652/ n erfüllt.
Aufgabe 24.8 • –
c) Die Organisation möchte erreichen, dass der Test mit
Wahrscheinlichkeit 0.9 zur Ablehnung von H0 führt,
Aufgabe 24.9 •• Die Neyman-Pearson-Tests sind Kon-
wenn die mittlere Füllmenge μ tatsächlich 999 ml be-
vexkombinationen zweier nichtrandomisierter NP-Tests.
trägt. Zeigen Sie, dass hierzu der Mindeststichproben-
umfang n = 53 nötig ist.
Aufgabe 24.10 •• –
Aufgabe 24.49 • Die folgenden Werte sind Reaktions-
Aufgabe 24.11 •• O.B.d.A. gelte X1 ∼ U(0, 1).
zeiten (in Sekunden) von 8 Studenten in nüchternem Zustand
(x) und 30 Minuten nach dem Trinken einer Flasche Bier (y).
Aufgabe 24.12 •• Nutzen Sie aus, dass
Unter der Grundannahme, dass das Trinken von Bier die Re-
(X1 − a, . . . , X2n − a) und (a − X1 , . . . , a − X2n )
aktionszeit prinzipiell nur verlängern kann, prüfe man, ob die
dieselbe Verteilung besitzen, was sich auf die Vektoren der
beobachteten Daten mit der Hypothese verträglich sind, dass
jeweiligen Ordnungsstatistiken überträgt. Überlegen Sie
die Reaktionszeit durch das Trinken einer Flasche Bier nicht
sich vorab, warum die Voraussetzung E|X1 | < ∞ gemacht
beeinflusst wird.
wird.
i 1 2 3 4 5 6 7 8
xi 0.45 0.34 0.72 0.60 0.38 0.52 0.44 0.54 Aufgabe 24.13 •• –
yi 0.53 0.39 0.69 0.61 0.45 0.63 0.52 0.67

Aufgabe 24.14 • –

Aufgabe 24.50 • Ein möglicherweise gefälschter


Würfel wird 200-mal in unabhängiger Folge geworfen, Beweisaufgaben
wobei sich für die einzelnen Augenzahlen die Häufigkeiten
Aufgabe 24.15 •• T kann – ganz egal, wie groß ϑ ist –
32, 35, 41, 38, 28, 26 ergaben. Ist dieses Ergebnis mit der
nur endlich viele Werte annehmen.
Hypothese der Echtheit des Würfels verträglich, wenn eine
Wahrscheinlichkeit von 0.1 für den Fehler erster Art toleriert
Aufgabe 24.16 •• –
wird?
Aufgabe 24.17 •• –
Aufgabe 24.51 • Es seien X1 , . . . , Xn unabhängige
Zufallsvariablen mit gleicher stetiger Verteilungsfunktion. Aufgabe 24.18 •• Verwenden Sie den Zentralen Grenz-
Wie groß muss n sein, damit das Intervall [X(1) , X(n) ] ein wertsatz von de Moivre-Laplace und Teil b) des Lemmas von
95%-Konfidenzintervall für den Median wird? Sluzki auf Seite 881.

Aufgabe 24.52 • Welches Resultat ergibt die Anwen- Aufgabe 24.19 • –


dung des Vorzeichentests für verbundene Stichproben in der
Situation von Aufgabe 24.49? Aufgabe 24.20 •• –
Lösungen zu Kapitel 24 221

Aufgabe 24.21 •• Nutzen Sie die Erzeugungsweise der Aufgabe 24.35 •• Verwenden Sie die Jensen’sche Un-
Verteilung aus. gleichung auf Seite 834.

Aufgabe 24.22 •• Es gilt für jedes k ∈ N und jedes Aufgabe 24.36 •• –


u ≥ 0 (Beweis durch Differenziation nach u) n
Aufgabe 24.37 •• Es gilt j =1 Xj ∼ "(n, ϑ) un-

ter Pϑ .
uj 1 u
e−u = e−t t k−1 dt.
j! (k − 1)! 0 Aufgabe 24.38 •• Es kann o.B.d.A. EX1 = 0 ange-
j =k
n √ nommen werden.
Setzen Sie ϕn := 1{ j =1 xj ≥ nλ0 + −1 (1 − α) nλ0 .}
Aufgabe 24.39 •• Nutzen Sie aus, dass die Summe der
2 -verteilt ist.
Aufgabe
√ 24.23 ••• Für X ∼ Po(λ) √ gilt P(|X − λ| ≤
Abweichungsquadrate bis auf einen Faktor χn−1
C λ) ≥ 1 − C −2 . Mit zk = (k − λ)/ λ ist
  Aufgabe 24.40 •• Vϑ (ϑn∗ ) = ϑ 2 /(n(n + 2))
√ zk2
λ exp − Aufgabe 24.41 •• –
2
k:|zk |≤C
Aufgabe 24.42 •• –
eine Riemann’sche Näherungssumme für das Integral
C 2
−C exp(−z /2) dz. Aufgabe 24.43 •• Beachten Sie Gleichung c) auf
Seite 831. Für die Berechnung der Varianz von X hilft Dar-
Aufgabe 24.24 •• – stellung (24.33).

Aufgabe 24.25 •• Es reicht, die Summe Tn in (24.64) Aufgabe 24.44 •• –


durch einen Summanden nach unten abzuschätzen und das
Gesetz großer Zahlen zu verwenden. Aufgabe 24.45 •• Beachten Sie (24.30).

Aufgabe 24.46 • –
Aufgabe 24.26 •• –
Aufgabe 24.47 •• –
Aufgabe 24.27 •• Verwenden Sie die σ -Subadditivität
von P und den Satz von Tonelli auf Seite 262. Aufgabe 24.48 •• –

Aufgabe 24.28 •• Verwenden Sie das Resultat von Aufgabe 24.49 • Legen Sie die auf Seite 935 gemach-
Aufgabe und den Zentralen Grenzwertsatz von de Moivre- ten Annahmen zugrunde.
Laplace.
Aufgabe 24.50 • –
Aufgabe 24.29 •• a) X besitzt die Varianz s/(s − 2). b)
√ Aufgabe 24.51 • –
Es gilt "(x + 1/2) ≤ "(x) x, x > 0.
Aufgabe 24.52 • Nach den auf Seite 957 gemachten
Aufgabe 24.30 •• Nutzen Sie die Summen-Struktur Voraussetzungen haben die Differenzen Zj = Yj − Xj eine
von Wm,n sowie die Tatsache aus, dass der Vektor symmetrische Verteilung mit unbekanntem Median μ.
(r(X1 ), . . . , r(Yn )) unter H0 auf den Permutationen von
(1, . . . , m + n) gleichverteilt ist. Beachten Sie auch, dass
die Summe aller Ränge konstant ist.
Lösungen
Rechenaufgaben
Verständnisfragen
Aufgabe 24.31 • Betrachten Sie die Fälle k = 0,
k = n und 1 ≤ k ≤ n − 1 getrennt. Aufgabe 24.1 •• α −1/n max(X1 , . . . , Xn ).

Aufgabe 24.2 •• Es gilt


Aufgabe 24.32 •• Betrachten Sie für 1 ≤ k ≤ n − 1
den Quotienten Lx (ϑ +1)/Lx (ϑ), wobei Lx die Likelihood- lim Pλ (Un ≤ λ ≤ On ) = 1 − α ∀λ ∈ (0, ∞),
n→∞
Funktion zu x ist. 
wobei mit h := −1 (1 − α/2) und Tn := n−1 nj=1 Xj
Aufgabe 24.33 •• – ? ?
h2 h h2 h2 h h2
Un = Tn + − √ Tn + , On = Tn + + √ Tn + .
Aufgabe 24.34 •• – 2n n 4n 2n n 4n
222 Lösungswege zu Kapitel 24

Aufgabe 24.3 • – Rechenaufgaben


Aufgabe 24.31 • –
Aufgabe 24.4 • Nein.
Aufgabe 24.32 •• –
Aufgabe 24.5 • Nein.
Aufgabe 24.33 •• –
Aufgabe 24.6 •• –
Aufgabe 24.34 •• –
Aufgabe 24.7 •• –
Aufgabe 24.35 •• a) ϑ(k) = 1/(k + 1). b) Nein.
Aufgabe 24.8 • –
Aufgabe 24.36 •• –
Aufgabe 24.9 •• –
Aufgabe 24.37 •• –
Aufgabe 24.10 •• –
Aufgabe 24.38 •• –
Aufgabe 24.11 •• –
Aufgabe 24.39 •• c = 1/(n + 1).
Aufgabe 24.12 •• –
Aufgabe 24.40 •• d) Der Schätzer @
ϑn .
Aufgabe 24.13 •• –
Aufgabe 24.41 •• –
Aufgabe 24.14 • –
Aufgabe 24.42 •• –

Beweisaufgaben Aufgabe 24.43 •• –


Aufgabe 24.15 •• –
Aufgabe 24.44 •• In b) muss n ≥ 49 gelten.
Aufgabe 24.16 •• –
Aufgabe 24.45 •• –
Aufgabe 24.17 •• –
Aufgabe 24.46 • –
Aufgabe 24.18 •• –
Aufgabe 24.47 •• Das Testniveau ist 0.6695 . . .
Aufgabe 24.19 • –
Aufgabe 24.48 •• –
Aufgabe 24.20 •• –
Aufgabe 24.49 • Die Hypothese wird auf dem 5%-
Niveau abgelehnt.
Aufgabe 24.21 •• –
Aufgabe 24.50 • –
Aufgabe 24.22 •• –
Aufgabe 24.51 • n muss mindestens gleich 6 sein.
Aufgabe 24.23 ••• –
Aufgabe 24.52 • Die Hypothese H0 : μ ≤ 0 wird auf
Aufgabe 24.24 •• – dem 5%-Niveau abgelehnt.

Aufgabe 24.25 •• –

Aufgabe 24.26 •• – Lösungswege


Aufgabe 24.27 •• –
Verständnisfragen
Aufgabe 24.28 •• – Aufgabe 24.1 •• Wir setzen kurz Mn :=
max(X1 , . . . , Xn ). Es gilt für jedes t mit 0 ≤ t ≤ ϑ
Aufgabe 24.29 •• –  n
t
Pϑ (Mn ≤ t) = Pϑ (X1 ≤ t)n =
Aufgabe 24.30 •• – ϑ
Lösungswege zu Kapitel 24 223

und somit – wenn wir zum komplementären Ereignis über- von H0 in einer langen Serie unabhängiger Testläufe unter-
gehen und t := ϑα 1/n setzen stellen. Dann würde man aber nicht testen!
 
Pϑ Mn > ϑα 1/n = 1 − α. Aufgabe 24.4 • Nein. Auch in diesem Fall handelt es
sich um den auf Seite 932 diskutierten Trugschluss, es exi-
Da wir hier das Kleiner- durch das Kleiner-gleich-Zeichen er- stiere eine „bedingte Wahrscheinlichkeit P(H0 gilt | Test führt
setzen können, ohne die Wahrscheinlichkeit zu ändern, folgt zur Ablehnung von H0 )“, und diese „Wahrscheinlichkeit“ sei
  höchstens α (= 0.05).
Pϑ (0, Mn α −1/n ] 6 ϑ = 1 − α ∀ϑ ∈ ,
Aufgabe 24.5 • Nein. Der Statistiker hat aufgrund sei-
ner Stichprobe die Hypothese H0 , der Auschussanteil betrage
und somit ist Mn α −1/n eine obere Konfidenzschranke für ϑ höchstens 2%, zum 5%-Niveau abgelehnt, weil der beobach-
zur Konfidenzwahrscheinlichkeit 1 − α. tete p-Wert 0.027 betrug. Dass in 70% aller Fälle, in denen
n ein Widerspruch zu H0 (d. h. eine Beanstandung) auftrat, in
Aufgabe 24.2 •• Es sei Sn = j =1 Xj = nTn . Da Wirklichkeit H0 zutraf, steht hierzu nicht im Widerspruch.
X1 , . . . , Xn unabhängig und identisch verteilt sind mit Nach den auf Seite 932 angestellten Überlegungen hätten es
sogar 100% aller Fälle sein können, wenn alle Sendungen der
Eλ (X1 ) = Vλ (X1 ) = λ,
Behauptung des Herstellers entsprochen hätten, d. h. stets H0
liefert der Zentrale Grenzwertsatz von Lindeberg-Lévy auf gegolten hätte.
Seite 887 zunächst für beliebiges h > 0:
Aufgabe 24.6 •• a) Es gilt
 √     
 n(Tn −λ)   Sn −nλ  Pϑ0 (Kϑ0 ) = Pϑ0 (C(X)  6 ϑ0 )
   
lim Pλ 
n→∞
√  ≤ h = n→∞lim Pλ  √
nλ 
≤h
λ = 1 − Pϑ0 (C(X) 6 ϑ0 )
= (h) − (−h) ≤ 1 − (1 − α)
= 2(h) − 1. = α,
Setzt man speziell h = −1 (1−α/2), so gilt 2(h)−1 = α. was die Behauptung zeigt.
Ferner transformiert sich das asymptotisch hochwahrschein-
liche Ereignis b) Die Voraussetzung besagt, dass es zu jedem ϑ0 ∈ 
 √  eine (messbare) Menge Kϑ0 ⊆ X mit der Eigenschaft
 n(Tn −λ) 
 √ ≤h Pϑ0 (Kϑ0 ) ≤ α gibt. Mit Aϑ0 := X \ Kϑ0 gilt dann
 λ 
Pϑ0 (Aϑ0 ) ≥ 1 − α. Setzen wir
wie folgt: Es gilt
C(x) := {ϑ ∈  | Kϑ 6 x},
√ √  
n|Tn − λ| ≤ h λ ⇔ n Tn2 − 2Tn λ + λ2 − λh2 ≤ 0 so folgt wegen
 
2 h2 x ∈ Aϑ ⇐⇒ Kϑ 6 x
⇔ λ − 2 Tn + λ + Tn2 ≤ 0
2n ⇐⇒ C(x) 6 ϑ
  2
h2 T n h2 h4 die Abschätzung
⇔ λ− Tn + ≤ + 2
2n n 4n Pϑ (C(X) 6 ϑ) = Pϑ (Aϑ ) ≥ 1 − α, ϑ ∈ ,
⇔ Un ≤ λ ≤ On was zu zeigen war.
mit den im Resultat angegebenen Größen Un und On , was Aufgabe 24.7 •• Teilt man in der Darstellung (24.51)
zu zeigen war. Zähler und Nenner durch σ , so ergibt sich
Für die Daten des Rutherford-Geiger-Experiments auf U +δ
Seite 787 nimmt Tn den Wert 3.87 an, und es ist n = 10 097. Tn = ,
W
Zu α = 0.05 ist h = 1.96. Damit ergeben sich die konkreten wobei
Werte für Un und On (auf 4 Nachkommastellen gerundet) zu √ √
3.8317 bzw. zu 3.9087. Man erhält also das konkrete Konfi- n(X n − μ) n(μ − μ0 ) Sn
U= , δ= , W = .
denzintervall [3.8317, 3.9087]. σ σ σ
Nach dem Satz von Student über die Eigenschaften der ML-
Aufgabe 24.3 • Hier liegt der in der Box auf Seite 932 Schätzer der Parameter der Normalverteilung auf Seite 911
angesprochene Trugschluss vor. Die Formulierung in 5% al- sind U und W stochastisch unabhängig, und es gelten U ∼
ler Fälle bezieht sich auf diejenigen „Fälle“ (Testergebnisse), N(0, 1) sowie (n − 1)Sn2 /σ 2 = (n − 1)W 2 =: V ∼ χn−1 2 .

in denen ein signifikanter Widerspruch zu H0 erhoben wird. Nach Definition der nichtzentralen tn−1 -Verteilung folgt die
Die Aussage hätte nur einen Sinn, wenn wir die Gültigkeit Behauptung.
224 Lösungswege zu Kapitel 24

Aufgabe 24.8 • a) Besitzt Q eine Fr,s -Verteilung, d. h., jeder NP-Test ist eine Konvexkombination von
so hat 1/Q nach Definition der Fr,s -Verteilung eine Fs,r - zwei nichtrandomisierten NP-Tests. Setzen wir für c ∈
Verteilung. Es gilt dann {−1, 0, 1, 2, 3}
#
1 − p = P(Q ≤ Fr,s;1−p ) 1, falls k > c,
  ψc (k) :=
1 1 0, falls k ≤ c,
= P ≥
Q Fr,s;1−p
  so ergeben sich wegen
1 1
= 1−P ≤ .
Q Fr,s;1−p j 0 1 2 3
27 27 9 1
Damit gilt 1/Fr,s;1−p = Fs,r;p , was zu zeigen war. P1/4 (X = j ) 64 64 64 64
1 9 27 27
P3/4 (X = j )
b) Mit ϑ = (μ, ν, σ 2 , τ 2 ) und κ := Fm−1,n−1;1−α gilt 64 64 64 64

  die Fehlerwahrscheinlichkeitspunkte (α(ψc ), β(ψc )) zu:


  1/σ 2 τ2
Pϑ Qm,n ≥ κ = Pϑ Q m,n ≥ κ j -1 0 1 2 3
1/τ 2 σ2
  37 10 1
τ2 α(ψc ) 1 64 64 64 0
= 1 − Fm−1,n−1 κ . 1 10 37
σ2 β(ψc ) 0 64 64 64 1

Das letzte Gleichheitszeichen gilt, weil die Fm−1,n−1 - Die Risikomenge ist nachstehend skizziert. Die Fehlerwahr-
Verteilung von scheinlichkeitspunkte der nichtrandomisierten Tests ψj für
j ∈ {−1, 0, 1, 2, 3} sind durch schwarze Kreise hervorgeho-
1/σ 2
Qm,n ben.
1/τ 2
unter ϑ nicht von ϑ abhängt. Dabei bezeichnet Fm−1,n−1 (·)
die Verteilungsfunktion der Fm−1,n−1 -Verteilung. Da
Fm−1,n−1(·) streng monoton
 wächst, wächst die Funktion
ϑ → Pϑ Qm,n ≥ κ streng monoton in σ 2 /τ 2 .

Aufgabe 24.9 •• Die Risikomenge ist gegeben durch

R = {(α(ϕ), β(ϕ)) | ϕ : X → [0, 1]}.

Für k ∈ X und j ∈ {0, 1} gilt


 
3 k
fj (k) = Pϑj (X = k) = ϑ (1 − ϑj )3−k
k j

und somit nach Einsetzen von ϑ0 = 1/4, ϑ1 = 3/4


 3−k
f1 (k) 1 9k
= 3k · = ,
f0 (k) 3 27
Aufgabe 24.10 •• Setzt man die ML-Schätzer σ E2 im
n
k ∈ X . Da dieser Quotient eine streng monoton wachsende Zähler und μ +
+n , σn im Nenner des verallgemeinerten
2
Funktion von k ist, ist ein NP-Test für H0 gegen H1 von der Likelihood-Quotienten ein, so ergibt sich nach Herauskür-
Gestalt ⎧ zen von (2π )n/2

⎨1, falls k > c,  
ϕ(k) = γ , falls k = c, 1 n
⎪ σ@n −n exp − E j =1 (X j − μ 0 ) 2
⎩ 2σn2
0, falls k < c, Q(X) =  
−n 1 n
mit c ∈ R, c ≥ 0. Setzen wir σFn exp − +2 j =1 (Xj − X n ) 2
2σn
# #  n/2
E2
@(k) := 1, falls k > c − 1,
1, falls k > c, σn
ψ(k) := , ψ =
0, falls k ≤ c, 0, falls k ≤ c − 1, +2
σn
 n n/2
j =1 (Xj − X n )2
so gilt = n .
@,
ϕ = (1 − γ )ψ + γ ψ j =1 (Xj − μ0 )2
Lösungswege zu Kapitel 24 225

Wegen und F (t) = p folgt


n
n s−1  

  n
(Xj − μ0 )2 = (Xj − Xn )2 + n(X n − μ0 )2 P X(r) ≤ Qp < X(s) = p j (1 − p)n−j ,
j =1 j =1
j
j =r

folgt und dies war zu zeigen. Wählt man r und s so, dass die obige
n(X n − μ0 )2 Summe mindestens gleich 1 − α ist, so ist [X(r) , X(s) ] ein
Q(X)−n/2 = 1 + n Konfidenzintervall für Qp zur Konfidenzwahrscheinlichkeit
j =1 (Xj − X n )
2
1 − α.
und damit die angegebene Darstellung.
Aufgabe 24.14 • Die Summe der sechs Gewinnzahlen
hat die gleiche Verteilung wie W6,43 unter H0 , wenn wir
Aufgabe 24.11 •• Nach Darstellung (24.81) und dem
unterstellen, dass beim Lotto jede Sechserauswahl der Zahlen
Hinweis besitzt Fn die gleiche Verteilung wie
1, 2, . . . , 49 die gleiche Ziehungswahrscheinlichkeit besitzt.
W := max(X1 , 1 − X1 ),

wobei X1 ∼ U(0, 1). Offenbar gilt P(0 ≤ W ≤ 1/2) = 1,


Beweisaufgaben
und für t ∈ [0, 1/2] gilt Aufgabe 24.15 •• Wir nehmen an, T sei ein erwar-
tungstreuer Schätzer für ϑ. Dann gilt für jedes ϑ ∈ 
P(W ≤ t) = P(X1 ≤ t, 1 − X1 ≤ t)
= P(1 − t ≤ W ≤ t) ϑ = Eϑ T
= t − (1 − t) = 2t − 1. n
= T (j ) Pϑ (X = j )
j =0
Folglich besitzt F1 eine Gleichverteilung auf dem Intervall   
[0, 1/2]. r s
n
j n−j
= T (j )   .
ϑ
Aufgabe 24.12 •• Sind allgemein X1 , . . . , Xk Zufalls- j =0
variablen mit existierenden Erwartungswerten, so existiert n
auch der Erwartungswert jeder Ordnungsstatistik Xj :k von
 Mit M := maxj =0,...,n T (j ) und der Normierungsbedingung
X1 , . . . , Xk , denn es gilt |Xj :k | ≤ kl=1 |Xl |.
  
Mit dem Hinweis gilt, dass (X1:2n − a, . . . , X2n:2n − a) die r s
gleiche Verteilung besitzt wie (a − X2n:2n , . . . , a − X1:2n ) n
j n−j
  = 1
(man beachte, dass sich durch das Minuszeichen die Reihen- ϑ
j =0
folge der Xj umkehrt). Somit besitzt auch die Summe der n
beiden „innersten Ordnungsstatistiken“ die gleiche Vertei-
lung. Es gilt also folgt ϑ ≤ M für jedes ϑ ∈ {r, r + 1, r + 2, . . .}, was nicht
möglich ist.
Xm:2n − a + Xm+1:2n − a ∼ a − Xm+1:2n + a − Xm:2n .
Anmerkung: Diese Aufgabe besitzt die folgende häufig zu
findende Einkleidung: In einem Teich befindet sich eine un-
Hieraus folgt bekannte Anzahl von Fischen. Es werden r Fische gefangen,
markiert und wieder ausgesetzt. Nach einer Weile werden n
E(Xm:2n + Xm+1:2n ) − 2a = 2a − E(Xm:2n + Xm+1:2n ) Fische gefangen. Bezeichnen s die Anzahl der unmarkierten
Fische und ϑ := r + s die Gesamtzahl der Fische im Teich,
und damit die Behauptung. so besitzt die Anzahl X der markierten Fische in dieser Stich-
probe die hypergeometrische Verteilung Hyp(n, r, s), wenn
Aufgabe 24.13 •• Wie im Fall p = 1/2 gilt für r und s man annimmt, dass jede n-elementige Teilmenge aller Fische
mit 1 ≤ r < s ≤ n die gleiche Wahrscheinlichkeit besitzt, diese Stichprobe zu
bilden.
     
P X(r) ≤ Qp < X(s) = P X(r) ≤ Qp − P X(s) ≤ Qp .
Aufgabe 24.16 •• a) Wir betrachten die beiden Seiten
Rechts stehen die Verteilungsfunktionen von X(r) und X(s) , der zu beweisenden Gleichung als Funktionen von ϑ und
ausgewertet an der Stelle Qp . Nach dem Satz über die Ver- nennen die linke Seite u(ϑ) und die rechte v(ϑ). Da k ≥ 1
teilung der r-ten Ordnungsstatistik auf Seite 832 mit t = Qp ist, gilt offenbar u(0) = v(0) = 0. Leitet man u und v nach
226 Lösungswege zu Kapitel 24

ϑ ab, so folgt c) Die Aussage (24.29) ist gleichbedeutend mit

n   
n l(x) < ϑ < L(x) ⇐⇒ a(ϑ) ≤ x ≤ A(ϑ).
u (ϑ)= j ϑ j −1 (1−ϑ)n−j − (n−j )ϑ j (1−ϑ)n−j −1
j
j =k Es gilt
n
n!
= ϑ j −1 (1−ϑ)n−j ϑ < L(x) = sup{ϑ | a(ϑ) = x} ⇐⇒ ∃ϑ1 > ϑ : a(ϑ1 ) = x
(j − 1)!(n − j )!
j =k ⇐⇒ a(ϑ) ≤ x.

n−1
n!
− ϑ j (1−ϑ)n−j −1 Dabei folgt die Richtung „⇐ “ der zweiten Äquivalenz aus
j !(n − j − 1)! der rechtsseitigen Stetigkeit von a(·) und der Tatsache, dass
j =k
die Funktion a(·) nur ganzzahlige Werte annimmt. Völlig

n−1
n! analog ergibt sich die Äquivalenz l(x) < ϑ ⇐⇒ x ≤ A(ϑ).
= ϑ i (1−ϑ)n−i−1
i!(n − i − 1)!
i=k−1
Aufgabe 24.17 •• a) Es ist l(0) = inf{ϑ | A(ϑ) = 0}.

n−1
n! Nach Definition von A(ϑ) gilt
− ϑ j (1−ϑ)n−j −1
j !(n − j − 1)! n  
j =k n α
n! A(ϑ) = 0 ⇐⇒ ϑ j (1 − ϑ)n−j ≤
= ϑ k−1 (1 − ϑ)n−k j 2
j =1
(k − 1)!(n − k)!
α
=v  (ϑ). ⇐⇒ 1 − (1 − ϑ)n ≤ .
2
Hieraus ergibt sich die Behauptung. Wegen 1 − (1 − 0)n ≤ α/2 folgt hieraus l(0) = 0.
b) Da die in (24.24) stehende Summe nach Teil a) streng
monoton in ϑ fällt, ist die Funktion a(·) monoton wachsend. Nach Definition ist L(0) = sup{ϑ | a(ϑ) = 0}. Nun ist
In gleicher Weise ist die Funktion A(·) monoton wachsend,
denn die in (24.25) stehende Summe wächst nach Teil a) 0  
n α
streng monoton in ϑ. Nach Definition von a(ϑ) und A(ϑ) a(ϑ) = 0 ⇐⇒ ϑ j (1 − ϑ)n−j >
j 2
gelten j =0
α
⇐⇒ (1 − ϑ)n >
 
a(ϑ)−1 .
n j α 1 2
ϑ (1−ϑ)n−j ≤ < ,
j =0
j 2 2 Hieraus folgt, dass L(0) die Gleichung (1 − ϑ)n = α/2

n   erfüllt, was zu
n j α 1
ϑ (1−ϑ)n−j ≤ < .  α 1/n
j =A(ϑ)+1
j 2 2 L(0) = 1 −
2
Hieraus folgt A(ϑ) + 1 − (a(ϑ) − 1) ≥ 2 und somit äquivalent ist. Völlig analog zeigt man die Gleichungen
a(ϑ) ≤ A(ϑ). l(n) = (α/2)1/n und L(n) = 1.
Sei kurz  
n j
pn,j (ϑ) := ϑ (1 − ϑ)n−j b) 1) Es ist l(x) = inf{ϑ ∈  | A(ϑ) = x}. Weiter gilt
j
A(ϑ) = x genau dann, wenn die Ungleichungen
gesetzt. Ist (ϑl ) eine Folge aus [0, 1] mit ϑl+1 ≤ ϑl ,

n  
l ≥ 1, und liml→∞ ϑl = ϑ, so konvergiert – da die in (24.24) n j α
ϑ (1 − ϑ)n−j ≤ ,
stehende Summe nach Teil a) streng monoton in ϑ fällt – die j 2
j =x+1
Summe
n  

a(ϑ)−1
n j α
pn,j (ϑl ) ϑ (1 − ϑ)n−j >
j 2
j =0 j =x

für l → ∞ von unten gegen erfüllt sind. Da die beide Summen streng monoton fallende
Funktionen von ϑ sind, erfüllt das Infimum aller ϑ mit der

a(ϑ)−1  α
pn,j (ϑ) ≤ . Eigenschaft A(ϑ) = x die Gleichung
2
j =0 n  
n α
ϑ j (1 − ϑ)n−j = ,
Hieraus folgt, dass a(ϑl ) wegen der Ganzzahligkeit der Funk- j 2
j =x
tion a(·) für hinreichend großes l gleich a(ϑ) sein muss. Dies
zeigt, dass a(·) rechtsseitig stetig ist. Analog folgt, dass A(·) was zu zeigen war. Ganz analog zeigt man die zweite Be-
linksseitig stetig ist. hauptung.
Lösungswege zu Kapitel 24 227

Aufgabe 24.18 •• Nach dem Zentralen Grenzwertsatz gleichen Ziel wie oben folgt
von de Moivre-Laplace auf Seite 889 gilt, wenn wir für das

dortige Sn die Zufallsvariable nTn einsetzen und durch n gn∗ (μ) = Pμ (|Gn | > h∗α )
kürzen,  √ 
√ = Pμ |X n − μ0 | > h∗α σ/ n
n(Tn − ϑ) Dϑ  √ 
Zn := √ −→ Z, = Pμ X n > μ0 + h∗α σ/ n
ϑ(1 − ϑ)  √ 
+ Pμ X n < μ0 − h∗α σ/ n
√ √ 
Pϑ Pϑ
wobei Z ∼ N(0, 1). Wegen Tn −→ ϑ gilt Wn −→ ϑ(1 − ϑ) n(X n − μ) n(μ0 − μ) ∗
= Pμ > + hα
und somit σ σ
? √ √ 
ϑ(1 − ϑ) Pϑ n(X n − μ) n(μ0 − μ) ∗
−→ 1. + Pμ < − hα
Wn σ σ
 √ 
∗ n(μ0 − μ)
Mit Teil b) des Lemmas von Sluzki auf Seite 881 folgt = 1 −  hα +
σ
?  √ 
√ n(μ0 − μ)
n(Tn − ϑ) ϑ(1 − ϑ) Dϑ + 1 −  h∗α −
√ = · Zn −→ Z. σ
Wn Wn  √ 
n(μ − μ0 )
= 2 −  h∗α −
σ
Damit ergibt sich  √ 
n(μ − μ0 )
 √   − h∗α + ,
 n(Tn − ϑ)  σ
lim Pϑ  √  ≤ hα = 1 − α.

n→∞ W n
was zu zeigen war.
Dies war zu zeigen, denn das hier stehende Ereignis ist iden-
tisch mit dem in der Aufgabenstellung. Man beachte, dass Aufgabe 24.20 •• Für x > 0 ist
wir auch bei den Symbolen für Verteilungskonvergenz und
 
stochastische Konvergenz den Parameter ϑ als Index hervor- 1 ∞
t2
gehoben haben. 1 − (x) = √ exp − dt
2π x 2
∞  
1 t t2
Aufgabe 24.19 • Wir betrachten zunächst den einsei- ≤ √ exp − dt
tigen Gauß-Test. Da die Hypothese H0 : μ ≤ μ0 genau 2π x x 2
  
dann zum Niveau α abgelehnt wird, wenn die Prüfgröße t2 ∞
√ 1 1 
Gn = n(Xn − μ0 )/σ größer als hα := −1 (1 − α) ist, = √ − exp − 
x 2π 2 x
gilt
ϕ(x)
= .
gn (μ) = Pμ (Gn > hα ) x
√ 
n(Xn − μ0 ) √
= Pμ > hα Setzen wir kurz q := −1 (1 − α) und δ := n(μ − μ0 )/σ ,
σ so gilt
 
σ hα
= Pμ X n > √ + μ 0
n 1 − gn (μ) = (q − δ) = 1 − (δ − q).
√ √ 
n(Xn − μ) n(μ0 − μ)
= Pμ > + hα Für δ > q (und somit für hinreichend großes n) folgt also
σ σ
 √   
n(μ − μ0 ) 1 1 1
= 1 −  hα − . 1 − gn (μ) ≤ 2
√ exp − (δ − q) .
σ δ − q 2π 2

Dabei wurde beim letzten Gleichheitszeichen verwendet, Nutzt man noch die aus dem Mittelwertsatz der Analysis

dass n(Xn − μ)/σ standardnormalverteilt ist, wenn μ der folgende Ungleichung
wahre Parameter ist. Alle Umformungen liefen darauf hin-
   
aus, diese Tatsache auszunutzen. 1 δ2
−1/2
exp − (δ − q)2 ≤ e exp − (δ − q)
Beim zweiseitigen Gauß-Test wird die Hypothese H0∗ : μ = 2 2
μ0 zugunsten der Alternative H1∗ : μ  = μ0 abgelehnt, wenn
|Gn | > h∗α gilt. Dabei ist h∗α := −1 (1 − α/2). Mit dem aus, so folgt die Behauptung.
228 Lösungswege zu Kapitel 24

Aufgabe 24.21 •• a) Nach Definition der Fr,s -Ver- c) Analog zu b) folgt mit E(R 2 ) = V(R) + E(R)2
teilung können wir  
R/r s2 1
Q = E(Q2 ) = 2
E(R 2
) E
S/s r S2
mit unabhängigen Zufallsvariablen R und S setzen. Dabei 2  
s 1
gelten R ∼ χr2 und S ∼ χs2 . Die Dichte der χk2 -Verteilung = 2 (2r + r 2 ) E .
r S2
besitzt nach (22.4) die Gestalt
1 k x Nun ist für s > 4
fk (x) := x 2 −1 e− 2 , x > 0.   ∞
2k/2 "(k/2) 1 1 1 s/2−1 −t/2
E = s/2 t e dt
Nach dem Satz auf Seite 824 (Methode Verteilungsfunktion) S2 2 "(s/2) 0 t 2
sind die mit gr bzw. gs bezeichneten Dichten von R/r bzw. ∞
2 · 2s/2−3
S/s durch = s/2 us/2−2−1 e−u du
2 "(s/2) 0
gr (u) = fr (ru) r 1 s 
r r/2 = " −2
= r/2 e−ur/2 ur/2−1 , 4"(s/2) 2
2 "(r/2) 1
gs (u) = fs (su) s = .
(s − 2)(s − 4)
s s/2
= s/2 e−us/2 us/2−1
2 "(s/2) Wegen V(Q) = E(Q2 ) − E(Q)2 folgt die Behauptung nun
für u > 0 und gr (u) = gs (u) = 0 sonst, gegeben. Nach Teil mit b) und direkter Rechnung.
c) des Satzes über die Dichte von Differenz, Produkt und
Quotient auf Seite 831 ergibt sich die Dichte von Q zu Aufgabe 24.22 •• Es sei
∞ 
fQ (t) = fr (tz)fs (z) z dz. cn := nλ0 + −1 (1 − α) nλ0
0
Setzt man hier die Ausdrücke für fr (tz) und fs (z) ein, zieht und Sn := X1 + . . . + Xn gesetzt. Zunächst gilt für die
Konstanten vor das Integral und führt anschließend die Sub- Gütefunktion Gϕn von ϕn mit dem Zentralen Grenzwertsatz
stitution u := z(tr + s)/2 durch, so folgt von Lindeberg-Lévy

r r/2 s s/2 t r/2−1 lim Gϕn (λ0 ) = lim Pλ0 (Sn ≥ cn )
fQ (t) = u(r+s)/2−1 e−u du. n→∞ n→∞
"(r/2)"(s/2)(tr + s)(r+s)/2 0  
Sn − nλ0
Da das Integral gleich "((r +s)/2) ist, ergibt sich mit (22.55) = lim Pλ0 √ ≥ −1 (1 − α)
n→∞ nλ0
und (22.56) nach Division von Zähler und Nenner durch
s (r+s)/2 die Behauptung. = 1 − (−1 (1 − α))
= α.
b) Aufgrund der Darstellung von Q und der Unabhängigkeit
von Zähler und Nenner gilt Setzen wir
  s
R
E(Q) = E ·E kn := 4cn 5 = min{k ∈ N | k ≥ cn },
r S
 
s 1 so gilt mit dem Hinweis für jedes λ ≤ λ0
= E(R) E
r S
  ∞

1 λj
= sE . Gϕn (λ) = e−λ
S j!
j =kn
Hierbei wurde E(R) = r ausgenutzt. Weiter gilt mit der λ
1
Substitution u := t/2 und der Funktionalgleichung "(x + = e−t t kn −1 dt
(kn − 1)! 0
1) = x"(x) für die Gamma-Funktion λ0
  ∞ 1
1 1 1 s/2−1 −t/2 ≤ e−t t kn −1 dt
E = s/2 t e dt (kn − 1)! 0
S 2 "(s/2) 0 t
∞ ∞
j
2 · 2s/2−2 λ0
= s/2 us/2−1−1 e−u du = e−λ0
2 "(s/2) 0 j!
j =kn
s 
1 = Gϕn (λ0 )
= " −1
2"(s/2) 2
1 und damit für jedes λ mit λ ≤ λ0
= ,
s−2
lim sup Gϕn (λ) ≤ α.
woraus die Behauptung folgt. n→∞
Lösungswege zu Kapitel 24 229

Die Testfolge (ϕn ) besitzt also das asymptotische Niveau α. Aufgabe 24.25 •• Es sei ϑ = (p1 , . . . , ps ) =
Um die Konsistenz der Folge (ϕn ) nachzuweisen, sei λ1 mit (π1 , . . . , πs ) = ϑ0 . Damit ist o.B.d.A. p1 = π1 sowie
λ1 > λ0 beliebig gewählt. Sei ε > 0 so gewählt, dass λ1 −
ε > λ0 . Wegen limn→∞ cn /n = λ0 gibt es ein n0 , sodass
s
(Xj − nπj )2
für jedes n ≥ n0die Ungleichung cn /n < λ1 − ε erfüllt ist. Tn =
nπj
Mit Xn := n−1 nj=1 gilt für solche n j =1

 (X1 − nπ1 )2
cn  ≥
Gϕn (λ1 ) = Pλ1 X n ≥ nπ1
 n   2
  n X1
≥ Pλ1 X n − λ1  < ε . = − π1 .
π1 n
Da die letzte Wahrscheinlichkeit nach dem Gesetz großer Wegen X1 ∼ Bin(n, p1 ) unter Pϑ konvergiert X1 /n nach
Zahlen für n → ∞ gegen eins konvergiert, folgt die Be- dem Gesetz großer Zahlen Pϑ -stochastisch gegen p1 , und
hauptung. somit gilt

Aufgabe 24.23 ••• Verwendet man den ersten, direkt  2


X1 Pϑ
aus der Tschebyschow-Ungleichung
√ folgenden Hinweis, so Yn := − π1 −→ δ := (p1 − π1 )2 > 0.
n
ergibt sich mit zk = (k − λ)/ λ
√ 1 Nach Definition der stochastischen Konvergenz gilt für
1≥ pλ (k) = P(|X − λ| ≤ C λ) ≥ 1 − 2 . ε := δ/2
C
k:|zk |≤C
lim Pϑ (|Yn − δ| < ε) = 1.
n→∞
Mit (24.62) folgt dann also
  Da das Ereignis {|Yn − δ| < ε} das Ereignis {Yn > δ/2} und
 
√ 1 zk2 1 somit das Ereignis {Tn > nδ/(2π1 )} zur Folge hat und für
1 ≥ Kλ λ √ exp − 1+O √ hinreichend großes n die Ungleichung
λ 2 λ
k:|zk |≤C
1 nδ
≥ 1− 2. 2
χs−1;1−α ≤
C 2π1
Es gilt (Riemann’sche Näherungssumme!) besteht, folgt
  C  
1 zk2 z2 2
lim √ exp − = exp − dz. lim Eϕn = lim Pϑ (Tn ≥ χs−1;1−α ) = 1,
λ→∞ λ 2 2 n→∞ n→∞
k:|zk |≤C −C

√ was zu zeigen war.


Da dieses Integral für C → ∞ gegen 2π konvergiert, folgt
die Behauptung.
Aufgabe 24.26 •• Wir bezeichnen mit  := {ϕ : X →
[0, 1] | ϕ messbar} die Menge aller Tests für das Zwei-
Aufgabe 24.24 •• Es sei A eine k × k-Matrix mit
−1 = A A. Nach dem Reproduktionsgesetz für die Nor- Alternativ-Problem.
malverteilung auf Seite 838 gilt dann
a) Für die Tests ϕ ≡ 1 und ψ ≡ 0 gelten α(ϕ) = 1, β(ϕ) = 0
Y := A(X − μ) ∼ Nk (0, A A ). und α(ψ) = 0, β(ψ) = 1.

Aus A A = −1 folgt A = −1 A−1 und somit b) Zu ϕ ∈  betrachten wir den Test ψ := 1 − ϕ ∈ . Für
diesen gelten α(ψ) = Eϑ0 (1 − ϕ) = 1 − α(ϕ) und analog
A A = A −1 −1
A = Ik . β(ψ) = 1 − β(ϕ).

Nach Aufgabe 22.46 sind die Komponenten Y1 , . . . , Yk von c) Sind (αj , βj ) ∈ R und δ ∈ [0, 1], so existieren Tests
Y stochastisch unabhängige und je N(0, 1)-verteilte Zufalls- ϕj ∈  mit (αj , βj ) = (α(ϕj ), β(ϕj )), j = 1, 2. Für die
variablen. Wegen Konvexkombination ϕ := δϕ1 + (1 − δ)ϕ2 gilt ϕ ∈  und


k
α(ϕ)=Eϑ0 (δϕ1 + (1 − δ)ϕ2 ) = δ Eϑ0 ϕ1 + (1 − δ)Eϑ0 ϕ2
(X − μ) −1
(X − μ) = Y  Y = Yj2
j =1
=δα1 + (1 − δ)α2

folgt die Behauptung aus der Erzeugungsweise der Chi- und analog β(ϕ) = δβ1 + (1 − δ)β2 . Folglich gehört der
Quadrat-Verteilung auf Seite 844. Punkt δ(α1 , β1 ) + (1 − δ)(α2 , β2 ) zu R.
230 Lösungswege zu Kapitel 24

Aufgabe 24.27 •• Die σ -Subadditivität von P liefert Da die Dichte symmetrisch um a ist, ist a der Median von
zunächst X. Die Ableitung Fs (Q1/2 ) ist also durch
⎛ ⎞  
4 " s+1
P⎝ {Xi = Xj }⎠ ≤ P(Xi = Xj ). 2
Fs (Q1/2 ) = fs (a) = √  
1≤i<j <∞ 1≤i<j <∞ π s " 2s
Wendet man den Satz von Tonelli auf Seite 262 mit 1 = gegeben. Da die Varianz σF2s nach Aufgabe 24.43 b) gleich
2 = R, A1 = A2 = B und μ1 = PXi sowie μ2 = PXj und s/(s − 2) ist, ergibt sich
f = 1 mit  = {(x, y) ∈ R2 | x = y} an, so folgt
AREFs (Qn,1/2 , X n ) = 4Fs (a)2 σF2s
P(Xi = Xj ) = 1 PXi (dx)PXj (dy)  
R2 4" 2 s+1
∞   =
2
 ,
= PXj (dy) PXi (dx) (s − 2)π " 2 2s
−∞ {x}
∞ was zu zeigen war.
= P(Xj = x) PXi (dx) b) Für die Fälle s = 3, s = 4 und s = 5 folgt unter Verwen-
−∞ √
dung von "(x + 1) = x"(x) und "(1/2) = π
= 0,
16
da aufgrund der Stetigkeit von Fj P(Xj = x) = 0 gilt. AREF3 (Qn,1/2 , X n ) = = 1.6211 . . .
π2
9
Aufgabe 24.28 •• Nach Aufgabe 24.13 gilt AREF4 (Qn,1/2 , X n ) = = 1.125,
8
n −1 
s  256
  n j AREF5 (Qn,1/2 , X n ) = = 0.9606 . . .
P Xrn :n ≤ Qp < Xsn :n = p (1 − p)n−j 27π 2
j
j =rn
Für den Fall s ≥ 6 verwenden wir die im Hinweis angegebene
= P(rn ≤ Sn ≤ sn − 1), Ungleichung
 
wobei Sn eine Zufallsvariable mit der Binomialverteilung 1 √
Bin(n, p) bezeichnet. Nach Standardisierung gilt also " x+ ≤ x "(x), x > 0,
2
 
  Sn − np
P Xrn :n ≤ Qp < Xsn :n = P an ≤ √ ≤ bn , die mit f (t) := e−t/2 t x/2 , g(t) := e−t/2 t x/2−1/2 aus der
np(1 − p) Cauchy-Schwarz’schen Ungleichung
wobei ∞  ∞ 1/2  ∞ 1/2
rn − np sn − 1 − np f (t)g(t)dt ≤ f (t)2 dt g(t)2 dt
an := √ , bn := √ . 0 0 0
np(1 − p) np(1 − p)
folgt. Hiermit ergibt sich
Wegen
lim an = −hα , lim bn = hα 4 2s " 2 ( 2s )
n→∞ n→∞ AREFs (Qn,1/2 , X n ) ≤  
liefern der Zentrale Grenzwertsatz von de Moivre-Laplace (s − 2)π " 2 2s
und Aufgabe 23.12 2s
=
  (s − 2)π
lim P Xrn :n ≤ Qp < Xsn :n = (hα ) − (−hα ) < 1, falls s ≥ 6,
n→∞
= 2(hα ) − 1
 α und damit insbesondere
= 2 1− −1
2 2
= 1 − α. lim sup AREFs (Qn,1/2 , X n ) ≤ .
s→∞ π
Wegen P(Qp = Xsn :n ) = 0 (vgl. die Selbstfrage auf Andererseits gilt
Seite 954) kann zu Beginn dieser Gleichungskette das    
Kleiner-Zeichen auch durch das Kleiner-gleich-Zeichen er- s+1 s−1
" = " +1
setzt werden, sodass die Behauptung folgt. 2 2
 
s−1 s−1
= " .
Aufgabe 24.29 •• a) Die Verteilungsfunktion Fs ist auf 2 2
R differenzierbar, mit der Ableitung (Dichte)
   Mit der obigen Ungleichung ergibt sich
s+1 −(s+1)/2  
1 " 2 (t − a)2 s−1 1 s 
fs (t) = √ s  1+ . " ≥ √ " ,
πs " 2 s 2 (s − 1)/2 2
Lösungswege zu Kapitel 24 231

und man erhält tere ist die Gleichverteilung auf den Paaren (i, j ) mit i, j ∈
{1, . . . , k} und i = j . Es folgt
2(s − 1)
AREFs (Qn,1/2 , Xn ) ≥ ,
(s − 2)π E(R1 R2 ) = i j P(R1 = i, R2 = j )
i=j
also insbesondere 1
= ij
2 k(k − 1)
i=j
lim inf AREFs (Qn,1/2 , X n ) ≥ . ⎛ ⎞
s→∞ π
1 k
= i⎝ j⎠
Zusammen mit der Abschätzung nach oben folgt die behaup- k(k − 1)
i=1 j :j =i
tete Grenzwertaussage

k  
1 k(k + 1)
2 = i −i
lim AREFs (Qn,1/2 , X n ) = . k(k − 1) 2
s→∞ π i=1
 
k 2 (k + 1)2 2
k
1
= − i
k(k − 1) 4
Aufgabe 24.30 •• Wir setzen kurz k := m + n sowie 
i=1

Ri := r(Xi ) für i = 1, . . . , m. Da aus Symmetriegründen 1 k 2 (k + 1)2 k(k + 1)(2k + 1)
jedes Ri die gleiche Verteilung besitzt und auch die Paare = −
k(k − 1) 4 6
und j mit i  = j identisch verteilt
(Ri , Rj ) für jede Wahl von i
sind, folgt wegen Wm,n = m (k + 1)(3k + 2)
i=1 Ri nach den Rechenregeln =
für Erwartungswert und Varianz 12
und damit
E(Wm,n ) = m E(R1 ),
Cov(R1 , R2 ) = E(R1 R2 ) − ER1 · ER2
V(Wm,n ) = mV(R1 ) + m(m − 1) Cov(R1 , R2 ).
(k + 1)(3k + 2) (k + 1)2
= −
12 4
Mit dem Hinweis folgt, dass R1 auf den Werten 1, 2, . . . , k
k+1
gleichverteilt ist. Damit ergibt sich = − .
12
k+1 k2 − 1 Hiermit ergibt sich b) durch direkte Rechnung.
E(R1 ) = , V(R1 ) =
2 12
Rechenaufgaben
(vgl. das Beispiel auf Seite 774 sowie (21.17)), woraus un-
mittelbar Aussage a) folgt. Aufgabe 24.31 • Wir betrachten zunächst die beiden
Fälle k = 0 und k = n. Im ersten gilt h(ϑ) = (1 − ϑ)n ,
Aussage b) ergibt sich am einfachsten, wenn man zu den oben und das Maximum wird für ϑ = 0 = k/n angenommen.
eingeführten Zufallsvariablen R1 , . . . , Rm die Rangzahlen Im zweiten Fall ist h(ϑ) = ϑ n , und diese Funktion wird für
Rm+j := r(Yj ), j = 1, . . . , n, hinzunimmt. Wegen ϑ = 1 = k/n maximal. Im verbleibenden Fall 1 ≤ k ≤
n − 1 gilt h(0) = h(1) = 0, sodass das Maximum von h

k
k
k(k + 1) im offenen Intervall (0, 1) angenommen wird. Differenziert
Rj = j =
2 man die Funktion h, so ergibt sich als notwendige Bedingung
j =1 j =1
für ein Extremum
  
gilt dann V( kj =1 Rj ) = 0. Andererseits ist mit Rechenre- n k−1
0 = h (ϑ) = ϑ (1 − ϑ)n−k−1 (k − nϑ)
geln für die Varianz k
⎛ ⎞ und damit ϑ = k/n. Wegen h (ϑ) > 0 für ϑ < k/n und

k
h (ϑ) < 0 für ϑ > k/n liegt an der Stelle k/n ein Maximum
V⎝ Rj ⎠ = k V(R1 ) + k(k − 1) Cov(R1 , R2 ) vor.
j =1
Aufgabe 24.32 •• Mit der Abkürzung
und somit
V(R1 ) t m := t (t − 1) . . . (t − m + 1)
Cov(R1 , R2 ) = − .
k−1 für t ∈ R und m ∈ N sowie t 0 := 1 ist die Likelihood-
Mit dem oben angegebenen Ausdruck für V(R1 ) folgt dann Funktion zu x durch
b) durch Einsetzen. ϑ k (N − ϑ)n−k
Lx (ϑ) = Pϑ (X = x) = ,
Alternativ (aber umständlicher) kann man Cov(R1 , R2 ) über Nn
die gemeinsame Verteilung von R1 und R2 berechnen. Letz- k = x1 + . . . + xn , gegeben.
232 Lösungswege zu Kapitel 24

Wir unterscheiden die Fälle a) k = 0, b) k = n und c) b) Mit (24.24) gilt für jedes ϑ ∈ 
1 ≤ k ≤ n − 1.

ϑ
k n+1 − (k − 1)n+1  
Zu a) Es ist Eϑ Tn = Pϑ Fϑn = k
(N − ϑ)n k − (k − 1)
n n
Lx (ϑ) = k=1
Nn
ϑ
k n+1 − (k − 1)n+1
und somit F
ϑ (x) = 0. =
ϑn
k=1
Zu b) Hier gilt  ϑ 
1 n+1
ϑ
ϑn
Lx (ϑ) = = n k − (k − 1) n+1
Nn ϑ
k=1 k=1
und folglich F
ϑ (x) = N. 1
= n ϑ n+1
Zu c) Im Fall 1 ≤ k ≤ n − 1 betrachten wir die Quotienten ϑ
= ϑ.
Lx (ϑ + 1) (ϑ + 1)k (N − ϑ − 1)n−k Nn
= · k
Lx (ϑ) N n
ϑ (N − ϑ)n−k
ϑ +1 N −ϑ +k−n Aufgabe 24.34 •• a) Die Likelihood-Funktion Lx zu
= · . x = (x1 , . . . , xn ) ∈ X := Nn0 ist durch
ϑ −k+1 N −ϑ
Eine direkte Rechnung liefert Lx (ϑ) = f (x, ϑ) = Pϑ (X = x)
n n 
 
Lx (ϑ + 1) k(N + 1) ϑ xj
> 1 ⇐⇒ ϑ < − 1, = Pϑ (Xj = xj ) = exp(−ϑ)
Lx (ϑ) n xj !
j =1 j =1
Lx (ϑ + 1) k(N + 1)  1 nj=1 xj
n
= 1 ⇐⇒ ϑ = − 1.
Lx (ϑ) n = exp(−nϑ) ϑ
xj !
j =1
Hieraus folgt die Behauptung.
n
gegeben. Ist j =1 xj = 0, so wird Lx offenbar für
Aufgabe 24.33 •• a) Aus dem Beispiel auf Seite 911
wissen wir bereits, dass EF ϑn ≤ ϑ für jedes ϑ ∈  gilt. Nun 1
n
ist für k = 1, . . . , ϑ F
ϑ (x) := 0 = xj
n
   n j =1
k
Pϑ max Xj ≤ k = n
j =1,...,n ϑ maximal. Im Fall j =1 xj > 0 betrachten wir die
Loglikelihood-Funktion
und somit
   n  
n
n
k k−1 n log Lx (ϑ) = −nϑ − log xj ! + xj log ϑ.
Pϑ max Xj = k = − . (24.24)
j =1,...,n ϑ ϑ j =1 j =1

Es folgt Ableiten nach ϑ liefert


 
1
ϑ n
d
Eϑ F
ϑn = k Pϑ max Xj = k log Lx (ϑ) = −n + xj , (24.25)
j =1,...,n dϑ ϑ
k=1 j =1
 
≥ ϑ Pϑ max Xj = ϑ und Nullsetzen dieses Ausdrucks liefert den ML-Schätzwert
j =1,...,n
 n   
ϑ ϑ −1 n 1
n
= ϑ − F
ϑ (x) = xj .
ϑ ϑ n
j =1
→ ϑ für n → ∞.

Somit ist die Schätzfolge (F


ϑn ) asymptotisch erwartungstreu Eine Betrachtung der Ableitung zeigt, dass an dieser Stelle in
für ϑ. Wegen der Tat ein Maximum der Loglikelihood-Funktion vorliegt.
Somit ist F
ϑ = X n der ML-Schätzer.
 
  ϑ −1 n
Pϑ F ϑn = ϑ = 1 − b) Aus (24.25) folgt wegen Lx (ϑ) = f (x, ϑ)
ϑ
→ 1 für n → ∞ d 1
n
Uϑ = log f (X, ϑ) = Xj − n.
dϑ ϑ
ist die Schätzfolge (F
ϑn ) auch konsistent für ϑ. j =1
Lösungswege zu Kapitel 24 233

n
Wegen j =1 Xj ∼ Po(nϑ) ergibt sich Wegen der Unabhängigkeit von X1 , . . . , Xn folgt somit
⎛ ⎞

n 4 ϑ2 − 1
1 Vϑ (@
ϑn ) = ·n·
If (ϑ) = Vϑ (Uϑ ) = 2 Vϑ ⎝ Xj ⎠ n 2 12
ϑ
j =1 ϑ2 − 1
1 n = .
= nϑ = . 3n
ϑ2 ϑ

c) Mit Aufgabe 24.37 •• a) Sei S := nj=1 Xj . Wegen X1 ∼
  Vϑ (X1 ) ϑ Exp(ϑ) gilt nach dem Additionsgesetz für die Gammavertei-
Vϑ X n = =
n n lung auf Seite 844 S ∼ "(n, ϑ). Die Zufallsvariable S hat
  also nach (22.52) unter Pϑ die Dichte
gilt Vϑ Xn = 1/If (ϑ), ϑ ∈ , was zu zeigen war.
ϑ n n−1 −ϑt
Aufgabe 24.35 •• a) Die Zufallsvariable X, deren Rea- g(t, ϑ) = t e
"(n)
lisierung k beobachtet wird, ist die Anzahl der Nieten vor
dem ersten Treffer. Sie besitzt also die geometrische Vertei- für t > 0 und g(t, ϑ) = 0 sonst. Es folgt für n ≥ 2
lung G(ϑ). Die Likelihood-Funktion ist
n
k Eϑ (F
ϑn ) = Eϑ
Lk (ϑ) = Pϑ (X = k) = (1 − ϑ) ϑ, 0 < ϑ < 1.
∞S
1
Diese nimmt ihr Maximum für den ML-Schätzwert = n g(t, ϑ) dt
0 t

F
1 ϑn
ϑ (k) := = n t n−2 e−ϑt dt
1+k "(n) 0

ϑn 1
an. = n un−2 e−u du
"(n) ϑ n−1 0
b) Da die durch g(t) := 1/(1 + t) definierte Funktion g :
ϑn 1
R≥0 → R strikt konvex ist und die Verteilung von X unter Pϑ = n "(n − 1)
nicht ausgeartet ist, folgt mit der Jensen’schen Ungleichung "(n) ϑ n−1
n
auf Seite 834 = ϑ.
  n−1
1
Eϑ (F
ϑ ) = Eϑ = Eϑ g(X) b) In gleicher Weise wie ergibt sich für n ≥ 3
1+X
> g (Eϑ X)  
n2
1 Eϑ (F
ϑn2 ) = Eϑ
= 1 = ϑ. S2
ϑ −1+1 ∞1
Der ML-Schätzer ist somit nicht erwartungstreu. = n2 2
g(t, ϑ) dt
0 t

ϑn
Aufgabe 24.36 •• Es ist = n2 t n−3 e−ϑt dt
"(n) 0

ϑn 1
1
ϑ
ϑ +1 = n2 un−3 e−u du
Eϑ (X1 ) = k = "(n) ϑ n−2 0
ϑ 2
k=1 ϑn 1
= n2 "(n − 2)
und somit "(n) ϑ n−2
n2
2
n = ϑ2
Eϑ (@
ϑn ) = Eϑ Xj − 1 (n − 1)(n − 2)
n
j =1
und somit
2 ϑ +1
= ·n· −1  2
n 2 n2 ϑ 2 nϑ
= ϑ, Vϑ (F
ϑn ) = −
(n − 1)(n − 2) n−1
was die Erwartungstreue von @
ϑn zeigt. ϑ 2 n2
= .
(n − 1)2 (n − 2)
Nach (21.17) gilt
c) Wegen limn→∞ Eϑ (F ϑn ) = ϑ und limn→∞ Vϑ (F ϑn ) = 0
ϑ2 − 1 für jedes ϑ folgt die Behauptung aus der Selbstfrage auf
Vϑ (X1 ) = .
12 Seite 969.
234 Lösungswege zu Kapitel 24

Aufgabe 24.38 •• Der Hinweis gründet auf der Glei- b) Wir bestimmen zunächst Erwartungswert und Varianz von
chung EX1 = EX n , es sei also im Folgenden EX1 := 0 ge- ϑn unter Pϑ . Unter Pϑ hat F
F ϑn die Verteilungsfunktion
setzt. Wegen der Linearität der Erwartungswertbildung und  
der identischen Verteilung der Xj gilt zunächst Gϑ (t) := Pϑ F ϑn ≤ t
 
⎛ ⎞ = Pϑ max Xj ≤ t

n j =1,...,n
1 n  2 ⎛ ⎞
E⎝ (Xj − X n )2 ⎠ = E X1 − X n .
n−1 n−1 <n
2 3
j =1 = Pϑ ⎝ Xj ≤ t ⎠
j =1
Weiter gilt
= Pϑ (X1 ≤ t)n
 n
2 t
(X1 − Xn )2 = X12 − 2X1 X n + X n =
ϑ
2 1
n n
= X12 − X1 Xj + 2 Xi Xj . für 0 ≤ t ≤ ϑ sowie Gϑ (t) = 0 für t ≤ 0 und Gϑ (t) = 1
n n
j =1 i,j =1 für t ≥ ϑ. Hieraus folgt, dass F
ϑn unter Pϑ die Dichte
 n−1
Wegen der Unabhängigkeit der Xj und EX1 = 0 gilt n t
gϑ (t) := , 0 < t < ϑ,
EX12 = σ 2 und E(Xi Xj ) = 0 für i  = j , und man erhält ϑ ϑ
und g(t) := 0 sonst, besitzt. Es ergibt sich
2 2 1
E(X1 − X n )2 = σ 2 − σ + 2 nσ2 ϑ ϑ
n n n n
F
Eϑ (ϑn ) = t gϑ (t) dt = n t n dt = ϑ,
n−1 2 ϑ n + 1
= σ 0 0
n   ϑ ϑ
n n
F 2
Eϑ ϑn = 2
t gϑ (t) dt = n t n+1 dt = ϑ2
und damit die Behauptung. 0 ϑ 0 n + 2
und somit
Aufgabe 24.39 •• Wegen  2
n n
Vϑ (F
ϑn ) = ϑ2 − ϑ

n n+2 n+1
(Xj − X n )2 ∼ σ 2 Y nϑ 2
j =1 = .
(n + 2)(n + 1)2

mit Y ∼ χn−1
2 gilt Wegen
n+1
ϑn∗ = F
ϑn
n
E(Sn (c) − σ 2 )2 = E(cσ 2 Y − σ 2 )2
  folgt
= σ 4 E c2 Y 2 − 2cY + 1
  n+1
Eϑ (ϑn∗ ) = Eϑ (F
ϑn ) = ϑ,
= σ 4 c2 EY 2 − 2cEY + 1 . n
 2
n+1 ϑ2
Vϑ (ϑn∗ ) = Vϑ (Fϑn ) = .
Die rechte Seite ist ein Polynom zweiten Grades in c, das für n n(n + 2)

EY n−1 1 c) Wegen μ := Eϑ (X1 ) = ϑ/2 gilt ϑ = 2μ. Somit ist der


c = = =
EY 2 (n − 1) + 2(n − 1)
2 n+1 Momentenschätzer für ϑ durch
1
n
seinen Minimalwert annimmt. @
ϑn = 2 X n = 2 Xj
n
j =1
Aufgabe 24.40 •• a) Die Dichte von X1 unter Pϑ ist
f1 (t) = 1[0,ϑ] (t). Die Dichte verschwindet also unabhängig gegeben. Es gilt
von ϑ, falls t < 0 gilt. Setzen wir X := [0, ∞)n , so ist für ϑ
x = (x1 , . . . , xn ) ∈ X die Likelihood-Funktion zu x durch Eϑ @
ϑn = 2Eϑ (X n ) = 2 = ϑ
2

n   sowie wegen
1
Lx (ϑ) = f1 (xj , ϑ) = 1[0,ϑ] max xj  
ϑn j =1,...,n ϑ2 ϑ2 ϑ2
j =1 Vϑ (X1 ) = Eϑ X12 − (Eϑ X1 )2 = − = ,
3 4 12
gegeben. Diese Funktion nimmt ihren Maximalwert an, wenn   4 ϑ2 ϑ2
Vϑ @
ϑn = = .
ϑ := maxj =1,...,n Xj gesetzt wird. n 12 3n
Lösungswege zu Kapitel 24 235

d) Da die Schätzer ϑn∗ und @


ϑn erwartungstreu sind, stimmen b) Im Fall der Poisson-Verteilung ist die Zähldichte auf
die mittleren quadratischen Abweichungen mit den Varian- X = N0 durch
zen überein. Wegen
ϑx 1
ϑ2 ϑ2 f (x, ϑ) = e−ϑ = e−ϑ exp(x log ϑ)
Vϑ (ϑn∗ ) = < = Vϑ (@
ϑn ) x! x!
n(n + 2) 3n
gegeben. Es liegt somit eine einparametrige Exponential-
für jedes ϑ ∈  und jedes n ≥ 2 ist der Schätzer @
ϑn gleich- familie mit b(ϑ) = exp(−ϑ), h(x) = 1/x!, T (x) = x und
mäßig besser als ϑn∗ , falls n ≥ 2. Q(ϑ) = log ϑ vor.

Aufgabe 24.41 •• Die Dichte der Gammaverteilung c) Die Dichte der Exponentialverteilung Exp(ϑ) auf
"(α, λ) ist X := [0, ∞) ist
λα α−1
f (t; α, λ) = t exp (−λt) f (x, ϑ) = ϑ exp(−ϑx).
"(α)
für t > 0 und f (t; α, λ) = 0 sonst. Die Likelihood-Funktion Es liegt also eine einparametrige Exponentialfamilie mit
zu x = (x1 , . . . , xn ) ∈ (0, ∞)n ist somit durch b(ϑ) = ϑ, h(x) ≡ 1, T (x) = x und Q(ϑ) = −ϑ vor.
⎛ ⎞α−1 ⎛ ⎞
 α n  n
n Aufgabe 24.43 •• a) Nach Gleichung c) auf Seite 831
λ ⎝
Lx (α, λ) = xj ⎠ exp ⎝−λ xj ⎠ ist die Dichte f des Quotienten zweier unabhängiger Zufalls-
"(α)
j =1 j =1 variablen X1 und X2 mit Dichten f1 bzw. f2 durch
gegeben. Für die Loglikelihood-Funktion folgt daher ∞
f (t) = f1 (ts) f2 (s) |s| ds, t ∈ R, (24.26)
log Lx (α, λ) = n (α log λ − log "(α)) −∞
n
n
+(α − 1) log xj − λ xj . √ Im Fall der tk -Verteilung gilt X1 ∼ N(0, 1) sowie
gegeben.
j =1 j =1
X2 ∼ Yk /k, wobei Yk ∼ χk2 . Wir bestimmen zunächst die
Verteilungsfunktion und dann die Dichte von X2 . Es ist für
Die Ableitungen dieser Funktion nach α und λ sind u>0
  n    
∂ d
log Lx (α, λ) = n log λ − log "(α) + log xj , P(X2 ≤ u) = P Yk /k ≤ u = P Yk ≤ ku2 .
∂α dα
j =1
Da Yk nach (22.4) die Dichte
∂ α
n
log Lx (α, λ) = n − xj , 1
∂λ λ g(t) = t k/2−1 exp(−t/2), t > 0,
j =1
2k/2 "(k/2)
sodass Nullsetzen dieser Ableitungen das zu zeigende Resul-
besitzt, hat X2 (Differenziation!) die Dichte
tat liefert.
Setzt man Fλn = Fαn /X n in die zweite Gleichung der Auf- f2 (s) = 2ks g(ks 2 ), s > 0,
gabenstellung ein, so ergibt sich als numerisch zu lösende
und f2 (s) = 0 sonst. Wegen
Bestimmungsgleichung für F αn
 
1 t 2s2
n 1
log F
αn − (F
αn ) = log X n − log Xj . f1 (ts) = √ exp −
n 2π 2
j =1

Dabei ist folgt durch Einsetzen in (24.26) unter Beachtung von


d f2 (s) = 0 für s ≤ 0 und Vorziehen aller Konstanten vor
(t) = log "(t)
dt das Integral sowie Zusammenfassen von Exponenten
die sogenannte Digamma-Funktion. ∞  
2k k/2 (t 2 + k)s 2
Aufgabe 24.42 •• a) Die Zähldichte der Binomialver- f (t) = √ exp − s k ds.
2k/2 "(k/2) 2π 0 2
teilung Bin(n, ϑ) auf X = {0, 1, . . . , n} ist
  Das Integral geht durch die Substitution
n x
f (x, ϑ) = ϑ (1 − ϑ)n−x
x t2 + k 2
   x u := s
n n ϑ 2
= (1 − ϑ)
x 1−ϑ
    in ∞
n ϑ 2(k−1)/2
= (1 − ϑ)n exp x log . e−u u(k−1)/2−1 du
x 1−ϑ (x 2 + k)(k−1)/2 0
n besitzt die Gestalt (24.18) mit b(ϑ) = (1 − ϑ) , h(x) = über. Da letzteres Integral gleich "((k + 1)/2) ist, folgt die
Sie n

x , T (x) = x und Q(ϑ) = log(ϑ/(1 − ϑ)). Behauptung durch direktes Ausrechnen.


236 Lösungswege zu Kapitel 24

b) Aufgrund der Gestalt der Dichte (24.35) gilt für r ∈ N b) Es ist


   
k+1 ϑ
1 " 2 ∞ |t|r Pϑ (ϑ ≤ 1.1 · Mn ) = 1 − Pϑ Mn <
E|X|r = √     dt. 1.1
πk " k −∞ 2 (k+1)/2  
2 1 + tk ϑ n
= 1 − Pϑ X1 <
1.1
Dieses Integral liefert genau dann einen endlichen Wert,  n
10
wenn k + 1 − r ≥ 2 gilt. Insbesondere folgt ≥ 1− .
11
E|X| < ∞ ⇐⇒ k ≥ 2, E|X|2 < ∞ ⇐⇒ k ≥ 3. Weiter gilt
 n  n
Im Fall der Existenz des Erwartungswertes gilt E(X) = 0, da 10 10
1− ≥ 0.99 ⇐⇒ ≤ 0.01,
die Verteilung von X symmetrisch um 0 ist. Nach Darstellung 11 11

(24.33 ist X verteilungsgleich mit N/ Z/k, wobei N und
was zu
Z stochastisch unabhängig sind und die Verteilungen N ∼ log 100
N(0, 1) und Z ∼ χk2 besitzen. Mit der Multiplikationsformel n ≥
log 11 − log 10
für Erwartungswerte und E(N 2 ) = 1 folgt
äquivalent ist. Die kleinste natürliche Zahl, die diese Unglei-
    chung erfüllt, ist n = 49.
1 1
E(X 2 ) = k E(N 2 ) E = kE .
Z Z
Aufgabe 24.45 ••
Da Z nach (22.4) die Dichte @
a) Mit (24.30) ist L(0) = 1 − 0.011/275 = 0.0166 . . . die
  gesuchte obere Konfidenzschranke.
1 t
f (t) =   t k/2−1 exp − b) Aus 1 − 0.011/n ≤ 10−4 folgt n ≥ log(0.01)/
2k/2 " k 2
2 log(0.9999), also n ≥ 46 050.
für t > 0 und f (t) = 0 sonst, besitzt, ergibt sich c) Unter den Annahmen wäre die Anzahl X von Mäusen mit
BSE-Symptomen unter insgesamt n Mäusen Bin(n, ϑ)-
  ∞  
1 1 t verteilt. Somit würde
E =   t k/2−2 exp − dt.
Z 2k/2 " k 0 2
2 E X = np = 10 000 000 · 0.0166 = 166 000

Mit der Substitution u := t/2 und der Definition der Gam- gelten.
mafunktion folgt dann
Aufgabe 24.46 • a) Wir verwenden ein Binomialmo-
dell, betrachten also die Ergebnisse der Befragungen als Rea-
∞ lisierungen von n = 1250 unabhängigen Zufallsvariablen
k 2k/2−1
E(X 2 ) =   uk/2−2 e−u du mit gleicher Binomialverteilung Bin(n, ϑ). Aufgrund des
k/2 k
2 " 2 0 großen Stichprobenumfangs machen wir Gebrauch von den
 
2k k k in (24.43) und (24.45) angegebenen Konfidenzgrenzen ln∗ und
=  " −1 = , L∗n . Einsetzen liefert, dass die Konfidenzgrenzen durch
" k 2 k−2
2
1.96 √
da "(x + 1) = x"(x) für x > 0. 0.25 ± √ 0.25 · 0.75 = 0.25 ± 0.024
n

Aufgabe 24.44 •• a) Zu zeigen ist gegeben sind. Das konkrete Konfidenzintervall ist somit
  [0.226, 0.274].
Pϑ ϑ ≤ α −1/n max Xj ≥ 1 − α ∀ϑ ∈ .
j =1,...,n
b) Wie in a) verwenden wir die in (24.43) und (24.45) ange-
Sei Mn := max(X1 , . . . , Xn ). Es ist gebenen Konfidenzgrenzen ln∗ und L∗n . Danach ist die Länge
    des Konfidenzintervalls durch
Pϑ ϑ ≤ α −1/n Mn = 1 − Pϑ Mn < ϑα 1/n 2hα 
 n √ Tn (1 − Tn )
= 1 − Pϑ X1 < ϑα 1/n n
 n gegeben. Da Tn der Mittelpunkt des Intervalls mit den Gren-
ϑα 1/n
≥ 1− zen ln∗ und L∗n ist und der Prozentsatz bis auf ±1% genau
ϑ geschätzt werden soll, muss die Läge des Intervalls 0.02
= 1 − α. betragen. Da für die Realisierung von Tn ein Wert in der
Lösungswege zu Kapitel 24 237


Nähe von 0.3 erwartet √ wird und sich die Werte der Funk- und somit wegen 0.9 = (1.282) die Gleichung n(μ0 −
tion t → g(t) := t (1 − t) bei kleinen Abweichungen μ1 )/2 − 2.326 = 1.282. Hieraus ergibt sich der Mindest-
von t zu 0.3 kaum ändern (so ist g(0.3) ≈ 0.458 und stichprobenumfang zu n = 53.
g(0.28) ≈ 0.449), setzen wir in die obige Darstellung der
(zufälligen) Intervalllänge für Tn den Wert 0.3 ein und lösen Aufgabe 24.49 • Wir sehen die Differenzen zi :=
mit hα = 1.96 die Ungleichung yi − xi als Realisierungen unabhängiger und je N(μ, σ 2 )-
verteilter Zufallsvariablen Z1 , . . . , Z8 an, wobei μ und σ 2
2 · 1.96 
√ 0.3(1 − 0.3) ≤ 0.02 unbekannt sind. Aufgrund der Aufgabenstellung testen wir
n die Hypothese H0 : μ ≤ 0 gegen die Alternative H1 : μ > 0.
nach n auf. Das kleinste (ganzzahlige) n, das dieser Unglei- Der Mittelwert z8 von z1 , . . . , z8 ist 0.06125, und es gilt
chung genügt, ist 8059.
1
8
(zj − z8 )2 = 0.002807 . . .
Aufgabe 24.47 •• Die Situation entspricht der einer 8−1
j =1
Urne mit r roten und s = 100 − r schwarzen Kugeln (diese
stehen für die defekten bzw. intakten Glühbirnen), aus wel- Wegen
cher n(= 10)-mal ohne Zurücklegen gezogen wird. Die An-
zahl X der gezogenen roten Kugeln besitzt die hypergeo- √
8 · z8
metrische Verteilung Hyp(10, r, s). Der Stichprobenraum ist * = 3.269 . . .
1 8
X := {0, 1, 2, . . . , 10}, und der Parameterbereich für r(= ϑ) 8−1 j =1 (z j − z 8 ) 2
ist  := {0, 1, . . . , 100}. Hypothese und Alternative lauten
H0 : r ≤ 10 bzw. H1 : r > 10. Der Händler wählt den kriti- wird H0 auf dem 5%-Niveau abgelehnt, denn nach Ta-
schen Bereich K := {1, 2, . . . , 10}. belle 24.2 gilt t7;0.95 = 1.895.
Mit s := 100 − r gilt
Aufgabe 24.50 • Wir führen einen χ 2 -Test durch. Die
s · (s − 1) · . . . · (s − 9) Teststatistik (vgl. (24.63)) nimmt mit s = 6, π1 = π2 =
Pr (X ∈ K) = 1−Pr (X = 0) = 1− .
100 · 99 · . . . · 91 . . . = π6 = 1/6 und k1 = 32, k2 = 35, k3 = 41, k4 = 38,
Diese Wahrscheinlichkeit ist monoton wachsend in r. Für k5 = 28 und k6 = 26 den Wert
r = 10, s = 90 ergibt sich P10 (X ∈ K) = 0.6695 . . ., d. h.,
6  
der Test besitzt das approximative Niveau 0.67. 6 200 2
χn2 (k1 , . . . , k6 ) = kj − = · · · = 5.02
200 6
j =1
Aufgabe 24.48 •• a) Wird H0 : μ ≥ μ0 als Hypothese
gewählt und ein Test zum Niveau 0.01 gegen die Alternative
an. Aus Tabelle 24.3 liest man den kritischen Wert zu
H1 : μ < μ0 durchgeführt, so dient diese Vorgehensweise 2
χ5;0.9 = 9.24 ab. Wegen 5.02 ≤ 9.24 wird die Hypothese
zum einen dem Schutz des Herstellers, denn man würde nur
der Echtheit bei einer zugelassenen Wahrscheinlichkeit von
mit der kleinen Wahrscheinlichkeit 0.01 zu einer falschen
0.1 für einen Fehler erster Art nicht verworfen.
Entscheidung gelangen, wenn in Wirklichkeit μ ≥ μ0 gilt.
Es bedeutet aber auch, dass man im Fall der Ablehnung der
Hypothese praktisch sicher sein kann, dass H0 nicht zutrifft. Aufgabe 24.51 • Mit (24.89) gilt
 n
b) Wegen σ = 2 ist die Prüfgröße des Gauß-Tests nach 1
√ P(X(1) ≤ Q1/2 ≤ X(n) ) = 1 − 2 · .
(24.46) durch Gn = n(X n − μ0 )/2 gegeben. Wegen 2
−1 (0.99) = 2.326 lehnt dieser Test H0 ab, falls Gn ≤
−2.326 gilt, was zur behaupteten Ungleichung äquivalent Wegen
ist. 1
1− ≥ 0.95 ⇐⇒ 2n−1 ≥ 20
2n−1
c) Es sei μ1 := 999. Nach Wunsch der Verbraucherorgani-

sation soll 0.9 = Pμ1 (X n ≤ μ0 − 4.652/ n) gelten. Da muss n mindestens 6 sein.

N := n(X n − μ1 )/2 eine N(0, 1)-Normalverteilung be-
sitzt, wenn μ1 der wahre Parameter ist, folgt Aufgabe 24.52 • Da sieben der acht Differenzen yj −
  xj in der Tabelle zu Aufgabe 24.49 positiv sind, nimmt die
4.652 
0.9 = Pμ1 X n ≤ μ0 − √ Vorzeichen-Testgröße T = 8j =1 1{Zj > 0} den Wert 7 an.
n
 √  Im Fall μ = 0 hat T die Binomialverteilung Bin(8, 1/2). Da
n(μ0 − μ1 )
= P μ1 N ≤ − 2.326 die Wahrscheinlichkeit P(T ≥ 7) monoton mit μ wächst,
2 gilt unter H0 (d. h. für jedes μ ≤ 0) P(T ≥ 7) = 9/256 =
√ 
n(μ0 − μ1 ) 0.0351 . . . Somit wird H0 : μ ≤ 0 auf dem 5%-Niveau ab-
=  − 2.326
2 gelehnt.

Das könnte Ihnen auch gefallen